Engineering Mathematis

You might also like

Download as pdf or txt
Download as pdf or txt
You are on page 1of 89

GATE – SLP

EC / EE / IN / CS / ME / CE
Engineering Mathematics
Module 1 : Linear Algebra, Calculus,
Probability and Statistics

Edition : 0617/0619800

Copyright © reserved with Vidyalankar Classes and Publications.

All rights reserved. No part of this work herein should be reproduced or used either
graphically, electronically, mechanically or by recording, photocopying, taping, web
distributing or by storing in any form and retrieving without the prior written permission of
the publisher. Anybody violating this is liable to be legally prosecuted.

Pearl Centre, S.B. Marg, Dadar (W), Mumbai 400 028.


Tel. No. : 022 – 4232 4232 / 022 – 2430 63 67
web site : www.vidyalankar.org
e-mail : gate@vidyalankar.org
GATE Syllabus
Engineering Mathematics
Electronics & Communications (EC)
Linear Algebra: Vector space, basis, linear dependence and independence, matrix algebra, eigen values and eigen vectors,
rank, solution of linear equations – existence and uniqueness.
Calculus: Mean value theorems, theorems of integral calculus, evaluation of definite and improper integrals, partial derivatives,
maxima and minima, multiple integrals, line, surface and volume integrals, Taylor series.
Dear GATE Aspirant, Probability and Statistics: Mean, median, mode and standard deviation; combinatorial probability, probability distribution
functions - binomial, Poisson, exponential and normal; Joint and conditional probability; Correlation and regression analysis.
The GATE program will help you to overcome the common feeling of confusion about the Electrical Engineering (EE)
exact scope of the syllabus. The course content will enable you to cover the course
Linear Algebra: Matrix Algebra, Systems of linear equations, Eigenvalues, Eigenvectors.
smoothly in the limited time at your disposal without wasting time on unnecessary details, Calculus: Mean value theorems, Theorems of integral calculus, Evaluation of definite and improper integrals, Partial
without omitting any useful part. Derivatives, Maxima and minima, Multiple integrals, Fourier series, Vector identities, Directional derivatives, Line integral,
Surface integral, Volume integral, Stokes’s theorem, Gauss’s theorem, Green’s theorem.
Probability and Statistics: Sampling theorems, Conditional probability, Mean, Median, Mode, Standard Deviation, Random
The program provides graded questions in every topic leading you to deeper and more variables, Discrete and Continuous distributions, Poisson distribution, Normal distribution, Binomial distribution, Correlation
analysis, Regression analysis.
intricate and basic concepts. This will help you to stimulate your own thinking and also
makes the process of learning, enjoyable and highly efficient. Instrumentation Engineering (IN)
Linear Algebra: Matrix algebra, systems of linear equations, Eigen values and Eigen vectors.
Calculus: Mean value theorems, theorems of integral calculus, partial derivatives, maxima and minima, multiple integrals,
The quick methods suggested to tackle questions and the practice that one has to put in Fourier series, vector identities, line, surface and volume integrals, Stokes, Gauss and Green’s theorems.
while solving the problems develops in you the required skill of tackling tricky problems Probability and Statistics: Sampling theorems, conditional probability, mean, median, mode and standard deviation, random
independently and confidently. We are sure you will be in a position to deal with every variables, discrete and continuous distributions: normal, Poisson and binomial distributions.

and any problem most successfully. Computer Science and Information Technology (CS)
Linear Algebra: Matrices, determinants, system of linear equations, eigenvalues and eigenvectors, LU decomposition.
I wish you all the best for your GATE. Calculus: Limits, continuity and differentiability. Maxima and minima. Mean value theorem. Integration.
Probability: Random variables. Uniform, normal, exponential, poisson and binomial distributions. Mean, median, mode and
standard deviation. Conditional probability and Bayes theorem.

Mechanical Engineering (ME)


Thank You
Linear Algebra: Matrix algebra, systems of linear equations, eigenvalues and eigenvectors.
Calculus: Functions of single variable, limit, continuity and differentiability, mean value theorems, indeterminate forms;
evaluation of definite and improper integrals; double and triple integrals; partial derivatives, total derivative, Taylor series (in one
Rashmi Deshpande and two variables), maxima and minima, Fourier series; gradient, divergence and curl, vector identities, directional derivatives,
Director line, surface and volume integrals, applications of Gauss, Stokes and Green’s theorems.

Vidyalankar Group of Educational Institutes Probability and Statistics: Definitions of probability, sampling theorems, conditional probability; mean, median, mode and
standard deviation; random variables, binomial, Poisson and normal distributions.

Civil Engineering (CE)


Linear Algebra: Matrix algebra; Systems of linear equations; Eigen values and Eigen vectors.
Calculus: Functions of single variable; Limit, continuity and differentiability; Mean value theorems, local maxima and minima,
Taylor and Maclaurin series; Evaluation of definite and indefinite integrals, application of definite integral to obtain area and
volume; Partial derivatives; Total derivative; Gradient, Divergence and Curl, Vector identities, Directional derivatives, Line,
Surface and Volume integrals, Stokes, Gauss and Green’s theorems.
Probability and Statistics: Definitions of probability and sampling theorems; Conditional probability; Discrete Random
variables: Poisson and Binomial distributions; Continuous random variables: normal and exponential distributions; Descriptive
statistics - Mean, median, mode and standard deviation; Hypothesis testing.

For EC/EE/IN/ME/CE : Topics in Vector Calculus are covered in different module.



Page
Contents Topics
No.
Pearl Centre, S.B. Marg, Dadar (W), Mumbai  400 028. Tel. 4232 4232 Chapter  2 : Calculus
EC / EE / IN / CS / ME / CE 2.1 Function of single variable 71
2.2 Limit of a function 71
ENGINEERING MATHEMATICS 2.3 Continuity 73
MODULE  1 2.4 Differentiability 76
2.5 Mean Value Theorems 79
INDEX 2.6 Maxima and Minima 80
Page 2.7 Integration 84
Contents Topics
No. 2.8 Definite Integration 93
Chapter  1 : Linear Algebra Notes 2.9 Double Integrals 97
1.1 Determinants 1 2.10 Triple Integrals 104
1.2 Matrices 4 Change of Variables in Double and Triple Integrals
2.11 106
and Jacobians
1.3 Rank of a Matrix 12
Notes 2.12 Application of Integration 111
1.4 System of Linear Equations 14 2.13 Partial and Total Derivatives 114
1.5 Eigen Values and Eigen Vectors 18 2.14 Taylor's Series and Maclaurin's Series 121
1.6 Vectors 21 2.15 Fourier Series 123
Assignment  1 24 List of Formulae 129
Assignment  2 27 Assignment  1 134
Assignment  3 30 Assignment  2 136
Assignment  4 34 Assignment  3 138
Assignment  5 37 Assignment  4 141
Assignments Assignment  6 40 Assignments Assignment  5 144
Assignment  7 43 Assignment  6 146
Assignment  8 46 Assignment  7 148
Assignment  9 48 Assignment  8 150
Assignment  10 51 Assignment  9 152

Assignment  11 54 Test Paper  1 154

Test Paper  1 57 Test Paper  2 156

Test Paper  2 60 Test Papers Test Paper  3 158

Test Paper  3 62 Test Paper  4 160


Test Papers Test Paper  5 162
Test Paper  4 64
Test Paper  5 66 Test Paper  6 164

Test Paper  6 68
Page Page
Contents Topics Contents Topics
No. No.
Chapter  3 : Probability and Statistics Solutions  Probability and Statistics
3.1 Basic Terms 166 Answer Key 308
Assignment
3.2 Definition of Probability 167 Model Solutions 310
3.3 Complement of an event 167 Answer Key 332
Test Paper
3.4 Independent Events 168 Model Solutions 333
3.5 Theorems of Probability 168
3.6 Random Variables 172
Notes
3.7 Probability Distribution Function 173
3.8 Expectation(Mean), Variance and Standard Deviation 173
3.9 Standard Distributions 174
3.10 Mean, Median, Mode and Standard Deviation 178
3.11 Correlation and Regression Analysis 180
List of Formulae 181
Assignment  1 184
Assignment  2 187
Assignment  3 190
Assignments
Assignment  4 193
Assignment  5 195
Assignment  6 198
Test Paper  1 200
Test Papers Test Paper  2 202
Test Paper  3 204
Solutions  Linear Algebra
Answer Key 206
Assignment
Model Solutions 209
Answer Key 240
Test Paper
Model Solutions 241
Solutions  Calculus
Answer Key 255
Assignment
Model Solutions 258
Answer Key 291
Test Paper
Model Solutions 293

Vidyalankar : GATE – Engineering Mathematics

Chapter - 1 : Linear Algebra Interchange of two adjacent rows and columns (Ri  Ri+1, Ci  Ci+1)

If two adjacent rows or columns of the determinant are interchanged, the value of the
1.1 Determinants
determinant so obtained is the negative of the value of the original determinant. Thus
a b
If a, b, c and d are any four terms then the representation   is called a determinant a1 a 2 a3 b1 b 2 b3
c d
b1 b2 b3    a1 a 2 a3 
and is denoted by D.
c1 c2 c3 c1 c2 c3
A determinant of order 2 is evaluated as follows:

a b Multiplication of row or column by factor [Ri (m), Cj(n)]


D=   = ad  bc
c d If the elements of any row or column are multiplied by the same factor, the value of the
A determinant of order 3 can be evaluated as follows: determinant so obtained is equal to the value of the original determinant multiplied by that
factor. Thus,
a1 a 2 a3
b b3 b b3 b b2 ma1 ma 2 ma 3 a1 a 2 a3
D = b1 b2 b 3  = a1  2  a2  1  a 3  1 
c2 c3 c1 c3 c1 c2  b1 b2 b3   m b1 b2 b3 
c1 c2 c3
c1 c2 c3 c1 c2 c3
1 2 3
1 3 2 3 2 1
For example: 2 1 3 = 1   2   3  
1 2 3 2 3 1 Sum of determinants
3 1 2
If any element in any row (or column) consists of the sum of two terms, the determinant
= 1(2  3)  2(4  9) + 3(2  3) = 1 + 10  3 = 6 can be expressed as the sum of two other determinants whose other rows (or columns)
Properties of Determinants remain the same, while the remaining row (or column) consists of these terms
respectively. Thus,
Interchange of rows and columns (Ri  Ci)
a1   1 a 2 a3 a1 a 2 a3 1 a 2 a3
The value of determinant is not affected by changing the rows into the corresponding
b1  1 b2 b3   b1 b2 b3   1 b2 b3 
columns, and the columns into the corresponding rows. Thus
c1   1 c2 c3 c1 c2 c3 1 c2 c3
a1 a 2 a3 a 1 b1 c1
b1 b2 b3   a 2 b2 c2  Change of row or column by multiples of other rows and columns Rij(p), Cij(p)
c1 c2 c3 a3 b3 c3
As the consequence of the properties 5, 4 and 2 we have the result.
Identical rows and columns (Ri = Rj , Ci= Cj). a1 a 2 a3 a1  pa 2 a 2  qa 3 a3
If two rows or two columns of a determinant are identical, the determinant has the value b1 b2 b3   b1  pb2 b2  qb3 b3 
c1 c2 c3 c1  pc2 c2  qc3 c3
zero. Thus,
a1 a 2 a3 a 1 a1 a 3
where care must be taken to leave at least one row or column unaltered in such changes
a1 a 2 a 3   0 b1 b1 b3   0
P and q being any positive or negative factors.
c1 c2 c3 c1 c1 c3

1 2
Notes on Linear Algebra Vidyalankar : GATE – Engineering Mathematics

1.2 Matrices
Note : 1. Area of a triangle whose vertices are (x1, y1) (x2, y2) and (x3, y3) is
A Matrix is a rectangular array of elements written as
x1 y1 1
1  a11 a12 . . . . a1n 
given by the absolute value of x 2 y 2 1 a
2  21 a 22 . . . . a2n 
x3 y3 1
 . . . . 
 
2. Area of a quadrilateral can be found by dividing it into two triangles. A=  . . . . 
 . . . . 
3. If the area of a triangle obtained from the three given points is zero,  
then the three points lie on a line.  . . . . 
a am2 . . . . amn 
 The condition for three points to be collinear is  m1
The above matrix A has m rows and n columns. So it is a m × n matrix or it is said that
x1 y1 1
1 the size of the matrix is m × n.
x2 y 2 1 = 0
2
x3 y3 1
Types of Matrices
Square Matrix :
Solved Example 1 : Solved Example 2 : It is a Matrix in which number of rows = number of columns
Find the area of a triangle whose vertices Find if the three points (1, 1), (5, 7) and
 1 2 3
are (2, 1), (4, 3), (2, 5). (8, 11) are collinear. For example: 4 5 6  is a square matrix of order 3.
 
Solution : Solution : 7 8 9 
The area of the triangle is If the points are collinear, area of the
Diagonal Matrix :
x1 y1 1 2 1 1 triangle formed by the three given points
1 1 It is a square matrix in which all non diagonal elements are zero.
A = x 2 y2 1 =  4 3 1 should be zero.
2 2  1 0 0
x3 y3 1 2 5 1 1 1 1
For example: 0 2 0 
1  
1 Area A=  5 7 1
= × abs [2(3  5)  4(1  5)  2(1 + 3)] 2 0 0 4 
2 8 11 1

1 1 Scalar Matrix :
= × abs [16 + 16  8] = 4 sq. units = [1(7  11)  5(1  11)
2 2 It is a diagonal matrix in which all diagonal elements are equal
+ 8 (1  7)] 4 0 0 
For example: 0 4 0 
1  
= [4 + 60  64] = 0 0 0 4 
2
The given points are collinear.
Unit Matrix :
It is a scalar matrix with diagonal elements as unity. It is also called Identity Matrix.
 1 0
Identity matrix of order 2 is I2 =  
0 1

3 4

Notes on Linear Algebra Vidyalankar : GATE – Engineering Mathematics

 1 0 0 Symmetric Matrix :
Identity matrix of order 3 is I3 = 0 1 0  If for a square matrix A, A = A then A is symmetric
0 0 1  1 4 5
For example: 4 2 8 
 
5 8 3 
Note : For any matrix A, AI = IA = A
Skew Symmetric matrix :
Upper Triangular Matrix : If for a square matrix A, A = A then it is skew  symmetric matrix.
It is a square matrix in which all the elements below the principal diagonal are zero.
 0 5 7
 1 2 0 For example:  5 0 3 
0 0 3   
For example:    7 3 0 
0 0 2 
Note : For a skew symmetric matrix, diagonal elements are zero.
Lower Triangular Matrix :
It is a square matrix in which all the elements above principal diagonal are zero. Orthogonal Matrix :
0 0 0  A square matrix A is orthogonal if AA = AA = I
For example: 2 1 0 
   cos  sin  
 1 2 4  For example: A=  
  sin  cos  
Column Matrix : Here AA = I
It is a matrix in which there is only one column.
Note : For orthogonal matrix A, A1 = A
1 
For example: 3 
  Conjugate of a Matrix :
1  Let A be a complex matrix of order m  n. Then conjugate of A is the matrix obtained by
Row Matrix : taking conjugate of every element in the matrix and denoted by A
It is a matrix in which there is only one row.  7  i 3  3i 4 
For example: if A= 
For example: [2 3 4] 9  2i i 8  4i

Transpose of a Matrix :  7  i 3  3i 4 
then conjugate of A = A  
9  2i i 8  4i
It is a matrix obtained by interchanging rows into columns
 1 3 5 Matrix A :
For example: If A =  
2 3 8  The transpose of the conjugate of a matrix A is denoted by A

 1 2  7  i 2  4i 4 
For example: Let A = 
A = transpose of A = 3 3  3  2i i 1  2i
5 8   7  i 2  4i 4 
then A  
3  2i i 1  2i

5 6
Notes on Linear Algebra Vidyalankar : GATE – Engineering Mathematics

 7  i 3  2i Periodic Matrix :


and A = A    2  4i
 i  A matrix A is called a periodic matrix. AK+1 = A where K is a +ve integer; if K is the least
 4 1  2i  +ve integer for which AK+1 = A, then K is the period of A.

Unitary Matrix : Note : If K = 1, we get A2 = A and it is idempotent matrix.


A square matrix A is said to be unitary if A A = I
Nil potent Matrix :
1 i 1  i 
 A matrix is called a Nilpotent matrix, if AK = 0 where K is a positive integer. If K is the
A=  2 2 
For example: 
1 i 1 i  least positive integer for which AK = 0, then K is the index of the nil potent matrix.
 2 2   ab b2 
For example: A=  2  has index 2
  1 0  a ab 
Here AA=   =I
0 1
Involutory Matrix :
Hermitian Matrix :
A matrix A is called involutory matrix if A2 = I
A square matrix A is called Hermitian matrix if aij = a ji
 0 1 1
 4 1  i 2  5i For example: A = 4 3 4 
For example: A =  1  i 3 1  2i   3 3 4 

2  5i 1  2i 8  Here A2 = I
The necessary and sufficient condition for a matrix A to be Hermitian is that A = A.
Note : I2 = I.  Identity matrix is always involutory.
Skew  Hermitian Matrix :
A square matrix A is skew Hermitian matrix if aij =  a ji Determinant of a square matrix
 2i 2  8i 1  2i Let A be a square matrix, then | A | = determinant of A.
For example:  (2  8i) 0 2i 
  1 2 3
 (1  2i) 4i 
2i For example: A = 2 1 3 
The necessary and sufficient condition for a matrix A to be skewHermitian is that A = A  3 1 2 

1 2 3
Note : All the diagonal elements of a skew Hermitian matrix are either zeroes or
|A| = 2 1 3
pure imaginary.
3 1 2
Idempotent Matrix : 1 3 2 3 2 1
= 1   2   3  
Matrix A is called idempotent matrix if A2 = A 1 2 3 2 3 1
 2 2 4  = 1(2  3)  2(4  9) + 3(2  3) = 1 + 10  3
For example: A =  1 3 4 
=6
 1 2 3 
 If | A |  0 then matrix A is called as nonsingular.
Here A2 = A
 If | A | = 0, A is called singular.

7 8

Notes on Linear Algebra Vidyalankar : GATE – Engineering Mathematics

Adjoint and Inverse of a Square Matrix Inverse of a Square Matrix :

Minor : Consider the determinant For a nonsingular square matrix A

a11 a12 a13 1


A1 = adj(A)
 = a 21 a 22 a 23  |A|
a 31 a 32 a 33 where A1 is called the inverse of square matrix.
To find minor leave the row and column passing through the element aij.
Note : A A1 = A1 A = I
a a13
The minor of the element a21 = M21 =  12 
a 32 a 33
Solved Example 3 : A31 = the cofactor of a31
a a13 Calculate the adjoint of A, 1 1
The minor of the element a32 = M32 =  11 
a 21 a 23 in | A | =   = 5
1 1 1  2 3
a a 23 where A =  1 2 3 
The minor of the element a11 = M11 =  22  A32 = the cofactor of a32 in
a 32 a 33  2 1 3 
1 1
Solution : |A|=    =4
1 3
Cofactor : The minor Mij multiplied by (1)i+j is called the cofactor of the element aij.
A11 = the cofactor of a11 in
A33 = the cofactor of a33 in
2+1 a a13 2 3
The cofactor of the element a21 = A21 = (1) M21 =   12  |A|=   =3 1 1
a 32 a 33 1 3 | A |=   =1
1 2
a a13 A12 = the cofactor of a12 in
The cofactor of the element a32 = A32 = (1)3+2 M32 =   11  Adj (A) = transpose of the matrix formed
a 21 a 23 1 3 by cofactor
|A|=   = 9
2 3  A11 A 21 A 31   3 4 5 
a a 23
The cofactor of the element a11 = A11 = (1)1+1 M11 =  22  =  A12 A 22 A 32    9 1 4 
a 32 a 33 A13 = the cofactor of a13 in
 A13 A 23 A 33   5 3 1 
And so on. 1 2
|A|=   = 5
2 1 Solved Example 4 :
Adjoint of a Matrix :
A21 = the cofactor of a21 in Find the inverse of the matrix by finding its
Adjoint of a square matrix A is the transpose of the matrix formed by the cofactors of the 1 3 3
1 1
| A |=    = 4
elements of the given matrix A. 1 3 adjoint where A = 1 4 3 
 a11 a12 a13  1 3 4 
A22 = the cofactor of a22 in
If A = a 21 a 22 a 23  Solution :
a 31 a 32 1 1
a 33  | A |=   =1 |A| = 1  0
2 3
 A11 A 21 A 31  A1 exists
A23 = the cofactor of a23 in
Then adj (A) =  A12 A 22 A 32   1 1 1
1 1
 A13 A 23 A 33  | A |=    =3 Now A = 3 4 3 
2 1
 3 3 4 

9 10
Notes on Linear Algebra Vidyalankar : GATE – Engineering Mathematics

The cofactors of the elements of A are  150 98 18  Solved Example 7 : Solved Example 8 :
(1) = 7 ; (1) = 3 ; (1) = 3 (adj B) (adj A) =  80 70 10  , Show that Show that
 15 14 1 
(3) = 1 ; (4) = 1 ; (3) = 0  cos  sin    8 4 1 
A=   is orthogonal 1
(3) = 1  150 98 18    sin  cos  A= 1 4 8  is orthogonal and
; (3) = 0 ; (4) = 1 9
adj (AB) =  80 70 10   4 7 4 
 7 3 3  Solution :
 15 14 1 
 Adj (A) =  1 1 0  find A1.
cos   sin  
 1 0 1  Hence we verify (adj B) (adj A) = adj (AB) A =   Solution :
 sin  cos  
 7 3 3   8 4 1   8 1 4 
 cos  sin   cos   sin   1 1
 A1 =
1
adj(A)   1 1 0  Solved Example 6 : AA =     AA = 1 4 8  4 4 7 
A   sin  cos    sin  cos   9 9
 1 0 1  Find the inverse of the matrix finding its  4 7 4   1 8 4 
 cos 2   sin2  0 
2 1 3  =    81 0 0   1 0 0 
sin2   cos2  
adjoint where A = 3 1 2 
0 1
Solved Example 5 :
 = 0 81 0   0 1 0  = I
81 
Find the adjoints of the matrices A and B  1 2 3   1 0  0 0 81 0 0 1
=   =I
Solution : 0 1 Similarly A A = I
1 2 3 
where A = 1 3 4  , 2 1 3  Similarly A A = I  A is orthogonal and therefore
1 4 3  | A | = 3 1 2  = 6  0  A1 exists Hence AA= AA = I
 8 1 4 
 1 2 3   A is orthogonal A1 = A =
1
4 4 7 
 0 4 5 9
B =  1 2 3  .  2 3 1  1 8 4 
 1 1 7  transpose of A = A =  1 1 2 
1.3 Rank of a Matrix
Verify the formula adj(AB) = (adj B ) (adj A)  3 2 3 
Sub-matrix
Solution : The cofactors of the elements of A are
Any matrix obtained by omitting some rows and columns form a given m  n matrix A is
We can find that (2) = 1 ; (3) = 3 ; (1) = 1
called a sub-matrix of A.
 7 6 1 (1) = 7 ; (1) = 3 ; (2) = 5
 a1 b1 c1 
adj A =  1 0 1 , (3) = 5 ; (2) = 3 ; (3) = 1 For example: a b contains
 2 c 2 
 1 2 1  2
 1 3 1
17 33 2  adj (A) =  7 3 5   a b1   a1
three 2 x 2 sub-matrices  1
c1   b1 c1 
  c 2  b2 c 3 
adj B = 10 5 5   5 3 1  a 2 b2   a 2
 1 4 4  two 1  3 sub matrices namely [a1 b1 c1] and [a2 b2 c2] and
1
 A1 = adj(A)
 1 5 32  |A |  a1  b1   c1 
three 2  1 sub matrix namely       and so on
AB =  1 6 42   1 3 1 a2  b2  c 2 
 1 9 38  1 
= 7 3 5  The rank of a matrix is r if :
6 
 5 3 1 i) It has atleast one non-zero minor of order r
ii) Every minor of A of order higher than r is zero

11 12

Notes on Linear Algebra Vidyalankar : GATE – Engineering Mathematics

The rank of a matrix in Row-Echelon form is equal to the number of non-zero rows. 1.4 System of Linear Equations
The rank of a matrix is also given by the number of linearly independent rows.
Consider a set of equations
a1x + b1y + c1z = d1
Note : 1. If A is zero matrix, then r(A) = 0
a2x + b2y + c2z = d2
2. IF A is not a zero matrix, r(A)  1
a3x + b3y + c3 z = d3
3. IF A is a nonsingular n  n matrix then r(A) = n (  | A |  0)
4. r(In) = n The equations can be written in the matrix form as
5. If A is an m  n matrix then r(A)  minimum of m and n  a1 b1 c1   x  d1 
a b c 2   y   d 
 2 2    2
Solved Example 9 : Solved Example 10 : a 3 b3 c 3  z  d3 

4 2 1 3  Find the rank of the matrix which is of the form AX = B


Find the rank of the matrix 6 3 4 7  .  3 2 1 5   a1 b1 c1 : d1 
 2 1 0 1 A = 5 1 4 2  . Augmented matrix, [A : B] = a 2 b2 c 2 : d2 
Solution :  1 4 11 19  a 3 b3 c 3 : d3 

This matrix contains four 3  3 matrices. Solution :


The matrix contains four 3  3 matrices Consistency conditions
 4 2 1 4 1 3 
 6 3 4 6 4 7  After reducing [A : B] to Row-Echelon form, find the ranks of A and [A : B]
     3 2 1 3 1 5 
 5 1 4 5 4 2 
 2 1 0  2 0 1     Case 1 : r(A)  r (A : D), then the system is inconsistent.
 1 4 11  1 11 19 
4 2 3  2 1 3  i.e. it has no solution.
 6 3 7 3 4 7 
    3 2 5   2 1 5 
 5 1 2  1 4 2 
 2 1 1  1 0 1    
Case 2 : r(A : D) = r(A) then the system is consistent and if

The determinants of all these are zero.  1 4 19   4 11 19  (i) r(A : D) = r(A) = Number of unknowns then the system is consistent

Then consider 2 nd
order sub matrices. It The determinants of all these are zero. and has unique solution.

Then consider 2nd order sub matrices. (ii) r(A : D) = r(A) < Number of unknowns then the system is consistent
2 1
can be seen that   has determinant
3 4  and has infinitely many solutions.
3 2 
It can be seen that   has a value 7
whose value is 5. i.e. not zero. Hence the  5 1
Solution of Linear Equations
rank of the matrix is 2. i.e. not zero.
Cramer’s Rule
Hence the rank of the matrix is 2.
The solutions of the equations
a1x + b1y + c1z = d1
a2x + b2y + c2z = d2
a3x + b3y + c3z = d3

13 14
Notes on Linear Algebra Vidyalankar : GATE – Engineering Mathematics

a1 b1 c1
Solved Example 11 : 3 1 5
Let  = a 2 b2 c2 
Solve 3x + y = 19 y = 2 6 0 
a 3 b3 c3
3x  y = 23 1 5 1
The solution is given by
Solution : = 3(6  0) + 1(2  0)  5(10  6)
x y z 1
   3 1 = 18 + 2  20 = 0
d1 b1 c1 a1 d1 c1 a1 b1 d1 a1 b1 c1 Here =   = 6
3 1 3 0 1
d2 b2 c2  a 2 d2 c2  a 2 b2 d2  a 2 b2 c2 
d3 b3 c3 a3 d3 c3 a3 b3 d3 a3 b3 c3 By Cramer’s rule z = 2 7 6 
1 1 5
x y z 1 19 1
   [  0]  
x y z  x 23 1 = 3 (35  6)  0 (10  6)  1(2  7)
x= 
 3 1 = 87  0 + 5 = 92
i.e.  
3 1
y x 138
x  Now x=  =3
x= ,y= ,z= z 19  23  46
   = =7
3  3
y 0
y=  =0
Method of Inversion 3 19  46
 
If the matrix form of the given equations is AX = B and if A  0, then the solution can be y 3 23 69  57 z 92
and y =   = 2 z=  =2
 3 1 3  3  46
obtained as  
3 1
X = A1B
Solved Example 12 : Solved Example 13 :
Non Zero Solutions of Linear Homogenous Equation Solve 3x  5z = 1 Test whether the following equations have
The homogenous equation in x, y, z are 2x + 7y = 6 nonzero solution. If they have such
a1x + b1y + c1z = 0 x+y+z=5 solution obtain the solutions.
a2x + b2y + c2z = 0 Solution : x + y  3z = 0, 3x  y  z = 0,
a3x + b3y + c3z = 0 3 0 5 2x + y  4z = 0
A system of simultaneous linear equations is said to have zero or trivial solutions if all the Here  = 2 7 0 Solution :
unknowns have zero values, and is said to have nonzero solution if at least one of the 1 1 1
1 1 3
unknowns has the nonzero value. = 3(7  0)  0(2  0)  5(2  7) Now 3 1 1 = 0
The necessary condition that the equations have nonzero solutions is = 21 0 + 25 = 46 2 1 4

a1 b1 c1 1 0 5 and hence the equations have nonzero


a 2 b2 c2  = 0 x =  6 7 0  solution. Solving the first two equations
a3 b3 c3 5 1 1 we get
= 1(7  0)  0(6  0)  5(6  35) x y z
 
= 7  0 + 145 = 138 4 8 4

15 16

Notes on Linear Algebra Vidyalankar : GATE – Engineering Mathematics

so that x = 4, y =  8, z = 4, where  1 1 1 Solved Example 17 : 5 3 7 : 4 


is a non-zero constant. These values Rank (A) = 2 since 2 4 3 = 0 Examine the consistency of [A : B] = 3 26 2 : 9 
satisfy the third equation and hence they 3 5 2 5x + 3y + 7z = 4 7 2 10 : 5 

are non-zero solutions. 1 1 3x + 26y + 2z = 9 [A : B] contains four 3 × 3 sub matrices:


and   =2 0
2 4 7x + 2y + 10z = 5 5 3 7  5 7 4 
Solved Example 14 : 3 26 2 3 2 9 
Rank [A : B] = 3 Solution :    
Test whether the following equations have
1 1 1 In the matrix form 7 2 10  7 10 5 
non zero solution
since 4 3 9  = 7  0 5 3 7   x  4  5 3 4   3 7 4
2x + 3y + 4z = 0 3 26 2  y   9  3 26 9 26 2 9 
5 2 11         
x  2y  3z = 0
r(A)  r(A : B) thus the system is 7 2 10  z  5  7 2 5   2 10 5 
3x + y  8z = 0
inconsistent i.e. it has no solution. 5 3 7  All the above 4 sub matrices have
Solution :
A = 3 26 2  determinant value 0
2 3 4
Solved Example 16 : 7 2 10   r (A) = r(A : B)
Now 1 2 3 = 63  0
Examine for consistency Rank of A = r(A) = 2 since | A | = 0 = 2 < 3 (number of unknowns)
3 1 8
3x + y + 2z = 3 5 3  System is consistent and has infinitely
Hence the equations have no nonzero and   = 121  0
2x  3y z = 3 3 26 many solutions.
solution. The only solution is x = 0, y = 0,
x + 2y + z = 4
z=0
Solution : 1.5 Eigen Values and Eigen Vectors
Solved Example 15 : In the matrix form Characteristic Equation
Discuss the consistency of
3 1 2   x  3  Let A be an n × n square matrix. Then [A  I] is characteristic matrix of A, where I is
x+y+z = 1 2 3 1  y    3 
      identity matrix.
2x + 4y  3z = 9  1 2 1  z  4 
|A  I| is characteristic polynomial.
3x + 5y  2z = 11
3 1 2  |A  I| = 0 is characteristic equation of A.
Solution : A = 2 3 1
In the matrix form  1 2 1 
Eigen values
1 1 1  Rank of A = r(A) = 3 since | A | = 8  0
A = 2 4 3  and The roots of the characteristic equation of a matrix are called its Eigen values.
3 1 2 3 
3 5 2 
[A : B] = 2 3 1 3 
Eigen vectors
1 1 1 1   1 2 1 4 
[A, B] = 2 4 3 9  Similarly r(A : B) is also 3.
If  is an Eigen value of A, then a non-zero vector X such that
3 5 2 11 AX = X or [A  I] [X] = 0 is called the Eigen vector of A corresponding to Eigen value .
r(A) = r(A : B)
 System is consistent and has unique
solution.

17 18
Notes on Linear Algebra Vidyalankar : GATE – Engineering Mathematics

Properties of Eigen values and Eigen vectors Solved Example 19 : 0 


Find the eigen vectors for the matrix :  X2 = 0 
 Sum of Eigen values of a matrix is equal to the trace of the matrix.
0 
 Product of Eigen values of a matrix is equal to its determinant. 2 1 1
2 3 2 
  But by definition we want nor zero X2. So
 Eigen values of A and AT are same.
3 3 4  we proceed as follows,
 If A is a triangular or diagonal matrix, Eigen values are the diagonal elements.
Solution : Expanding R1
1
 If an Eigen value of A is , then is an Eigen value of A1. From example (1) we get x1 + x 2 + x 3 = 0

1 = 7, 2 = 1, 3 = 1 We assure any element to be zero say x1
 If Eigen values of A are 1, 2….., then Eigen values of A are k
1k ,  k2 ………
Matrix equation of A in  is (A  I) x = 0 and give any convenient value say 1 to x2
 Eigen vectors of a real symmetric matrix corresponding to different Eigen values are and find x3.
2 1  x1   0 
1
 
orthogonal.   2 3 2   x 2   0  Let x1 = 0, x2 = 1 x3 = 1
 If X is an Eigen vector of A corresponding to an Eigen value, the kX is also an Eigen 3 3 4    x 3  0  0
vector of A corresponding to Eigen value X, where k is a non-zero scalar. x2 = 1 
Case (1) for 1 = 7, matrix equation is
1 
Cayley  Hamilton Theorem  5 1 1   x1  0 
 2 4 2  x   0  Case 3 : for 3 = 1
Every square matrix satisfies its characteristic equation i.e.,    2  
 3 3 3   x 3  0   1 1 1  x1  0 
|A  I| = 0 2 2 2  x  0 
n n1 n2 By Cramer’s rule    2  
  + a1 + a2 …….. + an = 0
3 3 3   x 3  0 
is satisfied by A, x1 x 2 x
   3
6 12 18 By Cramer’s rule we get
 An + a1An1 + a2An2 + …… anI = 0
x1 x x x1 x2 x
  2  3   3
1 2 3 0 0 0
Solved Example 18 : Instead of evaluating the determinant
1  0 
1. Find the eigen values of the matrix. directly we use the formula for its
 x1 = 2   x3 = 0 
2 1 1 expansion which is as follows : 0 
2 3 2  3 
  3  (sum of diagonal elements of A) 2 +
Case (2) for 2 = 1 Matrix equation is Again consider x1 + x2 + x3 = 0
3 3 4  (sum of minors of diagonal elements of A)
Now let x2 = 0 x1 = 1 and x3 = 1
Solution :   |A| = 0  1 1 1  x1  0 
2 2 2  x   0  1 
Characteristic equation of A in  is 3  92 + 15  7 = 0    2  
3 3 3   x 3  0   x3 = 0 
| A  I | = 0  (  1) (  7) (  1) = 0  1
By Cramer’s rule, we get
2 1 1  1 = 7, 2 = 1, 3 = 1
x1 x 2 x
 2 3 2  =0   3
0 0 0
3 3 4

19 20

Notes on Linear Algebra Vidyalankar : GATE – Engineering Mathematics

Solved Example 20 : 1 1 8 Note : Length of the vector is also called as Norm of the vector
 1 2 1 Now A2 = 5 5 2
Verify whether the matrix A =  1 0 3  5 3 0 Normal vector
 2 1 1 A vector whose length is 1 is called a normal vector
16 6 12
satisfies its characteristic equation. A 3 = 14 8 8 i.e. if X = (x1 , x2, ……xn) is a normal vector then x12 + x22 + ……..xn2 = 1

Solution : 2 10 14 If a vector is not a normal one, then it can be converted to a normal vector as follows.

1  2 1 0 0 0  Let X = (1, 3,  7) be a 3vector.


| A   I | =  1  3   A3  2A2 + 4A  18 I = 0 0 0  Let d = 12  32  (7)2  59
2 1 1   0 0 0 
 1 3 7 
Then x   , ,  is a normal vector.
= 3  22 + 4  18 d d d 

1.6 Vectors Orthogonal vector


A vector X is said to be orthogonal to Y if the inner product of X and Y is zero i.e. XY = 0
An ordered set of n number is called an nvector or a vector of order n.
i.e. if X = (x1, x2, …….xn) and
For example: X = (x1 , x2, …..xn ) is an nvector.
Y = (y1, y2, ……..yn) and if X and Y are orthogonal then
The numbers x1, x2, …..xn are called as the components of X.
x1y1 + x2y2 + ……….+ xn yn = 0
The components x1, x2, ……..xn of a vector may be written in a row or a column.
i.e. X = (x1, x2, x3, ………xn) and Linear dependence and independence of vectors
 x1  A system of n vectors x1, x2, ….xn of the same order are said to be linearly dependent if
 
x2  there exists n numbers K1, K2, ….Kn (where all of them are not zero) such that
X = :  are n vectors
  K1X1 + K2X2 + …. + KnXn = 0
: 
x  (where 0 is a null vector of the same order)
 n
The vector are linearly independent only if K1 = K2 = ………Kn = 0
Operation on Vectors
Inner product of two vectors Solved Example 21 : 1  3  0 
Let X = (x1, x2, ……..xn) and Examine for linear dependence i.e. c1  2  + c2  7  =  0 
   
Y = (y1 , y2, …….yn) be two nvectors X1 = (1 2 4)T , X2 = (3 7 10)T  4  10   0 

Then the product XY = x1y1 + x2y2 + …..+ xn yn is called the inner product of two Solution :  c1  3c 2  0 
  2c  7c  =  0 
nvectors We have  1 2   
 4c1  10c 2   0 
Length of a vector 1  3 
X1 =  2  X2 =  7  c1 + 3c2 = 0
Let X = (x1 , x2, ……xn ) be a vector. Then the length of a vector is the positive square
 4  10  2c1 + 7c2 = 0
root of the expression
Let c1x1 + c2x2 = 0 4c1 + 10c2 = 0
x12  x 2 2  .........xn2
(Note 0 on R. H. S. is Zero Vector) Consider first two equations in Matrix form.

21 22
Notes on Linear Algebra Vidyalankar : GATE – Engineering Mathematics

 1 3   c1   0   c1  2c 2  0c1 2c1  c 2  3c 3 Assignment  1


2 7  c   0   3c  3c  3c 2c1  4c 2  0c 3
  2    1 2 3
 c1  2c 2  0c 3 3c1  2c 2  4c 3 Duration : 45 Min. Max. Marks : 30
|A| = 1  0
 system has zero solution 3c1  3c 2  3c 3  0 0 0  Q 1 to Q 10 carry one mark each (B) All the eigen values are positive.
4c1  2c 2  6c 3  0 0 0 
  1. Given that the determinant of the (C) All the eigen values are distinct.
 c1 = c 2 = 0
2c1  c 2  3c 3   0 0 0 
(D) Sum of all the eigenvalues is zero.
 X1, X2 are linear independent.  1 3 0
 c1 + 2c2 = 0 matrix  2 6 4  is 12, the
5. With reference to the conventional
2c1 + c2 + 3c3 =0  1 0 2  Cartesian (x, y) coordinate system, the
Solved Example 22 :
c1 + c 2 + c 3 = 0  2 6 0
Show that vertices of a triangle have the following
Solving we get determinant of the matrix  4 12 8  coordinates: (x1, y1) = (1, 0); (x2, y2)
 1 2 3  2 1 3  0 3 3 
3 2 4  3 4 2  3 0 6  c1 = 2c2 and c2 = c3  2 0 4  = (2, 2); and (x3, y3) = (4, 3). The area
     
 1 3 2   2 2 1 0 4 3  Let c2 = c3 = 1 is [ME  2014] of the triangle is equal to [CE  2014]
 c1 = 2 (A) 96 (B) 24
are linearly dependent. 3 3
(A) (B)
We see that x1 x2 x3 are linearly dependent (C) 24 (D) 96 2 4
Solution :
Consider  1 2 3  2 1 3  0 3 3  4 5
2. Consider a 3 x 3 real symmetric matrix (C) (D)
2  3 2 4    3 4 2   3 0 6  = 0 5 2
 1 2 3 2 1 3  S such that two of its eigen values are
 1 3 2   2 2 1 0 4 3 
c1  3 2 4   c 2  3 4 2  a  0, b  0 with respective eigen 6. At least one eigen value of a singular
 1 3 2   2 2 1 matrix is [ME  2015]
 x1   y 1 
0 3 3  0 0 0  vectors  x 2  , y 2  . If a  b then x1y1 + (A) positive (B) zero
c 3  3 0 6   0 0 0   x 3   y 3  (C) negative (D) imaginary
 0 4 3  0 0 0 
x2y2 + x3y3 equals [ME  2014] 7. We have a set of 3 linear equations in
(A) a (B) b 3 unknowns. 'X  Y' means X and Y
 (C) ab (D) 0 are equivalent statements and 'X ≢ Y'
3. A real (4 X 4) matrix A satisfies the means X and Y are not equivalent
equation A2 = I, where I is the (4 X 4) statements. [EE  2015]
identity matrix. The positive eigen P : There is a unique solution.
value of A is ______. [EC  2014] Q : The equations are linearly
independent.
4. Which one of the following statements
R : All eigen values of the coefficient
is true for all real symmetric matrices?
matrix are nonzero.
[EE  2014]
S : The determinant of the coefficient
(A) All the eigen values are real.
matrix is nonzero.

23 24

Assignment on Linear Algebra Vidyalankar : GATE – Engineering Mathematics

Which one of the following is TRUE? 12. A system matrix is given as follows. 16. Let the eigen values of a 2 x 2 matrix 19. Given that
(A) P  Q  R  S 0 1 1 A be 1, -2 with eigen vectors x1 and x2  5 3   1 0
A  and I , the
(B) P  R ≢ Q  S A =  6 11 6  respectively. Then the eigen values 2 0 0 1
 6 11 5  and eigen vectors of the matrix 3
(C) P  Q ≢ R  S value of A is [EC, EE, IN  2012]
(D) P ≢ Q ≢ R ≢ S The absolute value of the ratio of the A2  3A+ 4I would, respectively, be (A) 15 A + 12 I (B) 19 A + 30 I
maximum eigen value to the minimum (A) 2, 14; x1, x2 [EE  2016] (C) 17 A + 15 I (D) 17 A + 21 I
8. Consider a system of linear equations:
eigen value is________. [EE  2014] (B) 2, 14; x1 + x2, x1  x2
x  2y + 3z = 1,
20. Consider the matrix
(C) 2, 0; x1, x2
x  3y + 4z = 1, and  1 5 3 7  T
13. If A =   and B = 8 4  . AB is (D) 2, 0; x1 + x2, x1 – x2  1 1 
6 2     0 
2x + 4y  6z = k. [EC  2015] 2 2

The value of k for which the system equal to [CE  2017] 17. Consider the matrix P=  0 1 0 .
 
has infinitely many solutions is _____. 2 1 1  1 1 
38 28   3 40  0
(A)   (B)      2 2 
A =  2 3 4  whose eigen values
32 56  42 8 
  1 1 2 
9. The larger of the two eigen values of   Which one of the following statements
43 27  38 32 
(C)   (D)   are 1, 1 and 3. Then Trace of about P is INCORRECT? [ME  2017]
4 5  34 50  28 56 
the matrix   is ____.[CS  2015]
2 1 (A3  3A2) is _______. [IN  2016] (A) Determinant of P is equal to 1
14. The value of x for which all the eigen (B) P is orthogonal
18. Consider the following system of
10. A real square matrix A is called skew- values of the matrix given below are (C) Inverse of P is equal to its
equations:
symmetric if [ME  2016] real is [EC  2015] transpose
2x1 + x2 + x3 = 0,
(A) AT = A (B) AT = A1 10 5  j 4  (D) All eigen values of P are real
x x2  x3 = 0,
(C) AT = A (D) AT = A + A1  20 2  numbers
x1 + x2 = 0.
 4 2 10 
This system has [ME  2011]
Q 11 to Q 20 carry two marks each (A) 5 + j (B) 5  j
(A) a unique solution
11. The system of linear equations (C) 1  5j (D) 1 + 5j
(B) no solution
2 1 3 a  5  (C) infinite number of solutions
     15. The two Eigen values of the matrix
 3 0 1  b  4  has (D) five solutions
 1 2 5   c   14  2 1
    
 1 p  have a ratio of 3:1 for p = 2.
[EC  2014]  

(A) a unique solution What is another value of p for which
(B) infinitely many solutions the Eigen values have the same ratio
(C) no solution of 3:1? [CE  2015]
(D) exactly two solutions (A) 2 (B) 1
(C) 7/3 (D) 14/3

25 26
Assignment on Linear Algebra Vidyalankar : GATE – Engineering Mathematics

Assignment  2 9. The solution to the system of 14. Consider the following 2  2 matrix A
 2 5  x   2  where two elements are unknown and
Duration : 45 Min. Max. Marks : 30 equations       is
 4 3  y  30  are marked by a and b. The eigen
Q 1 to Q 10 carry one mark each  3 2 1 [ME  2016] values of this matrix are 1 and 7.
1. The matrix form of the linear system 4. Given the matrices J = 2 4 2  and (A) 6, 2 (B) 6, 2 What are the values of a and b?
 1 2 6 
dx dy (C) 6, 2 (D) 6, 2  1 4
= 3x  5y and = 4x + 8y is A=   [CS  2015]
dt dt  1 b a 
K =  2  , the product KT JK is
[ME  2014]   10. Let M4 = I, (where I denotes the (A) a = 6, b = 4 (B) a = 4, b = 6
 1
d x   3 5  x  identity matrix) and M  I, M2  I and (C) a = 3, b = 5 (D) a = 5, b = 3
(A)    
dt y   4 8  y  _______. [CE  2014] M3  I. Then, for any natural number k,
15. The number of linearly independent
d x  3 8  x  M1 equals: [EC  2014]
(B)     eigen vectors of matrix
dt y  4 5  y  5. The determinant of matrix
(A) M 4k+1
(B) M 4k+2

0 1 2 3 2 1 0 
d x   4 5  x  (C) M4k+3 (D) M4k
(C)     1 0 3 0  A = 0 2 0  is ______. [ME  2016]
dt y   3 8  y   is ______. [CE  2014] 0 0 3 
2 3 0 1
d x  4 8  x    Q 11 to Q 20 carry two marks each
(D)     3 0 1 2
dt y  3 5  y  11. The maximum value of the 16. Let A be a 4 × 3 real matrix with rank

determinant among all 2 x 2 real 2. Which one of the following


6. The lowest eigen value of the 2  2
symmetric matrices with trace 14 is statement is TRUE? [EE  2016]
2. Which one of the following equations
4 2  (A) Rank of AT A is less than 2.
is a correct identity for arbitrary 3 x 3 matrix   is ______. [ME  2015] _______. [EC  2014]
 1 3
(B) Rank of AT A is equal to 2.
real matrices P, Q and R? [ME  2014]
12. The rank of the matrix [CE  2014] (C) Rank of AT A is greater than 2.
(A) P(Q + R) = PQ + RP 7. For what value of p the following set of
2 2 2 (D) Rank of AT A can be any number
(B) (P  Q) = P  2PQ + Q equations will have no solution? 6 0 4 4 
 2 14 8 18  is _________. between 1 and 3.
(C) det (P + Q) = det P + det Q 2x + 3y = 5 [CE  2015]  
14 14 0 10 
(D) (P + Q)2 = P2+ PQ + QP + Q2 3x + py = 10 17. The system of equations
x+y+z=6
8. The value of p such that the vector 13. The smallest and largest eigen values
3. The determinant of matrix A is 5 and x + 4y + 6z = 20
of the following matrix are:
the determinant of matrix B is 40. The  1 x + 4y + z = 
 2  is an eigen vector of the matrix 3 2 2 
determinant of matrix AB is   4 4 6  [CE  2015] has NO solution for values of  and 
_________. [EC  2014]  3   
2 3 5  given by [EC  2011]
4 1 2
p 2 1  is ______. [EC  2015] (A) 1.5 and 2.5 (B) 0.5 and 2.5 (A)  = 6,  = 20 (B)  = 6,   20

14 4 10  (C) 1.0 and 3.0 (D) 1.0 and 2.0 (C)   6,  = 20 (D)   6,   20

27 28

Assignment on Linear Algebra Vidyalankar : GATE – Engineering Mathematics

18. Let A be the 2  2 matrix with elements 0 xy x y 2 2


Assignment  3
a11 = a12 = a21 = +1 and a22 = 1. Then (C) 0 y  z y 2  z2 
1 z z2 Duration : 45 Min. Max. Marks : 30
the eigen values of the matrix A19 are
[CS  2012] 2 x  y x2  y2 Q 1 to Q 10 carry one mark each 4. The sum of eigen values of the matrix,
(A) 1024 and 1024 (D) 2 y  z y 2  z2  1. One of the eigen vectors of the matrix [M] is [CE  2014]
1 z z2  5 2 
(B) 1024 2 and 1024 2 215 650 795 
 9 6  is [ME  2014]
  Where [M] = 655 150 835 
(C) 4 2 and 4 2
485 355 550 
50 70  1 2 
(D) 512 2 and 512 2 20. Consider the matrix A =   (A)   (B)  
70 80   1 9 (A) 915 (B) 1355

whose eigen vectors corresponding to (C) 1640 (D) 2180


2 1
19. Which one of the following does NOT (C)   (D)  
eigen values 1 and 2 are 1 1 5. If any two columns of a determinant
equal [CS  2013]
 70   2  80  2. For matrices of same dimension M, N 4 7 8
1 x(x  1) x  1 x1 =   and x2 =  ,
(A) 1 y(y  1) y  1 1  50   70  and scalar c, which one of these P = 3 1 5 are interchanged,
1 z(z  1) z  1 properties DOES NOT ALWAYS hold? 9 6 2
respectively. The value of x1T x 2 is
(A) (MT )T = M [EC  2014] which one of the following statements
1 x  1 x2  1 _________. [ME  2017]
2
(B) 1 y  1 y  1 (B) (cM)T = c(M)T regarding the value of the determinant

1 z  1 z2  1 (C) (M + N)T = MT + NT is CORRECT? [ME  2015]


(D) MN = NM (A) Absolute value remains
 3. Given a system of equations: unchanged but sign will change
x + 2y + 2z = b1 (B) Both absolute value and sign will
5x + y + 3z = b2 change
Which of the following is true regarding (C) Absolute value will change but

its solutions? [EE  2014] sign will not change

(A) The system has a unique solution (D) Both absolute value and sign will

for any given b1 and b2 remain unchanged

(B) The system will have infinitely


6. The eigen value of the matrix
many solutions for any given b1
 1 1 5 
A = 0 5 6  are
and b2
[IN  2017]
(C) Whether or not a solution exists 0 6 5 
depends on the given b1 and b2
(A) 1, 5, 6 (B) 1, 5  j6
(D) The system would have no
(C) 1, 5  j6 (D) 1, 5, 5
solution for any values of b1 and b2

29 30
Assignment on Linear Algebra Vidyalankar : GATE – Engineering Mathematics

 1 1 1 Q 11 to Q 20 carry two marks each 13. If the characteristic polynomial of a  5 1
17. Consider the matrix   . Which
7. Let P = 2 3 4  and 11. Which one of the following statements 3 × 3 matrix M over  (the set of real 4 1 
3 2 3  is NOT true for a square matrix A? numbers) is 3  42 + a + 30, a   , one of the following statements is TRUE
 1 2 1 [EC  2014] and one eigen value of M is 2, then the for the eigen values and eigen vectors of
Q =  6 12 6  be two matrices. (A) If A is upper triangular, the eigen largest among the absolute values of this matrix? [CE  2017]
 5 10 5  the eigen values of M is ________.
values of A are the diagonal (A) Eigen value 3 has a multiplicity of
Then the rank of P + Q is _________. elements of it [CS  2017] 2, and only one independent eigen
[CS  2017] (B) If A is real symmetric, the eigen vector exists.
14. Perform the following operations on
values of A are always real and (B) Eigen value 3 has a multiplicity of
 3 4 45 
 1 tanx  positive
8. For A =  , the the matrix  7 9 105  . 2, and two independent eigen
  tanx 1  (C) If A is real, the eigen values of A vectors exist.
13 2 195 
determinant of ATA1 is [EC  2015] and AT are always the same (C) Eigenvalue 3 has a multiplicity of
(i) Add the third row to the second row.
(A) sec2 x (B) cos 4x (D) If all the principal minors of A are 2, and no independent eigen
(ii) Subtract the third column from the
(C) 1 (D) 0 positive, all the eigen values of A vector exists.
first column.
9. Consider the following simultaneous are also positive. (D) Eigenvalues are 3 and 3, and two
The determinant of the resultant matrix
equations (with c1 and c2 being is ____________. [CS  2015] independent eigen vectors exist.
constants): 4  3i i 
12. For given matrix P =  ,
 i 4  3i a 0 3 7
3x1 + 2x2 = c1 2 5 1 3  5 3 
 18. For the matrix A    , ONE of the
15. The matrix A = has  1 3
4x1 + x2 = c2 where i = 1 , the inverse of matrix P 0 0 2 4
 
The characteristic equation for these is [ME  2015] 0 0 0 b normalized eigen vectors is given as
simultaneous equations is [CE  2017] 1 4  3i i  det (A) = 100 and trace (A) = 14. [ME  2012]
(A)
(A) 2  4  5 = 0 24  i 4  3i The value of a  b is ___. [EC  2016]  1   1 
  
(B) 2  4 + 5 = 0 2  2
1  i 4  3i (A)  (B) 
(B) 16. Consider the following linear system.  3  1 
(C) 2 + 4  5 = 0 25 4  3i i     
x + 2y  3z = a  2   2
(D) 2 + 4 + 5 = 0 1 4  3i i 
(C) 2x + 3y + 3z = b  3   1 
24  i 4  3i    
5x + 9y  6z = c 10  5
10. The value of for which the matrix (C)  (D) 
1 4  3i i   1   2 
(D) This system is consistent if a, b and c    
3 2 4  25  i 4  3i  10   5
satisfy the equation [CE  2016]
A =  9 7 13  has zero as an
 6 4 9  x  (A) 7a  b  c = 0
(B) 3a + b  c = 0
eigen value is _______. [EC  2016]
(C) 3a  b + c = 0
(D) 7a  b + c = 0

31 32

Assignment on Linear Algebra Vidyalankar : GATE – Engineering Mathematics

19. The minimum eigen value of the 20. The eigen values of the matrix given Assignment  4
below are
following matrix is
0 1 0  Duration : 45 Min. Max. Marks : 30
3 5 2  0 0 1  [EE  2017]
 5 12 7  
[EC  2013]
   0 3 4  Q1 to Q6 carry one mark each 5. A matrix A has x rows and x + 5
 2 7 5  (A) (0, 1, 3) (B) (0, 2, 3) 1. A square matrix each of whose columns, matrix B has y rows and
(C) (0, 2, 3) (D) (0, 1, 3)
(A) 0 (B) 1 diagonal elements are ‘1’ and 11y columns. Both AB and BA exists.
(C) 2 (D) 3 nondiagonal elements are '0’ is called Then values of x and y respectively
(A) Null matrix are

 (B) Skew symmetric matrix (A) 3, 8 (B) 5, 11


(C) Identity matrix (C) 5, 11 (D) 3, 8
(D) row matrix
6. For what values of x, the matrix

2. If product of two matrices AB exists  3  x 2 2 


 2 4x 1  is singular
does BA exists ? 
 2 4 1  x 
(A) Always exist (B) May exist
(C) Never exist (D) None of these (A) 0, 3 (B) 0, 3
(C) 1, 4 (D) 3, 4
3. If a square matrix A is real and
Q7 to Q18 carry two marks each
symmetric, then the eigen values
7. Evaluate
(A) are always real
1  2
(B) are always real and positive
 =  2 1  where  is one of
(C) are always real and non-negative
2 1 
(D) occur in complex conjugate pairs
the imaginary cube root of unity
4. If A and B are two matrices such that (A) 0 (B) 1
AB and A + B are both defined then A, (C) 2 (D) None of these
B are
8. If a given matrix [A] mn has r linearly
(A) Scalar matrices of same order
independent vectors (rows or columns)
(B) square matrices of same order
and the remaining vectors are
(C) matrices of different order
combination of these r vectors. Then
(D) cannot predict
rank of matrix is
(A) m (B) n
(C) r (D) m  n

33 34
Assignment on Linear Algebra Vidyalankar : GATE – Engineering Mathematics

9. Are following vectors linearly dependent 8 7 9  15. Whether the following equations are  cos  sin  
17. If A =   &
x1 = (3, 2, 7) x2 = (2, 4, 1) 12. The inverse of a matrix 5 10 15  is inconsistent   sin  cos  
 1 2 3 
x3 = (1, 2, 6) x + y + z = 3  sin   cos 
B=   then
(A) dependent (B) independent 15 7 8  3x + y  2z = 2 cos  sin  
1
(C) can’t say (D) none of above (A) 10 5 3  2x + 4y + 7z = 7 cos  (A) + sin  (B) =
2
 5 2 1
(A) Yes 1 0   1 0
(A)   (B)  
10. If A is nonzero column matrix and B  15 7 8  (B) No 0 1 0 1
is a nonzero row matrix then rank of (B)  10 5 3  (C) Can’t say
0 1  0 1
 5 2 1 (D) Can’t be determined. (C)   (D)  
matrix AB is
 1 0  1 0 
(A) always 1  15 7 8 
(B) always = No. of elements in row (C)  10 5 3   5 4
 1 6 3   3 5 
 5 2 1 16. If A =  3 2  & B = 
2
(C) always = No. of elements in  18. If A =   & A = kA + 14I, then
 2 5 1  4 2 
column (D) none of these  2 3 
k=....
(D) does not depends on rows & & A + B  X = 0 then X =
columns 0 2 2 4 2  4 2  (A) 3 (B) 5
13. Rank of matrix  7 4 8  is (A) 9 7  (B) 9 7 
a h g  7 0 4  (C) 1 (D) 5
 1 4   1 4 
11. If A = [x y z], B =  h b f  ,
 g f c  (A) 3 (B) 2 4 2  4 2 
(C) 1 (D) none of these (C) 9 7  (D) 9 7 
x 
 1 4   1 4 
C =  y 
z  14. Eigen values of matrix
 1 0 0 
Then ABC =
A = 2 3 1
(A) ABC is not possible
0 2 4 
(B) [ax2 + by2 + cz2 + 2hxy + 2gzx
(A) 1, 2, 4 (B) 1, 3, 4
+ 2fyz]
(C) [a2x + b2y + c2z + 2abc + 2xyz (C) 1, 2, 5 (D) 1, 3, 4

+ 2fgh]
ax 2  byz  fcz2 
(D)  
2hxy  2gzx  2fyz 

35 36

Assignment on Linear Algebra Vidyalankar : GATE – Engineering Mathematics

Assignment  5 8. The inverse of the matrix 4 2 


11. For the matrix   the eigen value
 1 2 3  2 4 
Duration : 45 Min. Max. Marks : 30  2 1 0  is
  corresponding to the eigen vector
 4 2 5 
Q1 to Q6 carry one mark each 5. Inverse of a matrix 101
101 is
1. If A be any matrix, then matrix B if it (A) exist if matrix is singular  5 4 3   
exists such that AB = BA = I, then it is (B) is unique (A) 10 7 6  (A) 2 (B) 4
 8 6 5 
called A (C) 6 (D) 8
(C)
(A) Transpose of A adj A  5 4 3
(B) Inverse of A (D) is not unique (B) 10 7 6  12. Characteristic root of matrix

(C) Cofactor matrix A  8 6 5   1 2 5


 p q  r s A = 0 3 5  are
(D) Adjoint of A. 6. If P =  10 7 6 
 , Q =  s r  , then
 q p    (C)  8 6 5  0 0 2 
2. The eigen vector corresponding to an  5 4 3 
PQ is equal to (A) 1, 3, 2 (B) 1, 3, 2
eigen values are
 pr  qs ps  qr  (C) 1, 2, 3 (D) 3, 1, 2
(A)   5 4 3 
(A) different (B) unique 
 qr  ps qs  pr  (D) 10 7 6 
(C) nonunique (D) none of these  8 6 5  1 1 1
 pq  rs pr  qs 
(B)   13. The eigen values of 1 1 1 are
qr  ps qs  pr 
3. Rank of matrix which is in echelon
0 1 2 2  1 1 1
form is equal to pr  qs ps  qr 
(C)  9. The rank of the matrix 4 0 2 5  (A) 0, 0, 0 (B) 0, 0, 1

(A) No. of non-zero rows in the matrix qr  ps qs  pr  2 1 3 1 
(C) 0, 0, 3 (D) 1, 1, 1
(B) No. of columns in the matrix  pq  rs pr  qs  (A) 4 (B) 3
(D)  
(C) It is independent of No. of rows  qp  rs qs  pr  (C) 2 (D) 1 1 1 2 3 
and columns 1 3 0 3 
14. The rank of matrix   is
(D) Can’t say Q7 to Q18 carry two marks each 10. The following equations have solutions 1 2 3 3 
 
0 1 1 x + 2y  z = 3 1 1 2 3 
7. Is the matrix 4 3 4  an involutory
4. Cayley Hamilton theorem is
2x  2y + 3z = 2 (A) 3 (B) 4
(A) A matrix can be expressed as sum
3 3 4  3x  y + 2z = 1 (C) 2 (D) 1
of symmetric and skewsymmetric
matrix ? x y+z = 1
matrices
(A) No (A) x = 1, y = 4, z = 4
(B) Every square matrix satisfies its
(B) Yes (B) x = 4, y = 1, z = 1
own characteristic equation.
(C) Can’t say (C) infinite solutions
(C) Inverse of a matrix exists if it is
(D) Cannot be determined (D) it is consistent
singular
(D) None of these

37 38
Assignment on Linear Algebra Vidyalankar : GATE – Engineering Mathematics

15. The following equations have  1 0 Assignment  6


17. If A =   , then the value of k for
solutions.  1 7 
x + 2y + 3z = 6 Duration : 45 Min. Max. Marks : 30
which A 2 = 8A + kI is . . . .
3x  2y + z = 2 (A) 5 (B) 5 Q1 to Q6 carry one mark each 3. If a square matrix A be such that A2 = I
4x + 2y + z = 7  1 2  then it is called as
(C) 7 (D) 7 1. Eigen values of Matrix   are
(A) x = 2, y = 2, z = ½  5 4  (A) nilpotent matrix

(B) x = y = z = 1 3 2   1 0 (A) 6, 1 (B) 6, 1 (B) idempotent matrix


18. If Y =   & Y  2X =  3 2  ,
(C) They are inconsistent  1 4   (C) 6, 1 (D) 6, 1 (C) involutary matrix

(D) Infinite solutions then X = . . . . (D) none of these


2. Given an orthogonal matrix
2 1 1 2 
5 1  4 3  (A)   (B)   1 1 1 1  4. If 1 , 2 , ……..n are the latent roots
16. If A =   & B =  1 2  then  1 1 1 1  1 1 1 1
3 2    of matrix A then A3 has latent roots
1 1  1 1 A 
the matrix X which satisfies the (C)  (D)   1 1 0 0  (A) 1 …….n
   
1 2  2 1 0 0 1 1
equation 3A + X = 2B is given by X = 1 1 1
(B) , , ........
23 9   23 9 
l 1  2 n
 AA  is
T
(A)   (B)  
 7 10   7 10  (C) 13, 23 , …….n3
1/ 4 0 0 0 
 23 9   0 1/ 4 0 0  1 1 1
(C)   (D) None of these  (D) , , ........ 3
(A) 13  2 3 n
 7 10   0 0 1/ 2 0 
 
 0 0 0 1/ 2 
 1/ 2 0 0 0  5 2  1
 0 1/ 2 0 5. If A =   , then A is equal to
 0  3 1
(B)
 0 0 1/ 2 0 
   1 2   1 2 
 0 0 0 1/ 2  (A)   (B)  
 3 5   3 5 
1 0 0 0
0  1 2   1 2
1 0 0  (C)   (D)  
(C)   3 5  3 5 
0 0 1 0
 
0 0 0 1

1/ 4 0 0 0  2x 0  1  1 0
6. If A =   and A =  1 2  ,
 0 1/ 4 0 0   x x  
(D) 
 0 0 1/ 4 0  then value of x is
 
 0 0 0 1/ 4 
(A) 1 (B) 2
(C) 1/2 (D) none of these

39 40

Assignment on Linear Algebra Vidyalankar : GATE – Engineering Mathematics

Q7 to Q18 carry two marks each 3 2  15. If A is orthogonal (A AT = I = AT A) then 17. For a second order matrix A if A 2 = I
12. The eigen value of A =   are
7. A matrix ‘P’ which diagonalises ‘A’ is 6 5  | A | is then A is equal to
called (A)  3 33 (B) 3, 1 (A)  0 i 0  1 0 
(A)   (B)  
(A) Spectral matrix (B) Modal matrix (B) 1 0 1 0 i
(C) 1  2 7 (D) None of these
(C) Square matrix (D) None of these (C) 1 or 1  i 0  1 0 
(C)   (D)  
 cos  sin  0  (D) can be any value 0 i  0 i
 1 1 0 13. Given A =   sin  cos  0  , indicate
8. If A = 0 1 1 , then A1 is equal to 2 0.1
 0 0 1 16. Let A =  and
0 0 1 0 3   5 4 0 
the statement which is not correct for A, 18. If A = [1, 2, 3], B =  0 2 1 , then
(A) A 2 2
(B) A  3A + 3I3 1/ 2 a 
(A) It is orthogonal A 1     1 3 2 
(C) A (D) A2 + 2A  2I3  0 b
(B) It is non singular AB =
Then (a + b) =
(C) It is singular
9. If the rank of the matrix  2 
(A) 7/20 (B) 3/20
(D) A1 exists (A)  1 (B)  2 1 4
 1  1 (C) 19/60 (D) 11/20
A=   is 1, then the value of  4 
 1 1  14. The eigen values and the
(C)  4 1 2 (D) None of these
 is corresponding eigen vectors of a 2  2
(A) 1 (B) 1 matrix are given by
(C)  1 (D) None of these Eigen value Eigen vector

1
10. Rank of a unit matrix of order n is 1 = 8 v1 =  
1
(A) 1 (B) 2
1 
(C) 0 (D) n 2 = 4 v2 =  
 1
11. For what value of , the system of The matrix is
equations 6 2  4 6 
(A)   (B)  
3x  y + z = 0 2 6  6 4 
15x  6y + 5z = 0 2 4  4 8 
(C)   (D)  
x  2y + 2z = 0 4 2  8 4 
has non  zero solution.
(A) 1 (B) 2
(C) 6 (D) None of these

41 42
Assignment on Linear Algebra Vidyalankar : GATE – Engineering Mathematics

Assignment  7 8. The eigen vectors of the matrix 11. The eigen values of a matrix
0 0   5 4 
0 0 0  ,   0 is (are)] A=   are
Duration : 45 Min. Max. Marks : 30    1 2
0 0 0  (A) 5, 2 (B) 1, 6
Q1 to Q6 carry one mark each 6. Find the eigen values of the matrix and
(i) (a, 0, ) (ii) (, 0, 0) (C) 4, 5 (D) 1, 5
1. Eigen value of an inverse of matrix is state that whether it is diagonal or not
(iii) (0, 0, 1) (iv) (0, , 0)
(A) Same as the matrix  8 8 2 
1 2 3 0
A = 4 3 2 
(A) (i), (ii) (B) (iii), (iv)
(B) Negative of matrix values 2
(C) (ii), (iv) (D) (i), (iii) 4 3 2 
(C) Inverse of the matrix values  3 4 1  12. The rank of matrix  is
3 2 1 3
(D) No any relation between them  
(A) Eigen value 1, 2, 3 & diagonal 9. The rank of the following 6 8 7 5
(B) Eigen value 1, 2, 3 and (n + 1)  (n + 1) matrix, where a is a (A) 1 (B) 2
2. Rank ‘n’ of nonzero matrix
non diagonal real number is (C) 3 (D) 4
(A) may be n = 0 (B) may be n > 1
(C) Eigen value 1, 1, 3 and 1 a a2 . . . an 
(C) may be n = 1 (D) may be n  1  
non diagonal 13. The following set of equations
1 a a2 . . . an 
(D) Eigen value 1, 1, 3 and diagonal   3x + 2y + z = 4
 1 0 . . . . 
3. The rank of matrix   is . . . .  xy+z =2
0 1  
Q7 to Q18 carry two marks each . . . . 
2x + 2z = 5 have
(A) 3 (B) 2 1 a a2 . . . an 
7. The inverse of matrix  (A) No solution
(C) 1 (D) 0
2 1 3  (A) 1 (B) Unique solution
4. For what value of  do the equations x A =  1 1 1 is (B) 2 (C) Multiple solution
 1 1 1 (C) n (D) An inconsistency
+ 2y = 1, 3x + y = 3
(D) Depends on the value of a
have unique solution  1 1 2 
(A)  = 6 (B)   6 (A)  0 1/ 2 1/ 2  14. The matrix form of quadratic equations

(C)  = 5 (D)   5  1 1/ 2 3 / 2  10. The characteristic equation of matrix 6x12 + 3x22 + 14 x32 + 4x2x3 + 18x3x1 +

 1 0 1  0 h g  4x1x2 …is
(B)  1 1/ 2 1/ 2  A = h 0 f  is 6 2 9 
5. The formula for A1 is given by
g f 0 
 2 1/ 2 3 / 2  (A) 2 3 2 
(A) A1 = Adj A | A |
(A) 3   (f2 + g2 + h2)  2fgh = 0 9 2 14 
(B) A1 = Adj A  | adj A |  1 1 2 
(C)  1 0 1/ 2  (B) 2   (f2 + g + h)  2fgh = 0  6 4 18 
|A|
(C) A1 =
adjA  1 1/ 2 3 / 2  (C) 3 +  (f2 + g2 + h2)  2f2gh = 0 (B)  4 3 4 
18 4 14 
(D) None of these  1 1 2  (D) 2 +  cf2 + g2 + h2)  2fgh = 0
(D)  1/ 2 1/ 2 1/ 2 
 3 / 2 1/ 2 3 / 2 

43 44

Assignment on Linear Algebra Vidyalankar : GATE – Engineering Mathematics

 6 2 9   x 1   y 1  1 2 Assignment  8
17. If  1 y   3 x  = 2 1 , then
(C)  2 3 2       
 9 2 14  Duration : 45 Min. Max. Marks : 30
(x, y) = . . . .
 6 4 18  (A) (1, 0) (B) (1, 1) Q1 to Q6 carry one mark each (C) A is any matrix
(D)  4 3 4  (C) (0, 1) (D) (2, 1) 1. Matrix A and B are square matrices of (D) It's inverse is unique
18 4 14 
order n. Then B is said to be similar to 3 4 5 
4 1 2   
15. If P and Q are both unitary matrices 0 a b A if there exists a nonsingular matrix 6. If    1 0 2 
0 5 3   3 4 7 
then product PQ is 18. If  = a 0 c , then . . . P such that  
b c 0
(A) nonunitary (B) unitary (A) B = PAP1 (B) P AP 8x  3y 6z 32 
=  , the
(C) can’t say (D) none of these (A)  = 2abc (B)  = 0 (C) B = P1A P (D) B = PAP  4 12 26x  5y 
(C)  = abc (D)  = a 2  b2  c 2 values of x, y, z are
 1 2 2. For m number of equations, n number
16. If A =   , then adj A is (A) x = 3, y = 4, z = 1
2 1 of columns, let r be rank of m  n
coefficient matrix, then for r < n, the (B) x = 0, y = 1, z = 4
 1 2  2 1
(A)   (B)   (C) x = 1, y = 3, z = 4
 2 1  2 1 solution of matrix is
(A) zero matrix (D) x = 1, y = 3, z = 4
 1 2   1 2 
(C)   (D)   (B) unit matrix Q7 to Q18 carry two marks each
 2 1  2 1
(C) infinite number of solutions 7. Find the rank of matrix
(D) solution not possible 1 3 4 3

A =  3 9 12 9 
1 bc a(b  c)
 1 3 4 3 
3.  = 1 ca b(c  a) is equal to
1 ab c(a  b) (A) 1 (B) 2 (C) 3 (D) 4

(A) 0 (B) 1 8. If the characteristic roots of matrix A


(C) a + b + c (D) ab + bc + cd are 1, 2, 3, 4….n, then the

4. The multiplication of matrix is characteristic roots of A2 are


(A) Squares of eigen values of A
(A) always commutative
(B) Square roots of eigen values of A
(B) may be commutative
(C) Reciprocal of eigen values of A
(C) not always commutative
(D) Does not have any relation.
(D) can’t say

5. If the matrix A has inverse, then which 9. Are following vectors linearly
is wrong dependent
(A) A is non singular x1 = (1, 1, 1, 3) , x2 = (1, 2, 3, 4)
(B) | A |  0 x3 = (2, 3, 4, 9)

45 46
Assignment on Linear Algebra Vidyalankar : GATE – Engineering Mathematics

(A) Yes (B) No 14. The value of determinant of matrix Assignment  9


(C) Can’t say (D) None of these 1 a a2 a 3  bcd 
  Duration : 45 Min. Max. Marks : 30
10. Consider the system of equations 1 b b2 b3  cda 
  is
1 c c2 c 3  dab  Q1 to Q6 carry one mark each cos   sin  
x + 2y + z = 6
  (C)  
1 d d2 3
d  abc  1. A square matrix U is called unitary if  sin  cos  
2x + y + 2z = 6
x+y+z=5 (A) a3 . b3 .c3 . d3  abcd (A) U = U1 (B) U = U1  sin  sin  
(D)  
This system has (B) a2 b2 c2 d2  abcd (C) U = U1 (D) U = U1  sin  cos  
(A) Unique solution (C) 0
2. If A is Hermitian matrix, then IA is
(B) Infinite no. of solution (D) 1 1 0 5 3
(A) Symmetric  2 1 6 1
 8 29 18 
(C) No solution 6. If A =  , then value
15. Eigen values of matrix  0 7 29  are (B) Hermitian 3 2 7 1
(D) exactly two solutions  
 0 0 18  (C) Skew  Hermitian  4 4 2 0
11. The eigen value of a matrix
(A) 8, 7, 29 (B) 8, 7, 18 (D) None of these of A + A
5 8 5 
0 7 12  are (C) 18, 7, 8 (D) 29, 18, 29  2 2 8 7 
  3. If A A = I, then | A | is equal to  2 2 8 3 
0 0 13  1 1 1 (A)  
(A) +1 (B) 1  8 8 14 1
16.  4 3 2 =.....  
(A) 5, 8, 5 (B) 6, 7, 13 (C)  1 (D) 0  7 3 1 0 
(C) 5, 7, 13 (D) 5, 12, 13 42 32 22
 2 2 8 7 
(A) 2 (B) 2 4. The matrix A and B are given below :  2 2 8 3 
12. Determine the eigen values of the (B)  
(C) 1 (D) 0 1 1  1 0  8 8 14 1
matrix A =  , B =  1 0  . The value of  
1 1 1 1 1    7 3 1 0 
 2 1  2i
A =   17.  a b c  =.... AB is  2 8 2 7 
1  2i 2   2 8 2 3 
a 2  bc b2  ca c 2  ab 0 0   1 0
(A) 2, 2, (B) 1, 1 (A)  (B)  (C)  
   8 14 8 1
(A) 1 (B) 0 0 0   0 1  
(C) 3, 3 (D) 1, 3  7 1 3 0 
(C) 1 (D) None of these 1 1  1 0 
(C)   (D)  
13. Find the rank of matrix 1 1  1 0   2 2 7 2 
18. The value of the determinant  2 2 3 2 
 2 2 0 6 (D)  
1 2 3 14 8 1 8 
4 2 0 2   cos  sin    
A=  0 sec x tanx  is . . . . 5. Inverse of a matrix   sin  cos   is 0 7 0 3
   
 1 1 0 3 0 tan x sec x
 
 1 2 1 2   sin  cos  
(A) 1 (B) 0 (A)  
(A) 1 (B) 2  cos  sin  
(C) tan2 x (D) sec 2 x
(C) 3 (D) 4   sin  sin  
(B)  
 cos  cos  


47 48

Assignment on Linear Algebra Vidyalankar : GATE – Engineering Mathematics

Q7 to Q18 carry two marks each (A) x = y = z = 1  3 1 where A  square matrix,


13. If A =   , the value
a  ic b  id  (B) x = 1, y = z = 0  1 2  L  lower triangular matrix
7. A =   is unitary matrix if
b  id a  ic  (C) x = y = z = 0 A2  5A + 7I U  Upper triangular matrix
and only if (D) none of these
9 1 1 9 
(A)   (B)  
(A) a2 + b2 + c2 = 0  1 4 1 4  16. Tp , Tq , Tr are the pth, qth & rth terms of
10. The matrix form of quadratic
(B) b2 + c2 + d2 = 0 0 0   1 0
expression (C)  (D)  Tp Tq Tr
(C) a2 + b2 + c2 + d 2 = 1  
2x12 + x22  3x32  8x2x3 4x3x1 0 0  0 1 an A.P. then  p q r  equals
(D) a2 + b2 + c2 + d 2 = 0
+ 12x1x2 is 1 1 1
 cos  sin  
8. The quadratic expression in matrix 14. If A =   , then value of (A) 1 (B) 1
 2 12 4    sin  cos  
form are (A) 12 1 8  (C) 0 (D) p + q + r
An is
x2 + 4y2 + 9z2 + t2  12yz + 6zx  4xy  4 8 3 
 ncos  nsin   17. If 1, , 2 are three cube roots of unity,
 2xt  6zt (A)  
 2 6 2   nsin  ncos  
 1 2 3 1 (B)  6 1 4  1 2 
 2 4 6 0   2 4 3    cosn  sinn 
(B)   then   1 2  is equal to
(A)  
 sinn  cosn 
 3 6 9 3  2  1
   2 6 4 
 1 0 3 1   cosn sinn 
(C)  6 1 4  (C)   (A) 1 (B) 
 4 4 3    sinn cosn 
 1 4 6 2  (C) 2 (D) zero
 4 4 12 0 
cosn  sinn 
(B)    2 6 2  (D)  
 6 12 9 6   sinn cosn 
  (D)  6 1 4  18. If 1, , 2 are three cube roots of unity,
 2 0 6 1  2 4 3 
1 n 2n
 1 2 3 1 15. Which of the following statement is
 2 4 6 then 2n n  has the value....
0  11. Find the rank of matrix 1
(C)  wrong ?
 3 6 9 3 n 2n 1
 1 1 1 (A) (ABC)1 = C1B1A1
 
1 0 3 1 A = 2 3 4  (A) 1 (B) 2
(B) (ABC)T = CT AT BT
 1 4 6 2  8 2 3  (C)  (D) zero
 4 4 1 1
 12 0  (A) 1 (B) 2 (C) A = (A + A) + (A  A)
(D) 2 2
 6 12 9 6 
  (C) 3 (D) 4 provided A is square matrix
 2 0 6 1
12. Find eigen value of the matrix (D) A = LU,

9. The solution of the equation  4 3 


 1 2  
x + 2y + 3z = 0  
3x + 4y + 4z = 0 (A) 1, 5 (B) 1, 5
7x + 10y + 12z = 0 (C) 1, 5 (D) 1, 5

49 50
Assignment on Linear Algebra Vidyalankar : GATE – Engineering Mathematics

Assignment  10 9. Write quadratic equation in matrix form  5 2 4 


2x12 + x22  3x32  8x2x3  4x3x1 + 12. The inverse of a matrix  2 1 1
Duration : 45 Min. Max. Marks : 30 12x1x2
 4 1 0 

Q1 to Q6 carry one mark each 3 1 2  2 6 2  is

3 2 9  5. P = 2 3 1 = 8 then (A)  6 1 4   1 4 6 


1 
1. Rank of matrix  6 4 18  is 1 2 1  2 4 3  (A)  4 16 13 
37
12 8 36 
1 3 2  2 12 4   6 13 1 

(A) 2 (B) 3 Q = 3 2 1 = ? (B) 12 1 8   1 4 6 


1 
2 1 1  4 8 3  (B)   4 16 13 
(C) 2 (D) 1 37
(A) 8 (B) 8  2 6 2   6 13 1 
2. The value of the determinant for an
(C) 16 (D) 16 (C)  6 1 4   1 4 6 
1 
upper triangular matrix is equal to  2 4 3  (C)  4 16 13 
6. The rank of matrix 36
(A) sum of elements along principal
 2 12 4   6 13 1 
1 2 3 0
diagonal 2 (D)  12 1 8 
 4 3 2   1 4 6 
1 
(B) product of the elements along A =
3 2 1 3
is  4 8 3  (D) 4 16 13 
  36 
principal diagonal
6 8 7 5  6 13 1
(C) sum of all elements 10. The characteristic roots of the matrix
(A) 1 (B) 2
(D) product of all elements  6 2 2   0 2 3
(C) 3 (D) 4
A =  2 3 1 are 13. For A =  2 0 5  matrix which of
3. The skew matrix always have rank ‘r’ Q7 to Q18 carry two marks each
 2 1 3   3 5 0 
equal to  2 1 
 0  (A) 2, 0, 8 (B) 2, 2, 8 following statement is false
(A) r = 1 (B) r > 1  6 3
 1 1 1  (C) 2, 2, 8 (D) 0, 2, 8 (A) A is square matrix
(C) r = 0 (D) can’t say 7. The matrix   is (B) A is skew symmetric matrix
 2 6 3
4. The quadratic from of the symmetrical  1 1 1  11. The rank of the matrix (C) A is symmetric matrix
 
matrix  2 6 3 (D) | A | = 0
1 4 8 7 
(A) orthogonal (B) not orthogonal 0 0 3 0 
diag. [1, 2, … n ] is   is
(C) singular (D) none of these 4 2 3 1 
(A) 1 + 2 + …. n    1 2 3
8. The eigen values of matrix
 3 12 24 21 14. The rank of matrix A =  4 5 8 
(B) 12x1 + 22x2 2
+ ….. n xn
(A) 1 (B) 2  3 2 1
(C) 1x12 + 2x22 + …. + nxn2 2 3 4 
 0 2 1 are (C) 3 (D) 4 is
(D) 1x1 + 2x2 + …… + nxn  
 0 0 1 (A) 1 (B) 2

(A) 2, 2, 1 (B) 2, 2, 1 (C) 3 (D) 4

(C) 3, 4, 1 (D) 3, 4, 1

51 52

Assignment on Linear Algebra Vidyalankar : GATE – Engineering Mathematics

15. The value of the determinant 17. The roots of the equation Assignment  11
1 2 3 4 5 ax b c
2 3 4 5 1  0 bx a  = 0 are Duration : 45 Min. Max. Marks : 30
1 2 3 4 5 is 0 0 cx
Q1 to Q6 carry one mark each 4. If A & B are arbitrary square matrices
4 5 1 2 3 (A) a & b (B) b & c 1  1  1 17  of the same order, then
5 1 2 3 4
(C) a & c (D) a, b & c 1. If x 1  y  1  z 0   15  , (A) (AB) = AB
(A) 1, 00, 000 (B) 10,000 1 0   1 13 
(B) (A) (B) = B A
(C) 1,000 (D) 0
1 x 2 3 x  (C) (A + B) = A  B
18. If  0 x 0 = 0, then its roots then  y  = . . . . . (D) (AB) = B A
16. If a + b + c = 0, one root of the
0 0 x  z 
ax c b
are . . . . 5. Which of the following matrices is not
equation  c bx a  =0 4 2
b a cx
(A) 1 only (B) 0 only (A) 11 (B)  4  invertible
(C) 0 & 1 (D) 0, 1 & 1  2  11  1 1  1 1
is . . . . (A)   (B)  
0 1  1 2 
(A) x = 1 (B) x = 2 11
(C)  4  (D) None of these 2 3   2 2 
(C) x = 0 (D) x = a 2  b2  c 2 (C)   (D)  
 2  4 6  1 1 

2. If A & B are matrices such that A + B 6. If A is a square matrix, then AA + AA

and AB are both defined then : is a

(A) A & B can be any matrices (A) Unit matrix

(B) A & B are square matrices, not (B) Null matrix

necessarily of the same order (C) Symmetric matrix

(C) A & B are square matrices of the (D) Skewsymmetric matrix

same order
Q7 to Q18 carry two marks each
(D) No. of columns of A = no. of rows
7. A = (aij) is a 3  2 matrix, whose
of B.
elements are given by
3. If the matrix AB = 0, then we must aij = 2i  j if i > j
have = 2j  i if i  j
(A) Both A & B should be zero Then the matrix A is
(B) At least one of A & B should be  1 0  1 3
zero (A)  3 2  (B)  3 2 
(C) Neither A nor B may be zero  1 1  1 1
(D) A + B = 0

53 54
Assignment on Linear Algebra Vidyalankar : GATE – Engineering Mathematics

 1 3  1 0  1 0 0 14. If , ,  are real numbers, then The eigen vector corresponding to this
(C)  3 2  (D)  3 2  (C)  2 3 1 1 cos(   cos() eigen value is . . .
 5 4   1 1  2 1 0  cos( - ) 1 cos() is 3 4
(D) None of these cos( -  ) cos( -  ) 1 (A)  2  (B)  2 
7 0  3 0   3   4 
8. If X + Y =   & X  Y = 0 3  , equal to
2 5   
p 0 0 0 (A) 1 5
then the matrices X & Y are
11. The value of 
a q 0 0
 is . . . (B) cos  cos  cos  (C)  2  (D) None of these
2 0  5 0  b c r 0  5 
(A) X =   , Y =  1 4 (C) cos  + cos  + cos 
 1 1   d e f s
(D) None of these
5 0  2 0  (A) p + q + r + s (B) 1
(B) X =   , Y =  1 4 2 22 23 1 2 4
 1 1   (C) ab + cd + ef (D) pqrs 15. The characteristic equation of the 17. If 1 = 3 32 33  &  = 1 3 9 ,
5 0  2 0 0 2 4 4 42 4 3
1 4 16
(C) X =  , Y =
1 
12. The determinant matrix A =  1 1 2  is . . .
 1 4 1 then
a b a  b  2 0 5 
2 0  5 0  (A) 1 = 2 (B) 1 = 3
(D) X =   , Y =  1 1  b c b  c = 0 if
(A) x 3  6x 2  11x  6  0
 1 4   a   b b  c 0 (C) 1 = 4 (D) 1 = 24
(B) x 3  6x 2  11x  6  0
 1 0  0 1 (A) a, b, c are in A.P.
9. If I =   , J =  1 0  & (C) x 3  6x 2  11x  6  0 18. Let a, b, c, d u, v be integers. If the
 0 1   (B) a, b, c are in G.P. or (x  ) is a
(D) None of these system of equations ax + by = u,
 cos  sin   factor of ax 2  2bx  c
B=   then B equals cx + dy = v has a unique solution in
  sin  cos  (C) a, b, c are in H.P.
16. The characteristic equation of the integers, then
(A) I cos  + J sin  (D)  is a root of ax 2  2bx  c = 0
 3 10 5  (A) ad  bc = 1
(B) I sin  + J cos   2 3 4 
matrix A = has a (B) ad  bc = 1
 
(C) I cos   J sin  13. If a1, a2, . . . . form a G.P. & ai > 0 for  3 5 7  (C) ad  bc = ±1
(D) I cos  + J sin  all i  1, then repeated root. (D) ad  bc need not be equal to ±1
1 log am logam 1 logam  2
 1 2 3 
logam  3 logam  5 
10.  0 1 2 
logam  4
=.....
logam  6 logam  7 logam  8 
 0 0 1 
is equal to
1 2 5 
(A)  0 0 2  (A) log am  8  log am
 0 0 1  (B) log am  8  logam
 1 2 7  (C) zero
(B)  0 1 2 
(D) log am  4 2
 0 0 1 

55 56

Test Paper on Linear Algebra Vidyalankar : GATE – Engineering Mathematics

Test Paper  1 (A) equal to 3 11. Given that f(x) = x2  5x + 6,


(B) less than 3  3 1
Duration : 30 Min. Max. Marks : 25 A=   then f(A) = ?
(C) equal to or less than 3  1 2 
Q1 to Q5 carry one mark each 4 1 (D) can’t say
1 0   1 0 
5. For the matrix   , the eigen (A)   (B)  
1. A square matrix having determinant  1 4
9. Consider the following equation 0 1  0 1
equal to zero is called values are  1 0  1 0 
x+ y+z=9 (C)  (D) 
(A) scalar matrix  
(A) 3, 3 (B) 3, 5 2x +5y + 7z = 52  0 1  0 1
(B) singular matrix (C) 3, 5 (D) 5, 0 2x + y z = 0
(C) nonsingular matrix
(A) They have a unique solution  2 2 4 
(D) skew symmetric matrix Q6 to Q15 carry two marks each (B) They posses multiple solutions 12. Is the given matrix  1 3 4 
 0  tan  / 2  (C) They have no solution  1 2 3 
2. The values of x, y, z and a which 6. If A =   then
 tan  / 2 0  (D) The coefficient matrix is idempotent matrix ?
satisfies the matrix equations
cos   sin   orthogonal (A) Yes
 x  3 2y  x  0 7   sin  cos   = . . . .
 z  1 4a  6   3 2a  are   (B) No
   
0 c b 
2 2 (C) Can’t say
(A) x = 3, y = 2, z = 4, a = 3 (A) (I  A) (B) (I  A) 10. If A =  c 0 a  ,
(D) Cannot be determined
(B) x = 3, y = 2, z = 4, a = 1/3 I A  b a 0 
(C) I  A 2 (D)
(C) x = 3, y = 2, z = 4, a = 3 I A
 a 2 ab ac 
   cos   sin  0 
(D) x = 3, y = 2, z = 4, a = 1/3 7. The relationship between following B = ab b2 bc  Then AB is 13. Inverse of a matrix  sin  cos  0 
 2
vectors  0 1
3. Transpose of the product of two  ac bc c  0
x1 = (1, 3, 4, 2), x2 = (3, 5, 2, 2 ),
matrices is  2bc abc abc   cos   sin  0 
(A) product of matrices in reverse
x3 = (2, 1, 3, 2) is (A) abc abc ac  (A)  sin  cos  0 
order (A) 2x1 + x2  2x3 = 0 abc ac abc   0 0 1
(B) x4 + x2 + x3 = 0
(B) product of transposes of matrices  2bc ab ac   cos  sin  0 
in same order (C) x1 + x2  2x3 = 0 (B)  ab 2ac ab  (B)   sin  cos  0 
(C) product of transpose of matrices in (D) x1  2x2 + x3 = 0  ac ab 2ba   0 0 1

reverse order 0 0 0   cos  sin  0 


8. The points (x1, y1), (x2, y2), (x3, y3) are
(D) product of matrices in same order (C)  0 0 0  (C)  sin  cos  0 
collinear if and only if the rank of
 0 0 0   0 0 1
4. Any matrix is said to be orthogonal if matrix
 0 ab ac   sin  cos  0 
its determinant is  x1 y1 1
x  ab 0 bc  (D)  cos  sin  0 
(A) 0 (B) 1  2 y 2 1 is (D)  
 x 3 y 3 1  ac bc 0   0 0 1
(C)  2 (D) None of these

57 58
Test Paper on Linear Algebra Vidyalankar : GATE – Engineering Mathematics

1 1 1  8 1 3 6 Test Paper  2
14. The eigen values of 1 1 1 are 15. The rank of matrix  0 3 2 2
 
1 1 1  8 1 3 4  Duration : 30 Min. Max. Marks : 25
(A) 0, 0, 0 (B) 0, 0, 1 is Q1 to Q5 carry one mark each  4 9 12 
(B)  
(C) 0, 0, 3 (D) 1, 1, 1 (A) 1 (B) 2 1. Every square matrix can be  8 8 28 
(C) 3 (D) 4 represented as  6 9 12 
(C)  
(A) sum of lower and upper triangular  8 8 28 
 matrix
4 9 8 
(D)  
(B) sum of symmetric and skew  8 8 28 
symmetric matrix
(C) product of symmetric and skew 5. Consider the following determinant

symmetric matrix. 1 a bc
 = 1 b ca which of the following
(D) product of Hermitian & skew
1 c ab
hermitian matrix.
is a factor of  ?
2. Sum of eigen values of a matrix is
(A) a + b (B) a  b
equal to
(C) a + b + c (D) abc
(A) sum of all elements
(B) sum of principal elements Q6 to Q15 carry two marks each
(C) sum of diagonal elements 6. Consider the following equations
(D) none of these 2x + 6y + 11 = 0

x 5  3 4   7 16  6x + 20y  6z + 3 = 0
3. If 2    
 7 y  3   1 2  15 14  6y  18z + 1 = 0

then (A) They have a unique solution


(A) x = 2, y = 9 (B) x = 2, y = 5 (B) No solution
(C) x = 4, y = 6 (D) None of these (C) They have multiple solutions
(D) The coefficient matrix is idempotent
2 3 4 
4. If A =   and  cos  sin  
 1 2 5  7. If A =  ,
  sin  cos  
0 1 2 
B=   , then 2A + 3B will be  cos  sin  
2 4 6  B=   . Then AB = ?
  sin  cos  
 4 9 12 
(A)   cos(  ) sin(  ) 
 8 8 28  (A)  
 sin(  )  cos(  )

59 60

Test Paper on Linear Algebra Vidyalankar : GATE – Engineering Mathematics

 cos(  ) sin(  ) 
(B)  
(C) It is singular Test Paper  3
  sin(  ) cos(  ) (D) A1 exists
Duration : 30 Min. Max. Marks : 25
 cos(  ) sin(  )  1 3 5 
(C) 
 sin(  ) cos(  )
 12. If A =  0 2 1 , the eigen values Q1 to Q5 carry one mark each (C) there are infinite number of
 0 0 3  1. For triangular matrix eigen values are solutions
 cos(  ) sin(  ) 
(D)  
  sin(  ) cos(  ) of the matrix I + A + A2 are (A) same as that of row elements (D) solution is possible is in the form
(A) 1, 2, 3 (B) 3, 7, 13 (B) same as of column elements of complex number
1 1 3
5 2 6 (C) 1, 7, 13 (D) 1, 1, 1 (C) same as of that principal diagonal 5. Consider the following four possible
8. Given matrix   is a
elements properties of a matrix A.
 2 1 3   cos   sin  
13 The matrices   (D) can not predict (i) It is square matrix
nilpotent matrix of index . . .  sin  cos  
(ii) aij = +aji
(A) less than 3 a 0  2. Rank of a matrix A is said to be ‘r’ if
and  commute under (iii) aij = aji (iv) All leading diagonal
(B) 3 0 b  (A) atleast one square matrix of A of
(C) higher than 3 elements are zero.
multiplication order ‘r’ whose determinant is not
(D) can not be determined If A is skew-symmetric, which of the
(A) if a = b2 ,  = n, in integer equal to zero.
following is True ?
 1 2 3  (B) always (B) the matrix A contains any square
9. The rank of the matrix  2 4 1 is (C) if a = b submatrix of order r + 1, then the
(A) (i), (ii) (iv) (B) (iii) & (iv)

 1 2 7  (D) never determinant of every square sub


(C) (i) , (iii), (iv) (D) None of these

(A) 3 (B) 2 matrix of order r + 1 should be Q6 to Q15 carry two marks each
14. The rank of the matrix given below is
(C) 1 (D) 0 zero. 6. If aij = 1 for all i , j then rank (A) is
1 4 8 7 
10. Which of following is correct ? 0 0 3 0  (C) both (A) and (B) true (A) 1 (B) 0
 
(A) (AB)1 = A1 B1 4 2 3 1  (D) only (B) true (C) n (D) anything
 
(B) (AB)t = At Bt  3 12 24 21 3. If A is m  n matrix and B is   n 7. | Adj A | is equal to
(C) (A1)1 = At (A) 3 (B) 1 matrix, then product AB is
(A) | A | n (B) | A | n1
(D) adj(AB) = (adj B) (adj A) (C) 2 (D) 4 (A) m  n matrix
(C) |A|n+1 (D) |A1|
 cos  sin  0  (B) n   matrix
15. For a square matrix A, if A2 = A then A
11. Given A =   sin  cos  0  . Indicate (C) n  n matrix
where n is the order of the matrix
 0 is called
0 1 8. A and B are idempotent then AB is
(A) idempotent matrix (D) product not possible
the statement which is not correct for A. also idempotent if
(B) nilpotent matrix 4. The equations are said to be
(A) It is orthogonal
(C) involuntary matrix inconsistent if (A) A = BT
(B) It is non singular
(D) none of above (A) the equations have no solution (B) A and B are nonsingular

(B) the solution is zero vector (C) AB is symmetric



(D) AB = BA

61 62
Test Paper on Linear Algebra Vidyalankar : GATE – Engineering Mathematics

0
4
0 0 0 13. Consider system of equations Test Paper  4
2 3 0  2x + y + 2z =1
9. The rank of A =  is
1 0 0 0 x+y =0 Duration : 30 Min. Max. Marks : 25
 
4 0 3 0
x  ky + 6z = 3 for the system to Q1 to Q5 carry one mark each Q6 to Q15 carry two marks each
(A) 0 (B) 1 have unique solution, the value of k is 1. The matrix A is said to be skew 6. Find the rank of matrix
(C) 2 (D) 3 (A) k = 2 (B) k  2 Hermitian if  1 2 1
10. What are values of  and  so that the (C) k = 1 (D) k = 0 (A) A = A (B) A =  A A =  6 12 6 
equations have no solution (C) A = A1 (D) A = Adj A  5 10 5 
14. The adjoint of matrix
2x + 3y + 5z = 9 (A) 1 (B) 2
 1 2 3  2. If  is eigen value of a matrix and also
7x + 3y 2z = 8 (C) 3 (D) 4
A =  2 1 1 is 1/ is eigen value of same matrix then
2x + 3y + z = 
 4 5 2  that matrix is 7. If A & B are 3  3 matrices then AB = 0
(A)  = 9,  = 5 (B)  = 5,  = 9
 7 8 6  (A) Skew  Hermitian matrix implies :
(C)   9,  = 5 (D)   9,   5
(A)  11 14 13  (B) Hermitian matrix (A) Both A = 0 and B = 0
 5 5 5  (C) Orthogonal matrix (B) | A | = 0 and | B | = 0
 5 3 14 4 
11. The rank of matrix  0 1 2 1 is 7 4 5  (D) None of these (C) Either | A | = 0 or | B | = 0
 1 1 2 0  (B)  5 14 6  (D) Either A = 0 or B = 0
3. If matrix A has rank ‘r’, then A
(A) 1 (B) 2  5 11 8 
(transpose of A) will have rank
8. The characteristic equation of the
(C) 3 (D) 4  7 11 5  (A) r + 1 (B) r  1
(C)  8 14 5  matrix
(C) r (D) can’t say
12. The characteristic equation of the  6 13 5   2 1 1 
 1 0 2
4. A square matrix A be such that AK = 0 A =  1 2 1 is
 7 11 13 
matrix A =  0 2 1 is (D)  5 9 11 
then it is called  1 1 2 
 2 0 3  (A) Hermitian matrix
 2 14 20  (A) 63 + 92  4 + 1 = 0
3 2
(A)  + 6  7 2 = 0 (B) idempotent matrix for index K (B) 3 + 62  9  4 = 0
(C) nilpotent for index K
(B) 3  62 + 7 + 2 = 0 15. Find the value of x if (C) 3  62 + 9 4 = 0
(D) singular matrix for index K
(C) 23 + 2  6 + 7 = 0 x  2 2x  3 3x  4 (D) 63 + 92 + 4 1 = 0
(D) 23  2 + 6  7 = 0 2x  3 3x  4 4x  5  = 0 5. For x12  18x1x2 + 5x22 quadratic form,
3x  5 5x  8 10x  17 9. If In is the identity matrix of order n,
matrix is
1
(A) 1, 1, 2 (B) 1, 1, 2 then In 
 1 9   1 9 
(A)   (B)  
(C) 1, 2 (D) 1, 1 9 5   9 5  (A) does not exist (B) = In

 1 18   1 18  (C) = 0 (D) = nIn


 (C)   (D)  
18 5   18 5 

63 64

Test Paper on Linear Algebra Vidyalankar : GATE – Engineering Mathematics

10. If A is the transpose of a square 12. If a matrix is singular then the Test Paper  5
matrix A, then characteristic root of the matrix is
(A) | A |  | A | (A) 0 Duration : 30 Min. Max. Marks : 25

(B) | A | = |A | (B) diagonal matrix Q1 to Q5 carry one mark each 5. If the eigen values of an n  n matrix
(C) | A | + | A  | = 0 (C) can’t say 1. If for a square matrix A, At = A, then are all distinct then it is always similar
(D) | A | = | A | only when A is (D) not possible the matrix A is to a
symmetric (A) Symmetric (A) square matrix
13. Find the rank of matrix (B) Orthogonal (B) triangular matrix

 0 1 1  1 3 4 3 (C) Hermitian (C) diagonal matrix


 3 9 12 9 
11. Inverse of matrix  1 0 1 is   (D) Skew symmetric (D) none of these
 1 1 0   1 3 4 1

(A) 1 (B) 2 2. The values of ‘a’, & ‘b’ from the matrix
 1 1 1  Q6 to Q15 carry two marks each
1 equation
(A) 1 1 1  (C) 3 (D) 4 6. If A & B are square matrices of order 3
2  a  3 b2  2  2a  1 3b 
 1 1 1 such that | A | = 1, | B | = 3 then
    
3 6   0 2   0 b2  3b 
 1 1 1  14. If A =  determinant of 3AB is equal to
 , then A is
(B) 2  1 1 1  2 4  are (A) 9 (B) 27
 1 1 1 (A) Singular (B) Nonsingular (A) a = 1, b = 1 (B) a = 2, b= 1 (C) 81 (D) 81
1 2  (C) a = 2, b = 2 (D) a = 1, b = 2
 1 1 1 (C) I (D) 6 
1 
(C) 1 1 1  1 2  7. Find the rank of matrix
2 3. Find the index for which the matrix
 1 1 1 1 2 3 
15. If A is 3  4 matrix and B is a matrix  ab b2  A =  2 4 7 
 1 1 1  2  is nilpotent
 3 6 10 
(D) 2  1 1 1  such that both AB & BA are defined,  a ab 
 1 1 1 then B is a . . . (A) 3 (A) 1 (B) 2

(A) 4  3 matrix (B) 3  4 matrix (B) 2 (C) 3 (D) 4

(C) 3  3 matrix (D) 4  4 matrix (C) 1


(D) It is not nil potent 8. Eigen values of the matrix
 a h g 
4.  is characteristic root of matrix A if
A =  0 b 0  are
and only if these exists a nonzero  0 0 c 
vector X such that
(A) a, b, c (B) 0, a, b
(A) A1  A = X (B) AX = X (C) b, a, g (D) h, b, g
(C) XA = X (D) A  A1 = XX

65 66
Test Paper on Linear Algebra Vidyalankar : GATE – Engineering Mathematics

9. If following vectors are linearly 3 4 2  Test Paper  6


dependent then what is relation 12. Is the given matrix  4 3 1
between x1, x2, x3, x4 ?  2 1 3  Duration : 30 Min. Max. Marks : 25
x1 = (1, 2, 4); x2 = (2, 1, 3); idempotent ? Q1 to Q5 carry one mark each  1  i 1 2  3i
x3 = (0, 1, 2); x4 = (3, 7, 2) (A) No 1. The value of the determinant of matrix (C)  2  i 2i 1  i 
(A) 9x1 + 12x2 + 15 x3 + x4 = 0 (B) Yes
a h g f  3i 0 3  4i

(B) 9x1  12x2 + 5x3  5x4 = 0 (C) Can’t say 0 b c e 


A=  is  1  i 1 2  3i
(C) 9x1 + 12x2 + 5x3 + 5x4 = 0 (D) Insufficient Data 0 0 d k (D)  2  i 2i 1  i 
 
(C) None of these 0 0 0 
 3i 0 3  4i 
13. The rank of the matrix (A) abd  (B) abd 
10. If A is a nonsingular matrix & A is the 1 1 1  (C) abck  abde (D) abd  abge 3 1 2 
transpose of A then A =  0 2 5  is 4. If A = 6 2 4  Then rank of matrix A

(A) | A . A |  | A2 |  0 2 5  3 1 2 
0 1 2   1 2 
(B) | A . A |  | A |2 (A) 1 (B) 2 2. If A =  1 2 3  , B =  1 0  , then
  is
(C) 3 (D) 4 2 3 4   2 1 (A) 1 (B) 2
(C) | A | + | A | = 0
(D) | A | = | A | the product BA is (C) 3 (D) 0
14. The determinant of the matrix
3 2   2 3 
11. The inverse of the matrix 6 0 0 0 (A) 5 5  (B)  5 8  5 3 
 8 5. Eigen value of matrix   are
 2 0 0  7 8   8 7  3 3 
 a  ib c  id  is
2 2 2 2 1 4 4 0
 c  id a  ib  if a + b +c + d = 1   0 0  (A) 6, 4 (B) 5, 3
  6 8 1
1
(C) 0 0  (D) does not exist. (C) 3, 5 (D) 4, 6
is
(A) 11 (B) 24 0 0 
a  ib c  id  (C) 48 (D) 0
(A)   Q6 to Q15 carry two marks each
 c  id a  ib 
6. The value of determinant
 1  i 1 2  3i
 a  ib c  id
(B)  
15. If a  b  c, one value of x which 3. Let A =  2  i 2i 1  i  , then the 3 1 1 1
 c  id a  ib  satisfies the equation 1 3 1 1
 3i 0 3  4i   is
 a  ib c  id  0 x a x b 1 1 3 1
(C)   matrix A is given by
 c  id a  ib  x  a 0 x  c = 0 is given by 1 1 1 3
xb xc 0  i  1 1 2  3i (A) 1 (B) 1
a  ib c  id 
(D)   (A)  2  i 2i 1  i 
 c  id a  ib  (A) x = a (B) x = b (C) 0 (D) 4
 3i 0 3  4i
(C) x = c (D) x = 0
 1 i 1 2  3i
 (B)  2  i 2i 1  i 
 3i 0 3  4i

67 68

Test Paper on Linear Algebra Vidyalankar : GATE – Engineering Mathematics

7. The rank of matrix 10. Determine eigen values of matrix 3 4  15. If the system of equations
2 1 2 
 2 3 1 1 a h g  14. If A =  1 1  & B =   then x + ay + az = 0, bx + y + bz = 0,
  1 3 4
A =  1 1 2 4  is A =  0 b 0  are 2 0  cx + cy + z = 0 where a, b, c are
3 1 3 2   0 0 c 
  AB = B A non zero & non unity has a non trivial
6 3 0 7 
(A) g, b, c (B) g, h, c (A) Yes solution then the value of
(A) 1 (B) 2
(C) a, b, g (D) b, a, c (B) No a b c
(C) 3 (D) 4   is
(C) Can’t say 1 a 1 b 1 c

11. If A + B + C = , then the value of (D) Insufficient Data (A) 0 (B) 1


 1 2 3
abc
8. Adjoint of matrix A = 0 5 0  is sin(A  B  C) sinB cos C (C) 1 (D)
 sinB 0 tanA  is a 2  b2  c 2
2 4 3 
cos(A  B)  tan A 0
 15 0 10  
(A)  6 3 0  equal to
 
 15 6 5  (A) 0
(B) 1
 15 6 15 
(B)  0 3 0  (C) 2 sin B tan A cos C
 
 10 0 5  (D) None of these

 15 3 5 
(C)  10 6 0  1 2 1 2
  1 3 1 2 
 0 15 0  12. The rank of matrix A = 
2 4 3 4
15 10 0   
3 7 4 6
(D)  3 6 15 

 5 0 0 
is
(A) 1 (B) 2
(C) 3 (D) 4
9. The characteristic roots of the matrix
0 1 2  13. If a3  b 2  c  d
 1 0 1 are
 
3  1  1
2 1 0 
=   3 2   2 be an identity
(A) 1, 2  3 (B) 3, 1  2   3   4 5
(C) 2, 1  3 (D) None of these in , where a, b, c, d are constants,
then the value of d is
(A) 5 (B) 6
(C) 9 (D) 0

69 70
Vidyalankar : GATE – Engineering Mathematics

Chapter - 2 : Calculus 
 f(x)   xlim
lim  
a
f(x)  1

lim g(x) 
2  0
x a
 g(x)   x a  2

2.1 Function of single variable  lim  f  x  


g x 
  1 
2

x a
A real valued function y = f(x) of a real variable x is a mapping whose domain S and
co-domain R are sets of real numbers. The range of the function is the set y  f(x) : x  R , Standard Formulae
which is a subset of R.
sinx ax  1
lim =1 …x in radians lim = na (a > 0)
x0 x x
2.2 Limit of a function x0

tanx e x 1
lim = 1 …x in radians lim =1
The function f is said to tend to the limit  as x  a, if for a given positive real number x0 x x0 x
1
 > 0 we can find a real number  > 0 such that limcosx = 1
x0 lim1  x) x = e
x0
f(x)     whenever 0 < |x  a| < 
xn  an n(1  x)
Symbolically we write lim f(x)   lim  nan1 lim 1
x a xa xa x0 x

Left Hand and Right Hand Limits Solved Example 1 :


Solution :
Let x < a and x  a from the left hand side.
 1 Let h > 0, we have
Show that lim sin   does not exist.
If |f(x)   1 | < , a<x<a or lim f(x)   1 x 0
x
x a
lim f(x  h)   4  h  1
2

Solution : h 0  
then  1 is called the left hand limit.
For different values of x in the interval = 17  h  h  8  
Let x > a and x  a from the right hand side.  1
0 < | x | <  the function sin   takes = 17 if h (h + 8) < 1
If |f(x)   2 | < , a < x < a +  or lim f(x)   2 x
x a  (h  4)2  17  h  17  4
values between 1 and 1.
then  2 is called the right hand limit.
and lim f(x  h)  (4  h)2  1
 1 h 0
Since lim sin   is not unique limit does
If  1   2 then lim f(x) exists. If the limit exists then it is unique. x 0
x = 17  h(h  8)
x a
not exist.
= 16 if h ( h  8) >  1
Properties of Limits
or (h  4)2  15 or h  4  15
Let f and g be two functions defined over S and let a be any point, not necessarily in S Solved Example 2 :
lim f(x)   1 and lim g(x)   2 exist, then  lim f(x)  17 and lim f(x)  16
And if Show that lim  x  1 does not exist,
2
x 4 x 4
x a x a x 4

The limit does not exist.


 lim Cf(x)  C lim f(x)  C 1 C a real constant. where  is the greatest integer function.
x a x a

 lim  f(x)  g(x)  lim f(x)  lim g(x)   1   2


x a x a x a

 lim  f(x)g(x)   lim f(x)  lim g(x)   1 2


x a  x a   x a 

71 72

Notes on Calculus Vidyalankar : GATE – Engineering Mathematics

2.3 Continuity Properties of continuous functions


Let f be a real valued function of the real variable x. Let x0 be a point in the domain of f  Let the functions f and g be continuous at a point x = x0 Then,

and let f be defined in some neighbourhood of the point x0. The function f is said to be i) cf, f  g and fg are continuous at x = x0, where c is any constant.
continuous at x = x0 if ii) f/g is continuous at x = x0, if g(x0)  0
i) lim f(x)   exists and  If f is continuous at x = x0 and g is continuous at f(x0), then the composite function
x  x0
g(f(g)) is continuous at x = x0.
ii) lim f(x)  f(x 0 )
x  x0
 A function f is continuous in a closed interval [a, b] if it is continuous at every point in
Types of discontinuity (a, b)

A point at which f is not continuous is called a point of discontinuity. lim f  x   f  a  and lim f  x   f  b 
x a x b

If lim f(x)   exists, but lim f(x)  f(x 0 )


x  x0 x  x0  If f is continuous at an interior point c of a closed interval [a, b] and f(c)  0, then there
then x0 is called the point of removable discontinuity. exists a neighbourhood of c, throughout which f(x) has the same sign as f(c).
In this case we can redefine f(x), such that f(x0) =  , so that the new function is  If f is continuous in a closed interval [a, b] then it is bounded there and attains its
continuous at x = x0. bounds at least once in [a, b].
For example :  If f is continuous in a closed interval [a, b], and if f(a) f(b) are of opposite signs, then
 sin x  / x ; x0 there exists at least one point c  [a, b] such that f(c) = 0.
The function, f(x) = 
4 ; x0  If f is continuous in a closed interval [a, b] and f(a)  f(b) then it assumes every value
has a removable discontinuity at x = 0, since lim f(x)   and a new function can be between f(a) and f(b).
x 0

defined as Solved Example 3 : x


 The function f(x) = is discontinuous
x
 sin x  / x ; x0 Determine the point of discontinuity of the
f(x) = 
1 ; x0 x at x = 0.
function f(x) =
x Note : |x| = x when x > 0 and |x| = x
If lim f(x) does not exist, then x0 is called the point of irremovable discontinuity.
x  x0
Solution : when x < 0.
For Example :
x
For x < 0, = 1 Solved Example 4 :
1 x
f(x) = at x = 0
x Find the limit if it exists, as x approaches
x zero for the function f(x) given by
Here limf(x) does not exist. Also f(0) is not defined. For x > 0, = +1
x 0 x
if x < 0, f(x) = x
1  f(0) = 1
 f(x) = has an irremovable discontinuity at x = 0. x = 0, f(x) = 1
x
and f(+0) = +1 x > 0, f(x) = x2
Note : In case of an irremovable discontinuity, it does not matter whether or Also the function is not defined at x = 0 Solution :
how the function is defined of the point of discontinuity. If x < 0, f(0) = lim f  x   lim x  0
x 0 x 0

73 74
Notes on Calculus Vidyalankar : GATE – Engineering Mathematics

If x > 0, f(+0) = lim f  x   lim x  0 2 Solution :


2.4 Differentiability
x 0 x 0
lim f(x)  2  (1)  3 for x < 2,
Thus f(0)  f(0) each being zero. x 2 Let a real valued function f(x) be defined on an I and let x0 be a point in I. Then, if

Since the limit on the left = the limit on the x2 f  x   f  x0  f  x 0  x   f  x 0 


= 1 lim or lim
|x2| x  x0 x  x0 x  0 x
right at x = 0 the function f(x) has limit 0 at
x = 0. lim f(x)  2  (1)  1 for x > 2, exists and is equal to  , then f(x) is said to be differentiable at x0 and  is called the
x 2

Note : The limit at x = 0 is not equal to derivative of f(x) at x = x0.


x2
f(0) = 1, and hence the function is =1 If f(x) is differentiable at every point in the interval (a, b) then f(x) is said to be
|x2|
discontinuous at x = 0. differentiable in (a, b).
 The one sided limits do not coincide.
Solved Example 5 : Thus the function is not defined at x = 2. Properties of differentiation
x 3  x 2  16x  20  x = 2 is the only point of discontinuity.  Let the functions f and g be differentiable at a point x0. Then
  x  2
2
Let f(x) = 
The jump at this point = 1  (3) = 2 i) (cf) x0 = cf(x0), c any constant.
k ifx  2

Solved Example 7 : ii) (f  g)x0 = f(x0)  g(x0)
if x  2
x  2 for x2 iii) (fg)(x0) = f(x0) g(x0) + f(x0) g(x0)
Find k if f(x) is continuous at x = 2. Show that f(x) =  2 is
x  1 for x2  f  g  x 0  f   x 0   f  x 0  g  x 0 
Solution : iv)    x 0   , g  x0   0
discontinuous at x = 2 and determine the g g2  x 0 
For x  2 we have
jump of the function at this point.  If f is differentiable at x0 and g is differentiable at f(x0) then the composite function
x 3  x 2  16x  20
f(x) = Solution : h = g(f(x)) is differentiable at x0 and
x  2
2

lim (x  2)  4 h(x0) = g(f(x0)) f(x0)


x  2
 x  2  x  5
2

= =x+5  If the function y = f(x) is represented in the parametric form as


 x  2
2
x  2

lim x  1  32

x = (t) and y = (t), and if (t), (t) exist, then
 lim f(x)  lim(x  5)  7  The limit on the left  the limit on the right.
dy / dt    t 
  t   0
x 2 x 2
f(x) = 
We have f(2) = k
 The function has discontinuity at the dx / dt   t 
point x = 2
 f(x) is continuous at x = 2.
 The jump of the function at x = 2 is Note : If a function is differentiable at x = x0, then it is continuous at x = x0.
Now lim f(x)  f(2)
x 2
f(2 + 0)  f(2  0) = 3  4 = 1 However the converse need not be true.
i.e. 7 = k
Note : If f(x0  0)  f(x0 + 0), then
Solved Example 6 : f(x0 + 0)  f(x0  0) is called a jump
x2 discontinuity of the function f(x) at x0.
For the function f(x) = x  find the
x2

point of discontinuity and determine the


jump of the function at this point.

75 76

Notes on Calculus Vidyalankar : GATE – Engineering Mathematics

Standard formulae  1   1   1  At x = 0, we have y = | x |


f(x) = 2x cos    x 2   sin      2 
x   x   x 
y x
 
Function Derivative Function Derivative  1  1 x x
 2x cos    sin  
x x
x 1 y
k(constant) 0 sin1 lim  1
 1 x  0 x
a a2  x2 Now lim f   x  does not exist as lim sin  
1
x 0 x 0
x y
x lim 1
xn nxn1 cos 1
a a2  x2 does not exist. Therefore, f(x) is not x  0 x
x a continuous at x = 0. Since the right and the left side derivative
logx 1/x tan1
a a2  x2 are not equal, the function f(x) = |x| is not
x a Solved Example 9 : differentiable at the point zero.
x x 1
e e cot
a a2  x2 y  3 x is defined and continuous for all x
x a
ax axloga sec1 investigate whether this function has a Solved Example 11 :
a x x2  a2
derivative at x = 0. d n
sin x cos x cosec 1
x a Show that
dx
 
x  nxn1
2 2 Solution :
a x x a
y = 3
x  x  3 x at x = 0 Solution :
cos x  sin x sin h x cos h x Let y = xn
y = 3
x Let x receive a small increment x and let
tan x sec2x cos h x sin h x
y x 3
1 the corresponding increment in y be y
1   
cot x  cosec2 x sin h1 x x x 3
 x 
2 then
2
x 1 y + y = (x + x)n
1 y 1
sec x sec x tan x cos h1 x  lim  lim   By subtraction, y = (x + x)n  xn
x  0 x
x2  1  x 
x  0 3 2

y  x  x   x  x  x   xn
n
n n
1
cosec x  cosec x cot x tan h x1
 There is no finite derivative.   
x2  1 x x x  x  x

 x  x   xn
n
y
Solved Example 8 :  f(x) is continuous at x = 0 Solved Example 10 :  lim  lim  nxn1
x  0 x x x  x x  x  x
Show that the function f(x)  f(0) Investigate the function f(x) = | x | for
dy d n
x 2 cos(1/ x), x0
Now f(0) = lim
x 0 x differentiability at the point x = 0 
dx
 nxn1 or
dx
 
x  nxn1
f(x) = 
0 x0   1  Solution :
 lim  x cos     0
x 0
  x 
is differentiable at x = 0 but f(x) is not y = f(x) = | x |
continuous at x = 0. Hence f(x) is differentiable at
y = | x + x |  | x |
Solution : x = 0 and f (0) = 0

We have lim f(x)  0  f(0) for x  0 we have


x 0

77 78
Notes on Calculus Vidyalankar : GATE – Engineering Mathematics

2.5 Mean Value Theorems Geometric Interpretation


f(b)f(a)
Rolle's Theorem f(c) =
ba
The theorem states that
f(b)
 if f(x) is continuous in the closed interval [a, b] and
 if f(x) exists in open interval (a, b) and
f(a)
 if f(a) = f(b)
Then there exists at least one value c in (a, b) such that f(c) = 0
a c b
Geometric Interpretation
There exists at last one point at which the tangent is parallel to the secant through the
y
f(c) = 0 end points.

Cauchy's Mean Value Theorem


y = f(x)
f(a) = f(b) The theorem states that
 if f(x) and g(x) are both continuous in closed interval [a, b] and
 if f(x) and g(x) both exist in open interval (a, b)
Then there exists at least one value c such that
a c b x f (c) f(b)f(a)

g(c) g(b)g(a)
There exists at last one point at which slope of the tangent is 0 or the tangent is parallel
to x-axis
2.6 Maxima and Minima
A function f(x) is said to have a maximum value at x = a if f(a) is larger than any other
Lagrange's Mean Value Theorem
values of f(x) in the immediate neighbourhood of ‘a’. It has a minimum value if f(a) is less
The theorem states that
than any other value of f(x) sufficiently near ‘a’.
 if f(x) is continuous in the closed interval [a, b] and
Finding Maximum and Minimum values of y = f(x)
 if f(x) exists in open interval (a, b)
Then there exists one value c such that dy d2 y dy
 Get and 2 . Solve = 0 and consider its roots. These are the values of x
dx dx dx
f(b)f(a)
f(c) = dy
ba which make = 0.
dx
 For each of these values of x, calculate the corresponding value of y and examine
d2 y
the sign of .
dx 2
 If the sign is ve the corresponding value of y is a maximum.
If the sign is +ve, the corresponding value of y is minimum.

79 80

Notes on Calculus Vidyalankar : GATE – Engineering Mathematics

In this case it is much more convenient to dy


Note : Maximum and minimum values of a function occur alternately in a Note : If the sign changes from
dy dx
continuous function. (The graph of a continuous function has no break find the changes of sign of than to
dx + to  the point is a maximum point; if
gaps)
d2 y the sign changes from  to +, the point
A maximum is not necessarily the greater value and a minimum is not work out the value of .
dx 2 is a minimum point.
necessarily the least value of the function. The denominator must be positive and

Stationary Points and Turning Points therefore only the sign of the numerator Solved Example 14 :
dy dy need be examined. Find the maximum and minimum values of
A stationary point on a graph is any point at which = 0. A value of x for which =0
dx dx Near x = 2 the function 4 cos x  3 sin x.
does not necessarily give either a maximum or minimum. A turning point is a maximum or If x is slightly less than 2, x2 < 4; Solution:
minimum point. dy
is ve Let y = 4 cos x  3 sin x
dx
Solved Example 12 : 1 dy
Hence x = gives a minimum value of y 2
If x is slightly greater than 2, x > 4; Then = 4 sin x  3 cos x
Find the maximum and minimum points on 3 dx
dy
3
the curve y = 2x + 5x  4x + 7 2
and x = 2 gives a maximum value of y is +ve d2 y
dx = 4 cos x + 3 sin x
Solution : 1 8 dx 2
for x  , y6 ; for x   2, y  19 dy
3 27 Here changes from ve to +ve dy
Differentiating the equation we have dx = 0 when  4 sin x  3 cos x = 0
1 8  dx
dy  Near x = 2
 6x 2  10x  4  3 , 6 27  is a minimum point
dx   3
If x is slightly less than 2, x2 > 4; or tan x =
(2, 19) is a maximum point. 4
d2 y dy
 12x  10 is +ve 3 4
dx 2 dx and then (i) sin x = , cos x 
Solved Example 13 : 5 5
2
Now If x is slightly greater than 2, x < 4;
Find the maximum and minimum values of 3 4
dy dy (ii) sin x = , cos x 
 0 if6x 2  10x  4  0 2
x  2x  4 is ve 5 5
dx the function 2 dx
x  2x  4  tan  is negative.
or 3x 2  5x  2  0 dy
Here changes from +ve to ve
Solution : dx d2 y 16 9
i.e. (3x  1) (x + 2) = 0 In the first case, =   5
x 2  2x  4  x = 2 gives a minimum value of y, dx 2 5 5
dy 1 Let y = 2
i.e.  0, when x  or  2 x  2x  4 1 and  is +ve
dx 3 y ;
3 d2 y 16 9
(x  2x  4)(2x  2) 
2
1 d y2
 2  In the second case, =   5
for x  ,  14 and  is positive dy (x  2x  4)(2x  2) and x = 2 gives a maximum value of y, dx 2 5 5
3 dx 2 Then =
dx (x 2  2x  4)2 y=3 and  is ve
d2 y
for x  2, 2  14 and  is negative dy This the maximum value is 3 and the  The minimum value
dx  = 0 when x 2  4
dx 1  4  3
minimum is . = 4    3    5
or x = 2 3  5  5

81 82
Notes on Calculus Vidyalankar : GATE – Engineering Mathematics

and the maximum value Let ABCD by rectangular sheet and let a 2.7 Integration
4  3  square of edge x m be cut from each
= 4  3    5 Integrand and element of integration
5  5  corner.
When the flaps are folded up, the The function under the sign of integration is called integrand. For e.g. in  x 3 dx ; x3 is
Solved Example 15 :
dimensions of the rectangular box called integrand. In the integral  f(x) dx , dx is known as the element of integration and it
Find the maximum and minimum values of
obtained are 8  2x, 3  2x and x meters.
f(x, y) = 7x 2  8xy  y 2 where x, y are indicates the variable with respect to which the given function is to be integrated.
Let V cubic meters be the volume of the
2 2
connected by the relation x  y  1 . box then Constant of integration :
Solution : V =  8  2x  3  2x  x We know that
Since x 2  y 2  1, 3 2
 4x  22x  24x ; d 2
(x )  2x  2x dx  x
2

dx
we can put x  cos  , y  sin  dV
 12x 2  44x  24
dx d 2
  2x dx  x
2
f(x, y) = 7 cos2   8 cos  sin   sin2  = F() Also (x  c)  2x  c , where c is any constant
2
dx
d V
1 cos 2 1 cos 2 and 2  24x  44 So we notice that x is an integral of 2x, then x2 + c is also an integral of 2x. In general if
2
F(  ) = 7  4 sin2  dx
2 2
dV  f(x)dx = (x) then
= 4  3 cos 2  4 sin2
dx
 
 0 if 4 3x 2  11x  6  0
= 4  5 sin  cos 2  5 cos  sin2  f(x)dx = (x) + c
(3x  2) (x  2) = 0
= 4  5 sin (2  )
i.e. when x = 2/3 or 3
Since sine functions takes its maximum Standard formulae
2 d2 v
for x  ,  28 and
and minimum value as +1 and 1 3 dx 2 xn  1 1 x
x   sin1
n
dx   c , (n  1) c
respectively.  is negative n1 a2  x2 a
Fmax = 4 + 5 ; Fmin = 4  5 2 1 dx x
Hence x  gives a maximum value of V  x dx  log x  c    cos1
a
c
 Fmax = 9 Fmin = 1 3 a2  x2
2 11 dx 1 x
e  a2  x2 tan1  c
x
For x = , V= 7 dx  e x  c 
Solved Example 16 : 3 27 a a
A rectangular sheet of a metal is 8 meters ax dx 1 x
a
x

by 3 meters; equal squares are cut out at


Thus the maximum volume is 7
11
cm.
dx 
log e a
+c  a2  x2 
a
cot 1  c
a
27
each of the corners and the flaps are then dx 1 x
x = 3 is inadmissible, since the breadth
 cos xdx  sin x  c  
a
s ec1  c
a
folded upto form an open rectangular box. itself of the sheet is 3m. x x2  a2
dx 1 x
Find its maximum volume.
 cos ecx cot xdx   cos ecx  c  
a
cos ec 1  c
a
x x2  a2
Solution: D C

x 3m
 sin x dx   cos x  c  sinh x dx = cos h x + c

A B
8m
83 84

Notes on Calculus Vidyalankar : GATE – Engineering Mathematics

 2 sin A sin B = cos (A  B)  cos (A + B)


 sec x tan x dx  sec x  c  cosh x dx = sin h x + c CD CD
 2 sin cos = sin C + sin D
dx 2 2
 sec x dx  tan x  c
2  = sin h1 x + c
x2  1 CD CD
 2 cos sin = sin C  sin D
dx 2 2
 cos ec
2
x dx = cot x + c  = cos h1 x + c
CD CD
x2  1  2 cos cos = cos C + cos D
2 2
dx
 x2  1
1
= tan h x + c
 cot xdx = log sin x + c
 2 sin
CD
sin
DC
= cos C  cos D
2 2

 tan xdx = log sec x + c  tan xdx =  log (cos x) + c


1  tan2 A
 cos 2A = cos2 A  sin2 A = 1  2sin2 A = 2 cos2A  1 =
 x 1  tan2 A
 sec xdx  log(sec x  tan x)c  sec xdx  logtan  4  2 c 2 tanA
 sin 2A = 2 sin A cos A =
 x 1  tan2 A
 cosecxdx   log(cosecx  cot x)c  cosec xdx log  tan 2  + c
2 tanA
 tan 2A =
1  tan2 A
Important Trigonometric Identities
 sin2 A + cos2 A =1  sin 3A = 3 sin A  4 sin3A

 sin (A + B) = sin A cos B + cos A sin B


 cos 3A = 4 cos3 A  3 cos A
 cos (A + B) = cos A cos B  sin A sin B
3 tanA  4 tan3 A
tanA  tanB  tan 3A =
 tan (A + B) = 1  3 tan2 A
1  tanA tanB
 sin (A  B) = sin A cos B  cos A sin B
 sin
m
Note : Integration of x cosn xdx where m and n are positive integers

 cos (A  B) = cos A cos B + sin A sin B (i) If m be odd and n be even, for integration put t = cos x
tan A  tanB (ii) If m be even and n be odd, for integration put t = sin x
 tan (A  B) =
1  tan A tanB (iii) If m and n are odd, for integration put either t = cos x or sin x
 sin2 A  sin2 B = sin (A + B) sin (A  B) (iv) If m and n are even, for integration put either t = cos x or sin x

 cos2 A  sin2 B = cos (A + B) cos (A  B) Solved Example 17 :  4  sin x dx =  (3 sin x  sin3x)dx


3

Evaluate  sin3 x dx
 2 sin A cos B = sin (A + B) + sin (A  B)  3  sinxdx   sin3xdx
Solution :
 2 cos A sin B = sin (A + B)  sin (A  B) 1 cos 3x 
 sin
3
sin 3x = 3 sin x  4 sin3x x dx  3 cos x  c
4  3 
3
 2 cos A cos B = cos (A + B) + cos (A  B) 4 sin x = 3 sin x  sin 3x

85 86
Notes on Calculus Vidyalankar : GATE – Engineering Mathematics

Solved Example 18 : Solved Example 21 : 1  sinx 1 sin x Solved Example 24 :


  
cos2 x cos 2 x cos2 x
 sin3x cos 2x dx  sin  sin
3
Evaluate Evaluate x cos3 x dx Evaluate 6
x cos3 x dx
1
Solution : Solution : or = sec2x  tan x sec x Solution :
1  sinx
We have
1  sin x cos3 xdx   sin6 x(1  sin2 x)cos xdx
6

 sin3x cos 2x dx  2  (sin5x  sinx)dx sin3 x cos3 x = (sin x cos x)3 


1
 1  sin x dx
1 1 1 =  (sin6 x  sin8 x)cos x dx
= cos 5x  cos x = (2 sin x cos x)3   (sec 2 x  tanx sec x)dx
10 2 8
=  (t 6  t 8 )dt
 sec xdx   tanx sec xdx
2
1 =
= sin3 2x
Solved Example 19 : 8  by putting
 tanx  sec x  c
Evaluate  sin2x sin3x dx =
1 1
 (3 sin 2x  sin 6x) sin x = t =
1 7 1 9 1 1
t  t = sin7 x  sin9 x
8 4 7 9 7 9
Solution :
[ 4 sin3x = 3 sin x  sin 3x] Solved Example 23 :
1 Solved Example 25 :
 sin2xsin3xdx  2  cos x  cos 5xdx =
1
(3 sin 2x  sin 6x)
Evaluate  sin
7
x cos6 x dx
 sin
4
Evaluate x cos2 x dx
32
1 1 Solution :
= sinx  sin5x Solution :
  sin
3
2 10 x cos3 xdx
 sin
7
x cos6 xdx
sin4 x cos2 x
1
32 
 (3 sin2x  sin6x)dx   sin6 x cos6 x sinxdx 1
Solved Example 20 : = (2 sin2x)2 (1+cos2x)
8
dx 3 1 =  (1  cos2 x)3 cos6 x sin x dx
Integrate 
32  32 
= sin2xdx  sin6xdx 1
cos2 x sin2 x = (1cos2x  cos22x + cos3 2x)
=  (1  3 cos2 x  3 cos4 x  cos6 x) 8
Solution : 3   cos 2x  1   cos 6x 
=
32    32  c cos6 x sinxdx 1  1  cos 4x
cos2 x  sin2 x 2   6  = 1  cos 2x 
Now,
1
 8  2
cos2 x sin2 x cos 2 x sin2 x =  (cos x  3 cos x  3 cos x  cos x)
6 8 10 12
 sin
3
Hence x cos xdx
3
cos 6x  3 cos 2x 
1 1  
  3 1 sinxdx 4 
sin2 x cos2 x  cos 2x  cos 6x  c
64 192 =   (t 6  3t 8  3t10  t12 )dt 1 1 1 1 1 
= cosec2 x + sec2x =  cos 2x  cos 4x  cos 6x 
2 2
8  2 4 2 4 
[ 1 = sin x + cos x] Solved Example 22 : by putting t = cos x
1 1 1 
dx 1 t7 t9 t11 t13 = 1  cos 2x  cos 4x  cos 6x 
  Evaluate  dx = 3 3  16  2 2 
cos2 x sin2 x 1  sin x 7 9 11 13
 sin
4
Solution : x cos2 xdx
=  (cos ec 2 x  sec 2 x)dx 1 1 3
=  cos7 x  cos9 x  cos11 x
1 1 1  sinx 7 3 11 1 1 1 
  1  cos 2x  cos 4x  cos 6x  dx
 cos ec x dx   sec =
2 2
= x dx
1  sinx 1  sin x 1  sinx 1 16  2 2 
 cos13 x
=  cot x + tan x + c 1  sin x 13
= 1 1 1 1 
= x  sin2x  sin4x  sin6x 
1  sin2 x 16  4 4 12 

87 88

Notes on Calculus Vidyalankar : GATE – Engineering Mathematics

t
3 2
Solved Example 26 : = (1  t ) dt 1 1 x
 x 2  a 2 dx 
2
x x 2  a 2  a 2 cosh1  c
2 a
 sin x cos x dx
3 5
Evaluate by putting sin x = t
Solution : =  (t 3  2t 5  t 7 ) dt dx 1 ax
 a2  x2 
2a
log
ax
c
 sin x cos x dx
3 5
4 6 8
t t t
=  
4 3 8 dx 1 xa
=  sin x (cos x)cos x dx
3 4
 x2  a2 
2a
log
xa
c
sin4 x sin6 x sin8 x
=  
 sin
3
= x(1  sin2 x)2 cos x dx 4 3 8

Solved Example 27 : 1  sin 


Integration by Substitution
cos x  sinx
I=  1  sin 
. 2 sin  cos  d

Some of the general substitutions are Integrate  dx


1  sin2x 1  sin 
Type Substitution Solution :  cos 
.2 sin  cos d

a2  x 2 x = a sin  or a cos  We know that   (2 sin   cos 2  1)d


a2 + x 2 x = a tan  or a cot  or a sin h  1 + sin 2x = sin2 x + cos2 x + 2 sin x cos x
sin2
= (sin x + cos x)2 = 2 cos  +  +c
x 2  a2 x = a sec  or a cosec  or a cos h  2
cos x  sin x
By means of above substitution some standard integrals are
Let I =  1  sin2x
dx =  2 cos  + sin  cos    + c

= 2 1  sin2   sin  1  sin2     c


dx x x cos x  sinx
 2 2
 sin1
a
or  cos1 c
a
=  (sinx  cos x)2 dx = 2 1 x  x 1  x  sin1 x  c
a x
Let cos x + sin x = u
dx x = ( x 2) 1  x  sin1 x  c
 2 2
 sinh1   or log(x  x 2  a 2 )c
a
 (cos x  sin x) dx = du
a x
du 1 1
Then I =  u2 
u
c 
cos x  sinx
c Solved Example 29 :
dx 1 
x
  co sh   or log(x  x 2  a 2 )c
a
Find the value of the integral
x2  a2 cos x  sinx 1
  dx  c sinx cos x dx
1  sin2x cos x  sin x  a2 sin2 x  b2 cos2 x
dx 1 x 1 x
 a2  x2 
a
tan1   or
a a
cot 1  c
a Solution :
Solved Example 28 :
Put a2 sin2 x + b2 cos2 x = t
1 1 x
 a  x dx  x a 2  x 2  a 2 sin1  c
2 2
1 x 2(a2 sin x cos x) dx + 2b2 cos x (sin x) dx = dt
2 2 a Evaluate  1 x
dx
dt
sin x cos x dx =
1 1 x Solution : 2(a 2  b2 )
 a 2  x 2 dx 
2
x a 2  x 2  a 2 sinh1  c
2 a Put x = sin  or x = sin2  sinx cos xdx
 dx = 2 sin  cos  d
  a2 sin2 x  b2 cos2 x

89 90
Notes on Calculus Vidyalankar : GATE – Engineering Mathematics

dt dt 1 t Integration by parts
  2(a2  b2 )t  I =  t2  ( 2)2

2
tan1
2
c
Integral of the product of two functions
1 dt 1  1
2(a 2  b2 )  t
=  log t  c 1 x   du 
2(a 2  b2 ) = tan  x c  uvdxu vdx  dx  vdx dx
2  
 2 
1
= log (a2 sin2x + b2 cos2x) + c
2(a 2  b2 ) 1  tan2   1 Rule to choose the factor of differentiation or the 1st function
= tan1  c
2  2 tan   Of the two functions the first function u is selected based on the following preference

Solved Example 30 : order

d Solved Example 31 : 1. Inverse function


Evaluate  4
sin   cos4  cos x 2. Logarithmic function
Integrate  dx
Solution : 1  sin2 x 3. Algebraic function
Solution : 4. Trigonometric function
d sec 4 
I=  4 =  tan4   1d
sin   cos4  Let sin x = z  cos x dx = dz 5. Exponential function

Dividing numerator and denominator by cos x dz


Now  dx =  1  z2 Solved Example 33 : x
cos4 1  sin2 x   sin1 xdx  x sin1 x   dx
1 x2
1
= tan z + c
Integrate  x cos xdx
sec  sec 
2 2
I=  d Put 1  x2 = z,  2x dx = dz
1  tan4  = tan1 (sin x) + c Solution :
Here F. D. = x F. I. = cos x 1 dz
 sin 2 z
1
(1  tan2 )sec 2   xdx  x sin1 x 
  1  tan4 
d
Solved Example 32 :
Applying the rules of integration by parts.
= x sin1 x  z
Put tan  = x so that sec2  d = dx sin1 x  x cos xdx
1
Integrate  1 x2
dx
  sin
1
xdx  x sin1 x  1  x 2
1  dx 
(1  x 2 )dx 2 = x  cos xdx     cos xdx  dx
 I=   x dx Solution :  dx 
1 x 4
2 1
x  2 Solved Example 35 :
x Let x = sin   dx = cos  d = x sin x   1.sinxdx
1 1 Find  log xdx
1 sin x sin (sin )cos  = x sin x + cos x + c
 x2 dx
1  1 x 2
dx =  1  sin  2
d
Solution :
2
 1 Solved Example 34 :
x  x   2 2
 We have  log xdx   1 log xdx
  =  d 
2
c
Integrate  sin
1
x dx
1  1 Integrating by parts,
Put x  = t,   1  2  dx = dt (sin1 x)2
x  x  = c Solution : d 
2
Here F. I. = 1 and F. D. = sin1 x
 log xdx  log x  1dx    dx log x   1dx  dx
Now  1.sin
1
x  sin1 x   1dx
  1
= x log x    xdx
x
d 
   sin1 x) 1.dx  dx = x log x  x  c  x  log x  1  c
 dx 

91 92

Notes on Calculus Vidyalankar : GATE – Engineering Mathematics

= x  log x  loge   c b e ax
b  a a
  cosbx   eax sinbxdx   0 f(x)dx0 f(a  x)dx
a a a 
( 1 = log e)
eax sinbx a a

Solved Example 36 : =
a
 a f(x)dx20 f(x)dx

f(x) = f(x) i.e. it is an even function

 e sinbx dx
ax
Evaluate b ax b 2
 e cosbx  2  eax sinbxdx =0 f(x) = f(x) i.e. it is an odd function
Solution : a2 a 2a a

eax  a sinbx  b cosbx  b2


 0 f(x)dx20 f(x)dx f(2a  x) = f(x)
e
ax
I = sinbx dx I=  I
a2 a2
eax eax b cosbx =0 f(2a  x) = f(x)
= sinbx   dx  b2  e  a sinbx  b cosbx 
ax
a a  I 1 2  
 a  a2
eax sinbx b ax Solved Example 37 : Solution :
=   e cosbxdx
a a eax  a sinbx  b cosbx  /2
dx
 I=
a 2  b2
Integrate 0 cos xdx We know that  x
 log x
eax sinbx
= b
a Solution : dx b b
 a x
 log x a  logb  log a = log
a
2.8 Definite Integration
We know that  cos xdx  sinx  c
/2 Solved Example 40 :
/2 
0 cos xdx  sinx 0
b
  sin  sin   1
The integral a f(x)dx is called definite integral and is defined to be b    a) where (x) 2 /2

0 1  cos 2x 
1/ 2
Evaluate dx

is the indefinite integral of  f(x)dx . It is read as integral from a to b of f(x), a is called the Solved Example 38 :
Solution :
1
lower limit and b is called the upper limit. dx
Integrate 0 1  x 2 /2

0 1  cos 2x 
1/ 2
Let I = dx
b
x b
 f(x)dx    x  x  a    b     a  Solution :
/2
a
dx = 0 
1  1  2 sin2 x dx 
This is also called the theorem of calculus.
We know that  1 x2  tan1 x  c
/2 /2
1
Properties of definite integral 
dx
0 1  x2  tan1 x  c
1

0
= 0 2 sinxdx  2 0 sinx
b b
/2
 a f(x)dxa f(t)dt   tan1 1  c    tan1 0  c  = 2  cos x 0

   
b a
= 0  = 2  cos   cos 
 a f(x)dx b f(x)dx 4 4  2

Solved Example 39 :  2 1  0   2
b c b
  f(x)dx f(x)dx f(x)dx b
1
a a c Evaluate a x dx

93 94
Notes on Calculus Vidyalankar : GATE – Engineering Mathematics

Solved Example 41 : 1 3
Solved Example 42 :  1
= log  3  4y  2 = n1  n t  n  
/3
cos x 4 0
1
1 x t
Evaluate the integral 0 3  4 sin x
dx Solve 0 1 x
dx
1  3  1
dx  1
Solution :
= log 3  4
4  2
 log 3 

 lim
t  0
t x
 lim  n   
t 0  t
Solution :
Put sin x = y then cos x dx = dy 32 3 The integrand becomes infinite at x = 1. The given integral diverges.
1
= log  
when x = 0, then y = 0 and 4  3
  1 x
The area under the curve y = f(x) =
3 1 x Solved Example 44 :
when x = /3 then y 
2 between the lines x = 0 and x = 1 is not 3
dx
/3 3 /2
well defined since the curve extends to
Solve   x  1 2/3
cos xdx dy 0
Let I = 0 3  4 sinx
 0 3  4y infinity as x tends to 1 from the left. Solution :
Nevertheless, we can define the area from
Improper Integrals 1
x = 0 to x = t where 0 < t < 1. The function f(x) = becomes
 x  1
2/ 3
b
1 t
 The integral a f(x)dx is said to be an improper integral of the first kind, if one or both
Then the integral 0 f(x)dx  tlim1 0 f(x)dx infinite at x = 1 which lies between the
 
limits of integration.
the limits of integration are infinite.
We also say that the improper integral For the integral to exist, we must have the
b
converges.
 The integral a f(x)dx is said to be improper integral of second kind, if f(x) becomes t t
convergence of the following integrals.
1 x 1 x 1 3
lim  dx  lim  dx dx dx
unbounded at one or more points in the interval of integration [a, b]. t 1
0
1 x t 1
0 1 x
2   x  1
0
2/ 3
and   x  1
1
2/ 3

Limit comparison Test for Convergence and Divergence of Improper Integrals : t


= lim  sin1 x  1  x 2  It can be easily verified the convergence of
t 1  0
If f(x) and g(x) are positive functions and if
f x  
= lim sin1 t  1  t 2  1
these two integrals are +3 and 3  2
3

lim  L, 0  L  , then  f  x  dx and  g  x  dx both converge or both diverge. t 1  


x  gx a a
respectively.
 Hence the given improper integral
 
= sin1 1  0  1  1
dx 1 1 1 2
Note : 1.  1  x2 converges because
1 x 2

x 2
and  x2 dx converges.
Solved Example 43 :
converges.
1 1

  1
dx Solved Example 45 :
1 1 1 1
2.  e2x dx converges because 2x
 x
and  ex dx converges. Solve 0 x
.

dx

dx
e e
1 1 Compare x 2
and  1 x 2
with limit
  Solution : 1 1
dx 1 1 1
3.  x
diverges because  for x > 1 and
x x
 x dx diverges For some positive number comparison test.
1 1
1
dx Solution :
t x  n x t
1
  t < 1,
1 1  1 1 1  1
4.   x  x2  dx diverges because  
x  x x 2 
and  x dx diverges. f(x) =
1
and g(x) 
1
1 1 x2 1 x2

95 96

Notes on Calculus Vidyalankar : GATE – Engineering Mathematics

1 Solved Example 47 :
lim x 2  lim 1  x
2

Note : 1.  dxdy represent the area of the region S.
1 S
x  1 x  x 2 Solve e 2x
 10e x
dx
1 x2 1 2. In an iterated integral the limits in the first integral are constants and
 1  Solution : if the limits in the second integral are functions of x then we must
= lim  2  1
x  x
  
1 first integrate w.r.t. y and the integrand will become a function of x
= 1 a positive finite limit. e
1
2x
 10e x
dx converges because
alone. This is integrated w.r.t. x.
 
dx dx 
1 3. If R cannot be written in neither of the above two forms we divide R
Hence  2 converges, therefore 1 1  x 2
1 x
1 e2x dx converges and into finite number of subregions such that each of the subregions
converges. 1 2x can be represented in one of the above forms and we get the double
lim
x  e 2x

e  10e x  integral over R by adding the integrals over these subregions.
Solved Example 46 :  10 
= lim  1  x   1 , a positive finite limit

3
x 
 e  Solved Example 49 : 1
 1   3 x3 
Solve e x
5
dx
12
=   x
2
   x    dx
3   3  
1 
   x  2  dydx
0
Evaluate I =
Solution : Solved Example 48 : 0 0 1
 1 4x 3 
  x
2
 =    dx

3

1 1 Solution : 3 3 
1 ex  5 dx converges because 1 ex dx Solve 
1 x
dx
1 1
0

  xy  2y    2x  4  dx
2 1
dx =  x3 x x4 
converges Solution : 0
0
0
=    
 
 3 3 3 0
3 ex 3 1 1 1
Now lim
x 
  lim
ex  5 1 x  1  5e  x

1 x
dx diverges because  x dx
1
diverges  2x 2 
=    4  x 0
1
=
1 1 1 1
  
 2 0 3 3 3 3
= 3 a positive finite limit. 1 1
and  for x > 1 =1+4=5
x x
Solved Example 51 :
Solved Example 50 :
2.9 Double Integrals 3 2
dxdy
1 1 Evaluate 
  x  dydx xy
2
Evaluate y 2
2 1
The double integral of a function f(x, y) over a region D is denoted by  f  x,y  dxdy .
D
0 x
Solution :
Solution :
Let f(x, y) be a continuous function defined on a closed rectangle 3 2
dxdy
3
dy dx
2
1 1
 =  y 1 x
R = { (x, y) / a  x  b and c  y  d }
 
0 x
x 2  y 2 dydx  2 1
xy 2

For continuous functions f(x, y) we have 3


dy
 y log x 
2
1
 1
 =
 
d 1
0  x x  y dy  dx
2 2
b
 d
  b
 = 2

 f  x,y  dxdy     f  x,y  dy  dx     f  x,y  dx  dy   3


dy
R a c   a  =  log 2  log1 
c 1

1
y3  y
=   x 2 y   dx 2

0
3 x
= log 2 log y 2
3

97 98
Notes on Calculus Vidyalankar : GATE – Engineering Mathematics

= log2 log3  log2 The region of integration 0  y  2 and 0  1 In this region x varies from 0 to 2 when

y2
= x 1  x 2 dx
0  x  1 for fixed x, y varies from 0 to x.
3 x can also be written as
0
= log 2 log 2
2 1  3/2 1
  1 cubic units. When 1  x  2, y varies from 0 to 2 x.
0  x  2 and 2x  y  2
=
3 
1 x2    0 3  The region D can be subdivided into
Solved Example 52 :  2 2  2 regions D1 and D2 as shown.
y
2
 I =   dy  dx Solved Example 55 :
 e dxdy  x 2  y 2  1    xydydx   xydydx   xydydx
x
Evaluate the double integral 0 2x  a cos 

 
R D D1 D2
2
Evaluate I = r sin drd
2
where the region R is given by
  x 2  y 2  1 In the region D1 for fixed x, y varies from
0 0
= dx
 2x
R : 2y  x  2 and 0  y  1. 0
Solution : y = 0 to y = x and for fixed y, x varies
2 a cos 
Solution :
=   x 2  5   x  1  dx

r2  from x = 0 to x = 1.
Integrating with respect to y first, we get 0
  I = 0 sin   2  d
0 Similarly for D2 the limit of integration for
2
 x / 2 x2  2
 x x2  5 5
 
x/2  y is y = 0 to y = 2  x
0 ye  0 dx
x2
I = 0  0 e dy  dx = = 
2
 log x  x 2  5
2
1
=  a 2 cos2  sin d
   20
 xydydx
D
2 2
1 1 2  1 4
 
2 
2
=  xe x dy   ex  = e 1 1 2 a 2
  x  1  cos2 d  cos  
2 0
20 = 1 x 2 2x
4 0 4 2 0    xydydx   
0 0 1 0
xydydx

Solved Example 53 :
5
2
1
2

= 3  log5  log 5   9  1  [d(cos ) = sin  d]
1
 xy 2 
x
2
 xy 2 
2x
a 2 a2
=

 cos3   = =    dx     dx
Evaluate the integral
=
5
log 5  1 6  0 3 0
2 0 1
2 0
y2 4 1 2
1 2 1
x dx   x  2  x  dx
2

2 0
2 2
y Solved Example 56 : =
 x2  y2  1
dxdy 21
0 0 Solved Example 54 : Evaluate I = D xydydx where D is the
1 2
Solution : The cylinder x 2  z 2  1 is cut by the  x3  1 x 4 4x 3 
=    2x 2   
region bounded by the curve x = y2, 6
 0 2  4 3 1
y planes y = 0, z = 0 and x = y. Find the
x = 2  y, y = 0 and y = 1.
volume of the region in the first octant. 1 1 16 8  1  1 4 
=  2  4   4  2  
y=2 Solution : Solution : 6 2 4 3  2  4 3
(2, 2)
In the first octant we have z  1  x 2 . The 1 4 11 9
=   
y2 = 2x 6 6 24 24
projection of the surface in the x  y plane 2
y =x
is bounded by x = 0, x = 1, y = 0 and y = x. (1, 1)
0 x Solved Example 57 :
1
x 
It would be easier to integrate it first with  V =  zdxdy     1  x 2 dy  dx Change the order of integration in the
R 0 0  D1
respect to y. D2 4 y
1 integral I =   f  x, y  dxdy
 y 0 dx
x

2
= 1 x (1, 0) (2, 0) 1 y/2
0

99 100

Notes on Calculus Vidyalankar : GATE – Engineering Mathematics

Solution : Solved Example 58 : Solved Example 59 :  32 128 8 2   1 1 1 1 


=        
The region of integration D is bounded by a a 4 2 5 21 3 3   5 21 3 3 
xdxdy
Evaluate   2 1   x  dxdy
2
Evaluate y 2
by changing
0 y x y
2
x = y/2 1006
y
=
x=y 105
by changing the order of integration. the order of integration.
y=1
Solution : Solution :
y=4 Solved Example 60 :
(2, 4) y=4 3 4y
y=a
y=4 (2, 4) (4, 4)
(a, a)
Evaluate 0 1  x  y  dxdy by changing

D3 y=x y=x 2 x=2


y = 2x
y=x
the order of integration.
x=a
D2 R (1, 1) y=1 Solution :
(0.5, 1) D1 (2, 2) The region is bounded by
y=1 (0, 0) (a, 0)
(1, 1) y=0
(0, 0)
(0, 0) Region R of integration y=3
The region of integration is bounded by
is bounded by x=1
Here, x varies from 0.5 to 4 y=1
x=y x2 = 4  y
y=4
1 x=a
when  x  2 , y varies from 1 to 2x 2
2 x =y
y=0
when 1  x  2, y varies from x to 2x x=2 (1, 3) y=3
y=a
when 2  x  4, y varies from x to 4 We change the order of integration as dy
We have to change the order of integration
For changing the order of integration we dx. In the region x varies from 1 to 2 and
as dy dx. x=1 y = 4  x2
must divide D into sub regions D1, D2, D3. for fixed x, y varies from 1 to x2.
Here in the region R, x varies from 0 to a
4 2
 I =  f  x, y  dxdy
D
and for fixed x, y varies from 0 to x. 1   x
2
 y 2 dxdy  (0, 0) (1, 0) (2, 0)
y
a a a x
xdxdy x
=  f  x,y  dxdy   f  x, y  dxdy 0 y x2  y2 =  2
0 0 x y
2
dydx 2x 2
We have to change the order of integration
1 1  x 
2
D1 D2
=  y 2 dydx
as dydx.
  f  x, y  dxdy a x
1  y 
0 x  x tan
1
D3
=  x   dx x2 In this region x varies from 1 to 2
 0 2
 y  3

1 x y 
2
=  dx and y varies from 0 to 4  x2
1 2x 2 2x
3 1
  f  x,y  dydx    f  x, y  dydx
a x
=  1  y  
= 0 tan  x  dx 3 4y

0 1  x  y  dxdy
1/ 2 1 1 x 2
 x6 1
1  x
0 4
=   x 2   dx
4 4
3 3
   f  x,y  dydx a

0 tan 1  tan1 0  dx
1
2 x
= 2 2 4  x2
 x5 x7 x3 x 
=      = 1 0  x  y  dydx

a
 a a  5 21 3 3 1
dx   x 0 
4 0
=
4 4

101 102
Notes on Calculus Vidyalankar : GATE – Engineering Mathematics

4  x2 We divide the region into two subregions Solution : 1


 2
 y2 
0 y  x 0
2 2 1 y
= dy
= 1 xy  
2 0
dx R1 and R2 (0, 1)
 R xydydx   xydydx   xydydx 1

   x=1
2
0 y
2
2 4  x2 R R = 1  y 2 dy

1 x 4  x  2   dx 1 2
2
= x=0
 We have to change the order of integration
  /2
as dxdy in R1 and R2.
0 sin
2
(0, 0) y=0 (1, 0) =  cos d
 x4
2

=    x 3  4x 2  4x  8  dx In region R1, y varies from a to 2a for a
1
2  2
x +y =1 2
fixed y, x varies from 0 to 2a  y Putting y = sin  
2  when y = 0  = 0 
 x 5 x 4 4x 3  In the region R2, y varies from 0 to a and  
=     2x 2  8x 
 10 4 3 1  y = 1  =  /2 
for a fixed y, x varies from 0 to ay . The region is bounded by
241 again put t = sin 
= a 2a  x y = 0 (x  axis)
60  0  xydydx  dt = cos  d
x2 / a y= 1  x 2 or x 2  y 2  1
When  = 0 t=0
Solved Example 61 : = 
R
xydydx   xydydx
R
x = 0 (y  axis)  = /2 t=1
1 2
a 2a  x x=1 1
0   t3 
1
Evaluate xydydx by changing the
 0 t dt   3 
2a 2a  y a ay 2
Here y varies from 0 to 1 and for fixed y
x2 / a = a 0 xydxdy  
0 0
 xydxdy 0

order of integration x varies from 0 to 1 y 2


1 1 x 2
2a  y 1
0 0
ay
Solution : 2a
 x2 y   x2 y  a
2  y 2 dydx 
= a   dy     dy 1 1 x 2 1 1 y
3
 2 0 0
2 0  0 0 y 2 dydx = 0 0 y 2 dxdy
(0, 2a) 2a a
y ay 2
 2a  y  dy   dy
2
2
x = ay = a 2 0
2
R1 2.10 Triple Integrals
(0, a) (a, a) 2a
 y3  a
a

a  2a y   2ay 2  dy   y 2 dy
2
= A triple integral of a function defined over a region R is denoted by
R2 2  20
y = 2a  x
2a a  f  x, y,z  dxdydz or  f  x,y,z  dV or  f  x, y,z  d  x,y,z 
 y 4 2ay 3   y3  R R
= a2 y 2    a 
(0, 0) (a, 0) (2a, 0)  8 3 a  6 0
Evaluation of Triple Integrals :
5a 4 a 4 9a 4
=  = If the region R can be described by
24 6 24
The region R is bounded by
x1  x  x 2 , y1  x   y  y 2  x  , z1  x,y   z  z2  x, y 
x=0
Solved Example 62 : then we evaluate triple integral as
x=a
1 1 x2
x2 y 2  x  z2  x,y  x2  y2  x   z2  x,y   
y = 2a  x
Evaluate 0 0 ydydx by interchanging the    f  x,y,z  dzdydx =  
x y x
  f  x,y,z  dz  dy  dx
  
 
x y x z x,y   1  1    z1  x,y 
x2 1 1 1
y=
a order of integration.

103 104

Notes on Calculus Vidyalankar : GATE – Engineering Mathematics

Note : There are six possible ways in which a triple integral can be evaluated
log a
 1 4x 3 2x x 
1
0y 36  4x 2
= 0  e  2 e  e  dx
2  3
(order of variables of integration). We choose the one which is simple to
log a 0x3
use. 1 3 
=  e4x  e2x  e x 
8 4 0 2 9
x2 
 3 3 
 I =  0  36  4x  9y  dy  dx
2 2
Solved Example 63 : x
1  xy 
a x a 4
1 1 4 3 2 3
0
2 0 0
= xy 3 dydx =    dx = a  a a
2 0  4 0 8 4 8  
2 3 2

   xy zdzdydx
2
Evaluate
0 1 1 a
1 x 
a 6 6 x2
1 5 a Solved Example 67 : 3

0 4  9  x  y  3y  0
2 9

8 0
Solution : = x dx     = 2 3 3
dx
8  6  0 48
Find the volume of the solid in the first
2
2 3 2 2 3
 xy z  2 2
octant bounded by the paraboloid.
0 1 1 xy zdzdydx  0 1 
2 3
 dydx  8 8 
0  3  9  x   
3/2 3/2
Solved Example 65 : = 2
 9  x2
2 1
9  dx
 / 2 1
z  36  4x 2  9y 2 
…(Integrating w.r.t. z keeping x, y constants)
0 0 0 r
2
Evaluate : I = sin drdd
Solution : 16
3

 
3/ 2

9 0

2 3
xy 2  = 9  x2 dx
=    2xy 2   dydx We have I=  dzdydx
2  Solution :
0 1 R
 /2 1 Substituting x = 3 sin  we get
2 3 r3  The projection of the paraboloid (in the first
3 I= 0 0 sin   3  dd
2 0 1
= xy 2 dydx 16
/2
octant) in the x  y plane is the region in 0  27 cos    3 cos   d
3
0 I =
9
 /2 
1 1 the first quadrant of the ellipse
  cos 0 d
3 /2

3 0 0 sin dd 3 0
3  xy 3 
2
= =
2 0  3 1
/2
=   dx 4x 2  9y 2  36
0 cos
4
= 144 d

1 1    The region R is
…(integrating w.r.t. y keeping x constant) =  d   0 
30 3 3 3 1 
2 0  z  36  4x 2  9y 2 = 144      27 cubic units
1 4 2 2
 27x  x  dx
2 0
=

2
Solved Example 66 :
= 13  xdx log a x x  y 2.11 Change of Variables in Double and Triple Integrals
0 0 0 e
xy z
0 Evaluate I = dzdydr
2
and Jacobians
 x2 
= 13    26 Solution : Consider the transformation given by the equation
 2 0
log a x
xy x = x(u, v, w) ;
0  e  0 dydx
xy z
I=
Solved Example 64 : 0 y = y(u, v, w) ;
a x y
log a x z = z(u, v, w)
0 0 0 xyzdzdydx 0 0  e 
Evaluate I = =
2 x  y 
 e x  y dydx
Where the functions x, y, z have continuous first order partial derivatives.
Solution : log a x
 1 2 x  y  
 xyz2 
a x y = 0 2 e

 e x  y  dx
0
I =   dydx
0 0
2 0

105 106
Notes on Calculus Vidyalankar : GATE – Engineering Mathematics

The Jacobian J of the transformation is defined by Solution : Solution :


x x x cos  r sin  0 The equations are
  x, y,z 
u v w J=   sin  r cos 0 = r AB  x  y = 
y y y  r, ,z 
J= 0 0 1 BC  x + y = 3
u v w
z z z and  f  x, y,z  dxdydz CD  x  y = 
u v w R
DA  x + y = 
  f r cos ,r sin ,z  rdrddz
  x, y,z  D(0, ) C(, 2)
The Jacobian is also denoted by J = R 
  u, v, w  x  y = 

x+y=
For a transformation in two variables x = x(u, v) and y = y(u, v) the Jacobian is given by a Solved Example 71 : x + y = 3

  x, y  2
If u = x  y and v = 2xy 2
xy=
determinant of order two. Hence J =
  u, v  Prove that
A(,0) B(2,)

x x Substitute y  x = u and y + x = v
Solved Example 68 : Solution :   x, y  u v 2x 2y Then   u   and   v  3
 =  
The transformation from cartesian Here 0     and 0    2   u,v  y y 2y 2x
v u v u
u v  x= and y 
coordinates (x, y) to polar coordinates (r, )   x,y,z  2 2
J=
is given by  r, ,   = 4 x y  2 2
 x x
x = r cos  y = r sin    x, y  u v
sin  cos  r cos  cos  r sin  sin  Solution : and I= 
Solution :   u,v  y y
  sin  sin  r cos  sin  r sin  cos 
cos  r sin  0

We have x 2  y 2  x 2  y 2   2xy     2
u v
x x
r  cos  r sin   u2  v 2 1 1
J=  r = r2 sin  
y y sin  r cos  2 2 1
= 
r  and  f  x,y,z  dxdydz  x 2
 y 2  u2  v 2 1 1 2
R 2 2
and   u,v 
  f r sin  cos ,r sin  sin ,r sin     4 u2  v 2 1
R f  x,y  dxdy  R f r cos ,r sin   rdrd R   x,y   |J|=
2
r 2 sin drdd   u,v  1
  x  y  cos2  x  y  dxdy
2
   I =
Solved Example 69 :   x, y  4 u2  v 2 R

The transformation from Cartesian 1


3 
Solved Example 70 :    u
2
cos2 vdudv
coordinates (x, y, z) to spherical polar Solved Example 72 : 2
The transformation from Cartesian
coordinates Evaluate the integral
coordinates (x, y, z) to cylindrical 3
3

 cos
2
= vdv
  x  y  cos2  x  y  dxdy , where R is the
2
(r, , ) is given by coordinates 3
R
x = r sin  cos  (r, , z) is : 3
parallelogram with successive vertices at 3 4
y = r sin  sin  x = r cos , y = sin , z = z
=
6  1  cos 2v  dv  3
(, 0), (2, ), (, 2) and (0, )
z = r cos 

107 108

Notes on Calculus Vidyalankar : GATE – Engineering Mathematics

Solved Example 73 : 
 r 2  1 2
 Solved Example 76 : The projection of the given region on the
=
2 0
e rdr =   e r d r 2
20 2
  Using triple integral find the volume of the x  y plane is the triangle bounded by the
Evaluate I = R x 2  y 2 dxdy by changing
 sphere x2 + y2 + z2 = a2 lines
  1 r 2   1 
to polar coordinates, where R is the =  e    
2  2 0 2  2  4
Solution : x = 0;
region in the x  y plane bounded by the V = 8  dxdydz where D is the region x y
 y = 0 and  1 0xa
circles x2 + y2 = 4 and x2 + y2 = 9.  I= D a b
2 2 2 2 2
Solution : bounded by the sphere x + y + z = a in In this region x varies from 0 to a.
x = r cos  Solved Example 75 : the first quadrant.
Using  x
For fixed x, y varies from 0 to  1   b.
Substitute x = r sin  cos , y = r sin  sin  a
y = r sin  Evaluate the integral  zdxdydz , where T is
T , z = r cos   x
We get dx dy = r dr d and i.e. 0  y   1   b
the hemisphere of radius a, x2 + y2 + z2 = a2, Jacobian | J | = r2 sin   a
2 3 2 3
r3 
I=   r r dr d   
0 2 0
  d
 3 2
z  0. and the limits r = 0 to a; 0    /2 For fixed (x, y), z varies from 0 to
Solution : 0    /2  x y
19
2
38 1 a  b  c
Changing to spherical coordinates  
= 0 d =
V = 8
a /2 /2
3 3
substitute   0 J dddr
  x y 
0  z   1  a  b  c 
0 0
x  r sin  cos , y  r sin  sin , z  r cos  a /2 /2    
Solved Example 74 :
0    2 and 0    /2 = 8  0 r
2
sin dddr
Evaluate the improper integral 0 0
 I = 
D
dxdydz
We get,
a /2

 r   cos 0 ddr
/2
= 8 2  x  x x
 x2 dx dy dz = r2 sin dr d d  1 b  1   c
I= 0 e dx a  a  a b

2 / 2 a
0 0
= 0 0 0 dzdydx
 0 0  r cos   r
2
Solution : I = sin  dr d d a  /2 a
r 2 d dr = 8  r 2  0 dr
/2
= 8   x
 1 b
I2 = I  I 2 / 4
0 0 0 a  a
 x y
a4 = c 0  1  a  b  dydx
  2    2     x2  y2  =
4 0 0 sin  cos dd a
r3 
a
 
=   e x dx    e  y dy     e
0
dxdy = 4 r dr 2
= 4  
0  0  00 2 / 2
0  3 0  x
 1 b
a4 a
 x y2  a
Put x = r cos  =
8 0 0 sin2dd 4a 3 = c   1   y  
a 2b  0
dx
= 0 
y = r sin  3
/2 a
x 
2 2 2
a4  cos 2   x 1
Hence J = r =
8 0   2  0
d
Solved Example 77 :
= bc   1     1    dx
 a  2  a 
0 
The region of integration is the entire first
2
Find by triple integral the volume of the
quadrant. Hence r varies from 0 to  and  a4 bc 
a
x
2
=
8 0 d tetrahedron bounded by the planes x = 0, =
2 0  a 
1 dx
varies from 0 to /2.
x y z
 /2 a 4 y = 0, z = 0 and   1 a
r 2 = a b c bc  a  x 
3
abc
 I2 = 0 0 e rddr 4 =  1  =
Solution : 2  3  a   6
0

109 110
Notes on Calculus Vidyalankar : GATE – Engineering Mathematics

Solved Example 78 : Solved Example 80 :


 uvwdudvdw
2 2 2 1 3
= ab c
R
= 0 4xdx    3  x  dx
Find the area of the region bounded by the
Evaluate I =  xyzdxdydz where D is the
Now substitute
1
R
graph of y = sin x and y = cos x between 1 3
 2   x2 
positive octant of the ellipsoid u = r sin  cos  x = 0 and x = /4. = 2  x 3/ 2   3x  
 3 0  2 1
x 2 y 2 z2 v = r sin  sin  Solution :
  1
a 2 b2 c 2 w = r cos  4 10
= 2 = sq. units.
3 3
Solution : Then J = r2 sin 
Substitute x = au, 1 /2 /2 Solved Example 82 :
y = bv I  a 2 b2 c 2   0 r
5
sin3  cos  cos  sin dddr
0 0
Find the area of the region bounded by the
z = cw 1 /2 /2 two parabolas y = x2 and x = y2.
 a 2b2 c 2  r 5 dr 0 sin
3
 cos d  sin  cos d
a 0 0
  x,y,z  0 0
J=  0 b 0  abc 1 /2 /2
  u,v,w  1  1  1 
= a b c  r 6   sin4    sin2  
2 2 2 /4
0 0 c
 6 0  4 0  2 0
 Required area = 0  cos x  sinx  dx
x = y2
Let R be the positive octant of the sphere P
a 2 b2 c 2 /4
u2 + v 2 + w 2 = 1
=
48
=  sinx  cos x  0 B
y = x2
 I=   abc uvw  abc  dudvdw
R
=
2

2
  0  1
A
2 2 0 M
=  
2  1 sq. units.
2.12 Application of Integration
Solution :
Solved Example 79 : The latus rectum of the parabola y2 = 4ax Solved Example 81 :
Let us first find the points of intersection
Calculate the area bounded by the is a line parallel to y  axis. The shaded Find the area above the xaxis bounded
2
solving y = x2 and x = y2
parabola y = 4ax and its latus rectum. portion is the desired area. As the by y2 = 4x and the line x + y = 3 y = y4
Solution : parabola is symmetric about the xaxis, Solution : 3
 y (y  1) = 0
therefore area y
y  y=0  x=0
L OLL = 2  Area OSL. (1, 2)
A y=1  x=1
a a a
= 2  ydx  2  4axdx  4 a  x1/ 2 dx For the curve x = y2
(3, 0)
0 0 0
S 1 1
0 C B
0 (a, 0) x x 8 3/2 a
Now the area OBPM = 0 ydx  0 xdx
= 4 a  a a3 / 2  0
3 / 2 x 0 3 1
 x3 / 2 
 
8a 2
=  3 / 2 0
3
Required area = area OCA + area ACB 2
 sq.units
3

111 112

Notes on Calculus Vidyalankar : GATE – Engineering Mathematics

2
For the curve y = x Solved Example 84 : and in the interval (0, 2) ; 32x + 3  2x5 + 3  x6 
0

2
x6 
=   16x 2    16x 2  
1 Find the area of one turn of the
 3  2  3  x 0
Area OAPM = 0 ydx archimedan spiral r = a.
Thus the required area is given by
0 128 128 256
  2x  =   sq. units.
5
1 Solution : A =  3  32x  3 dx 3 3 3
1
 x3  1
  x dx     sq. units.
2 2

0  3 0 3 2

2 1 1 
  32x  3  2x 5  3 dx 
The required area =   sq. units. 0
3 3 3 O
A
Solved Example 83 :
2.13 Partial and Total Derivatives
Find the area bounded by the cardioid Functions of two variables
2 2
1 2 1
2 0
r = a(1 + cos ) The area is A = r d  a 2  2 d  If three variables x, y, z are so related that the value of z depends upon the values of x
2 0
Solution : and y, then z is called the function of two variables x and y and this is denoted by z = f(x, y)
a 2  2 
3
4
=  3 a 2 Partial derivatives of first order
2 3 3
O A Let z = f(x, y) be a function of two independent variables x and y. If y is kept constant and
x alone is allowed to vary then z becomes a function of x only. The derivative of z with
Solved Example 85 :
respect to x, treating y as constant, is called partial derivative of z with respect to x and is
Find the area bounded by the curves :
5
denoted by
y = 2x + 3 and y = 32x + 3
A cardioid is symmetrical about the initial z f
Solution : or or fx
line. x x
y
 z f(x  h, y)  f(x,y)
1 C  lim
Hence the required area = 2  r 2 d since x h0 h
0
2
Similarly the derivative of z with respect to y, treating x as constant, is called partial
 varies from 0 to  for the upper part.
(0, 3) y=3 derivative of z with respect to y and is denoted by

 a 1  cos   d B
2
= 2
z f
x or or fy
0 x y y
 /2
 z f(x,y  k)  f(x,y)
= 4a 2  cos4 d  4a 2 0 cos
4
t2dt A Thus  lim
0
2 y k 0 k

 y = 32x + 3 z z
(putting  t) y and are called first order partial derivatives of z
2 x y
/2 Two curves will intersect when
3 1
= 8a 2 0 cos
4
tdt  8a 2
422 2x 5  3  32x  3  2x  x 4  16   0

3a 2  In the interval (2, 0) ; 2x 5  3  32x  3


=

113 114
Notes on Calculus Vidyalankar : GATE – Engineering Mathematics

Note : (i) If z = u + v where u = f(x, y) , v   (x, y) , then z is a function of x and y. Solved Example 86 : Solved Example 88 :

z u v z u v First order partial derivative of u = y x is If u = log (tan x + tan y) show that


   ;  
x x x y y y Solution : u u
sin 2x + sin 2y =2
u = yx x y
(ii) If z = uv, where u = f(x, y) , v = (x, y)
Treating y as constant Solution :
z  v u
then  (uv)  u v
 x x x x u u = log(tan x + tan y) [ u is symmetrical
= y x log y
x with respect to x and y]
z  v u
 (uv)  u v
y y y y treating x as constant u 1 
= . (tan x  tany)
u x tan x  tany x
u = x yx1
(iii) If z = , where u = f(x, y), v =(x, y) y
v sec 2 x
=
u v tanx  tany
v u
z  u x x Solved Example 87 :
then     u
dx x  v  v 2 Similarly
Find first order partial derivative of y
u v x2  y2
v u u = tan1 sec 2 y
z  u y y xy =
  tanx  tany
y y  v  v 2
Solution :
u u
(iv) If z = f(u) where u =  (x, y) = sin 2x + sin 2y
u 1   x2  y2  x y
z dz u z dz u = .  
then  . ;  . x  x 2  y 2  x  x  y 
2
1
x du x y du y 1   = [sin 2x sec2x + sin 2y sec2y]
 x  y  tanx  tany

(x  y)2 1
Partial derivatives of higher order = =
(x  y)2  (x 2  y 2 )2 tanx  tany
  z   z 2
 or fxx  1 
x  x  x 2  2  1
(x  y)
x
(x  y 2 )  (x 2  y 2 ) (x  y)
x 2 sinx cos x. cos2 x  2 siny cos y cos2 y 
  
  z   2 z (x  y)2
  or fyy (tan x  tany)
y  y  y 2 (x  y).2x  (x 2  y 2 ).1 =2 =2
= tanx  tan y
(x  y)2  (x 2  y 2 )2
  z   z 2

   or fxy
x  y  xy u x 2  2xy  y 2
  Solved Example 89 :
x (x  y)2  (x 2  y 2 )2
  z  2z
 or fyx 3y 3 y
y  x  yx u y 2  2xy  x 2 If u = xy show that 
Similarly  x 2 y xyx
2z 2z y (x  y)2  (x 2  y 2 )2
In general  or fxy = fyx Solution :
xy yx
u = xy
u
= xy log x
y

115 116

Notes on Calculus Vidyalankar : GATE – Engineering Mathematics

2
 x   u  1  y 2
  y  Solved Example 90 : Solved Example 91 :
     yx y 1 log x  x y   x y 1  yx y 1 log x
xy x  y  x yx y  x  1 1 log x  log y x2 y2
If f(x, y) = 2   If u= show that
= xy1 (y log x + 1) = xy1 (y log x + 1) x xy x2  y2 xy

3 y   2 x   y 1  3u    3u   y 1 f f  2u  2u u
    [x (y log x  1)]      x (y log x  1) show that x  y + 2f (x, y) = 0 x y  2
x 2 y x  xy  x xyx x  yx  x x y x 2 x y x
...(1) …(2) Solution : Solution :
u  3u  3u 1 1 log x / y
= y xy1 from (1) and (2)  f(x, y) =   2 x2 y2
x x 2 y xyx u = is a homogenous function of
x2 xy x  y2 xy

Euler’s theorem on Homogenous functions tx degree 3 in x, y


log
1 1 ty
If u is a homogenous function of degree n in x and y then f(tx, ty) = 2 2
 2  2 2 2 2  By Euler’s theorem we have
t x t xy t x t y
u u
u u x +y =3u
x +y = nu  x  x y
1 log 
x y = 21 1 y
t  2   2  Differentiating (1) partially with respect to x
 x xy x  y 2 
Since u is a homogenous function of degree n in x and y it can be expressed as  2u u 2 y u
= t2 f (x, y) x  1.  y  3
u = xn f x 2 x xy x
f (x, y) is a homogenous function of degree
u y  y   y   2u  2u u
 = n xn1f    xn f '   .  2  or x y  2
x x x x  2 in x and y
x 2 xy x
u By Euler’s theorem, we have
y y
x = n x n f    x n 1 y f '   ...(1)
x x x f f
x y = 2 f (x, y)
x y
u y 1 y
Also = xn f    . = xn1 f   
y x x x Composite functions
u y (i) If u = f (x, y) where x = (t), y = (t)
y = xn y f    …(2)
y x du
then u is called a composite function of (the single variable) t and we can find .
dt
Adding (1) and (2) we get
u u (ii) If z = f(x, y) where x = (u, v), y = (u, v)
y
x +y = n xn f   = nu
x y x then z is called a composite function of (2 variables) u & v so that we can find
z z
Note : Euler’s theorem can be extended to a homogenous function of any &
u v
number of variables. Thus if u is a homogenous function of degree n in x,
Total derivative of composite functions
y and z then
If u is a composite function of t, defined by the relations
u u u
x +y +z = nu u = f(x, y) ; x = (t), y = (t) then
x y z
du u dx u dy
 .  .
dt x dt y dt

117 118
Notes on Calculus Vidyalankar : GATE – Engineering Mathematics

Note : 1. If u = f (x, y, z) and x, y, z are functions of t then u is a composite 1 z z


= .3 = x y
1  (x  y) x y
function of t and
u dx u dy u dz 1
du
 .  .  .  (1) . 12 t 2 Solved Example 95 :
dt x dt y dt z dt 1  (x  y)2
If  (x, y, z ) = 0 show that
2. If z = f (x, y) and x, y are functions of u and v, then 3(1  4t 2 ) 3(1  4t 2 )
=   y   z   x 
z  z x z  y 1  (x  y)2 1  (3t  4t 3 )2  z   x   y    1
 .  .   x   y  z
u  x u  y  u
3(1  4t 2 ) Solution :
=
 z z x z y 1  9t 2  24t 4  16t 6 The given relation defines y as a function
 .  .
v x v y v
3(1  4t 2 ) of x and z. Treating x as constant.
3. If u = f (x, y) where y = (x) =
(1  t 2 )(1  8t 2  16t 2 ) 
then since x = (x), u is a composite function of x  y  z
  
3(1  4t ) 2
3  z  x 
du u dx u dy = 
  .  . (1  t 2 ) (1  4t 2 )2 1 t2 y
dx x dx y dx
The given relation defines z as a function
du u u dy
   . of x and y. Treating y as constant.
dx x y dx Solved Example 94 :
If z is a function of x and y 
du  z 
is called the total differential coefficient of u, to distinguish it from its x
dx where x = eu + ev and y = eu  ev,  x   
 y
u show that z
partial derivative .
x z z z z 
 x y  x 
u v x y y
Similarly   
 y z 
Solved Example 92 : = 2a3t3(5t + 8) Solution : x
du Also u = xy2 + x2y = at2 . 4a2t2 + a2 t4. 2at Here z is a composite function of u and v
Find when u = x when u = xy2 + x2 y, Multiplying we get the desired result.
dt = 4a3t4 + 2a3 t5 z z x z y
  .  .
x = at2, y = 2at. du u x u y u
= 16a3 t3 + 10a3 t4 = 2a3 t3 (5t + 8) Solved Example 96 :
Solution : dt z u z
= e  ( e  u ) If u = f (y  z, z  x, xy) prove that
The given equations define u as a x y
u u u
composite function of t. Solved Example 93 : z z x z y   =0
and  .  . x y z
1 3
du u dx u dy If u = sin (x  y), x = 3t, y = 4t show that v x v y v
  .  . Solution :
dt x dt y dt du 3 z z
= = (e v )  (e v ) Here u = f (X, Y, Z)
= (y2 + 2xy). 2at + (2xy + x2) . 2a dt 1 t2 x y
2 2 2
where X = y z, Y = z  x, Z = xy
= (4a t + 2at . 2at) 2at Solution : subtracting
2. 2 4  u is a composite function of X, Y and Z
+ (2at 2at + a t ).2a du u dx u dy z z z z
 .  .   (eu  e  v )  (e u  e v )
= 8a3t3 + 8a3 t4 + 8a3t3 + 2a3 t4 dt x dt y dt u v x y

119 120

Notes on Calculus Vidyalankar : GATE – Engineering Mathematics

u u X u Y u Z u u X u Y u Z x2 x4 x6 
(1)n 2n
 .  .  .  .  .  .  cos x = 1   .... x
x X x Y x Z x z X z Y z Z z 2! 4! 6! n  0 (2n)!

u u u u u u x3 2 5
= (0)  (1)  (1) = (1)  (1)  (0)  tan x = x  x ....
X Y Z X Y Z 3 15
u u X u Y u Z Adding
 .  .  . Taylor's series in two variables
y X y Y y Z y u u u
  =0 Taylor's series of a function in two variables, f(x, y) about (a, b) is
u u u x y z
= (1)  (0)  (1)
X Y Z f(x, y) = f(a, b) + (x  a) fx (a, b) + (y  b) fy (a, b)
1
+ (x  a)2 fxx (a,b)2(x  a)(y  b)fxy (a,b)(y  b)2 fyy (a,b) ....
2! 
2.14 Taylor's Series and Maclaurin's Series
Taylor's series is a representation of a function as an infinite sum of terms that are Solved Example 97 : Solved Example 98 :
calculated from the values of the functions derivatives at a single point. Expand cos z in a Taylor’s series about Obtain the Taylor’s series for sin x.

f (a)(x  a) (x  a)2 
f (n) (a)(x  a)n z/4 Solution :
f(x) = f(a) +
1!
f (a).
2!
..... =  n! Solution : f(x)  sin x
n 0
f (x)  cos x
f  z   cos z
f (x)   sinz
Note : Putting x = a + h, Taylor’s series becomes f   z    sinz
f (x)   cos z
h 2
h n n f   z    cos z
f  a  h   f  a   hf   a   f   a   ...  f a  … By Maclaurin’s series
2! n! f   z   sinz
x2
 1 f(x)  f(0)  xf (0)  f (0)
If the Taylor's series is centered at zero, that series is also called Maclaurin’s series. f   2!
4 2
If a = 0 in Taylor series of f(a + h), then the series becomes xn n
 1 ...  f (0)  ...
f     n!
h2 hn n 4 2
f  h   f  0   hf   0   f   0   ...  f  0   ...  1 x 2k 1
k
2! n! x3 x5
 1 sinx  x    ...   ...
f      3! 5!  2k  1 !
4 2
This is called as Maclaurin’s series about a = 0.
 1
f     Solved Example 99 :
Taylor's series expansion of some functions 4 2
Obtain the Taylor’s series for cos hx.
x x2 x3 By using Taylor’s series
 ex = 1   .... Solution :
1! 2! 3!    
f  z   f     z   f   x2 x3
1 1 1 4  4 4 ex  1  x    ...
 e = 1   .... = 2.7182…. 2! 3!
1! 2! 3! z   / 4
2

 f     … x2 x3
x3 x5 x7 
(1)n 2! 4 e x  1  x    ...
 sin x = x   ..... x 2n1 2! 3!
3! 5! 7! (2n  1)!
1    
n 0 2
 1  ex  e x x2 x4
= 1   z     z   ... coshx   1   ....
2   4  2!  4   2 2! 4!

121 122
Notes on Calculus Vidyalankar : GATE – Engineering Mathematics

Solved Example 100 : h2


h3 Similarly if f(x) is defined in (0, 2L) then Fourier series is given by
f  x   f  2   hf   2   f   2   f   2 
Find the Taylor’s series expansion for the 2! 3! 
 nx nx 
f(x)  a 0    an cos  bn sin
n 1  L L 
function f  x   1  2x  2x 2  3x 3 about the h2 h3
= 29  h  42   40   18 
2 6 1
2L
point x = 2.
f  x   29  42h  20h2  3h3
a0 
2L  f(x)dx
0
Solution :
f  x   1  2x  2x 2  3x 3 f  2   29
2L
where h = x  2 1 nx
an   f(x)cos dx
L 0 L
f   x   2  4x  9x 2 f   2   42
f   x   4  18x f   2   40 1
2L
nx
L 0
bn  f(x)sin dx
f   x   18 L

Odd and Even Functions :


2.15 Fourier Series
Let f(x) be defined in (C, C)
Periodic function :
1. If f(x) = f(x), for all x in (C, C) then f(x) is even function.
A function f(x) is said to be periodic if f(x + T) = f(x) for all values of x where T is the same
2. If f(x) = f(x), for all x in (C, C) then f(x) is an odd function.
positive number.
For Example:
Note : Unless ‘0’ is midpoint of the given interval, we cannot talk of the function
1. sin x = sin (x + 2)
being even or odd.
= sin (x + 4) and so on.
For Example: f(x) = x2 in (0, 3) is neither even nor odd, since if x  (0, 3)
sin x is periodic in x with period 2.
then x  (0, 3) and hence f(x) is not defined.
2. The period of tan x is .
But if, f(x) = x2 in (3, 3) then f(x) is even function.
Fourier series (General form)
Let f(x) be defined in (L, L) then Fourier series is given by Graphical representation of even and odd function :
 i) If f(x) is even, then its graph is symmetrical about f(x) axis
 nx nx 
f  x   a 0    an cos  bn sin
n  1 L L  For Example: If f(x) =  x,  < x < 0
=x 0<x<
L
1 then,
f  x  dx
2L L
where a0  f(x)

L
1 nx
f  x  cos
L L
an  dx
L

1
L
nx x
f  x  sin  
L L
bn  dx
L

123 124

Notes on Calculus Vidyalankar : GATE – Engineering Mathematics

ii) If f(x) is odd then its graph is symmetrical about origin. f(x) Solved Example 101 :  sin2x sin3x 
Hence x    2  sin x    ...
For Example: if f(x) = a , if  < x < 0 (0, a)  2 3 
Find the Fourier series representing
=a , if 0 < x < 
f(x) = x, 0 < x < 2 and draw its graph from
x Solved Example 102 :
 
x = 4 to x = 4.
Find Fourier Series for f(x) = x2 in  ,  
(0, a) Solution :
and show that
Fourier expansion of an even function f x  x 
ao 
   an cosnx  bn sinnx 
2 n 1 1 1 1 2
   .... 
Let f(x) be defined in (, ) and let f(x) be even function. Fourier series is given by 12 22 32 12
 where
f(x)  a 0    an cosnx  bn sinnx 
n 1
Solution :
2 2
1 1 1 4 2
1 1
  a0 
  f  x  dx    xdx   2
 2 We have f(x) = x2 is an even function in
where, a 0   f(x)dx   0 f(x)dx
2 
f(x) is even 0 0

2
 ,  
  1
an 
1 2
 f(x)cosnxdx   0 f(x)cosnxdx [ f(x) and cos nx are even functions]
an =
  x cosnxdx
  0 
nx
f  x   a 0   an cos
n 1 

1   x sinnx  
2 2
1 1
bn   f(x)sinnxdx  0
 
[f(x) sin nx is odd] = 
   n  0
 0 n sinnxdx  
1 2 2

 0
i) a0  x dx 
 3
f(x)  a 0   an cosnx [Half range cosine series in (0, )] 1  cosnx 
2
n 1 = x sinnx  =0
n  n  0 
2 2 nx
 0
ii) an  x cos dx
Fourier expansion of an odd function 
2
Let f(x) be an odd function in (, ) 1
bn =
  x sinnxdx
0 Integrating by parts

f(x)  a 0   an cosnx  bn sinnx
n 1 2
1  x cosnx sinnx  2 2  nx 
=  =  x
1

  n n2  0    n
sin
 
ao   f(x)dx  0
2 
f(x) is odd function

1 2 2 
 =     2 nx    3 nx  
1  n n 2x  2 2 cos   2  3 3 sin 
an   f(x)cosnx dx  0
 
f(x) cos nx is odd
n    n    0

 
1 2 2  2 3 n 4 2
bn   f(x)sinnxdx =  0 f(x)sinnxdx
  =   1   2 2  1
n

  n2 2  n

4 2 0 2 4
f(x)   bn sinnx dx {Half range sine series in (0, )}
n 1

125 126
Notes on Calculus Vidyalankar : GATE – Engineering Mathematics

 1
n
1   cosnx x cosnx sinnx 
2 Solved Example 105: Solved Example 106 :
 2 4 2 
 nx 
 f x 
3
 2


n 1 n2
cos 
  
 …(1) =
  n

n

n2  0 Find the Fourier series for f(x) = sin h ax in Find the Fourier series for

putting x = 0 in equation (1) we get 1  2 cos 2n  2 (, ) 2 x 2


f(x) =  in  < x <  and show
=
  n n Solution : 12 4

 1
n
 2 4 2 
0
3
 2

 n2 Clearly f(x) is an odd function of x 1 1 1 2
n 1 that   .... 
Solved Example 104 : f(x) = sin h a(x) =  sin h ax =  f(x) 12 22 33 12
2  1 1 1 1  Find a Fourier expansion for
      ... Solution :
12  12 22 32 42


f(x) = x2 for  < x < 
Let, sin h ax = 
n 1
bn sinnx
 Note that f(x) is an even function of x
1 1 1 1 2 Solution :
     ...   2 ( x)2  x 2
12 22 32 42 12 2  f(x)      f(z)
 0
x2 is an even function where, bn = sinhax  sinnx dx 12 4 12 4
  
1 1 2 1 3 2 Let f(x) = a 0   an cosnx
 x dx   0 x dx  3   3
2
Solved Example 103 : a0  
2 eax  e ax
 0
2  = sinnx dx n 1
Find a Fourier series to represent 2

1

2 2
 1  2 2 
 0  12 4 
f(x) =   x for 0 < x < 2. a0 =    dx
 x cosnxdx   0 x cosnxdx
2
an  1  ax
 

  0
 e sinnx dx   e sinnx dx 
 ax
  =
Solution : 0  
2 2
The indefinite integral 1  2 x3 
1 1 1 1 =  x  0
a0 
  f  x  dx       x  dx 2  x sinnx
2
sinnx  = 
  a 2  n2
eax  a sinnx  ncosnx    12 12  0
0 0 =   2xdx 
 n n  
 2  2 x 2 
1  0  12 4 
x  
2
2 1  an =    cosnxdx
=  x 0    
2
 e ax  a sinnx  ncosnx  
 2  x 2 sinnx 2x cosnx cosnx  a  n2
2
0
  2 0  =     2dx 
 n n n n2  Integrating by parts
1 ea  n  1n  ne a  1n 
1 =
=

2  2   0 2  x 2 sinnx 2x cosnx 2 sinnx 
=   

 a 2  n2   
=
2  2 x 2   sinnx 
  
 n n2 n3 
   12 4   n 
n  1
2 n
1
an =  f  x  cosnxdx  = e a
 e  a  
 0
2  x 2 sinnx 2x cosnx 2 sinnx 
 an      
 a 2  n2    x    cosnx   1   sinnx  
  2    2  3 
 n n2 n3  0  2  n    n  0
2
1 2n  1 sinha 
n
=
     x  cosnxdx 4 4 =  sinha 
e a  e  a 
 2    (1)
n
(1)n1
0 =   cosn  2 cosn  a 2  n2   2   =
  2n2
cosn  
n2

n2
n2  n 
2
1  sinnx x sinnx cosnx  bn = 0
   2n  1
n 1
= sinha 2 x 2 
(1)n1
  n n n2  0 =    cosnx

Integrating by parts.
4 4 4
a1  2 ,a 2  2 ,a 3  2 etc 
 a 2  n2  12 4 n1 n2
1 2 3 Put x = 0 we get
1 Hence    1n   1 1n   1n1   1n1 
= 0  0   2 
(1)n1  1 1 1 1 
    2  2  2  2  ...
2 sinha  n  1 sinnx
n 1
2 4 4 4 12 n1 n2 1 2 3 4 

2
2 x   cos x  2 cos 2x  2 cos 3x  ...  f(x) =
1 3 12 2 3  a 2  n2
bn =
     x  sinnx dx
0
n 1

127 128

Notes on Calculus Vidyalankar : GATE – Engineering Mathematics

List of Formulae d 1 dx 1 x
dx
cosh1x =
x2  1
a 2
 x2
 tan1  c
a a
 Limits d x
a  a x log e a d 1 dx 1 x
sinx dx (tanh1 x)  2 a 2
 x2

a
cot 1  c
a
lim =1 …x in radians dx x 1
x 0 x d
sin x = cos x dx 1 x
lim
tanx
=1 …x in radians
dx x x 2  a2

a
s ec 1  c
a
x 0 x  Integration
d
cosec x =  cosec x cot x dx 1 x
dx x n 1
limcosx = 1
x 0  x dx  n  1  c ,
n
(n  1) x x 2  a2

a
cos ec 1  c
a
d
x n  an cos x =  sin x
lim
x a xa
nan1 dx 1
 x dx  log x  c  sinh x dx = cos h x + c
d
ax  1 sec x = sec x tan x  cosh x dx = sin h x + c
 e dx  e
x x
lim = na (a > 0) dx c
x 0 x
dx
d
tan x = sec2x ax  = sin h1 x + c
ex 1  a dx  log
x
dx +c x2  1
lim =1 e a
x 0 x
d dx
1
dx
cot x =  cosec2x  cos xdx  sin x  c  x2  1
= cos h1 x + c
lim1  x) x = e
x 0
d x 1  cos ecx cot xdx   cos ecx  c dx
lim
n(1  x)
1 dx
sin1 
a a2  x2
x 2
1
= tan h1 x + c
x 0 x  sin x dx   cos x  c
d
cos 1
x

1  tanxdx =  log (cos x) + c

 Continuity
dx a a2  x2  sec x tanx dx  sec x  c
 x
The function f is said to be continuous d x a  sec
2
x dx  tanx  c  sec xdx  logtan  4  2 c
tan1  2
at x = x0 if dx a a  x2
 x
 cos ec x dx
2
= cot x + c
i) lim f(x)   exists and d x a  cosec xdx log  tan 2  +c
x  x0 cot 1  2
dx a a  x2  cot xdx = log sin x + c dx x x
ii) lim f(x)  f(x 0 )
d x a   sin1 or cos 1 c
a a
 tanxdx
x  x0
sec 1  = log sec x + c a2  x2
dx a x x2  a2
 Derivatives dx x
d d x a  sec xdx  log(sec x  tan x)c   sinh1  
a
k(constant)0 cosec 1  a2  x 2
dx a x x2  a2
dx
 cosecxdx   log(cosecx  cot x)c or log(x  x 2  a 2 )c
d n d
x nxn1 cos hx = sin hx 1 x
dx dx   sin1 c dx x
a2  x2 a  x 2  a2
 cosh1  c
a
d 1 d
log x  sin hx = cos hx dx x
dx x dx  a2  x 2
  cos 1
a
c orlog(x  x 2  a 2 )c
d x d 1
e  ex sin h1x =
dx dx x2  1

129 130
Notes on Calculus Vidyalankar : GATE – Engineering Mathematics

dx 1 x Some Important Substitutions In Rule 8: The presence of 2ax  x 2 2 tanA


a 2
 tan1  
 x2 a a Integration
tan 2A =
1  tan2 A
suggests the substitution
1 1  x  Rule 1: If the numerator is equal to or sin 3A = 3 sin A  4 sin3A
or cot  c x = a (1  cos)
a a a multiple of the derivative of the cos 3A = 4 cos3 A  3 cos A
1 denominator, then put the denominator 3 tanA  4 tan3 A
 a 2  x 2 dx  x a 2  x 2
2 equal to some other variable say z. It
 Important Trigonometric Identities tan 3A =
1  3 tan2 A
2 2
sin A + cos A =1
1 x is then reduced to standard form after
 a 2 sin1  c sin (A + B) = sin A cos B + cos A sin B
2 a simplification.  Definite Integration
cos (A + B) = cos A cos B  sin A sin B
1

b
a 2  x 2 dx  x a2  x2 Rule 2: If there are two factors in the x b
2 tan (A + B) =
tanA  tanB
 f(x)dx    x  x a
  b     a 
integrand one of which is of the type 1  tanA tanB a
1 x
 a 2 sinh1  c sin (A  B) = sin A cos B  cos A sin B
2 a  [f(x)] and other of the type f (x) then Properties of definite integral
put f(x) = z. cos (A  B) = cos A cos B + sin A sin B
1 b b

 x  a dx  x x 2  a 2
2 2

2 tan A  tanB  f(x)dx f(t)dt


Rule 3: If the integrand is of the tan (A  B) = a a
1  tan A tanB
1 x
 a 2 cosh1  c type a 2  x 2 , then put x = a sin  or
b a

 f(x)dx  f(x)dx
2 2
2 a sin A  sin B = sin (A + B) sin (A  B)
x = a cos cos2 A  sin2 B = cos (A + B) cos (A  B)
a b
dx 1 ax
a 2
 x2

2a
log
ax
c
2 sin A cos B = sin (A + B) + sin (A  B)
b c b
Rule 4: If the integrand is of the type  f(x)dx f(x)dx f(x)dx
dx 1 xa x2 + a2 or some power of it then put 2 cos A sin B = sin (A + B)  sin (A  B)
a a c

x 2
 a2

2a
log
xa
c
a a
x = a tan  or x = a cot 2 cos A cos B = cos (A + B) + cos (A  B)
 f(x)dx f(a  x)dx
2 sin A sin B = cos (A  B)  cos (A + B) 0 0
Rule 5: If the integrand is of the
Integration by parts a a
CD CD
 du  type x 2  a 2 , then put x = a sec  or 2 sin
2
cos
2
= sin C + sin D  f(x)dx2 f(x)dx …f(x) = f(x)
 uvdxu vdx  dx  vdx dx x = a cosec 
a 0

CD CD =0 …f(x) = f(x)


2 cos sin = sin C  sin D
Of the two functions the first function u Rule 6: If the integrand is of the 2 2 2a a
is selected based on the following
ax 2 cos
CD
cos
CD
= cos C + cos D
 f(x)dx2 f(x)dx …f(2a  x) = f(x)
type , then put x = a cos 2 and 0 0
preference order ax 2 2
1. Inverse function =0 …f(2a  x) = f(x)
CD DC
a2  x2 2 sin sin = cos C  cos D
2. Logarithmic function if it is of the type , then put 2 2
a2  x 2
3. Algebraic function cos 2A = cos2 A  sin2 A = 1  2sin2 A  Euler’s theorem on Homogenous
x2 = a2 cos 2
4. Trigonometric function 2
1  tan A functions
= 2 cos2A  1 =
5. Exponential function Rule 7: The presence of ax 1  tan2 A u u
x +y = nu
suggests the substitution of a + x = t2 x y
2 tanA
2 sin 2A = 2 sin A cos A =
or x = a tan  or x = a cos 2. 1  tan2 A

131 132

Notes on Calculus Vidyalankar : GATE – Engineering Mathematics

 Total derivative of composite  Maclaurin's Series Assignment  1


functions h2
f  h  f  0   hf   0   f   0   ... Duration : 45 Min. Max. Marks : 30
du u dx u dy 2!
 .  .
dt x dt y dt n
h n Q 1 to Q 10 carry one mark each 1
 f  0   ... (A) 0 (B)
n! xsinx 2
 Taylor's series 1. Lt  is [ME  2014]
x0 1cosx 1
f (a)(x  a) (C) (D) undefined
f(x) = f(a) +   Fourier Series (A) 0 (B) 1 4
1!

 nx nx  (C) 3 (D) not defined
f  x   a 0    an cos  bn sin 6. If a continuous function f(x) does not
f (a). 
(x  a)2
.... n 1  L L 
2! have a root in the interval [a, b], then
L
2. If a function is continuous at a point,
1 which one of the following statements
f  x  dx
2L L
f (n) (a)(x  a)n
 where a0  (A) the limit of the function may not
= 
n0 n! exist at the point is TRUE? [EE  2015]
L
f(x, y) = f(a, b) + (x  a) fx (a, b) 1 nx (A) f(a)f(b) = 0 (B) f(a)f(b)0
an   f  x  cos dx (B) the function must be derivable at
L L L
+ (y  b) fy (a, b) the point (C) f(a)f(b)0 (D) f(a) f(b)0
L
1 nx (C) the limit of the function at the point
(x  a)2 f (a,b)  f  x  sin
L L
bn  dx a y
1  
 
xx
L 7. The double integral f(x,y)dxdy is
tends to infinity
+ 2(x  a)(y  b)fxy (a,b)   .... 0 0
2!  
2 (D) the limit must exist at the point and equivalent to [IN  2015]
(y  b) fyy (a,b) 
the value of limit should be same x y

as the value of the function at that


(A)  
0 0
f(x,y)dxdy

 a y
point [ME  2014] (B)  
0 x
f(x,y)dxdy
x
 1 a a
3. The value of lim  1  is [EC  2014]
x
(C)  
0 x
f(x,y)dydx
x 

a a
(A) n2 (B) 1.0 (D)  
0 0
f(x,y)dxdy
(C) e (D) ∞ n

 1
1
8. The value of n 
n 0  2 
is _________.


4. The series n 0
converges to
n!
[EC  2015]
[ME  2014]
loge (1  4x)
9. Lt is equal to
(A) 2 In 2 (B) 2 x 0 e3x  1
(C) 2 (D) e [ME  2016]
1
(A) 0 (B)
1cos(x 2 ) 12
5. The value of limx 0  is
2x 4 4
(C) (D) 1
[ME  2015] 3

133 134
Assignment on Calculus Vidyalankar : GATE – Engineering Mathematics

3
x  sin(x) 17. A political party orders an arch for the Assignment  2
10. The value of limx 0  is
x entrance to the ground in which the
[ME  2017] Duration : 45 Min. Max. Marks : 30
annual convention is being held. The
(A) 0 (B) 3
(C) 1 (D) 1 profile of the arch follows the equation Q 1 to Q 10 carry one mark each (C) f(x0) = 0 and f(x0) < 0
y = 2x  0.1x2, where y is the height of  e2x  1  (D) f(x0) = 0 and f(x0) > 0
Q 11 to Q 20 carry two marks each
the arch in meters. The maximum
1. Lt 
x  0 sin(4x)
 is equal to [ME  2014]
11. The minimum value of the function  
7. A function f(x) = 1  x2 + x3 is defined
possible height of the arch is
1 (A) 0 (B) 0.5 (C) 1 (D) 2
f(x) = x(x 2  3) in the interval [CS, ME-2012] in the closed interval [1, 1]. The value
3
(A) 8 meters (B) 10 meters 2. The value of the integral of x, in the open interval (1, 1) for
100 ≤ x ≤ 100 occurs at
(C) 12 meters (D) 14 meters
2
(x  1)2 sin(x1) which the mean value theorem is
x = ________. [EC  2017] 0 (x1)2 cos(x1) dx is satisfied, is [EC  2015]
18. Consider the function f (x) = |x| in the
12. The maximum value of (A) 1/2 (B) 1/3 (C) 1/3 (D) 1/2
interval 1  x  1. At the point x = 0, (A) 3 (B) 0 [ME  2014]
f(x) = 2x3  9x2 + 12x  3 in the interval
f (x) is [ME  2012] (C) 1 (D) 2
8. Consider the function f(x) = 2x3  3x2
0  x  3 is _____. [EC  2014]
(A) Continuous and differentiable 3. The maximum value of the function in the domain [1, 2]. The global
13. Consider a spatial curve in three-
(B) Non-continuous and differentiable f(x) = In(1 + x)  x (where x > 1) minimum of f(x) is ______ [ME  2016]
dimensional space given in parametric
(C) Continuous and non-differentiable occurs at x = _________. [EC  2014]
form by [ME  2015] (D) Neither continuous nor 9. Given the following statements about a
x(t) = cos t, y(t) = sin t, differentiable 4. Let f(x) = x ex. The maximum value of function f:  , select the right
2  the function in the interval (0,∞) is option: [EC  2016]
z(t) = t, 0  t  . 19. The maximum value of
 2 (A) e1 (B) e [EE  2014]
f(x) = x3  9x2 + 24x + 5 in the interval P: If f(x) is continuous at x = x0, then
The length of the curve is __________
(C) 1 e1 (D) 1+ e1 it is also differentiable at x = x0.
2 [1, 6] is [EE  2012]
cos(1 x)
14.  dx = ______. [CS  2015] (A) 21 (B) 25 5. At x = 0, the function f(x) = |x| has Q: If f(x) is continuous at x = x0, then it
1 x2
(A) a minimum [ME  2015] may not be differentiable at x = x0.
(C) 41 (D) 46
15. The region specified by (B) a maximum R: If f(x) is differentiable at x = x0,

 (C) a point of inflexion then it is also continuous at x = x0.


 20. A parametric curve defined by
{(, , z): 3 ≤   5,  ,
8 4  u   u  (D) neither a maximum nor minimum (A) P is true, Q is false, R is false
x = cos   , y = sin   in the
3  z  4.5} in cylindrical coordinates  2   2  (B) P is false, Q is true, R is true
range 0 ≤ u ≤ 1 is rotated about the 6. While minimizing the function f(x), (C) P is false, Q is true, R is false
has volume of _______ [EC  2016]
X-axis by 360 degrees. Area of the necessary and sufficient conditions for (D) P is true, Q is false, R is true
16. The area between the parabola x = 8y 2
surface generated is [ME  2017]
a point, x0 to be a minima are:
and the straight line y = 8 is _______.  1
dx
(A) (B) 
2
[CE  2015] 10. The integral  (1  x)
is equal to ____
[CE  2016] (C) 2 (D) 4
0
(A) f(x0) > 0 and f(x0) = 0
[EC  2016]
 (B) f(x0) < 0 and f(x0) = 0

135 136

Assignment on Calculus Vidyalankar : GATE – Engineering Mathematics

Q 11 to Q 20 carry two marks each Assignment  3


17. At x = 0, the function f (x) = x3 + 1 has
11. The value of the integral is
(A) A maximum value [ME  2012]
2 x Duration : 45 Min. Max. Marks : 30
 e
xy
dydx is [ME  2014] (B) A minimum value
0 0
Q 1 to Q 10 carry one mark each 6. Given, i = 1 , the value of the
1 1 2 (C) A singularity
(A) (e  1) (B) (e  1)2 1. Consider a function f(x,y,z) given by 2
2 2 (D) A point of inflection cos xisin x
f(x,y,z) = (x2 + y2  2z2) (y2 + z2)
definite integral, I =  cos x  isinx
dx
1 2 1 1
2
 1  cos x  0

(C) (e  e) (D)  e  18. lim  2  is [ME  2012] The partial derivative of this function
2 2 e
x 0
 x  is: [CE  2015]
(A) 1/4 (B) 1/2 (C) 1 (D) 2 with respect to x at the point, x = 2,
12. The minimum value of the function (A) 1 (B) 1
y = 1 and z = 3 is _____. [EE  2017]
f(x) = x3  3x2  24x + 100 in the 19. Which one of the following functions is (C) i (D) i
continuous at x = 3? [CS  2013]
interval [3, 3] is [EE  2014] 2. For 0  t < ∞, the maximum value of
 7. The contour on the x-y plane, where
(A) 20 (B) 28 (C) 16 (D) 32  2, if x3 the function f(t) = et  2 e2t occurs at
 the partial derivative of x2 + y2 with
13. Consider an ant crawling along the (A) f(x) =  x1, if x3 (A) t = loge 4 (B) t = loge 2
x  3 respect to y is equal to the partial
curve (x  2)2 + y2 = 4, where x and y are  , if x3 (C) t = 0 (D) t = loge 8
 3 derivative of 6y + 4x with respect to x,
in meters. The ant starts at the point [EC  2014]
 4, if x3 is [EC  2015]
(4, 0) and moves counter-clockwise with (B) f(x) = 
8x if x3 (A) y = 2 (B) x = 2
3. If z = xy In (xy), then [EC  2014]
a speed of 1.57 meters per second. The
x  3, if x3
(C) f(x) =  z z (C) x + y = 4 (D) x  y = 0
time taken by the ant to reach the point  x  4, if x3 (A) x y =0
x y
(2, 2) is (in seconds) ____ [ME  2015] 1
(D) f(x) = , if x3 8. The values of x for which the function
x 3 27 z z
(B) y x
14. limx  x 2  x  1  x is [ME  2016] x y x 2  3x  4
20. Let I = c  xy 2 dxdy , where R is the f(x) is NOT continuous
(A) 0 (B)  (C) 1/2 (D) 
R
z z x 2  3x  4
region shown in the figure and (C) x y
15. The area of the region bounded by the x y are [ME  2016]
c = 6 × 104. The value of I equals (A) 4 and 1 (B) 4 and 1
parabola y = x2 + 1 and the straight z z
(D) y x =0
_______. (Give the answer up to two x y
line x + y = 3 is [CE  2016] (C) 4 and 1 (D) 4 and 1
decimal places) [EE  2017]
59 9 10 7  xsinx 
(A) (B) (C) (D)
6 2 3 6 4. lim 
x  equals to [CE  2014] 9. As x varies from −1 to + 3, which one
x 
 
16. Let f :[1, 1]   , of the following describes the
(A)  ∞ (B) 0
3 4 behaviour of the function
where f(x) = 2x  x  10. The minimum (C) 1 (D) ∞
f(x) = x3  3x2 + 1? [EC  2016]
value of f(x) is _____. [IN  2016]
  sin x 
5. The value of limx 0   (A) f(x) increases monotonically.
 2 sin xx cos x 
(B) f(x) increases, then decreases and
is ___________ [ME  2015]
increases again.


137 138
Assignment on Calculus Vidyalankar : GATE – Engineering Mathematics

(C) f(x) decreases, then increases and 15. The angle of intersection of the curves 19. The value of the definite integral 20. Consider the following definite integral:
decreases again. x2 = 4y and y2 = 4x at point (0, 0) is e 1
(sin1 x)2

1
x l n(x)dx is [ME  2013] I=  dx [CE  2017]
(D) f(x) increases and then decreases. [CE  2016] 0 1 x2
4 3 2 2 3 4
10. The maximum value attained by the (A) 0 (B) 30 (A) e  (B) e  The value of the integral is
9 9 9 9
function f(x) = x(x  1) (x  2) in the (C) 45 (D) 90 2 3
2 3 4 4 3 2 (A) (B)
(C) e  (D) e  24 12
interval [1, 2] is _____. [EE  2016] 9 9 9 9
16. The cost function for a product in a 3 3
Q 11 to Q 20 carry two marks each (C) (D)
firm is given by 5q2, where q is the 48 64
11. The Taylor series expansion of amount of production. The firm can

3 sin x + 2 cos x is [EC  2014] sell the product at a market price of
x3 Rs. 50 per unit. The number of units to
(A) 2 + 3x  x2  + .....
2 be produced by the firm such that the
2x3 profit is maximized is [CS, ME  2012]
(B) 2  3x + x  + .....
2 (A) 5 (B) 10
2x3 (C) 15 (D) 25
(C) 2 + 3x + x + + .....
2
x3 17. The area enclosed between the
(D) 2  3x  x2 + + .....
2 straight line y = x and the parabola
x  1 y = x2 in the x  y plane is [ME  2012]
12. The expression lim0  is equal
 (A) 1/6 (B) 1/4
to [CE  2014] (C) 1/3 (D) 1/2
(A) log x (B) 0
(C) x log x (D) ∞ 18. Consider the function f(x) = sin(x) in

13. The maximum area (in square units) of the interval x[/4, 7/4]. The number

a rectangle whose vertices lie on the and location (s) of the local minima of

ellipse x2 + 4y2 = 1 is _________. this function are [CS  2012]

[EC  2015] (A) One, at /2


(B) One, at 3/2
1
2 D
14. The integral (x  y  10)dxdy , (C) Two, at /2 and 3/2
2 2 (D) Two, at /4 and 3/2
where D denotes the disc: x + y  4,
evaluates to ________ [EC  2014]

139 140

Assignment on Calculus Vidyalankar : GATE – Engineering Mathematics

Assignment  4 9. First rule for maximum and minimum 12. f (x) = (x  2)3 is
values of y = f(x) are as follows
(A) maximum at x = 2
Duration : 45 Min. Max. Marks : 30
dy d2 y dy (B) minimum at x = 2
Get and 2 . Solve  0 and
Q1 to Q6 carry one mark each dx dx dx
 2
 (C) neither maximum nor minimum at
ex   1  x  x 
1  cos x  2 = consider its roots. These are the value
6. lim x=2
1. Lim equals to x 0 x3
x 0 x2 dy (D) none of these
of x which make = 0. For each of
(A) 1/2 (B) 1/6 dx
1 1
(A) (B)
3 2 (C) 1/3 (D) 1 these values of x, calculate the 13. The function sin x (1 + cos x) is

1 1 corresponding value of y, and examine maximum in the interval (0, ), when
(C) (D)
4 5 Q7 to Q18 carry two marks each d2 y  
the sign of . Now pick the correct (A) x = (B) x =
7. Which of the following is incorrect dx 2 4 2
sin 
2. What is lim equal to?
 0  statement ? statement from the following  2
(C) x = (D) x =
(A) A stationary value is always an alternatives. 3 3
(A)  (B) sin 
(C) 0 (D) 1 extreme value (A) If the sign is +, the corresponding
14. The absolute minimum of
(B) A maximum value is always a value of y is a maximum
x 2 1/ 3
f (x) = 4x3  8x2 + 1 in [ 1, 1] is at
3. The value of lim is stationary value (B) If the sign is , the corresponding
x  8 (x  8)
(A) 1 (B) 3
(C) A minimum value is always a value of y is a minimum
1 1 (C) 1 (D) 8
(A) (B) stationary value (C) If the sign is , the corresponding
16 12
(D) A minimum value may be greater value of y is a maximum 15. A spherical snowball is melting at rate
1 1
(C) (D) than maximum value (D) None of these
8 4 of 4 cubic centimeter per hour. How

10. Let f(x)  x 3  6x 2  24x  4. Then f(x) fast is the diameter changing when it is
ax  b 8. Which of the following is correct
4. Lim is
x  cx 20 centimeters ?
statement ? has
(A) 0 (B) a (A) 0.1 cm/hr (B) 0.02 cm/hr
(A) A stationary point on a graph is (A) a maximum value at x = 2
a a (C) 0.04 cm /hr (D) 0.01 cm/hr
dy (B) a maximum value at x = 5
(C) (D) any point at which =0
b c dx (C) a minimum value at x = 5 16. If a mothball evaporates at a rate
5. The discontinuity of the function (B) A stationary point on a graph is (D) neither maximum nor minimum at proportional to its surface area 4r2,
ex  1 dy any point then its radius
f(x) at the point x = 0 is known any point at which =
x dx 11. Let a number 30 be divided into two (A) decreases at constant rate
as (C) A stationary point on a graph is parts such that cube of one multiplied (B) decreases logarithmically
(A) first kind of discontinuity any point at which there are gaps with the other is maximum, then the (C) decreases exponentially
(B) second kind of discontinuity or breaks parts are (D) None of these
(C) mixed discontinuity (D) None of these (A) 12.4, 17.6 (B) 13, 17
(D) removable discontinuity (C) 10.5, 19.5 (D) 22.5, 7.5

141 142
Assignment on Calculus Vidyalankar : GATE – Engineering Mathematics

17. The absolute maximum of 18. The absolute minimum of Assignment  5


f(x) = x4  2x3  x2  4x + 3 on the x3
f(x) = on the interval [ 1, 1] is Duration : 45 Min. Max. Marks : 30
interval [0, 4] is. (x  2)
(A) 80 (B) 16 at Q1 to Q6 carry one mark each cot 
6. The value of L im is
(C) 28 (D) None of these (A) 0 (B) 1   (  / 2) ( / 2)  
1. L im ( x 2  1  x  1) equals to
x
(C) 1/3 (D) None of these (A) 1 (B) 2
(A) 0 (B) e
1
 (C) 1 (D)  (C) 0 (D)
2
2. It is given that Q7 to Q18 carry two marks each
ax  b 7. If y = 7x  x3 and x increase at rate of
f(x) = ,L im f (x)  2 and
x 1 x0
4 units/sec how fast is slope of graph
L im f (x)  1 , then value of f ( 2) is changing when x = 3 to
x

(A) 0 (B) 1 (A) 72 (B) 80

(C) e (D)  (C) 64 (D) 64

3. The limiting value of 8. An object moves on the parabola


2 2 2 2 3y = x2 when x = 3, the x coordinate of
1 2 3 n
   .............  3 as n  
n3 n3 n3 n the object is increasing at the rate of

is 1 foot/min. How fast is y coordinate


increasing of the moment
1
(A) 1 (B)
2 (A) 1 (B) 2

1 1 (C) 3 (D) 4
(C) (D)
3 4 9. A snowball is increasing in volume of
2n the rate of 10 cm3/h. How fast is the
 1
4. What is the value of lim  1   ?
n
 n surface area growing at the moment

(A) 0 (B) e2 where radius of snowball is 5 cm ?


(A) 4 (B) 2
(C) e1/2 (D) 1
(C) 12 (D) 16
1  cos 2x
5. The value of L im is
x  (  / 2) (  2x)2 10. The height h and radius r of cylinder of
greatest volume that can be cut from a
1 1
(A) (B) sphere of radius b is
4 3
1 2b 2
(C) (D) 0 (A) h = ,r  b
2 3 3

143 144

Assignment on Calculus Vidyalankar : GATE – Engineering Mathematics

b 2
/4
Assignment  6
(B) h =
3
,r= b
3
14.  (1  tanx) / (1  tanx) dx evaluates to
0
Duration : 45 Min. Max. Marks : 30
b b (A) 0 (B) 1
(C) h = ,r 
2 3 6 (C) In 2 (D) ½ In 2 Q1 to Q6 carry one mark each 4. The maximum value of
2b 1 15. The following plot shows a function y 1. The discontinuity of the function f (x) = 2 | x  1| + 3 | x  2 |
(D) h = ,r  b
3 3 |x| for all x  R is
which varies linearly with x. The value f (x) = , x0
x (A) 2 (B) 4
11. A pipe with length 3m and radius of 2
of the integral I   1
y dx is
= 0, x = 0 (C) 1 (D) None of these
10cm has an outer layer of ice that is y
at the origin is known as
melting at the rate of 2cm3 /min. How 3 5. Pick out the correct statement
(A) Discontinuity of 1st kind
fast is the thickness of ice decreasing 2 (A) logarithmic function is decreasing
(B) Discontinuity of 2nd kind
when ice is 2cm thick? wherever it is defined
1 (C) Mixed discontinuity
1 x (B) logarithmic function is increasing
(A) cm / min (D) None of these
200 1 1 2 3 whenever it is defined
1 (A) 1.0 (B) 2.5 2. The value of the coefficients ‘a’ such (C) logarithmic function is neither
(B) cm / min
800 (C) 4.0 (D) 5.0 that the function f defined as increasing nor decreasing
1 16. The value of the quantity P, where f (x) = ax 2  2 x  1 (D) None of these
(C) cm / min
1800 1
= 1 x  1
 xe dx, is equal to 6. f (x) = log sin x. This function is
x
1 P=
(D) cm / min 0 continuous is
3600  
(A) increasing on  0,  and
(A) 0 (B) 1 (A) a = 1 (B) a = 2  2
12. The maximum area of any rectangle (C) e (D) 1/e (C) a = 0 (D) a = 3
 
which may be inscribed in a circle of decreasing on  ,  
17. The sum of the infinite series, 2 
radius 1 is 3. If f (x) = 1, if x < 3
1 1 1  
(A) 1 (B) 2 1+    ....is = ax + b, if 3  x < 5 (B) increasing on  ,   and
2 3 4 2 
(C) 3 (D) 4 =7 if 5  x
(A)  (B) infinity  
Determine the value of ‘a’ and ‘b’ so decreasing on  0, 
13. A point on a curve is said to be an 2  2
(C) 4 (D) that f (x) is continuous.
extremum if it is a local minimum or a 4 (C) increasing on [0, 0] and
local maximum. The number of distinct   (A) a = 4, b =  7
 x2  y2 decreasing on [, ]
extrema for the curve
18. The value of the integral   e e (B) a = 3, b =  8
0 0 (D) increasing on [, 2] and
3x 4  16x 3  24x 2  37 is (C) a = 1, b =  9
dx dy is decreasing on [0, 0]
(D) a = 2, b = 3
(A) 0 (B) 1  
(A) (B)  (C)  (D)
(C) 2 (D) 3 2 4


145 146
Assignment on Calculus Vidyalankar : GATE – Engineering Mathematics

Q7 to Q18 carry two marks each 1 1 1 Assignment  7


13. The series 1    +……… is
1 1 1 3 9 27
7. The series 1     .....is Duration : 45 Min. Max. Marks : 30
2 22 23 (A) convergent (B) divergent
(A) convergent (B) divergent (C) oscillatory (D) None of these Q1 to Q6 carry one mark each 6. The maximum of f(x) = (x + 2)/(x  1) is
(C) oscillatory (D) None of these
14. The series whose nth term is 1. Function f (x) = e 1/ x ; x > 0 is a (A) 4 (B) 1

(A) increasing function (C) 2 (D) does not exist


8. The series 1 + 2 + 3 + …… is n 1 n
3 3
is
(A) convergent (B) divergent (A) convergent (B) divergent (B) decreasing function Q7 to Q18 carry two marks each
(C) oscillatory (D) None of these (C) oscillatory (D) None of these (C) neither increasing nor decreasing 7. Use Lagrange’s mean value theorem
(D) none of these
for f(x) = ex . Which of the following is
9. The series 1  2 + 3  4 + 5  6 + . . . . 1 th
15. The series whose n term is sin is
n 1 true?
is 2. f (x) = x  is
(A) convergent (B) divergent x (A) 1 + x < ex < 1 + xex
(A) convergent (B) divergent
(C) oscillatory (D) None of these (A) maximum at x = 1 (B) 1 + xex < ex < 1 + x
(C) oscillatory (D) None of these
(B) maximum at x = 2 (C) ex < 1 + x ex < 1 + x
16. The series
1 1 1 (C) maximum at x =  1 (D) none of these
10. The series 1     .......is 2 3 4 5 n1
2 3 4     2  ........is (D) maximum at x =  2
1 4 9 16 n 8. x1/X is decreasing function if
(A) convergent (B) divergent
(A) convergent (B) divergent 3. f (x) =  x 2 + 2x + 1 is (A) x < e (B) x > e
(C) oscillatory (D) None of these
(C) oscillatory (D) none of these (C) x = e (D) x > 1/e
(A) maximum at x = 1
1 2 3 17. The interval in which Lagrange’s (B) maximum at x = 2
11. The series   +…. 9. The Maclaurin series of ex is
1  21 1  2 2 1  23
theorem is applicable for function (C) maximum at x =  1 
x n 1 
xn
is
f(x) = 1/x is (D) maximum at x =  2
(A)  n  1 !
n 0
(B)  n!
n0
(A) convergent (B) divergent
(A) [3, 3] (B) [2, 2]
(C) oscillatory (D) None of these 4. The maximum value of 5 cos  + 3 cos

xn
(C) [ 2, 3] (D) [1, 1] (C) 
n 1 n!
(D) none of these
12. Which of the following to true?  
   3  + 3 is
18. For f(x) = x3 suppose   10. Taylor series for sin x about /4 is
1 1 1
(A) 1    is convergent f(b)  f(a) = (b  a) f (c) holds then c is (A) 5 (B) 10
x    
n
21/ 3 31/ 3 41/ 3 1 

1 2 ab (C) 11 (D)  1


(A) 
2 n 0 n!
1 1 1 (A) a  b2 (B)
(B) 1     is convergent 2 2
2 3 4
 1  x   / 4 
n n
3 2 
5. f (x) = x  12 x  45 x  11 is (B) 2
1 1 1 a 2  b2  ab a 2  b2  ab n!
(C) 1  2  2  2 is convergent (C) (D) n 0
2 3 4 3 3 (A) minimum at x = 5
1 1  x   / 4  x   / 4 
2
(B) minimum at x =  5 (C)   ...
1 1 1
(D) 1     is divergent 2  1! 2! 
2k 3k 4k (C) minimum at x = 4

 (D) minimum at x =  4 (D) none of these

147 148

Assignment on Calculus Vidyalankar : GATE – Engineering Mathematics

11. The Maclaurin series for ln (1  x) is x


Assignment  8
14. If f(x) = 2 n
xn , then f33(0) is
x 2 x 3 n 0
(A) x  ..... Duration : 45 Min. Max. Marks : 30
2 3 (A) (33!) 333 (B) (32!) 333
32 33
x2
x 3 (C) (3!) 2 (D) (33!) 2 Q1 to Q6 carry one mark each 6. Evaluate lim e  nlogn
(B) x   ..... x 0
2 3 1. For what value of k will f(x) = x  kx1
1 99 (A) 1 (B) 0
x 2 x 3 15. If f(x) = tan x, then f (0) is have a relative maximum of x = 2
(C) x   ..... (C)  (D) does not exist
2 3 (A) 97! (B) 98! (A) 2 (B) 1
(D) none of these (C) 99! (D) none of these (C) 4 (D) 8
Q7 to Q18 carry two marks each
12. The Taylor series for 1/x about 1 is 16. The n th
non zero term of Maclaurin 7. The first three nonzero terms of
 2. A wire 16 feet long is bent to form a
series for ln (1 + x2) is Taylors series for  n x around 2 is
  1  x  1
n 1 n
(A)
n0
rectangle. If the area of the rectangle
1
 1
n 1
 x  2
2n 2
x x 2n
(A)  n2 + (x  2 ) 
 1 is to be maximized, the dimensions of
n
 (A) (B) 8
  1  x  1
n n
(B) n n1
n0
the rectangle should be
1 1
(x - 2) -  x - 2 
2
x 2n x 2n1 (B) n2 +
(C)  1 (D)  1
n 1 n
 (A) 2, 6 (B) 5, 3 2 8
  1  x  1
n 1
(C)
n
n1 n
(C) 4, 4 (D) None of these
n 1 1 1
(x - 2) +  x - 2 
2
(C) n2 -
(D) none of these 1 2 4
17. If f(x) = 2 , then f36(0) is |x2|
x  x 1 3. lim is equal to 1
x2  x - 2
x 2 2
13. The first non zero terms of Maclaurin (D) n2 + (x - 2) -
(A) 36! (B) 36! 4
series for sec x is (A) 1 (B) 1
(C) 236 (D) none of these (C) 0 (D) does not exit
1 5x 4 8. The Maclaurin series for 2x is
(A) 1  x 2 
2 24
  n2    n2 
n n
18. The first three nonzero terms of f x  2
x x

(B) x 
x 2 54 3
 x 4. If lim  0 then lim f  x  is equal
(A)  n  1! x n
(B)  n!
x n 1
3 16 Maclaurin series for tan x are x 1 f x  2 x 1 n 0 n 0

x3 2 5 to x
  n2 
n
x 2 5x 4 4 (A) x   x  .....
(C) 1 
2

16
x 3 15 (C)  n!
xn (D) None of these
(A) 1 (B) 1 n 0

2 4 x2 4 3
x 5 4 (B) 1   x  ..... (C) 2 (D) 2
(D) x   x 2 15
2 24 2
9. If f(x) = ex then f100 (0) is
x2 2 5 5. Which of the following limit exist ?
(C) 1   x  ..... (A) 100! (B) 2100
2 15
1/ x 1
(D) none of these (A) lim e (B) lim 100!
x 0 x  0 1  e1/ x (C) (D) 100!  50!
50!
1 2
 (C) lim (D) lim1/ x
x 0 x x 0

149 150
Assignment on Calculus Vidyalankar : GATE – Engineering Mathematics

10. If f(x) satisfies f(0) = 2, /2


Assignment  9
14. Evaluate I = 0
sin2x log tanx dx
f   0   1,f (0)  4,f (0)12
(A) 0 (B) 1 Duration : 45 Min. Max. Marks : 30
andf n (0)  0 for n > 3 (C) /2 (D) /2 log 2 Q1 to Q6 carry one mark each 6. The limiting value of
then f(x) is 2x
1 2 n
 x    .......  as n   is
1. lim   is
(A) x + 2x3 15. The area bounded by curve x2 = 4y x  1  x
  n n n
(B) 2 + 2x2 and straight line x = 4y  2 is (A) 2/3 (B) 1/2
1
(A) e (B)
(C) 2 + x + x2 (A) 1/8 (B) 3/8 e2 (C) 1/3 (D) does not exist
(D) 2 + x + 2x2+ 2x3 (C) 9/8 (D) 2 1 Q7 to Q18 carry two marks each
(C) (D) 2e
e /2

11. The Taylor series for f(x) = 2x +4x  3 2


16. The area bounded by x axis, tanx
7. I =  0
cos4 x sin3 dx
 1
about 1 is 2. lim   is equal to 1 2
x 0 x
8   (A) (B)
y = 1+ ordinate at x = 2 and x = 4 is 35 35
(A) 3 + 8 (x 1) + 2 (x  1) 2 x2 (A) 0 (B) 1
(B) 2 + 4x + 2x2 (A) 2 (B) 4 3 4
(C) 1 (D) 0 (C) (D)
35 35
(C) 2 + 4 (x  1) + 4 (x  1)2 (C) 8 (D) 1
lim  cos ecx  is equal to
x
3.
(D) None of these x 0 1 x2
8. Evaluate I =  1 x2
dx
17. The area bounded by curve (A) 1 (B) e
 sin2kx y = 2x + x2  x3 the x axis and lines (C) r1 (D) 0 3 1 1
12. Evaluate I = 0 sinx
dx (A)
2
sin x  x 1  x 2  C
2
x = 1 and x = 1 1 2 3 1
4. lim  2  2  ......  
(A) 0 (B) 1/2 (A) 1/2 (B) 1 n  n2 n n n 1
 (B) x 1 x2  C
(C) 1 (D) /2 2
(C) 3/2 (D) 2 (A) 0 (B) 1
(C) sin 1 + 1 x2  C
(C) ½ (D) 

log 1  cos   d x  sin x (D) none of these
13. Evaluate I = 0 18. Evaluate I =  1  cos x dx  ax 2  b x2
 dx
(A)

log2 (B) log2 x x
5. The function f(x) =  2

x2 9. Evaluate I =  cos x  sinx
2 (A) x  cot  C (B)  x cot  C 2ax  b x2
2 2 x 
(C) log2 (D) /4 log 2 is continuous at x = 2 if (A) log tan     C
x x 2 8
(C) x cot  C (D) x + tan  C (A) a = 0, b = 1/2
2 2 x
(B) a = 1/2 , b = 0 (B) log cot C
2
 (C) a = 1/2, b = 0 (C) log cot x + C
(D) b = 1/2, a = 0 (D) log tan x + C

151 152

Assignment on Calculus Vidyalankar : GATE – Engineering Mathematics

10. Evaluate I =  1  3 sin


dx
2
14. The value of lim 
 nsinx 
 , where [.] is
Test Paper  1
x x 0
 x 
1 Duration : 30 Min. Max. Marks : 25
(A) tan1  tan x   C the greatest integer function
2
(A) n + 1 (B) n Q1 to Q5 carry one mark each Q6 to Q15 carry two marks each
(B) tan1 (tan x) + C
(C) n  1 (D) None of these sin3x  sinx 
1
(C)
1
tan1  2 tanx   C
15. Let f   xy,  yz,z  and
1. Lim
x 0 sinx
is equal to 6. x 2
dx
2 1

(A) 0 (B) 2 (A) Converges


(D) None of these

C : sin  2t  , t, t 2 .  (C) 1 (D) 3
d (B) Diverges
11. Evaluate I =  sin 4
  cos4 
Then the value of the line integral (C) Converges and diverges

1  tan2   1
1
 fd r from (0, 1, 1) to (0, 1, 1) is
2. Lim sin
1
is
(D) None of these
(A) tan   C x 0 x
2  2 tan  
(A) 1 (B) 1 (A)  (B) 0 
dx
 tan2   1 
(B) tan1  
(C) 0 (D) None of these (C) 3 (D) does not exist 7. 0 ex  e x is equal to
 tan   

e
-x
16. If IN = xn-1logx dx , then (A) /2 (B) /4
1  1  sin3x
(C) tan1  
0
3. Lim is equal to (C) /3 (D) 
2  2 tan   2 x 0 5x
(A) In+2 + (2n + 1) In+1 + n In = 0
(D) None of these 3 5
(B) In+2  (2n+1) In+1  n2 In = 0 (A) (B) 
5 3 dx
12. I = 
sin x
dx (C) In+2 + (2n + 1) In+1  n In = 0 2 8.  x n x 2
is equal to
sin3x 1 1 2
(D) In+2  (2n+1) In+1 + n2 In = 0 (C) (D)
 3  tan x  5 3 1 1
(A) log  C (A) (B)
 3  tan x  17. The function f(x) = 1  1  x 2 is  n4  n3
 
1 3  tanx 4. If [x] denotes the greatest integer not 2 1
(A) not continuous of x = 0 (C) (D)
(B) log C  n2  n2
2 3 3  tanx greater than x, then Lim[x] is
(B) is continuous but not differentiable x 2

1
 
5
(C) log 3  tan x  C at x = 0 (A) 0 (B) 1 dx
3 (C) differentiable at x = 0
9.  3
x2
is equal to
(C) 2 (D) does not exist 1

1  3  (D) None of these


(D) log  C
 3  tanx  9 1
 
3
3
2 3   (A) (B) 3
9 1
18. The function (1 x)n  1 2 2
5. Lim is equal to
 
/2
13. I = 0
x 2 sinx dx z = 5xy 4x2 + y2  2x  y + 5 has at x 0 x (C) 3 3
9 1 (D) None of these
1 18 (A) n (B) n  1
2 x= ,y=
(A) (B) 2  2 41 41
2 (C) 0 (D) does not exist
(A) maximum (B) saddle point
(C)  + 2 (D)   2
(C) minimum (D) None of these


153 154
Test Paper on Calculus Vidyalankar : GATE – Engineering Mathematics

10. The area under curve y =


x
for
13. The area between the curves y = 1/x Test Paper  2
1 x2 1
and y = to right of line x = 1 is
x 1 Duration : 30 Min. Max. Marks : 25
0  x  1 is
(A) 1/4 (B) 1/2 (A) n 3 (B) n 2 Q1 to Q5 carry one mark each (B) Both A and R are true and R is not
(C) 1 (D) 16 (C) 2 n 3 (D) 2 n 2 the correct explanation of A
1. Let f (x) = 2x if x < 2
=2 if x = 2 (C) A is true but R is false
3
11. The volume of solid obtained by dx (D) A is false but R is true
14. x2 = x2 if x > 2
2x 1
revolving area under y = e about the discontinuity of f (x) at x = 2 is
(A) converges 4. If a function ‘f’ is defined on R by
x axis is known as
(B) diverges f (x) = 1 for x a rational number
(A) /2 (B) /4 (A) first kind of discontinuity
(C) both =  1 for x an irrational number,
(C) 2 (D)  (B) second kind of discontinuity
(D) neither of the above then
(C) mixed discontinuity
9 (D) removable discontinuity at x = 2 (A) f (x) is continuous at x = 1
dx
12.   x  1 2/3
is equal to 
dx (B) f (x) is continuous at x =  1
0 15.    nx  p
is _________ for p  1
e
2. If f (x) = x  1 1x2 (C) f (x) is continuous at x = 0
(A) 3 (B) 6
(A) convergent = 2x  3, 2x3 (D) f (x) is not continuous at any point
(C) 7 (D) 9
(B) divergent Tick the following alternative which is
(C) convergent as well as divergent appropriate for the above function. 5. If f (x) = 4x + 3, x4
(D) None of these (A) continuous at x = 1 = 3x + 7, x=4

(B) continuous at x = 2 the function ‘f’ is


 (C) discontinuous at x = 1 (A) continuous at x = 4

(D) discontinuous at x = 2 (B) discontinuous at x = 4


(C) continuous at x =  4
3. Consider the following statement (D) discontinuous at x =  4
Assertion (A) :
The function f (x) = x  [x], x  z is Q6 to Q15 carry two marks each
discontinuous at x = 1 /2

Reason (R) :
6. I = 0 cot xdx is

Left L im f (x)  Right L im f (x)


x 1 x 1 (A) 2 (B) /4
Of these statements (C)  (D) None of these
(A) Both A and R are true and R is the
correct explanation of A

155 156

Test Paper on Calculus Vidyalankar : GATE – Engineering Mathematics

7. The area in first quadrant under curve 11. The minimum value of x  5x  2 is 2
Test Paper  3
1
y= is (A) 5 (B) 0
x 2
 6x  10  (C) 1 (D) 2
Duration : 30 Min. Max. Marks : 25

 Q1 to Q5 carry one mark each 5. The function


(A) /2 (B)  2 tan1 3
4 (x16  1) 1
12. The maximum value of 1. The Lim 5 is f (x) = x 2 sin , if x  0
  x 1 (x  1) x
(C)  tan1 3 (D)  tan1 3 sinx  cos x
2 2 f(x)  is = 0, if x = 0 is
2 5 2
(A) (B)
16 17 (A) differentiable at x = 0 and f ‘(0) = 0
(A) 1 (B) 2
1 16 17 (B) not differentiable at x = 0 since
8. The area under y = (C) (D)
x 2
a 2
 (C) 1/ 2 (D) 3 5 2 1
 , cos x  0
x
for x  a + 1 is
13. The maximum value of 1/xx is x2  4 (C) differentiable at x = 0 and the
1 1 2. L im equal to
(A)  n  a  1 (B)  n  a  1 (A) e (B) ee
x2
x  2  3x  2 derivative is continuous at x = 0
a 2a
(C) e1/e (D) e1/e (A) 8 (B)  8 (D) not differentiable at any x since it
1
(C)  n  a  1 (D) None of these (C) 4 (D)  4 is not continuous for any x
2
14. The function f(c) = x5  5x4+5x3  1
has a x  bx Q6 to Q15 carry two marks each
9. For what values of k, with k  1 and 3. L im equals to
x0 x 6. The maximum point on curve x = exy is
1 (A) 1 minima and 2 maxima
1
k > 0 does 0 xk dx converges ? (B) 2 minima and 1 maxima
a
(A) log  
b
(B) log   (A) (1, e) (B) (1, 1/e)
b a
(C) 2 minima and 2 maxima (C) (e, 1) (D) (1/e, 1)
(A) 2 (B) 1
x b
(D) 1 minima and 1 maxima (C) log   (D) log  
(C) 3 (D) k > 1 a x 7. If f(x, y) is such that fx = ex cos y and
fy = ex sin y then which of following is
15. The greatest and least value of |x|
a
dx 4. The function f (x) = at x = 0 has true
10. 0 a2  x 2 where a > 0 is x 3 3x 2
  2x in  0,2  are
x
3 2 (A) f(x, y) = ex + ysin (x + y)
(A) discontinuity of second kind
1 (B) f(x, y) = ex sin
(A) n a (B)   na  1 5 2 11 1 (B) discontinuity of first kind
2 (A) and (B) and
6 3 6 6 (C) f(x, y) does not exist
(C) mixed discontinuity
1
(C)  na  1 (D) None of these
(C)
8
and
2
(D)
13
and
1 (D) continuous
(D) None of these
2
3 3 3 3
8. If f(x, y) = ln (x2 + y2) then

(A) fxx + fyy = 0 (B) fxx  fyy = 0
(C) fxx +2fyy = 0 (D) 2fxx + fyyt = 0

157 158
Test Paper on Calculus Vidyalankar : GATE – Engineering Mathematics

4 2
9. If z = x + 3xy  y and y = sin x then z z Test Paper  4
(C) x y
dz x y
is
dx z z Duration : 30 Min. Max. Marks : 25
(D)  z
3
(A) (4x + 3 sin x) + (3x  2 sin x) cos x x y
Q1 to Q5 carry one mark each 5. Which of following is false ?
(B) (2x2 3 sin x) + (3x  2 cos x)sin x f 1. Maxima and minima of a continuous (A) If f (a) = 0 and f (a) >0 then f (a)
13. Find the general solution of =1
(C) (4x3  3 sin x)  (3x + 2 sin x) cos x xy function occur is minimum value
(D) none of these is (A) simultaneously (B) If f (a) = 0 and f(a)  0 then f(a)
(A) f(x, y) = A (x) + B(y) + xy (B) once is extreme value
10. A point P is moving along curve of (B) f(x, y) = yA (x) + xB(y) (C) alternately (C) a maximum value is greater than
(C) f(x, y) = A (x) + B(y) (D) rarely greatest values
x2 y2
intersection of paraboloid  z
16 9 (D) none of these (D) f(a) is minimum value if f(a) is least
2. Let f(x) = | x | than
and cylinder x2 + y2 = 5. If x is value of f(x) in immediate
(A) f (0) = 0
increasing at the rate of 5cm/s how 14. The equation of tangent plane sphere neighborhood of point [a, f(a)]
(B) f(x) is maximum at x = 0
fast is z changing when x = 2m and phase x2 + y2 + z2 = 1 at point
(C) f(x) is minimum of x = 0 Q6 to Q15 carry two marks each
y = 1 cm ? 1 1 1  6. The area included between curves
 , ,  is (D) none of these
25 125 2 2 2 
(A) (B) y = 3x2  x  3 and y = 2x2 + 4x +7 is
36 36 (A) x + y + z = 2 3. Find the maximum speed with which a
(A) 10 (B) 45/2
144 18 snail moves if the equation of distance
(C) (D) (B) x + y + 2z=2 (C) 15 (D) 30
25 25 covered (s) in terms of time (t) is
(C) 2x  y + 32 = 5 7. The area included between curve
2
t
(D) 2x + y = 2 s = 3t 3   15 units ? r = a (sec  + cos ) and its asymptote
2
2 f
11. The general solution of = 0 is r = a sec  is
x 2 1
15. The altitude h of right circular cone is (A) speed units
9 5a 2
(A) f(x, y) = x g(y) + h (y) (A) a2 (B)
decreasing at rate of 3 mm/s while 4
(B) f(x, y) = y g(x) + h (x) 1
radius is increasing at 2mm/s. How (B) speed units
18 2a 2
(C) f(x, y) = x g(y) + h (x) (C) (D) 2a2
fast is volume v changing when 3
(D) f(x, y) = g(x) + h (y) 1
altitude and radius are 100mm and (C) speed units
27 8. The mass distributed over the area
50mm respectively ? (D) None of these bounded by curve 16y2 = x3 and line
y/x
12. If z = xe then
2200 2500 2y = x assuming that density of a point
(A) (B)
z z 3 3 f(x)
(A) x y z 4. The function g (x) = , x  0 has an of area varies as distance of point from
x y x
12500 1600 x axis. Take k as constant of
(C) (D) extreme value then
z z 3 3
(B) x y z proportionality
y z (A) f (x) = f (x) (B) g (x) = f(x)
(A) k (B) 1/3k
 (C) f(x) = 0 (D) g(x) = f (x)
(C) 2/3k (D) 2k

159 160

Test Paper on Calculus Vidyalankar : GATE – Engineering Mathematics

9. The density at any point in circular  2


(C) a 3 log2   Test Paper  5
lamina of radius a varies as its  3
distance from a fixed point on its (D) a3 Duration : 30 Min. Max. Marks : 25
circumference. Take k as proportional Q1 to Q5 carry one mark each 5. If z is defined as function of x and y by
constant, the mass of lamina is 13. The volume of solid formed by 1 z
1. The minimum value of x + , x > 0 xy  yz + xz = 0 then is
1 3 revolution of the curve (a  x)y2 = a2x x x
(A) ka 2 (B) ka
9 about its asymptote (A) 1 (B) 3/2 yz yz
(A) (B)
8 3 32 3 2 a 3 (C) 5 (D) 2 yx yx
(C) ka (D) ka (A) (B)  a 2 3
9 9 2 yx xz
2. The triangle of maximum area (C) (D)
3 a 2 3
2 3
yz xy
(C) (D) 4 a
10. The volume of sphere x2 + y2 + z2 = a2 2 inscribed in a circle of radius r is
cut off by plane z = 0 and cylinder (A) a right angled  with hypotenuse 2r Q6 to Q15 carry two marks each
x2 + y2 = ax is (B) an equilateral  6. If f(x) = ex and g (x) = ex then value of
14. The surface of reel formed by
a 3
2a 3
(C) an isoceles  of height r c in interval [a, b] by Cauchy’s mean
(A) (B) revolution of cycloid x = a ( + sin ),
3 3 (D) does not exist value theorem
y = a(1 cos ) round a tangent at its
(C) a3 (D) 2a3 vertex is
ab
(A) (B) ab
log x 2
a 2
2a 2
3. The maximum value of in (0,  )
11. The volume cut off from paraboloid (A) (B) x 2ab
3 3 (C) (D) none of these
1 2 is ab
x2 + y  z  1 by plane z = 0 is 2 2
4 16a 32a
(C) (D) (A) e (B) 1/e
3 3 7. If f and F be both continuous in [a, b]
  (C) 1 (D) none of these
(A) (B)
4 2 and are derivable in (a, b) and if

3 15. The volume generated by revolving f (x) = F (x) for all x in [a, b] then f(x)
(C) (D)  4. A manufacturer sells ‘x’ items at a
4 area below the xaxis between curve and F (x) differ
 x 
price of Rs.  3  per item and it
12. The volume of solid obtained by y8  1000  (A) by 1 in [a, b]
 x  2 and x axis about line
2 2 x (B) by x in [a, b]
revolution of loop of curve y = x x 
x + 5 = 0 is costs Rs.   200  to produce them. (C) by constant in [a, b]
ax 2 
about x axis is (A) 18 (B) 32 (D) none of these
ax Find the production level for maximum
(C) 432 (D) 216 profit. 8. If f(x) = 4x2 then the value of c in [1, 3]
3
(A) 2a
(A) 1250
 2 f  3   f  1
(B) 2a 3 log2   for which f (c) = is
 3 (B) 1500
4
(C) 1750
(A) 0 (B) 1
 (D) Data insufficient
(C) 2 (D) 3

161 162
Test Paper on Calculus Vidyalankar : GATE – Engineering Mathematics

a 1 cos   a 1 cos  


9. I =

 
0 0
r 4 cos drd 13. I =

 
0 0
2r 4 sin3 dr d Test Paper  6

(A)
5
a 5 (B)
21 5
a
2 24 Duration : 30 Min. Max. Marks : 25
(A) a 5 (B) a 5
16 16 35 35
Q1 to Q5 carry one mark each Q6 to Q15 carry two marks each
3 1 4 26 a 5 28 a 5
(C) a 5 (D) a (C) (D)
R y dA
2
16 4 35 35 x 2 2 6. If I = where R is region
1. lim is
x 8 x8
y
4 2 bounded by y = 2x, y = 5x and x = 1.
10. I = 2 y2 / 4 x dx dy a a  a2  y 2 xy log  x  a  0 1
  (A) (B)
2
14. I = dx dy Then I is equal to
0 0
x  a
2 0 2
24 36 39 39
(A) (B) 1 1 (A) (B)
5 5 a2 a2 (C) (D) 2 4
(A) log a (B) log a  1 4 2 2
72 108 2 8
(C) (D) 39 39
5 5 (C) (D)
a2 x  sin x 8 16
(C) 8a log a (D) 2log a  1 2. lim is
b b 8 x 0 x3
11. I =   y  2b  y  dx dy 1
0 0
(A) 1/6 (B) 1/3 7. The value of I = R 2y  y 2
dA where
a 2a  x
b4 2b4
(A) (B) 15. I =  
0 x /a2
xy dx dy (C) 1/3 (D) none of these
3 3 R is region in the first quadrant
3 4 5 4
4b 4 (A) a (B) a 1 bounded by x2 = 4  2y is
(C) b 4
(D) 8 8 3. lim  x  a  cos is
3 x a xa (A) 0 (B) 2
7 4
/4 2 sin  3 (C) a (D) a4 (A) 0 (B) a (C) 4 (D) 1
12. I = 2  
0 0
r dr d 8
(C) cos a (D) does not exist
3 3 8. The volume V under plane z = 3x + 4y
(A) (B) 2
4 4 1  cos x and over the rectangle
4. The function f(x) = (x  0) be
3 5 x2 R : 1  x  2, 0  y  3 is
(C) 2 (D)
4 2 made continuous at x = 0 by defining 49
(A) 25 (B)
f(0) to be 2
 (A) 1 (B) 1/2 63
(C) (D) 16
(C) 0 (D) e 2

5. The differentiability of a function 9. The volume V in first octant bounded


continuity is by z = y2, x = 2 and y = 4 is
(A) sufficient 64 128
(A) (B)
(B) necessary 3 3

(C) sufficient and necessary 16 32


(C) (D)
(D) none of these 3 3

163 164

Test Paper on Calculus

10. The area of region R bounded by


13. I =
/4
 / 4 d0
a cos 2  r dr
Chapter - 3 : Probability and Statistics
xy = 1 and 2x + y = 3 is a2  r 2
(A) n 2 (B) 2 + n 2     3.1 Basic Terms
(A) 2a 1   (B) 2a 2  
3  4  4
Random Experiment
(C)   n2 (D) None of these
4
    Consider an action which is repeated under essentially identical conditions. If it results in
(C) 2a 1   (D) 2a  2  
 4  4
any one of the several possible outcomes, but it is not possible to predict which outcome
11. The volume in first octant bounded by
will appear. Such an action is called as a Random Experiment. One performance of such
2x + 2y  z + 1 = 0 y = x and x = 2 is 1 x 2x y

(A) 10 (B) 28
14. I = 0 x 0
2
dx dy dz an experiment is called as a Trial.

(C) 12 (D) 16 11 13 Sample Space


(A) (B)
30 30 The set of all possible outcomes of a random experiment is called as the sample space.
1 All the elements of the sample space together are called as ‘exhaustive cases’. The
12. The area of region enclosed by (C) (D) 16
8 number of elements of the sample space i.e. the number of exhaustive cases is denoted
cardioide r = 1 + cos  is
by n(S) or N or n.
 3
1 y 1 x 
(A) (B) 15. I =
2 1 2 2
 y dx dy Event
2 2
(C)  (D) 2 Any subset of the sample space is called as an ‘Event’ and is denoted by some capital
1 1
(A) (B) letter like A, B, C or A1, A2, A3,.. or B1, B2, ... etc.
3 3
2 2 Favourable cases
(C) (D)
3 3 The cases which ensure the happening of an event A, are called as the cases favourable
to the event A. The number of cases favourable to event A is denoted by n(A) or NA or nA.

Mutually Exclusive Events or Disjoint Events
Two events A and B are said to be mutually exclusive or disjoint if A  B = 
i.e. if there is no element common to A & B.

Equally Likely Cases


Cases are said to be equally likely if they all have the same chance of occurrence i.e. no
case is preferred to any other case.

Permutation
A permutation is an arrangement of all or part of a set of objects. The number of
permutations of n distinct objects taken r at a time is

n n!
Pr =
n  r  !

165 166
Notes on Probability and Statistics Vidyalankar : GATE – Engineering Mathematics

(ii) Let B: number on the die is a prime number


Note: The number of permutations of n distinct objects is n! i.e., nPn = n!
The number of permutations of n distinct objects arranged in a circle  B = {2, 3, 5}  B = {1, 4, 6}

is (n  1)!
Note: P(A) + P( A ) = 1 i.e. P(A) = 1  P( A )
The number of distinct permutations of n things of which n1 are of one
n!
For any events A and B, P  A  = P  A  B  + P A  B  
kind, n2 of a second kind … nk of a kth kind is
n1! n2! ......nk!
3.4 Independent Events
Combination Two events A & B are said to be independent if
A combination is selection of all or part of a set of objects. The number of combinations of
P(A  B) = P(A).P(B)
n distinct objects taken r at a time is

n n!
Cr = Note: If A & B are independent then
r!  n  r  !
A & B are independent
Note: In a permutation, the order of arrangement of the objects is important.
A & B are independent
Thus abc is a different permutation from bca.
A & B are independent
In a combination, the order in which objects are selected does not matter.
Thus abc and bca are the same combination. 3.5 Theorems of Probability
3.2 Definition of Probability Addition Theorem
Consider a random experiment which results in a sample space containing n(S) cases If A and B are any two events then

which are exhaustive, mutually exclusive and equally likely. Suppose, out of n(S) cases, P(AB) = P(A) + P(B) – P(AB)
n(A) cases are favourable to an event A. Then the probability of event A is denoted by
P(A) and is defined as follows. Note: 1. A  B : either A or B or both i.e. at least one of A & B

P(A) =
n(A)
=
number of cases favourable to event A A  B : neither A nor B i.e. none of A & B
n(S) number of cases in the sample space S
AB & A  B are complement to each other

3.3 Complement of an event  P( A  B ) = 1 – P(A  B)

The complement of an event A is denoted by A and it contains all the elements of the 2. If A & B are mutually exclusive, P(A  B) = 0

sample space which do not belong to A. P(A  B) = P(A) + P(B)

For example: Random experiment: an unbiased die is rolled. 3. P  A1  A2  A3  = P  A1  + P  A2  + P  A3 P  A1  A2 


S = {1, 2, 3, 4, 5, 6} P  A2  A3   P  A3  A1  + P  A1  A2  A3 
(i) Let A: number on the die is a perfect square

 A = {1, 4}  A = {2, 3, 5, 6}

167 168

Notes on Probability and Statistics Vidyalankar : GATE – Engineering Mathematics

Multiplication Theorem Solved Example 3 : 2 1 2 2 7


P  A  B     
A die is loaded in such a way that an even 9 9 9 9 9
If A & B are any two events then
number is twice as likely to occur as an 2
P(A  B) = P(A).P(B/A) = P(B).P(A/B) and P  A  B 
odd number. If E is the event that a 9

1. Conditional probability of occurrence of event B given that event A has already number less than 4 occurs on a single toss
Solved Example 5 :
occurred. of the die. Find P(E).
A mixture of candies 6 mints, 4 toffees and
Solution :
P A  B 3 chocolates. If a person makes a random
P(B/A) = S = {1, 2, 3, 4, 5, 6 }
P A selection of one of these candies, find the
We assign a probability of w to each odd
2. Conditional probability of occurrence of event A given that event B has already probability of getting (a) a mint, or
number and a probability of 2w to each
occurred (b) a toffee or a chocolate.
even number. Since the sum of the
Solution :
P A  B probabilities must be 1, we have 9w = 1 or
P(A/B) = (a) Since 6 of the 13 candies are mints,
P B  w = 1/9.
the probability of event M, selecting
Bayes’ Theorem Hence probabilities of 1/9 and 2/9 are mint at random, is
assigned to each odd and even number 6
Suppose that a sample space S is a union of mutually disjoint events B1, B2, B3, ..., Bn, P(M) =
respectively. 13
suppose A is an event in S, and suppose A and all the Bi’s have nonzero probabilities. If
k is an integer with 1 ≤ k ≤ n, then  E = {1, 2, 3}
1 2 1 4 (b) Since 7 of the 13 candies are toffees
and P(E) =   
P  A / Bk  P Bk  9 9 9 9 or chocolates it follows that
P(Bk / A) =
P  A / B1  P B1  + P  A / B 2  P B 2  + ... + P  A / Bn  P Bn  7
P  T  C 
Solved Example 4 : 13

Solved Example 1 : Solved Example 2 : In the above example let A be the event
that an even number turns up and let B be Solved Example 6 :
A single die is tossed. Find the probability A coin is tossed twice. What is the
the event that a number divisible by 3 In a poker hand consisting of 5 cards, find
of a 2 or 5 turning up. probability that at least one head occurs ?
occurs. Find P(A  B) and P(A  B). the probability of holding 2 aces and
Solution : Solution :
Solution : 3 jacks.
The sample space is S = {1, 2, 3, 4, 5, 6} The sample space is
A = {2, 4, 6} and Solution :
1
P(1) = P(2) = … = P(6) = S = {HH, HT, TH, TT} The number of ways of being dealt 2 aces
6 B = {3, 6}
from 4 is
The event that either 2 or 5 turns up in Probability of each outcomes = 1/4 We have,
4 4!
C2  6
indicated by 2  5. Thus A  B = {2, 3, 4, 6} and 2!2!
Probability of atleast one head occurring is The number of ways of being dealt 3 jacks
1 1 1 A  B = {6}
P 2  5  P 2  P 5    1 1 1 3 from 4 is
6 6 3 P(A) =    By assigning a probability of 1/9 to each
4 4 4 4 4 4!
C3  4
odd number and 2/9 to each even number 3!1!

169 170
Notes on Probability and Statistics Vidyalankar : GATE – Engineering Mathematics

 Total number of ways n = 6.4 = 24 points. Since all sample points are equally Solution : Solution :
hands with 2 aces and 3 jacks. likely, we have P(A) = 1/6 and P(B) = 1/18. Let E be the event that at least 5 cars are Suppose there are 100 flowers

The total number of 5card poker hands all The events A and B are mutually exclusive serviced. Now Number of roses = 40; Number of

of which are equally likely, is since a total of 7 and 11 cannot both occur P(E) = 1  P(E), where E is the event carnations = 60

52! on the same toss. that fewer than 5 cars are serviced. 25% of 40 = 10 roses are red and
N= 52
C5  = 2, 598, 960
5!47!  P  A  B   P  A   P B  Since P(E) = 0.12 + 0.19 = 0.31 10% of 60 = 6 carnations are red
 The probability of event C of getting 2  P(E) = 1  0.31 = 0.69 Let A be the event that the flower is red
1 1 2
aces and 3 jacks in a 5card poker =  = and B the event that the flower is a rose.
6 18 9
hand is Solved Example 11 :  A  B is the event that the flower is a
Solved Example 9 :
24 In a garden 40% of the flowers are roses red rose.
P(C) =  0.9  105 If the probabilities are respectively 0.09,
2,598,960 and the rest are carnations. If 25% of the 16
0.15, 0.21 and 0.23 that a person n(A) = 16  P(A) =
roses and 10% of the carnations are red, 100
Solved Example 7 : purchasing a new automobile will choose
the probability that a red flower selected at 10
The probability that Paula passes the colour green, white, red or blue. What n(A  B) = 10  P (A  B) =
random is a rose. 100
mathematics is 2/3 and the probability that is the probability that a given buyer will
(A) 5/6 (B) 1/4 P(B/A) = probability that a selected flower
she passes English is 4/9. If the purchase a new automobile that comes in
(C) 4/5 (D) 5/8 is a rose red is colour
probability of passing both courses is 1/4, one of those colours ?
P  A  B 10 / 100 5
what is the probability that Paula will pass Solution : P(B/A) =  
PA 16 / 100 8
at least one of these courses ? Let G, W, R and B be the events that a buyer
Solution : selects, respectively, a green, white, red or 3.6 Random Variables
If M is the event “passing Mathematics” blue automobile. Since these four events are A random variable is a function that associates a real number with each element in the
and E the event “passing English” then mutually exclusive the probability is sample space. It is a set of possible values from a random experiment.
P G  W  R  B For example: Suppose that a coin is tossed twice so that the sample space is S = {HH,
P(M  E) = P  M   P E   P  M  E 
 P  G   P  W   P R   P  B  HT, TH, TT}. Let X represent the number of heads that can come up. With each sample
2 4 1 31  0.09  0.15  0.21  0.23 = 0.68
=    point we can associate a number for X as shown in the table.
3 9 4 36
Solved Example 10 :
Solved Example 8 : SAMPLE POINT HH HT TH TT
If the probabilities that an automobile
What is the probability of getting a total of X 2 1 1 0
mechanic will service 3, 4, 5, 6, 7 or 8
7 or 11 when a pair of dice are tossed ?
more cars on any given workday are
Solution : Discrete Random Variable
respectively 0.12, 0.19, 0.28, 0.24, 0.10
Let A be the event that 7 occurs and B the A random variable is discrete if it can take only discrete or separate values. Value of a
and 0.07. What is the probability that he
event that 11 comes up. Now a total of 7 discrete random variable is often obtained by counting.
will service at least 5 cars on next day at
occurs for 6 of the 36 sample points and a For example: Number of heads appearing when three coins are tossed.
work?
total of 11 occurs for only 2 of the sample

171 172

Notes on Probability and Statistics Vidyalankar : GATE – Engineering Mathematics

Continuous Random Variable Variance of random variable is given by

A random variable is continuous if can take any value in a given range. Value of a Var(X) = E((X  )2) = E(X2)  E(X)2 = E(X2)  2
continuous random variable is often obtained by measuring.
Standard deviation is square root of variance
For example: Time required to travel certain distance.

= Var(X) 
3.7 Probability Distribution Function
In general sense, probability distribution function may refer to: Note: 1. Expected value  = E(X) is a measure of central tendency.
 Probability mass function 2. Standard deviation  is a measure of spread.
 Probability density function
 Cumulative distribution function Solved Example 12 : 4, 5, 6, 7, 8, 9, 10, 10, 10, 10 each having
Thirteen cards are drawn simultaneously 1
Probability mass function the same probability so that
from a deck of 52 cards. If aces count 1, 13
It is a function that gives probability that a discrete random variable is exactly equal to
face cards 10 and other count by their 1
some value.
p(x) = 1  2  ...  10  10  10  10 
denomination, find the expectation of the 13

Probability density function total score on 13 cards. 85



Solution : 13
It is used to specify the probability of a continuous random variable falling within a
Let x denote the number corresponding to Hence the required expectation is 85.
particular range of values and the probability is given by the integral of the probability
th
density function over that range. the i card. Then x takes the value, 1, 2, 3,

Cumulative distribution function 3.9 Standard Distributions


It is a probability that random variable X will take a value less than or equal to x. Binomial Distribution
i.e., F(x) = P(X ≤ x)
Probability of exactly k successes in n trials is given by

3.8 Expectation(Mean), Variance and Standard Deviation P(X=k) = nCk pk (1  p)n  k

Expected value for a discrete random variable is given by where p is the probability of success in each trial

n Here the trial are Bernouilli trials, where each trial can have exactly two outcomes viz.
E(X) = 
i=1
xi P(xi )
success and failure.
If p is the probability of success
Expected value for a continuous random variable is given by then q = 1  p is the probability of failure.
b Then
E(X) =  x f(x) dx
a
P(X=k) = nCk pk qn  k

173 174
Notes on Probability and Statistics Vidyalankar : GATE – Engineering Mathematics

For Binomial Distribution,


Note: Standard Normal Distribution
Mean = np
Special case with  = 0 and  = 1
Variance = npq = np(1  p)
x2

Variance < Mean e 2
Probability density function is f(x) =
k 2
Cumulative distribution function is, F(x) =  n
Ci pi qni
i 0 This graph is symmetric about x = 0 where it attains its maximum value

Poisson Distribution 1
2
Probability of k events occurring in an interval is given by

 k e Uniform Distribution


P(k events) =
k! Probability density function for uniform distribution is
1
where  is the average number of events per interval. a≤x≤b
ba
For a Poisson Distribution, f(x) =
0 x < a or x > b
Mean = 
Variance =  For Uniform Distribution,
Variance = Mean ab
Mean =
2
i k
Cumulative distribution function is, F(x) = e  



i  0 i! (ba)2
Variance =
12
0 xa
Normal Distribution
xa
Cumulative distribution function is, F(x) = axb
Probability density function of normal distribution is ba
1 xb
(x  )
1 
f(x) = e 2 2
Exponential Distribution
 2
Probability density function for uniform distribution is
where  is the mean and 2 is the variance
ex x≥0
f(x) =
Graph of f(x) is symmetric about x =  and is a bell shaped curve. 0 x<0
1  x    For exponential distribution
Cumulative distribution function is, F(x) = 1erf  
2   2  1
Mean =

1
Variance =
2
Cumulative distribution function is, F(x) = 1  ex

175 176

Notes on Probability and Statistics Vidyalankar : GATE – Engineering Mathematics

Solved Example 13 : 8 Solved Example 18 :  x e 


The probability of getting exactly 2 heads
b) P(3  X 8) =  15 Ci (0.4)i (0.6)15i If the probability that an individual suffers a
P (X = x) =
x!
i 3

in 6 tosses of a fair coin is = 0.8779 bad reaction from injection of a given where  = np = 2000 (0.001) = 2

Solution : c) P (X = 5) = 15
C5 (0.4)5 (0.6)15  5 serum is 0.001, determine the probability 23 e2
a) P (X = 3) =  0.180
2 62 = 0.1859 that out of 2000 individuals 3!
 1  1
P (X = 2) = 6 C2     a) exactly 3
2 2 Solved Example 16 : b) P (X > 2)
b) more than 2, individuals will suffer a
6!  1   1 
2 62
15 The probability is 0.02 that an item = 1  P  X  0   P  X  1  P  X  2  
=  bad reaction.
2!4!  2   2  64 produced by a factory is defective. A
Solution :  20 e2 21e 2 22 e2 
shipment of 10,000 items are sent to its = 1    
Solved Example 14 : Let X denote the number of individuals  0! 1! 2! 
warehouse. Find the expected number E
The probability that a certain kind of suffering a bad reaction. X is poisson = 1  5e2 = 0.323
of defective items and the standard
component will survive a given shock test distributed.
deviation.
is 3/4. Find the probability that exactly 2 of
the next 4 components tested survive. Solution :
 = np = 10000  0.02 = 200
3.10 Mean, Median, Mode and Standard Deviation
Solution :
Assuming the tests are independent and = npq  10000     Mean
p = ¾ for each of the 4 tests we get Mean of a data generally refers to arithmetic mean of the data. However, there are two
= 196 = 14
2 2 2 more types of mean which are geometric mean and harmonic mean. The different types
4  3  1 4! 3 27
C2     = = Solved Example 17 : of mean for set of values and grouped data are given in the following table.
4 2 2!2! 44 128
Ten percent of the tools produces in a
Solved Example 15 : certain manufacturing turn out to be Set of Values Grouped Data
The probability that a patient recovers from defective. Find the probability that in a
 xi  fi xi
a rare blood disease is 0.4. If 15 people sample of 10 tools chosen at random Arithmetic Mean
n  fi
are to have contracted this disease, what exactly two will be defective.
is the probability that Solution :   f log xi 
Geometric Mean (x1 . x 2 . x 3 ....xn )1/n Antilog  i 
a) at least 10 survive We have  = np = 10 (0  1) = 1   fi 
b) from 3 to 8 survive Then according to the Poisson distribution
n
c) exactly 5 survive e  x  fi
P (X = x ) = Harmonic Mean  1 1 1 1
Solution : x!  + +    fi xi
 x1 x 2 x 3 xn 
Let X be the number of people that survive. 12 e1
or  0.1839 or 0.18
P(X = 2) =
a) P(X  10) = 1  P(X  10) 2!
In general the approximation is good if
9
= 1   15 Ci (0.4)i (0.6)15 i P  0.1 and  = np  5
i 0

= 1  0.9662 = 0.0338

177 178
Notes on Probability and Statistics Vidyalankar : GATE – Engineering Mathematics

Median 3.11 Correlation and Regression Analysis


For an ordered set of values, median is the middle value. If the number of values is even,
Correlation
median is taken as mean of the middle two values.
Correlation analysis deals with finding linear association between two variables i.e. if the
For a grouped data, median is given by
two variables are linearly related to each other.
N  Correlation coefficient between variables x and y can be found as follows.
 2  C
Median = L +   h
f  (xi  x) (yi  y) n x i y i   x i  y i
r= =
 (xi  x)  (yi  y)
2 2
where, L = Lower boundary of the median class n xi2  (  xi )2  n yi2  (  yi )2 

N =  fi
where, x = mean of x
C = Cumulative frequency upto the class before the median class
y = mean of y
h = Width of median class
f = Frequency of the median class Regression
Mode Regression analysis involves identifying the relationship between a dependent variable
For a set of values, mode is the most frequently occurring value. For a grouped data, and one or more independent variables.
mode is given by If X and Y are the variables then equation for regression line of Y on X is given by

 fi  fi-1  Y = a + bX
Mode = L + h
 2fi  fi-1  fi+1  where, b=
n xy   x  y
n  x 2  (  x)2
where, L = Lower boundary of the modal class
and a can be found by using
fi = Frequency of the modal class (Highest frequency) y = a + bx
fi1 = Frequency of the class before the modal class
fi+1 = Frequency of the class after the modal class
Note: Correlation coefficient measures linear association between two variables.
h = Width of the modal class
r can vary between 1 to 1 i.e. 1 ≤ r ≤ 1.
Note: Relation between Mean, Median and Mode :
If there is no linear association between the two variables, r will be zero.
Mean  Mode = 3(Mean  Median)

Standard Deviation
Standard deviation is given by,

  xi  x 
2

2 = For a Set of Values


n
 fi  xi  x 
2

2 = For a Grouped Data


 fi

179 180

Notes on Probability and Statistics Vidyalankar : GATE – Engineering Mathematics

List of Formulae Cumulative distribution function is, Cumulative distribution function is,
i k
F(x) = 1  ex
 Permutation and Combination  Bayes’ Theorem F(x) = e   


i  0 i!
n n! 
P(Bk )P(A / Bk )  Mean
Pr = P(Bk/A) =
n  r ! n  Normal Distribution
 P(Bi P(A/Bi  (x  )
Arithmetic Mean
i 1
n! 1 
2 2  xi
n
Cr = f(x) = e = …. for set of values
r!  n  r  !  Expectation(Mean), Variance and  2 n
 Probability Standard Deviation Mean =   fi xi
= …. for grouped data
n(A)
n Variance = 2  fi
P(A) = E(X) =  xi P(xi )
i 1
n(S)
Cumulative distribution function is, Geometric Mean
…. For Discrete Random Variable
1  x    = (x1.x2.x3….xn)1/n …. for set of values
b
 P(A) + P( A ) = 1 F(x) = 1erf  
E(X) =  x f(x) dx 2   2    f log xi 
= Antilog  i
a 
  fi 
 P  A  = P  A  B + P A  B  ….. For Continuous Random Variable  Uniform Distribution
…. for grouped data
2 2 2 1
Var(X) = E((X  ) ) = E(X )  E(X) a≤x≤b
 P(AB) = P(A) + P(B) – P(AB) ba Harmonic Mean
= E(X2)  2 f(x) =
0 x < a or x > b n
=
 P  A1  A 2  A 3  = P  A1  + P  A 2   1 1 1 1
= Var(X)       
ab 
+P  A 3   P  A1  A 2 P  A 2  A 3  Mean =  x1 x 2 x 3 xn 
 Binomial Distribution 2
…. for set of values
P  A 3  A1  + P  A1  A 2  A 3  P(X=k) = nCk pk (1  p)n  k (ba)2
Variance =  fi
12 = …. for grouped data
Mean = np  fi xi
 Two events A & B are said to be Cumulative distribution function is,
Variance = npq = np(1  p)
independent if  Median
Variance < Mean 0 xa
P(A  B) = P(A).P(B) xa For an ordered set of values, median
Cumulative distribution function is, F(x) = axb
ba is the middle value. If the number of
 P(A  B) = P(A).P(B/A) = P(B).P(A/B) k 1 xb
F(x) = 
i 0
n i
Ci p q n i
values is even, median is taken as
 Exponential Distribution mean of the middle two values.
 Conditional probability of occurrence  Poisson Distribution
ex x≥0 For a grouped data,
of event B given that event A has f(x) =
k e  N 
P(k events) = 0 x<0
already occurred k!  2 C 
Median = L   h
P(A  B) Mean =  1 f
P(B/A) = Mean =
P(A) 
Variance = 
1
Variance = Mean Variance =
2

181 182
Notes on Probability and Statistics Vidyalankar : GATE – Engineering Mathematics

where, fi = Frequency of the modal class Assignment  1


L = Lower boundary of the median class (Highest frequency)
N = fi fi1 = Frequency of the class before the Duration : 45 Min. Max. Marks : 30
C = Cumulative frequency upto the modal class Q 1 to Q 10 carry one mark each 5. Among the four normal distributions
class before the median class fi+1 = Frequency of the class after the 1. A sample of 15 data is as follows: 17, with probability density functions as
h = Width of median class modal class 18, 17, 17, 13, 18, 5, 5, 6, 7, 8, 9, 20, shown below, which one has the
f = Frequency of the median class h = Width of the modal class 17, 3. The mode of the data is lowest variance? [ME  2015]
IV
[ME  2017]
 Relation between Mean, Median and
 Mode (A) 4 (B) 13
Mode : III
For a set of values, mode is the most (C) 17 (D) 20
Mean  Mode = 3(Mean  Median) II
frequently occurring value.
2. In a housing society, half of the I
For a grouped data,  Standard Deviation
families have a single child per family,
 fi fi1    xi x 
2
while the remaining half have two
2 1 0 1 2
Mode = L  h 2 = For a Set of Values
 2fi fi1fi1  n (A) I (B) II
children per family. The probability that
(C) III (D) IV
where,  fi  xi x 
2
2 a child picked at random, has a sibling
 = For a Grouped Data
L = Lower boundary of the modal class  fi is _______. [EC2014] 6. A random variable X has probability
density function f(x) as given below:
3. An unbiased coin is tossed an infinite
 a  bx for0x1
number of times. The probability that f(x) = 
the fourth head appears at the tenth  0 otherwise

toss is [EC2014] If the expected value E[X] = 2/3, then

(A) 0.067 (B) 0.073 Pr[X < 0.5] is _______. [EE2015]

(C) 0.082 (D) 0.091 7. Consider a Poisson distribution for the


tossing of a biased coin. The mean for
4. The probability density function of
this distribution is . The standard
evaporation E on any day during a
year in a watershed is given by deviation for this distribution is given
by [ME2016]
1
 0E5mm / day
f(E) =  5 (A)  (B) 2
 0 otherwise
(C)  (D) 1/
The probability that E lies in between 2
and 4 mm/day in a day in the
watershed is (in decimal) _________
[CE2014]

183 184

Assignment on Probability and Statistics Vidyalankar : GATE – Engineering Mathematics

8. The probability of getting a “head” in a 12. The number of accidents occurring in 15. The chance of a student passing an 18. The variance of the random variable X
single toss of a biased coin is 0.3. The a plant in a month follows Poisson exam is 20%. The chance of a student with probability density function
coin is tossed repeatedly till a “head” is distribution with mean as 5.2. The passing the exam and getting above 1
f(x) = | x |e|x| is _____. [EC2015]
obtained. If the tosses are independent, probability of occurrence of less than 2 90% marks in it is 5%. GIVEN that a 2
then the probability of getting “head” for accidents in the plant during a student passes the examination, the 19. Two random variables ܺ and ܻ are
the first time in the fifth toss is _______ randomly selected month is[ME2014] probability that the student gets above distributed according to
[EC2015] (A) 0.029 (B) 0.034 90% marks is [ME2015] FX,Y(x, y)

9. X and Y are two random independent (C) 0.039 (D) 0.044 1 1 (x  y), 0  x  1, 0  y  1,
(A) (B) = 
events. It is known that P(X) = 0.40 and 18 4 0, otherwise.
13. Parcels from sender S to receiver R 2 5 The probability P(X + Y  1) is ______.
P(X  YC) = 0.7. Which one of the (C) (D)
pass sequentially through two post- 9 18 [EC2016]
following is the value of P(X  Y)?
offices. Each post-office has a 16. The probability density function of a
[CE2016]
1 random variable, x is 20. An urn contains 5 red and 7 green
(A) 0.7 (B) 0.5 probability of losing an incoming
5 x
(C) 0.4 (D) 0.3 f(x) = (4x 2 ) for 0  x  2 balls. A ball is drawn at random and its
parcel, independently of all other 4
10. Suppose that a shop has an equal colour is noted. The ball is placed back
parcels. Given that a parcel is lost, the = 0 otherwise
into the urn along with another ball of
number of LED bulbs of two different
probability that it was lost by the The mean, x of the random variable is the same colour. The probability of
types. The probability of an LED bulb
second post-office is ____. [EC2014] ___________. [CE2015] getting a red ball in the next draw is
lasting more than 100 hours given that
it is of Type 1 is 0.7, and given that it is 17. Suppose a fair six-sided die is rolled [IN2016]
14. A traffic office imposes on an average
of Type 2 is 0.4. The probability that once. If the value on the die is 1, 2, or 65 67
5 number of penalties daily on traffic (A) (B)
3, the die is rolled a second time. What 156 156
an LED bulb chosen uniformly at violators. Assume that the number of
random lasts more than 100 hours is is the probability that the sum total of 79 89
penalties on different days is (C) (D)
values that turn up is at least 6? 156 156
________. [CS2016] independent and follows a Poisson
[EC2015]
Q 11 to Q 20 carry two marks each distribution. The probability that there
(A) 10/21 (B) 5/12
11. In the following table, x is a discrete will be less than 4 penalties in a day is
(C) 2/3 (D) 1/6
random variable and p(x) is the ________. [CE2014]
probability density. The standard

deviation of x is [ME2014]

x 1 2 3

P(x) 0.3 0.6 0.1

(A) 0.18 (B) 0.36


(C) 0.54 (D) 0.6

185 186
Assignment on Probability and Statistics Vidyalankar : GATE – Engineering Mathematics

Assignment  2 8. Types II error in hypothesis testing is Q 11 to Q 20 carry two marks each


(A) acceptance of the null hypothesis 11. Consider an unbiased cubic dice with
Duration : 45 Min. Max. Marks : 30 when it is false and should be opposite faces coloured identically and
Q 1 to Q 10 carry one mark each 1 rejected each face coloured red, blue or green
(A) 0 (B)
1. A group consists of equal number of 2 (B) rejection of the null hypothesis such that each colour appears only

men and women. Of this group 20% of 4 1 when it is true and should be two times on the dice. If the dice is
(C) (D)
the men and 50% of the women are 5 5 accepted thrown thrice, the probability of

unemployed. If a person is selected at (C) rejected of the null hypothesis obtaining red colour on top face of the
5. Three vendors were asked to supply a
random from this group, the probability when it is false and should be dice at least twice is ____. [ME2014]
very high precision component. The
of the selected person being employed rejected
respective probabilities of their 12. Let X be a real-valued random variable
is_______ [ME2014] (D) acceptance of the null hypothesis
meeting the strict design specifications with E[X] and E[X2] denoting the mean
when it is true and should be
are 0.8, 0.7 and 0.5. Each vendor values of X and X2, respectively. The
2. Vehicle arriving at an intersection from accepted [CE2016]
supplies one component. The relation which always holds true is
one of the approach roads follow the 9. A voltage V1 is measured 100 times
probability that out of total three (A) (E[X])2 > E[X2] [EC2014]
Poisson distribution. The mean rate of
and its average and standard deviation
components supplied by the vendors, (B) E[X2]  (E [X])2
arrival is 900 vehicles per hour. If a
are 100 V and 1.5 V respectively. A
at least one will meet the design (C) E[X2] = (E[X])2
gap is defined as the time difference
second voltage V2, which is
specification is ________. [ME2015] (D) E[X2] > (E[X])2
between two successive vehicle
independent of V1, is measured
arrivals (with vehicle assumed to be 6. Consider the following probability 13. A fair coin is tossed n times. The
200 times and its average and
points), the probability (up to four mass function (p.m.f.) of a random probability that the difference between
standard deviation are 150 V and 2 V
decimal places) that the gap is greater variable X: the number of heads and tails is
respectively. V3 is computed as:
than 8 second is _______. [CE2017]  q ifX0 V3 = V1 + V2. Then the standard (n  3) is [EE2014]

p(x, q) = 1  q ifX1 deviation of V3 in volt is ________. (A) 2n (B) 0
3. Let X be a zero mean unit variance  0 otherwise
 [IN2016] (C) nCn3 2n (D) 2n + 3
Gaussian random variable. E[|X|] is
If q = 0.4, the variance of X is ______.
10. Suppose p is the number of cars per 14. An observer counts 240 veh/h at a
equal to _______. [EC2014]
[CE2015] specific highway location. Assume that
minute passing through a certain road
4. A fair (unbiased) coin was tossed four 7. The area (in percentage) under junction between 5 PM and 6 PM, and the vehicle arrival at the location is
times in succession and resulted in the standard normal distribution curve of p has a Poisson distribution with mean Poisson distributed, the probability of
following outcomes: (i) Head, (ii) Head, random variable Z within limits from 3. What is the probability of observing having one vehicle arriving over a
(iii) Head, (iv) Head. The probability of −3 to +3 is __________ [ME2016] fewer than 3 cars during any given 30second time interval is _________.
obtaining a ‘Tail’ when the coin is minute in this interval? [CS2013] [CE2014]
tossed again is [CE2014] (A) 8/(2e3) (B) 9/(2e3)
(C) 17/(2e3) (D) 26/(2e3)

187 188

Assignment on Probability and Statistics Vidyalankar : GATE – Engineering Mathematics

15. Two players, A and B, alternately keep 18. The probability that a screw Assignment  3
rolling a fair dice. The person to get a six manufactured by a company is
first wins the game. Given that player A defective is 0.1. The company sells Duration : 45 Min. Max. Marks : 30
starts the game, the probability that A screws in packets containing 5 screws Q 1 to Q 10 carry one mark each 4. If {x} is a continuous, real valued
wins the game is [EE2015] and gives a guarantee of replacement 1. A nationalized bank has found that the random variable defined over the
(A) 5/11 (B) 1/2 if one or more screws in the packet are daily balance available in its savings interval (∞, + ∞) and its occurrence is
(C) 7/13 (D) 6/11 found to be defective. The probability accounts follows a normal distribution defined by the density function given as:
16. The probability that a thermistor that a packet would have to be with a mean of Rs. 500 and a standard 1 x a 
2

1  
2 b 

replaced is _______. [ME2016] f(x) = e where 'a' and 'b'
randomly picked up from a production deviation of Rs. 50. The percentage of 2  b
unit is defective is 0.1. The probability savings account holders, who maintain are the statistical attributes of the
that out of 10 thermistors randomly 19. If a random variable X has a Poisson an average daily balance more than random variable {x}. The value of the
picked up, 3 are defective is distribution with mean 5, then the Rs. 500 is________ [ME2014] 2
1  xa 
a 1   
expectation E[(X + 2)2] equals 
2 b 
[IN2015] integral e dx is

2  b
_________. [CS2017] 2. Let X be a random variable which is
(A) 0.001 (B) 0.057
uniformly chosen from the set of [CE2014]
(C) 0.107 (D) 0.3
positive odd numbers less than 100. (A) 1 (B) 0.5
20. Consider the following experiment.
17. Let X  {0, 1} and Y  (0, 1) be two The expectation, E[X], is _________. 
Step 1. Flip a fair coin twice. (C)  (D)
2
independent binary random variables. Step 2. If the outcomes are (TAILS, [EC2014]
If P(X = 0) = p and P(Y = 0) = q, the HEADS) then output Y and stop.
3. Ram and Ramesh appeared in an 5. If P(X) = 1/4, P(Y) = 1/3, and
P(X + Y  1) is equal to [EC2015] Step 3. If the outcomes are either
interview for two vacancies in the P(X  Y) = 1/12, the value of P(Y/X) is
(A) pq + (1  p) (1  q) (HEADS, HEADS) or (HEADS,
same department. The probability of [ME2015]
(B) pq TAILS), then output N and stop.
Ram's selection is 1/6 and that of 1 4
(C) p(1  q) Step 4. If the outcomes are (TAILS, (A) (B)
Ramesh is 1/8. What is the probability 4 25
(D) 1  pq TAILS), then go to Step 1.
that only one of them will be selected? 1 29
The probability that the output of the (C) (D)
[EC2015] 3 50
experiment is Y is (up to two decimal
(A) 47/48 (B) 1/4
places_______. [CS2016]
(C) 13/48 (D) 35/48



189 190
Assignment on Probability and Statistics Vidyalankar : GATE – Engineering Mathematics

6. Suppose A and B are two independent Q 11 to Q 20 carry two marks each 14. The probability of obtaining at least 18. Three cards were drawn from a pack
events with probabilities P(A)  0 and 11. A machine produces 0, 1 or 2 two "SIX" in throwing a fair dice 4 of 52 cards. The probability that they

P(B)  0. Let A and B be their defective pieces in a day with times is [ME2015] are a king, a queen, and a jack is

complements. Which one of the associated probability of 1/6, 2/3 and (A) 425/432 (B) 19/144 [ME2016]

following statements is FALSE? 1/6, respectively. The mean value and (C) 13/144 (D) 125/432 16 64
(A) (B)
the variance of the number of 5525 2197
[EC2015]
defective pieces produced by the 15. Two coins R and S are tossed. The 4
(A) P(A  B) = P(A) P(B) 3 8
(C) (D)
machine in a day, respectively, are joint events HRHS, TRTS, HRTS, TRHS 13 16575
(B) P(A|B) = P(A)
[ME2014] have probabilities 0.28, 0.18, 0.30,
(C) P(A  B) = P(A) + P(B)
0.24, respectively, where H represents 19. Probability density function of a
(A) 1 and 1/3 (B) 1/3 and 1
(D) P(AB) = P(A)P(B)
 head and T represents tail. Which one
(C) 1 and 4/3 (D) 1/3 and 4/3 random variable X is given below.
7. The second moment of a Poisson- of the following is TRUE? [EE2015]
0.25 if1x5
f(x) = 
distributed random variable is 2. The 12. A box contains 4 red balls and 6 black (A) The coin tosses are independent  0 otherwise
mean of the random variable is _____. balls. Three balls are selected (B) R is fair, S is not
P(X  4) is [CE2016]
[EC2016] randomly from the box one after (C) S is fair, R is not
3 1
another without replacement. The (D) The coin tosses are dependent (A) (B)
8. The spot speeds (expressed in km/hr) 4 2
observed at a road section are 66, 62, probability that the selected set
1 1
16. The probability density function of a (C) (D)
45, 79, 32, 51, 56, 60, 53, and 49. The contains one red ball and two black 4 8
x
random variable X is pX(x) = e for
median speed (expressed in km/hr) is balls is [EC2014]
x  0 and 0 otherwise. The expected
______ (Note: answer with one 1 1 20. Consider the random process
(A) (B)
20 12 value of the function gX(x) = e3x 4 is X(t) = U + Vt
decimal accuracy) [CE2016]
where U is a zero-mean Gaussian
3 1 ___________. [IN2015]
9. A probability density function on the (C) (D) random variable and V is a random
10 2
interval [a, 1] is given by 1/x2 and variable uniformly distributed between
17. A fair die with faces {1, 2, 3, 4, 5, 6} is
outside this interval the value of the 13. Let X be a random variable with 0 and 2. Assume that U and V are
thrown repeatedly till '3' is observed for
function is zero. The value of a is probability density function statistically independent. The mean
the first time. Let X denote the number
______. [CS2016] 0.2, for| x |1 value of the random process at t = 2 is
 of times the die is thrown. The
10. Let U and V be two independent zero f(x) 0.1,for1| x |4 ________. [EC2017]
 0, expected value of X is __________.
 otherwise.
mean Gaussian random variables of
[EC2015]
1 1 The probability P(0.5 < x < 5) is
variances and respectively. The
4 9 ________. [EE2014]
probability P(3V  2U) is [ME2013] 
(A) 4/9 (B) 1/2
(C) 2/3 (D) 5/9

191 192

Assignment on Probability and Statistics Vidyalankar : GATE – Engineering Mathematics

Assignment  4 12. In a race where 12 horses are running, 15. A man and his wife appear for an
the chance that horse A will win is 1/6, interview for two posts. The probability
Duration : 45 Min. Max. Marks : 30 that B will win is 1/10 and that C will of the husband's selection is 1/7 and
Q 1 to Q 6 carry one mark each Q 7 to Q 18 carry two marks each win is 1/8. Assuming that a dead heat that of the wife’s selection is 1/5. The
1. Probability of getting an even number 7. Probability than a leap year selected at is impossible the chance that one of probability that only one of them will be
in a single throw with a die is random will have 53 Sundays. them will win selected is
(A) 1/2 (B) 2/3 (A) 2/5 (B) 1/8 (A) 1/390 (B) 47/120 (A) 2/7 (B) 4/5
(C) 1/4 (D) 1/3 (C) 2/7 (D) 1/7 (C) 3/20 (D) 1/54 (C) 4/35 (D) 6/35
8. A card is drawn from an ordinary pack
2. Probability of getting tail in a throw of a 13. There are two bags, one of which 16. There are two bags. One bag contains
of playing cards and a person bets that
coin is contains 5 red and 7 white balls and 4 white and 2 black balls. Second bag
it is a spade or an ace. Then the odds
(A) 1 (B) 1/3 the other 3 red and 12 white balls. A contains 5 white and 4 black balls.
against his winning this bet is
(C) 1/4 (D) 1/2 ball is to be drawn from one or other of Two balls are transferred from first bag
(A) 9 to 4 (B) 8 to 5
the two bags, find the chance of to second bag. Then one ball is taken
3. A bag contains 6 white balls, 9 black (C) 7 to 6 (D) none of above
drawing a red ball. from the second bag. The probability
balls. The probability of drawing a black 9. In a horse race the odds in favour of that it is white is
(A) 37/120 (B) 1/10
ball is four horses H1, H2, H3, H4 are 1 : 3, (A) 42/165 (B) 95/165
(C) 1/13 (D) 1/96
(A) 2/5 (B) 3/5 1 : 4, 1 : 5; 1 : 6 respectively not more (C) 5/165 (D) 48/165
(C) 1/5 (D) 4/5 than one wins at a time. Then the 14. What is the probability of a particular
chance that one of them wins is person getting 9 cards of the same suit 17. In a single throw of two dice find the
4. Probability of a card drawn at random probability that neither a doublet
(A) 320/419 (B) 319/420 in one hand at a game of bridge where
from an ordinary pack of cards to be (same number on the both dice) nor a
(C) 419/520 (D) 520/519 13 cards are dealt to a person ?
club card is 13 total of 9 will appear.
10. In a garden 40% of the flowers are C9  4
(A) 3/4 (B) 2/4 (A) 39 (A) 5/1 (B) 1/9
roses and the rest are carnations. If C4
(C) 1/4 (D) 1/5 (C) 13/18 (D) 1/4
25% of the roses and 10% of the 13
C9  13
C9  13
C2  4
(B)
5. Probability of throwing a number carnations are red, the probability that a 52
C9 18. A speaks truth in 75% and B in 80% of
greater than 3 with an ordinary die is red flower selected at random is a rose. 13 39 the cases. In what percentage of
C9  C4  4
(A) 1/2 (B) 1/3 (A) 5/6 (B) 1/4 (C) 52
C13 cases are they likely to contradict each
(C) 1/4 (D) 2/3 (C) 4/5 (D) 5/8 other narrating the same incident ?
13
C9  4
11. Find the chance of throwing more than (D) (A) 75% (B) 80%
52
6. Probability of getting a total of more C13
15 in one throw with 3 dice (C) 35% (D) 100%
than 10 in a single throw with 2 dice
(A) 1/54 
(A) 1/6 (B) 1/8 (B) 17/216
(C) 1/12 (D) 2/3 (C) 5/108
(D) cannot be determined

193 194
Assignment on Probability and Statistics Vidyalankar : GATE – Engineering Mathematics

Assignment  5 8. Three machines A, B and C produce 12. If 10 coins are tossed 100 times, how
respectively 60%, 30% and 10% of the many times would you expect 7 coins
Duration : 45 Min. Max. Marks : 30 total number of items of a factory. The to fall head upward ?

Q 1 to Q 6 carry one mark each 5. 4 coins are tossed. Then the probability percentages of defective output of (A) 12 (B) 11

1. A die is rolled. The probability of that at least one head turns up is these machines are respectively 2%, (C) 10 (D) 9

getting a number 1 or 6 on the upper (A) 15/26 (B) 1/16 3% and 4%. An item is selected at
13. If the probability of a defective bolt is
face is (C) 14/16 (D) 15/16 random and found defective. Find the
0.1, the mean and standard deviation
(A) 1/3 (B) 1/2 probability that the item was produced
6. In a throw of 3 dice the probability that for the distribution of defective bolts in
(C) 2/3 (D) 1/4 by machine C.
a total of 500 are
at least one die shows up 1 is (A) 2/25 (B) 1/25
2. The probability of the horse A winning (A) 5/6 (B) 1/6 (A) 50, 6.7 (B) 40, 7.3
(C) 4/25 (D) 3/25
the race is 1/5 and the probability of (C) 91/216 (D) 90/215 (C) 20, 3.1 (D) none of these
the horse B winning the same race is 9. A fair die is tossed 180 times. The
Q 7 to Q 18 carry two marks each 14. Two cards are drawn with replacement
1/6, then the probability of one of the expected number of sixes is
7. Match the following : (A) 40 (B) 25 from a well shuffled deck of 52 cards.
horse to win the race is
List  I (C) 180 (D) 30 The mean and standard deviation for
(A) 11/29 (B) 11/28
(a) P(),  is the empty set the number of aces are
(C) 11/30 (D) 11/31
10. In a population having 50% rice
(b) P(A/B)P(B)
3. A card is drawn from a pack of 52 consumers, what is the probability that X 0 1 2
cards and then a second is drawn.
(c) P A   three or less out of 10 are rice
144 24 1
P(x)
Probability that both the cards drawn 
(d) P A  B  consumers? 169 169 169
are queen is (A) 17% (B) 10% Probability distribution table
(e) P (A  B)
(A) 1/219 (B) 1/13 (C) 40% (D) 50%
2 1
List  II (A) ,0.377 (B) ,0.277
(C) 1/17 (D) 1/221 13 13
(i) 1  P(A) 11. The chances of a person being alive
4. A bag contains 5 white and 3 black who is now 35 years old, till he is 75 3
(ii) P(A  B) (C) ,0.477 (D) none of these
13
balls. Two balls are drawn at random are 8 : 6 and of another person being
(iii) 1  P(A  B)
one after the other without alive now 40 years old till he is 80 are 15. In a Binomial distribution, the mean
(iv) 0
replacement. The probability that both 4 : 5. The probability that at least one and standard deviation are 12 and 2
(v) P(A) + P(B)  P(A  B)
the balls drawn are black is of these persons would die before respectively. The values of n and p are
(A) 3/28 (B) 3/8 (A) a  iv, b  ii, c  i, d  iii, e  v completing 40 years hence is respectively.
(C) 2/7 (D) 3/29 (B) a  iii, b  ii, c  i, d  iv, e  v (A) 8/14 (B) 16/63 1
(A) 12, 2 (B) 9,
(C) a  ii, b  iii, c  i, d  iv, e  v (C) 4/9 (D) 47/63 3
(D) a  i, b  ii, c  iii, d  iv, e  v 2 1
(C) 18, (D) , 18
3 3

195 196

Assignment on Probability and Statistics Vidyalankar : GATE – Engineering Mathematics

16. For a biased die the probabilities for 18. Aishwarya studies either computer Assignment  6
different faces to turn up are given science or mathematics everyday. If
below. she studies computer science on a Duration : 45 Min. Max. Marks : 30

Face Probability day, then the probability that she Q1 to Q6 carry one mark each (A) 1 / 16 (B) 7 / 663
1 0.10 studies mathematics the next day is 1. The probability that the birthday of a (C) 8 / 663 (D) none of these
0.6. If she studies mathematics on a child is Saturday or Sunday is
2 0.32 Q7 to Q18 carry two marks each
day, then the probability that she (A) 1 / 2 (B) 2 / 7
3 0.21 7. The probability that a man aged 50
studies computer science the next day (C) 1 / 3 (D) 1 / 7
4 0.15 years will die within a year is 0.01125.
is 0.4. Given that Aishwarya studies
5 0.05 2. The probability of getting a multiple of The probability that out of 12 such
computer science on Monday, what is
6 0.17 2 in the throw of a die men at least 11 will reach their fifty first
the probability that she studies
(A) 1 / 3 (B) 1 / 4 birthday.
The die is tossed and you are told that computer science on Wednesday ?
(C) 1 / 6 (D) 1 / 2 (A) 1 (B) 0
either face 1 or face 2 has turned up. (A) 0.24 (B) 0.36
3. Probability that the sum of the score is (C) 0.9923 (D) 0.8823
The probability that it is face 1 is (C) 0.4 (D) 0.6
(A) 2/11 (B) 3/21 odd in a throw of two dice is 8. The probability that a certain beginner
(C) 11/13 (D) 5/21 (A) 1 / 4 (B) 1 / 5 at golf gets a good shot if he uses the
(C) 1 / 2 (D) 1 / 3 correct club is 1/3 and the probability
17. There are three events A, B and C,
4. A committee of 5 students is to be of a good shot with an incorrect club is
one of which must occur and only one
chosen from 6 boys and 4 girls. The 1/4. In his bag are 5 different clubs,
can happen, the odds are 8 to 3
probability that the committee contains only one of which is correct for the
against A, 5 to 2 against B; find the
exactly 2 girls is shot in question. If he chooses a club
probability of C.
(A) 9 / 20 (B) 1 / 6 at random and takes a stroke, the
(A) 41/57 (B) 3/11
(C) 10 / 21 (D) 1 / 9 probability that he gets a good shot is
(C) 34/77 (D) 2/7
(A) 2 / 3 (B) 4 / 15
5. A bag contains 7 red, 5 blue, 4 white
(C) 5 / 9 (D) 7 / 36
 and 4 black balls. Then the probability
that a ball drawn at random is red or 9. From a pack of cards two are drawn,
white is the first being replaced before the
(A) 11 / 20 (B) 1 / 7 second is drawn. The probability that
(C) 1 / 11 (D) 3 / 20 the first is a diamond and the second
is a king is
6. Two cards are drawn at random from a (A) 1 / 52 (B) 4 / 13
pack of 52 cards. The probability that (C) 57 / 64 (D) 11 / 52
one may be a jack and other an Ace is

197 198
Assignment on Probability and Statistics Vidyalankar : GATE – Engineering Mathematics

10. One of the two mutually exclusive 15. Find the probability of drawing one Test Paper  1
events must occur, if the chance of rupee coin from a purse with two
one is 2/3 of the other, then odds in compartments one of which contains 3 Duration : 30 Min. Max. Marks : 25
favour of the other are fifty paisa coins and 2 onerupee Q1 to Q5 carry one mark each 5. From a bag containing 4 white and
(A) 1 : 1 (B) 1 : 2 coins and the other contains 2 fifty 1. In a shooting competition, the 5 black balls a man draws 3 at
(C) 2 : 3 (D) 3 : 2 paisa coins and 3 one rupee coins. probability of hitting the target by A is random. Then the probability that all 3
1 (A) 1 / 2 (B) 2 / 5
11. Given P(A  B ) = and 2/5, by B is 2/3 and by C is 3/5. If all are black is
3
(C) 3 / 5 (D) none of these of them fire independently at the same (A) 5/42 (B) 1/42
2
P(A  B) = then P(B) is
3 16. If A and B are two events such that target, calculate the probability that (C) 1/20 (D) 1/15
(A) 2 / 3 (B) 1 / 5 only one of them will hit the target.
P(A  B) = 5/6 . P(A  B) = 1/3 P( B ) Q6 to Q15 carry two marks each
(C) 1 / 3 (D) 4 / 5
= 1/ 2 then the events A and B are (A) 2/5 (B) 2/3 6. Urn A contains 6 red and 4 black balls
12. There are 64 beds in a garden and 3
(A) dependent (C) 3/5 (D) 1/3 and urn B contains 4 red and 6 black
seeds of a particular type of flower are
(B) independent balls, one ball is drawn at random from
sown in each bed. The probability of a 2. A box contains 6 white balls and
(C) mutually exclusive urn A and placed in urn B. Then one
flower being white is 1/4. The number 3 black balls and another box contains
(D) none of these ball drawn at random from urn B and
of beds with 3, 2, 1, and 0 white 4 white balls and 5 black balls. The
placed in urn A. If one ball is now
flowers is respectively 17. A determinant is chosen at random from probability that a ball selected from
drawn from urn A, the probability that it
(A) 1, 9, 27, 27 (B) 27, 9, 1, 27 the set of all determinants of order 2 with one of the box again selected at
is found to be red is
(C) 27, 9, 1, 1 (D) 27, 9, 9, 1 elements 0 or 1 only. The probability that random is a white ball.
(A) 3 / 11 (B) 1 / 17
value of the determinant chosen is (A) 6/18 (B) 4/18
13. A sample space has two events A and (C) 32 / 55 (D) 31 / 56
positive is (C) 1/2 (D) 5/9
B such that probabilities
7. Two persons each make a single
(A) 2 / 17 (B) 1 / 17
P(A  B) = 1/2 , P(A) = 1/3, 3. Two sisters A and B appeared for an throw with a dice. The probability they
(C) 1 / 16 (D) 3 / 16
P(B) = 1/3. What is P(A  B) ? Audition. The probability of selection get equal value is P1. Four persons
18. One hundred identical coins each with
(A) 11/12 (B) 10/12 of A is 1/5 and that of B is 2/7. The each make a single throw and
probability p of showing up heads are
(C) 9/12 (D) 8/12 probability that both of them are probability of three being equal is P2.
tossed. If 0 < p < 1 and the probability
selected is Then
14. A man takes a step forward with
of heads showing on 50 coins is equal
(A) 2/35 (B) 1/5 (A) P1 = P2 (B) P1 < P2
probability 0.4 and backward with
to that of the heads showing in 51
(C) 2/7 (D) none of these (C) P1 > P2 (D) none of these
probability 0.6. The probability that at
coins, then the value of p is
the end of eleven steps he is one step 4. Two balls are to be drawn from a bag 8. A bag has 13 red, 14 green and 15
(A) 1 / 2 (B) 49 / 101
away from the starting point. containing 5 red and 7 white balls. Find black balls. The probability of getting
(C) 50 / 101 (D) 51 / 101
(A) 0.3678 (B) 0.25 the chance that they will both be white exactly 2 blacks on pulling out 4 balls is
(C) 11c2(0.24)3 (D) none of these (A) 5/108 (B) 5/7 P1. Now the number of each colour ball
 (C) 7/22 (D) 2/35 is doubled and 8 balls are pulled out.

199 200

Test Paper on Probability and Statistics Vidyalankar : GATE – Engineering Mathematics

The probability of getting exactly 4 13. A student appears for tests I, II and III. Test Paper  2
blacks is P2. Then The student is successful if he passes
(A) P1 = P2 (B) P1 > P2 either in tests I and II or tests I and III. Duration : 30 Min. Max. Marks : 25
(C) P1 < P2 (D) none of these The probabilities of the student Q1 to Q5 carry one mark each 5. A bag contains 3 black and 5 white
passing in tests I, II and III are p, q and 1. What is the chance of throwing a balls. One ball is drawn from the bag.
9. If A and B are arbitrary events, then
1/2 respectively. If the probability that number greater than 4 with an ordinary What is the probability that the ball is
(A) P ( A  B)  P(A) + P(B) 1
the student is successful is 1/2. Then die whose faces are numbered from not black ?
(B) P(A B)  P(A) + P(B)  1
(A) p = q = 1 (B) p = q = 1/2 1 to 6 ? (A) 3/8
(C) P(A  B) = P(A) + P(B)  1
(C) p = 1, q = 0 (D) p = 1, q = 1/2 (A) 1/6 (B) 1/3 (B) 5/8
(D) none of above
(C) 1/4 (D) none of these (C) 2/7
14. The figure below shows the network
10. One mapping is selected at random (D) none of the these
connecting cities A, B, C, D, E and F. 2. Find the chance of throwing at least
from all the mappings from the set
The arrows indicate permissible one ace in a simple throw with two
S = 1,2,3,...n into itself. The probability Q6 to Q15 carry two marks each
directions of travel. If Deepak from city dice numbered 1 to 6.
that the selected mapping is one to one 6. A single letter is selected at random
A wants to visit city F, what is the (A) 1/11 (B) 1/36
is from the word “PROBABILITY”. The
probability he will pass through city C (C) 11/18 (D) 11/36
(A) 1/nn (B) 1/n! probability that it is a vowel is
B D 3. What is the probability that a digit (A) 1/3 (B) 2 / 21
(C) n/n! (D) none of these
A selected at random from the logarithmic (C) 0 (D) 2 / 9
F
11. For two events A and B P(A  B) is
table is 1 ?
(i) not less than P(A) + P(B) 1 (ii) C E 7. A and B throw a coin alternately till
(A) 1/2 (B) 1/5
not greater than P(A) + P(B) one of them gets a ‘head’ and wins the
(A) 3/5 (B) 4/5 (C) 1/3 (D) 1/10
(iii) equal to P(A) + P(B)  P(A  B) game. Their respective probabilities of
(C) 1/2 (D) 3/4
4. A problem in mathematics is given to winning are
(iv) equal to P(A) + P(B) + P(A  B)
15. Mean of 200 observations was found
three students Dayanand, Ramesh (A) 1/3, 2/3 (B) 1/2, 1/2
(A) i, ii, iii (B) i, iv
to be 90. However, when the (C) 1/4, 3/4 (D) 2/3, 1/3
(C) i, iii (D) i, ii, iii, iv and Naresh whose chances of solving
observations were being made, two
1 1 
12. Two persons A and B have observations were wrongly taken as it are , , respectively. The 8. Two dice are thrown the probability of
2 3 4
respectively n + 1 and n coins, which 15 and 80 instead of 40 and 87. Then getting an odd number on the one and
probability that the problem is solved is
they toss simultaneously. Then the the correct mean is a multiple of 3 on the other is
(A) 1/4 (B) 1/2
probability that A will have more heads (A) 90 (B) 91.16 (A) 11/36 (B) 2/17
(C) 3/4 (D) 1/3
than B is (C) 90.16 (D) 91.6 (C) 10/36 (D) 12/36
(A) 1 / 2 (B) > 1 / 2
(C) < 1 / 2 (D) none of these


201 202
Test Paper on Probability and Statistics Vidyalankar : GATE – Engineering Mathematics

9. In a certain college, 4% of the men 13. If A and B are two events such that Test Paper  3
and 1% of the women are taller than P(A) = 0 and P(B)  1, then P( A / B ) is
Duration : 30 Min. Max. Marks : 25
1.8m. Further 60% of the students are equal to
woman. If a student is selected at Q1 to Q5 carry one mark each 5. A bag contains 8 green and 10 white
(A) 1  P(A/ B ) (B) 1  P( A /B)
random and is taller than 1.8m, the 1. An urn contains 25 balls numbered 1 balls. Two balls are drawn. What is the
(C) 1  P(A / B) (D) P(A)
probability that the student is women is P(B) P(B) through 25. Two balls are drawn from probability that one is green and the
(A) 3/11 (B) 2/11 the urn with replacement. The other is white ?
14. Ten points are marked on a straight
(C) 4/11 (D) 1/11 probability of getting at least one odd 80 10
line and 11 points are marked on (A) (B)
is 153 17
10. A and B throws two dice; if A throws 9 another straight line. If any three
(A) 144/625 (B) 312/625 8 81
then B’s chance of throwing a higher points are chosen, what is the (C) (D)
(C) 12/25 (D) 481/625 17 157
1 probability that it forms a triangle?
number is , state whether the above
6 11 21
(A) (B) C3 2. Four persons are chosen at random Q6 to Q15 carry two marks each
statement is true or false. 14  10
C3  11C3  from a group containing 3 men, 6. Fifteen coupons are numbered
(A) True
6 10
C3  11C3
2 women and 4 children. The chance 1, 2, 3, …….., 15. Seven coupons are
(B) False (C) (D)
7 1045 that exactly 2 of them will be children selected at random one at a time with
(C) Can not say
is replacement. The probability that the
(D) Data insufficient 15. Three machines A, B and C produce (A) 10/21 (B) 5/8 largest number appearing on selected
11. A pack of cards contains 4 aces, identical items of their respective (C) 13/32 (D) 3/32 coupon is 9, is
4 queens and 4 jacks. Two cards are output 5%, 4% and 3% items are
6 8
 9   7 
drawn at random. The probability faulty. If A produces 25% of the total 1 (A)   (B)  
3. Let A and B be events with P(A) = ,  16   15 
that at least one of them is an ace is output, B produces 30% and C the 2
7
remainder and an item selected at 1 1 2
19 3 P(B) = and P(A  B) = . Then (C)   (D) none of these
(A) (B) 3 4 5
33 16 random is found to be faulty, what is

1 1 the probability that it was produced by P(A/B) is


(C) (D) 7. A bag contains 2 white and 3 black
6 9 machine C? (A) 1/4 (B) 1/3
(C) 1/2 (D) 3/4 balls. A ball is drawn 5 times, each
12. x is a random variable with mean  (A) 0.155 (B) 0.255
being replaced before another is
(C) 0.355 (D) 0.455
and S. D.  > 0
4. How many different words can be drawn. Find the probability that exactly
x 4 of the balls drawn are white.
x = ; then var(x) = _______ formed from the letters of the word

Ganeshpuri when all the letters taken 2 48
(A) 0 (B) 2 (A) (B)
(A) 2! (B) 10! 5 625
(C) can’t say (D) 1
(C) 9! (D) 5! 31 3
(C) (D)
 725 5

203 204

Test Paper on Probability and Statistics

8. From a pack of 52 cards two are


drawn at random. The probability that
12. For a Binomial distribution, the mean
Solutions  Linear Algebra
is 6 and the standard deviation is 2 .
one is a king and the other a queen is The Binomial distribution is
(A) 3/ 8 (B) 8 / 663
Answer Key on Assignment  1
9 9
 1 2  1 1
(C) 7/ 663 (D) 9 / 663 (A)    (B)   
3 3 2 2 1. (A) 2. (D) 3. 0.99 to 1.01 4. (A)
9 9
9. In three throws of two dice, the  6 1  4 1 5. (A) 6. (B) 7. (A) 8. 2
(C)    (D)   
7 7 5 5
probability of throwing doublets not 9. 6 10. (C) 11. (B) 12. 2.9 to 3.1
more than two times is
13. If 3 squares are chosen at random on 13. (A) 14. (B) 15. (D) 16. (A)
(A) 1/216 (B) 1/6
a chess board the probability that they
(C) 4/216 (D) 215 / 216 17. 6 to 6 18. (C) 19. (B) 20. (D)
should be in a diagonal line is

10. If the sum of the mean and the (A) 5/744 (B) 3/744

variance of a Binomial distribution for (C) 7/744 (D) 9/744 Answer Key on Assignment  2
18 trials is 10, find the distribution.
14. A man has 9 friends (4 men and 1. (A) 2. (D) 3. 199 to 201 4. 23 to 23
18
 1 2
(A) (p + q)n (B)    5 women). In how many ways can he
3 3 5. 88 to 88 6. 2 7. 4.49 to 4.51 8. 16.5 to 17.5
invite them, if there have to be exactly
18 11
 1 1  1 2 9. (D) 10. (C) 11. 48.9 to 49.1 12. 2.0 to 2.0
(C)    (D)    3 women in the invitees ?
2 2 3 3
(A) 320 (B) 160 13. (D) 14. (D) 15. 2 to 2 16. (B)
(C) 80 (D) 200
11. If the sum of the mean and the 17. (B) 18. (D) 19. (A) 20. 0 to 0
variance of a Binomial distribution for
15. Given two events A and B and
5 trials is 1.8 find the distribution.
5 5
P(A) = 1/4, P(B/A) = 1/2, P(A/B) = 1/4. Answer Key on Assignment  3
 1 2  1 1
(A)    (B)    Which of the following is true?
3 3 2 2 1. (D) 2. (D) 3. (B) 4. (A)
(A) P(A B) = 1/2
9 5
 6 1  4 1 5. (A) 6. (C) 7. 2.0 to 2.0 8. (C)
(C)    (D)    (B) A is subevent of B
7 7 5 5
(C) P(A/B) + P(A B) = 1 9. (A) 10. 0.95 to 1.05 11. (B) 12. (A)
(D) None of these
13. 5.0 to 5.0 14. 0 15. 2.9 to 3.1 16. (B)

17. (A) 18. (B) 19. (A) 20. (A)



205 206
Solutions  Linear Algebra Vidyalankar : GATE – Engineering Mathematics

Answer Key on Assignment  4 Answer Key on Assignment  8


1. (C) 2. (B) 3. (A) 4. (B) 1. (C) 2. (C) 3. (A) 4. (C)
5. (D) 6. (A) 7. (A) 8. (C) 5. (C) 6. (C) 7. (A) 8. (A)
9. (A) 10. (A) 11. (B) 12. (D) 9. (B) 10. (C) 11. (C) 12. (C)
13. (B) 14. (C) 15. (A) 16. (D) 13. (C) 14. (C) 15. (C) 16. (B)
17. (B) 18. (B) 17. (B) 18. (A)

Answer Key on Assignment  5 Answer Key on Assignment  9

1. (B) 2. (C) 3. (A) 4. (B) 1. (B) 2. (B) 3. (C) 4. (A)

5. (B) 6. (A) 7. (B) 8. (A) 5. (C) 6. (B) 7. (C) 8. (A)

9. (B) 10. (A) 11. (C) 12. (B) 9. (C) 10. (B) 11. (C) 12. (C)

13. (C) 14. (A) 15. (B) 16. (C) 13. (C) 14. (C) 15. (B) 16. (C)

17. (D) 18 (D) 17. (D) 18. (D)

Answer Key on Assignment  6 Answer Key on Assignment  10


1. (B) 2. (C) 3. (C) 4. (C) 1. (D) 2. (B) 3. (B) 4. (C)
5. (B) 6. (C) 7. (B) 8. (B) 5. (B) 6. (C) 7. (A) 8. (A)
9. (B) 10. (D) 11. (C) 12. (C) 9. (A) 10. (B) 11. (C) 12. (B)
13. (C) 14. (A) 15. (C) 16. (A) 13. (C) 14. (C) 15. (D) 16. (C)
17. (C) 18. (B) 17. (D) 18. (C)

Answer Key on Assignment  7 Answer Key on Assignment  11


1. (C) 2. (D) 3. (B) 4. (B) 1. (C) 2. (C) 3. (C) 4. (D)
5. (D) 6. (A) 7. (A) 8. (C) 5. (C) 6. (C) 7. (C) 8. (C)
9. (A) 10. (A) 11. (B) 12. (C) 9. (A) 10. (B) 11. (D) 12. (B)
13. (B) 14. (A) 15. (B) 16. (A) 13. (C) 14. (D) 15. (B) 16. (C)

17. (A) 18. (B) 17. (D) 18. (D)



207 208

Solutions  Linear Algebra Vidyalankar : GATE – Engineering Mathematics

Model Solution on Assignment  1 9. 6  max = 1 and min = 3


The given matrix is max 1 1
1. (A) 1 2 4    =   =
1 4 5  min 3 3
2 6 0 1 3 0   0 2 3 A=  
2 2 1
 4 12 8 = (2)3   2 6 4 1 1 1 13. (A)
Inorder to find eigen values of above
2 0 4 1 0 2 1  1 5 3 7 
= 1[2
 3]2[03]4[02] matrix. A=   B=  
= 8  (12) = 96 2 6 2  8 4 
|A  I| = 0
1 3
2. (D) = 1  6  8 4 5
 1 5  3 8 
ABT =  
38 28 
 = 32 56 
2 2    =0 6 2  7 4   
We know that the Eigen vectors 2 1 
corresponding to distinct Eigen values 6. (B)  (4  ) (1  )  10 = 0 14. (B)
of real symmetric matrix are  4  5 + 2  10 = 0 10 5  J 4 
orthogonal.
7. (A) Let A =  x 20 2 
 2  5  6 = 0
 4 2 10 
 x1   y1   2  6 + 1  6 = 0
 x  y  = x y + x y + x y = 0 8. 2
 2  2 1 1 2 2 3 3 Given that all eigen values of A are real.
x  2y + 3z = 1  (  6) + 1(  6) = 0
 x 3   y 3 
 A is Hermitian
x  3y + 4z = 1   + 1 = 0 or 
 
T
3. 0.99 to 1.01 2x + 4y  6z = k   = 1 or  = 6  A = A i.e. A =A
2
Given, A = I Arugmented matrix (A/B) is given by  Larger eigen value of matrix is 6.
 10 x 4  10 5  J 4 
We know that above equation is  5  j 20 2  =  x 20 2 
 1 2 3 1 10. (C)   
satisfied by identity matrix. (A/B) =  1 3 4 1   4 2 10   4 2 10 
1 0 0 0  1 0 0 0  2 4 6 k  11. (B)
 x=5j
0 1 0 0  0 1 0 0 
i.e.,   =I Now applying row reduction technique  12. 2.9 to 3.1
0 0 1 0  0 0 1 0
    R2  R2  R1 Characteristic equation is given by 15. (D)
0 0 0 1 0 0 0 1
R3  R3 + 2R1 |A  I| = 0 The characteristic equation is 2 
Eigen value of identity matrix is 1.
We get 0 1 1 (sum of diagonal elements)  + |A| = 0
4. (A)  1 2 3 1   6 11   6  =0
 2  (2 + p)  + (2p  1) = 0
 0 1 1 2  6 11 5
 (2  p) (44pp2 )4(2p  1)
5. (A)  0 0 0 k  2  0 1 1  =
2
Area of triangle For infinite solution   6 11   6  =0
x1 x 2 x3 6 11 5 (2  p) (84pp2 )
(A/B) = (A) = r =
1 2
=  y1 y 2 y3  3 2
  + 6 + 11 + 6 = 0
2  k2=0
1 1 1 (2  p)  (84pp2 ) 3
 k=2   = 1, 2, 3 are the eigen values  
(2  p) (84pp2 ) 1
of A

209 210
Solutions  Linear Algebra Vidyalankar : GATE – Engineering Mathematics

2 (84pp2 ) 2 Multiply by ‘A’, We get, Model Solution on Assignment  2


 
2(2  p) 4 A3 = 5A2  6A
1. (A) 5. 88 to 88
= +5 [5A + 6I]  6A
 2 84pp2 = (2 + p) dx
3 0 1 2 3
A = 19A + 30I Given that = 3x  5y 1
 4(8  4p + p2) = (4 + 4p + p2) dt 0 3 0 
20. (D) A= 
2 3 0 1
 3p2  20p + 28 = 0 dy  
= 4x + 8y
 1 1  dt 3 0 1 2
 (3p  14) (p  2) = 0  0 
 2 2
d x  3 5   x  1 3 0 1 0 0
 p = 2, 14/3   = 
P=  0 1 0  Matrix form
dt y     = 1 2 0 1 2 2 3 1
  4 8   y 
 1 0
1 
3 1 2 3 0 2
16. (A)  2 2  2. (D)
1 0 3
17. 6 to 6 1  1  1  1 
|P| =  0 0   3. 199 to 201 3 2 3 0
2 2  2 2
18. (C) 3 0 1
1 1 4. 23 to 23
2 1 1  =  1 = 1[1(1)  3(4  3)] + 2[1(6)]
0 1 1 2 2
  Given Matrices,  3[1(3) + 3(9)]
 1 1 0   1 1   1 1 
 0   0  3 2 1  1 = 1[ 1  3] + 12  3[3  27]
r < n. Hence, infinite number of  2 2  2 2
J = 2 4 2  and K = 2
T
P.P =  0 1 0  0 1 0    = 4 + 12  3 [24]
solutions.      1 2 6   1
 1 0
1   1
0
1  = 4 + 12 + 72 = 88.
19. (B)  2 2   2 2  KT = 1 2 1

|A  I| = 0 …characteristic equation  1 0 0  3 2 1  1  6. 2


 5 3   0  = 0 1 0   KTJK = 1 2 12 4 2  2  4 2 
     0 0 0 1  1 2 6   1 Let A=  
 2 0  0    1 3
 P is an orthogonal matrix  1 Characteristic equation of A is
5   3
 
2 
 0 (A) Is correct =  3  4  1 2  8  2 1  4  6   2 
A  I  0
 1
 Inverse of P is its transpose only
(5 + ) + 6 = 0 4 2
 (B) and (C) both are correct  1    =0
2 + 5 + 6 = 0 1 3
 (D) is incorrect =  6 8 1  2  = 6 + 16 + 1
 By Cayley Hamilton’s Theorem, 2  7 + 10 = 0   = 2, 5
 1
A2 + 5A + 6I = 0
KTJK = 23
 A2 = 5A  6I


211 212

Solutions  Linear Algebra Vidyalankar : GATE – Engineering Mathematics

7. 4.49 to 4.51  2x = 12 0 4 4  eigen values are 1 and 2


2 3 : 5   x=6 and  14 8 18   smallest is 1 and largest is 2.
[A : B] =  
3 p : 10  14 0 10
 (6, 2)
14. (D)
2C2  3C1 = 4(140 + 252) + 4(112)
Sum of Eigen values = 1 + a
2 3 : 5 10. (C) = 4(112) + 4 (112) = 0
0 2p9 : 5   6=1+a
  Given : M4 = I Hence (A)  2.
4k 4 k k
 a=5
There is no solution if  M = (M ) = I = I 6 0
Now   = 84  0 and product of Eigen values = det(A)
rank (A : B)  rank(A). (A) M 4k + 1
=M 2 14
7 = a  4b
Now, rank (A : B) = 2. So rank(A) = 1 (B) M4k + 2 = M2 Hence (A) = 2
4k + 3 4k 4 1 1  7 = 5  4b
only if 2p  9 = 0 i.e. p = 9/2 = 4.5 (C) M =M M M =M  Rank of given matrix is 2
 7  5 = 4b

13. (D)  b=3


8. 16.5 to 17.5 11. 48.9 to 49.1

AX = X a b  The characteristic equation is


Let A =  15. 2 to 2

 c d (3  ) 2 2
4 1 2  1  1 It is an upper triangular matrix. Hence
 P 2 1  2  =  2   4 (4  ) 6   0 ;
      a + d = 14 and bc ≥ 0 eigen values are the diagonal
14 4 10  3  2 3 (5   )
3  Now |A| is maximum when ad is elements i.e. 2, 2, 3. Hence there are
maximum and bc = 0 i.e. C2 = C2 + C3 gives
 12   2 linearly independent eigen vectors.
 P  7  = 2  when a = d = 7 (3  ) 0 2
   
 36  3   4 (2  ) 6  =0 16. (B)
 Maximum value of |A| = 7 × 7 = 49
2 (2  ) (5   )
  = 12 ….(1) 17. (B)
12. 2.0 to 2.0 (3   ) 0 2
2 = P + 7 ….(2) 1 1 1 : 6 
The given matrix is  (2  )  4 1 6  = 0;
and 3 = 36  146:20 
6 0 4 4  2 1 (5   )
 = 12 1 4  :  
A =  2 14 8 18 
i.e.,
R3 = R3  R2 gives
 Equation (2) gives P + 7 = 24 14 14 0 10  R3  R3  R2
(3  ) 0 2
 P = 17 The matrix is of order 3  4 (2  ) 4 1 6  =0 1 1 1 : 6 
Hence (A)  3 2 0 (1  )  14 6 : 20 
9. (D)
0 0   6 :   20 
2x + 5y = 2 ….(1) 6 4 0  (2  ) {(3  )[(1  )  0]  0
2 8 = 6(0 + 112)
14 For  = 6,   20 System is
4x + 3y = 30 ….(2) + 2[0  (2)]} = 0
14 14 0
 (2  ) {3  2 + 2 + 4} = 0 inconsistent
2 × (1) + (2) gives 13y = 26
+ 4(28  196)  It has no solution.
 y = 2  (2  ) (2  2 + 1) = 0
= 672  672 = 0
 2x  10 = 2  (2  ) (  1)2 = 0

213 214
Solutions  Linear Algebra Vidyalankar : GATE – Engineering Mathematics

18. (D) 19. (A) Model Solution on Assignment  3


1 1  If matrix B is obtained from matrix A by
A  1. (D) 5. (A)
1 1 replacing the lth row by itself plus k
th
times the m row, for /  m then  5 2 
32 0  Eigen values of the matrix  
A 5 .A 5    9 6  6. (C)
 det(B) = det(A). With this property
 0 32  Characteristic equation is
given matrix is equal to the matrices are 4, 3
4 0  1 1 5
4
A    The eigen vector corresponding to
0 4  given in options (B), (C) and (D).
 0 5 6  0
eigen vector  is Ax = x (verify the
128 128  0 6 5
A10 .A 5    20. 0 to 0 options)
128 128   (1  ) [(5  )2 + 36] = 0
 50 70  1
A=     is eigen vector corresponding to
4 4   70 80  1  = 1; 2  10 + 61 = 0
A5   
 4 4  10  100  224
Eigenvectors are eigen value  = 3 =
512 512  2
A15 .A 4     70    2  80 
512 512  X1 =  ;X2   2. (D) 1012j
 1  50   70  = 5j6
512 512  MN is not always equal to NM. 2
 A 19
  where 1, 2 = Eigenvalues of A
512 512  3. (B)
   80  7. 2.0 to 2.0
512   512 X1T X2  70 1  50  2 
  0  70   1 2 2 : b1 
512 (512   ) [A:B] =    0 1 2 
= 70(2  80) + (1  50) 70  5 1 3 : b2  P + Q =  8 9 10 
 
   512    2   512   0
2 2
= 702  5600 + 701  3500 R2  R2  5R1  8 8 8 

 2 = 2  (512)2 = 70(1 + 2)  9100 1 2 2 : b1   8 9 10 


 0 9 7 : b  5b 
= 70(130)  9100  2 1 R1R2  0 1 2 
 = 2 (512)
 1 1 1 
= 9100  9100 = 0  (A) = (A/B) < number of unknowns,
If eigen values of A are 1, 2,…, then
k  sum of eigenvalues  1   2  for all values of b1 and b2. 8 9 10 
eigen values of A are
 
 The equations have infinitely many 8R3 R10 1 2 
 Trace5080130 
 ,  ,…(k > 0)
k
1
k
2 0 1 2 
solutions, for any given b1 and b2.
8 9 10 

R3  R2 0 1 2 
4. (A)
 215 650 795  0 0 0 
Given, Matrix [M] =  655 150 835 
 Rank is 2
 485 355 550 

Sum of the eigenvalues


= Trace of matrix
= 215 + 150 + 550 = 915

215 216

Solutions  Linear Algebra Vidyalankar : GATE – Engineering Mathematics

8. (C) 11. (B) Now |C| = 42(2145  600)  17. (A)


 1 tanx  4(54600 + 54600) + 45(560 + 2002) 5 1
A=  12. (A) Let A = 
  tanx 1  |C| = 42(1545)  4(0)

4 1 
|P| = (16 + 9)  (1) = 24
 1  tanx  + 45(1442) Characteristic equations is 2  6 + 9
AT =  1  4  3i
1  1 i 
 tan x P = 1
adj(P) = 64890 + 64890 = 0 = 0   = 3, 3
|P | 24  i 4  3i
1 1 Eigen value 3 has multiplicity 2.
Now A = (Adj(A))
|A| 13. 5.0 to 5.0 15. 2.9 to 3.1 Eigen vectors corresponding to  = 3

1  1  tanx  Since one eigenvalue of M is 2 sum of diagonal elements is (A  3I) X = 0


A= 
sec 2 x  tan x 1   23  4(2)2 + a(2) + 30 = 0 = sum of eigen values  5  3 1   x   0 
   
T
 |A .A | = 1 1
 a = 11  a + b + 7 = 14  4 1 3   y   0 

 Characteristic polynomial is a+b=7  2 1  x   0 


9. (A)    
3  42  11 + 30 = 0 (a  b)2 = (a + b)2  4ab = 49  40 = 9  4 2   4   0 
Given systems
(  2) (  5) ( + 3) = 0  |a  b| = 3  2 1  x   0 
3x1 + 2x2 = c1 R2  R2  2R1     
  = 2, 5, 3 0 0  4 0
4x1 + x2 = c2
Largest absolute value of '' is 5 16. (B) e(A) = 1
 3 2   x1   c1 
Matrix Form is      1 2 3 : a  Number of linearly independent
 4 1   x 2  c 2  14. 0 [A : B] =  2 3 3 : b  eigenvectors corresponding to
AX = B The given matrix is  5 9 6 : c 
eigenvalue  = 3 is n  r = 2  1 = 1
Characteristic equations of above  3 4 45  R2  R2  2R1
where n = no. of unknowns,
system is A =  7 9 105   1 2 3 : a 
 [A : B] =  0 1 9 : b  2a  r = rank of (A  I)
|A  I| = 0 13 2 195 
 5 9 6 : c   One linearly independent
3 2 Now according to question
  0 R3  R3  5R1 eigenvector exists corresponding
4 1  (i) R2  R2 + R3 (Adding third row to  1 2 3 : a 
to  = 3
By expanding 2  4  5 = 0 the second row)  [A : B] =  0 1 9 : b  2a 
 0 1 9 : c  5a  18. (B)
 3 4 45 
10. 0.95 to 1.05 R3  R3  R2
 B =  20 11 300  5 3
Matrix A has zero as an eigen value if  1 2 3 : a  |A  I| = 0  =0
13 2 195  1 3
 [A : B] =  0 1 9 : b  2a 
|A| = 0
(ii) C1  C1  C3 (Subtracting third  0 0 0 : c  b  3a   (5  ) (3  ) = 3 = 0
 3[(63 + 7x) + 52]  2[(81 + 9x)
column from first column)  Equation are consistent if  2  8 + 15  3 = 0
+ 78] + 4[36 + 42] = 0
 42 4 45  c  b  3a = 0 i.e. 3a + b  c = 0  2  8 + 12 = 0   = 2,  = 6
 3(7x  11)  2(9x  3) + (4 × 6) = 0
 C =  280 11 300  (A  2I)  X = 0
 21x  33  18x + 6 + 24 = 0  182 2 195 
At,  = 2
 3x = 3  x = 1

217 218
Solutions  Linear Algebra Vidyalankar : GATE – Engineering Mathematics

 3 3   x1   0  Therefore atleast one of the Eigen Model Solution on Assignment  4


 1 1 x   0  value is ‘0’
   2  
1. (C) c1  c1 + c2 + c3, we get
 x1 + x 2 = 0 As the choices are non negative, the
minimum Eigen value is ‘0’ 2. (B) 1    2  2
 x1 = x2
2 2
If [A] 3  4 & [B] 4  5  =  1     1
 1 
  20. (A) AB will exist, BA does not exist 1    2 1 
2
Hence the required vector is 
 1  Characteristic equation is 3. (A)
  0  2
 2  1 0 Every square, real symmetric matrix is =  0 2 1 =0
19. (A) 0  1  Hermitian and therefore all its 0 1 
0 3 4  
3 5 2   5 5 2 eigenvalues are real.
 5 12 7   as 1 +  + 2 = 0
 
C1  C3
 12 12 7   (4 + 2 + 3) = 0
4. (B)
 2 7 5   7 7 5 
 ( + 1) ( + 3) = 0 8. (C)
5. (D)
 determinant = 0,   = 0, 1, 3 are the eigenvalues. For a matrix rank is equal to no. of
[A] x ( x + 5) [B] y  (11 y)
So the matrix is singular independent vectors.
AB exists  x+5=y …..(1)
9. (A)
BA exists  x = 11  y …..(2)
 We find relation between than as
Solving (1) and (2) gives x = 3, y = 8
1x1 + 2x2 + 3x3 = 0
6. (A)
1 (3, 2, 7) + 2 (2, 4, 1)
A is singular matrix means | A | = 0
+ 3 (1, 2, 6) = 0
3x 2 2
which gives 31 + 22 + 3 = 0
  2 4x 1  =0
21 + 42  23 = 0
2 4 1  x
71 + 2 + 63 = 0
R2  R2 + R 3
which gives 1 = 1, 2 = 1,3 = 1
3x 2 2
 x1  x2  x3 = 0 is linear equation
 0 x x  = 0
2 4 1  x  They are dependent

 x(3  x) (x  3) = 0 10. (A)

 x = 0, 3 11. (B)

7. (A) By performing

1   2 a h g x 
2 ABC = [x y z]  h b f  y 
 
  1
 g f c  z 
2 1 

219 220

Solutions  Linear Algebra Vidyalankar : GATE – Engineering Mathematics

ax  hy  gz  We have Model Solution on Assignment  5


= [x y z] hx  by  fz   1 1 1   x   3 
gx  fy  cz  1. (B) R3  R3  2R1, R4 R4  R1
AX = 3 1 2   y    2  = B
2 2 2 2 4 7  z  7  As we know A1 A = I = A A1 and subsequently.
= [ax + by + cz + 2hxy
+ 2gzx + 2fyz]  B must be inverse of A R2  R2  R3, R3  R3  6R2 ,
Augmented matrix
12. (D) R4  R4 3R2 ,
 1 1 1 : 3  2. (C)
As | A | = 0 [ A B ] ~ 3 1 2 : 2  3. (A) R3 
1 1
R3, R4  R4 &
5 2
So inverse does not exist. 2 4 7 : 7 
13. (B) 4. (B) R4  R4  R3.
We reduce [A B] to Echelon form by
0 2 2 Consequently we get
applying successively. 5. (B)
Let A =  7 4 8   1 2 1 : 3
R2  R2  3R1, R3  R3  2R1
6. (A) 0 1 0 : 4 
 7 0 4 
transformations [A : B] = 
R2  R2 + R 3  p q  r s  0 0 1 : 4
 1 1 1 : 3 
PQ =     
0 2 2  q p   s r  0 0 0 : 0
 [A : B] ~ 0 2 5 : 7 
A =  0 4 4   pr  qs ps  qr  which is Echelon form with rank
0 2 5 : 13   
 7 0 4   qr  ps qs  pr  Number of nonzero rows = 3
Apply R3  R3 + R2
 | A | = 0 ….. rank < 3 1 2 1
7. (B) 0 1
0 2 
 1 1 1 : 3  0 
Now minor  [A : B] ~ 0 2 5 : 7  Matrix is involutory if it satisfies condition Also AI = 
 0 0 1
7 4  0 0 0 : 20   
that A2 = I and given matrix satisfies this
0 2 0 0 0
   = 14  0  Rank [A : B] = 3 condition.
7 4 Here rank = 3
(No. of non-zero rows in Echelon form.) 8. (A)
 rank = 2 As rank [A : B] = rank A
1 1 1  adj A
14. (C) A1 = we calculate inverse using  It has solution, which is unique
 A ~ 0 2 5  |A|
Eigen values are obtained by These equations are equivalent to
0 0 0  this formula.
|AI| =0 x + 2y  z = 3
Rank A = 2
1  0 0  y = 4
9. (B)
 2 3 1  =0 Since Rank A  Rank [A : B]
z = 4
0 2 4  Inconsistent equations. 10. (A)
which gives x = 1, y = 4, z = 4
which gives 1 = 1, 2 = 2, 3 = 5 16. (D) By writing augmented matrix
15. (A) 11. (C)
17. (B)  1 2 1 :3 
3 1 2 :1 Eigen values of given matrix = 2, 6
Given equations are AX = B 
18. (B)
It will be consistent if 2 2 3 :2  For  = 6,
 
Rank A = Rank [A : B]  1 1 1 : 1 1 101
eigen vector =  or 
 performing R2  R2  3R1 , 1 101

221 222
Solutions  Linear Algebra Vidyalankar : GATE – Engineering Mathematics

12. (B) 15. (B) Model Solution on Assignment  6


For triangular matrix eigen values are Augmented matrix
1. (B) 7. (B)
same as diagonal elements  1 2 3 : 6
[A : B] = 3 2 1 : 2 
Characteristic equation is
1 = 1, 2 = 3, 3 = 2.
4 2 1 : 7  1   2 8. (B)
  =0
13. (C) 5 4   Characteristic Equation:
By performing R2  R2  3R1 ,
1 1 1 (1  ) (4  )  10 = 0 |A  I| = 0
1
Explanation: Given matrix is, 1 1 1 R3  R3  4R1 and  R2
8 4  5 + 2  10 = 0 (1   ) 1 0
1 1 1 2  5  6 = 0   0 (1   ) 1  =0
1 2 3 : 6 
0 1 0 0 1 
1    1 1
 1 : 2  (  6) (  + 1) = 0
Now  1 1    1  0 0 6 11 : 17   1 = 6, 2 = 1  (1  )3 = 0
1 1 1    Performing R3  R3 + 6R2 1  3 + 32  3 = 0

3   3   3   2. (C)  I  3A + 3A2  A3 = 0
1 2 3 : 6 
or  1 1    1  =0 0 1 1 : 2  …. Cayley-Hamilton Theorem
 
1 1 1    0 0 5 : 5  3. (C)  I  3A + 3A.A  A2A = 0

1 1 1 we get x = y = z = 1 Post multiplying by A1,


or  3    1 1    1  0 4. (C) I.A1  3AA1 + 3AAA1  A2AA1 = 0A1
1 1 1    16. (C)  A1  3I + 3AI  A2I = 0

1 0 0 5. (B)  A1  3I + 3A  A2 = 0
or  3    1  0   0 17. (D) 5 2   A1 = A2  3A + 3I
1 0 
A=   ,| A | = 1
3 1
 2 (3  ) = 0 18. (D) adjA 9. (B)
 A1 =
Hence eigenvalues are 0, 0, 3. |A| For given matrix of rank = 1
 1 2   = 1  rank = 1
14. (A)  3 5 
  =  1 2 
Reducing to Echelon form, 1  3 5 
  10. (D)
R4  R4  R1
As | A |  0 for unit matrix
We have number of nonzero rows = 3
6. (C)  rank equal to number and order
 rank = 3
As A A1 = I
2x 0   1 0
  0 2x   0 1
    
 2x = 1
x = 1/2

223 224

Solutions  Linear Algebra Vidyalankar : GATE – Engineering Mathematics

11. (C) 15. (C) Model Solution on Assignment  7


 3 1 1 | AT A | = | I | = | AT | . | A | = 1
A = 15 6 5 
1. (C) 6. (A)
 |A|.|A| =1
  2 2  As per theorem for inverse matrix, that 8 8 2
 (|A|)2 =1 eigen values of inverse matrix are A =  4 3   2  = 0
For nontrivial solution P(A) < 3
 |A| =1 inverse of eigen values of the matrix 3 4 1 
 | A | = 0  6 +  (0 + 1) = 0
1 R1  R1  (R2 + R3)
 =6 16. (A) i.e. if  is eigen value of A, then is

A.A 1  I 1   1   1  
12. (C) eigen value of A1
 4 3   2  =0
 1
Solving characteristic equation, 2 10  . 1/ 2 a    1 0  2. (D) 3 4 1 
   0 b  0 1
| A  I | = 0 0 3   For non zero matrix rank must be 1 1 1
3 2 greater than or equal to 1, but not (1  ) 4 1   2  = 0
  =0  b
6 5    1 2a  10    1 0  equal to zero. 3 4 1  
  0 1
Gives  = 1  2 7 0 3b  
3. (B) (1  ) [(1  ) (4  ) + 2] = 0
comparing the corresponding terms, (1  ) (2  5 + 6) = 0
1 0
13. (C) we get As determinant   =1
0 1  (1  ) (  2) (  3) = 0
As determinant of A i.e. |A|  0 1 b b
b , 2a  0  2a   rank equal to no. of rows = 2 Roots of equation are 1, 2 and 3.
3 10 10
14. (A) As the eigen values of the matrix A are
b 1 4. (B)
The sum of eigen values of a matrix is  a  a  all distinct hence A is similar to diagonal
20 60 For given equations
the sum of diagonal elements matrix.
1 1 1  20 21 7  1 2 x  1 
1 + 2 = 12.  a  b     3    y   3  7. (A)
60 3 60 60 20      
Only option (A) has sum of diagonal Adj A
By using formula A1 =
17. (C) R2  R2  3R1 |A|
elements of 12.
1 2  x  1   1 1 2 
0   6   y   0 
18. (B)       We find A1 =  0 1/ 2 1/ 2 
 1 1/ 2 3 / 2 
i.e. (  6) y =0
 8. (C)
i.e. 6 =0
|AI| =0
 = 6 will give no solution to have
 0 
unique solution   6
 0  0  = 0  3 = 0
5. (D) 0 0 

Adj A   =0
As A1 =
|A|

225 226
Solutions  Linear Algebra Vidyalankar : GATE – Engineering Mathematics

x  13. (B) Model Solution on Assignment  8


Hence if x =  y  is eigen vector then, By solving,
z  1. (C) 7. (A)
 3 2 1  x  4 
AX =  X  1 1 1  y   2  , Here P diagonals matrix B by pre & As the rows are similar if common is
     
0 0   x  0   2 0 2  z  5 
post multiplication such that taken from each row
 0 0 0   y   0 
      B = P1AP 8. (A)
We get the values only one for each
0 0 0  z  0 
2. (C) Hint : AX = X
parameter, so it has unique solution.
 z=0
Hint : As per the properties of the A(AX) = A(X)
14. (A)
  0  matrices.  A2X = (AX) =  (X) = 2X
 The vectors must be 0  or   a h g  x 
0  0   x y zh b f y  3. (A) A2X = 2 X
 g f c   z  1 bc  ca  ab a(b  c) 9. (B)
x 
= ax2 + by2 + cz2 + 2hxy + 2fyz + 2gzx  = 1 bc  ca  ab b(c  a) Given vectors are
As the vectors of the form  y 
1 bc  ca  ab c(a  b)
0  i.e. XT AX gives, 1x1 + 2x2 + 3x3 =0
9. (A) ax2 + by2 + cz2 + 2hxy + 2fyz + 2gzx 1 1 a(b  c) Which gives
Hint : After taking the common from  a11 a12 a13  = (ab + bc + ca) 1 1 b(c  a) 1x1 + 2x2 + 23x3 = 0
each column, we get matrix having all  If A = a 21 a 22 a 23  1 1 c(a  b)
1 x1 + 22x2 + 33x3 =0
values = 1 and we get after reducing a 31 a 32 a 33  = (ab + bc + ca ) 0 = 0 1x1 + 32x2 + 43x3 = 0
all rows and columns to zero the Then for 31x1 + 42x2 + 93x3 = 0
4. (C)
matrix of rank 1 only ax2 + by2 + cz2 + 2hxy + 2fyz + 2gzx which on solving gives 1 = 2 = 3 = 0
As per the property of multiplication of
10. (A) a11 = a a22 = b a33 = c
matrix.  They are not linearly dependent
For characteristic equation 1
a12 = a21 = h = × (co-efficient of xy) 5. (C) 10. (C)
 h g 2
A =  h  f  1 6. (C)  1 2 1 : 6
g f  a23 = a32 = f =
2
× (co-efficient of yz) Now 2 1 2 : 6 
8 4 5 
= 3  (f2 + g2 + h2)  2fgh = 0 4 1 2     1 1 1 : 5 
1 0 5 3  1 0 2 
a31 = a13 = g = × (co-efficient of zx)   3 4 7 
11. (B) 2   Now, R3  R1
5 4 15. (B) 17 29 32  1 2 1 : 6 
By solving,   =0  2 1 2 : 6 
1 2  
Hint : The property of the unitary  4 12 11  
We get  = 1 and  = 6 matrix 0 1 0 : 1
17 24 32 
12. (C)   4 12 11
16. (A)   R2  2R1
Hint : By reducing the given matrix to
17. (A) 8x  3y 6z 3z  1 2 1 : 6 
echelon form, we come to know that,  0 3 0 : 6 
18. (B)  4 12 26x  5y    
the rank of given matrix is 3.
0 1 0 : 1
  x = 1, y = 3, z = 4

227 228

Solutions  Linear Algebra Vidyalankar : GATE – Engineering Mathematics

R2 + 3R3 14. (C) Model Solution on Assignment  9


1 2 1 : 6  We have
 0 0 0 : 3  1. (B) A + A
  1 a a2 a3 1 a a2 bcd
0 1 0 : 1 As per the definition of unitary square 3   1 2
1 0 5 3 4
1 b b2 b3 1 b b2 cda  2 1 6
Now the left part of dashed line have =      matrix. 1 0 1 2 4 
1 c c2 c3
1 c c2 dab =  +
3 2 7 1 5 6 7 2 
rank 2 and right part have all elements    
1 d d2 d3 1 d d2 abc  4 4 2 0  3 1 1 0
present so, it does not have solutions. 2. (B)
= 1 + 2 (say) As per the definition of Hermitian
11. (C)  2 2 8 7 
Multiplying R1, R2, R3, R4 of 2 by matrix  2 2 8 3 
Hint : If the matrix is in the form of
a, b, c, d =  
upper / lower triangular matrix , then  8 8 14 1
 
the diagonal elements are the eigen a a2 a3 abcd 3. (C)  7 3 1 0 
2
values of the element. 1 b b b3 abcd
Now 2 =  
abcd c c 2 c3 abcd 4. (A) 7. (C)
12. (C)
d d2 d3 abcd a  ic b  id
 2 1  2i A=   =1
1  2i 2  have eigen values as 5. (C) b  id a  ic
  a a2 a3 1
cos  sin 
2   1  2i b b2 b3 1   = cos2  + sin2  = 1  a2 + b 2 + c 2 + d 2 = 1
  =0 =    sin  cos 
1  2i 2   c c2 c3 1
T
(  2) ( + 2)  (1 + 2i) (1  2i) = 0 d d2 d3 1  cos   sin  
& Adjoint of A =   8. (A)
4
  4  (1 + 4) = 0   sin  cos  
1 a a2 a3 a h g  x 
2  9 = 0 1 b b2 b3
cos   sin  
=  x y z  h b f  y 
=   =  1 
 The eigen values are  = 3,  = 3  sin  cos    g f c   z 
1 c c2 c3
13. (C) 1 d d 2
d 3 cos   sin   = ax2 + by2 + cz2 + 2hxy + 2fyz + 2gzx
 A1 =  
 sin  cos   i.e. XT AX gives,
2 2 0 6   = 1 + (1) = 0
4 2 0 2 ax2 + by2 + cz2 + 2hxy + 2fyz + 2gzx
  =0
1 1 0 3 15. (C) 6. (B)  a11 a12 a13 
1 2 1 2 For triangular matrix eigen values are 1 0 5 3  If A = a 21 a 22 a 23 
same as major principal diagonal  2 1 6 1 a 31 a 32 a 33 
Rank < 4
A=  &
Now minor of order 3 element i.e. 18, 7, 8. 3 2 7 1
  Then for
 4 4 2 0
4 2 0 16. (B) ax2 + by2 + cz2 + 2hxy + 2fyz + 2gzx
1 1 0 = 1(4  2) = 6  0  1 2 3 4 a11 = a a22 = b a33 = c
17. (B) 0 1
1 2 1 2 4 
18. (A) A =  1
5 6 7 2  a12 = a21 = h = × (co-efficient of xy)
Rank = 3   2
3 1 1 0


229 230
Solutions  Linear Algebra Vidyalankar : GATE – Engineering Mathematics

1 11. (C) As cos2  sin2  = cos 2 a  (p  1)d a  (q  1)d a  (r  1)d


a23 = a32 = f = × (co-efficient of yz)
2 | A | 33 = 5 & 2 sin  cos = sin 2  p q r
1 1 1 1
a31 = a13 = g = × (co-efficient of zx)  Rank of A is 3. Similarly , we get
2 R2  R2  R3
 cosn sinn 
Extension of this method 12. (C) An =   a  (p  1)d a  (q  1)d a  (r  1)d
  sinn cosn 
Hint : Eigen values are the values of  p 1 q 1 r 1
9. (C) 1 1 1
diagonal for diagonal matrix. So
As the number of equations & 15. (B)
a  (p  1)d a  (q  1)d a  (r  1)d
reduce given matrix to diagonal matrix. 1
unknowns are same and they are As (ABC)T = CT BT AT  (p  1)d (q  1)d (r  1)d
Or sum of the eigen values of a matrix d
equal to zero, so their values are also  (B) is wrong. 1 1 1
= trace of the matrix
zero. …..(d0 )

Or | A | ≠ 0 13. (C) 16. (C) R1  R1  R2


 the system will have trivial solution. Characteristic Equation: Tp Tq Tr a a a
1
|A  I| = 0 Let   p q r  = (p  1)d (q  1)d (r  1)d
10. (B) d
1 1 1 1 1 1
(3  ) 1
a h g  x     =0
1 (2  ) Here, Tp, Tq and Tr are pth qth and rth 1 1 1
x y z  h b f  y  a
term of an AP. = (p  1)d (q  1)d (r  1)d
 g f c   z   (3  ) (2  )  (1) = 0 d
Let the first term of the AP be a and 1 1 1
= ax2 + by2 + cz2 + 2hxy + 2fyz + 2gzx  6  5 + 2 + 1 = 0
2 common difference of the AP be d.  =0
i.e. XT AX gives,    5 + 7 = 0
Then
2 2 2
ax + by + cz + 2hxy + 2fyz + 2gzx Cayley-Hamilton Theorem,
Tp = a +(p  1)d 17. (D)
 a11 a12 a13  A2  5A + 7I = 0
Tq = a +(q  1)d
 If A = a 21 a 22 a 23 
a 31 a 32 14. (C) Tr = a + (r  1)d 18. (D)
a 33 
 cos  sin  
Then for A=  
  sin  cos   
ax2 + by2 + cz2 + 2hxy + 2fyz + 2gzx
a11 = a a22 = b a33 = c  cos  sin    cos  sin  
A2 =   
  sin  cos     sin  cos  
1
a12 = a21 = h = × (co-efficient of xy)
2  2 2 cos  sin  
cos   sin   sin  cos  
1 = 
a23 = a32 = f = × (co-efficient of yz)   sin  cos  
2 cos 2   sin2  

1   sin  cos  
a31 = a13 = g = × (co-efficient of zx)
2  cos 2 sin2 
=  
  sin2 cos 2 

231 232

Solutions  Linear Algebra Vidyalankar : GATE – Engineering Mathematics

Model Solution on Assignment  10 9. (A) 6   2 0


a h g  x  (2  )  4 4   0 = 0
1. (D) 6. (C)
 x y zh b f y  2 1 1
As R2 = (2)  R1 By performing R4  R4  R3  R2  R1  g f c   z 
(2  ) . [(6  ) . (4  )  8] = 0
 | A| 3  3 = 0 & R2 R2  2R1, R3  R3  3R1.
= ax2 + by2 + cz2 + 2hxy + 2fyz + 2gzx [(2  ) (2  10 + 16) ] = 0
 rank  0 Sequentially we get
i.e. XT AX gives, (2  ) (  2) (  8) = 0
3 2 1 2 3 0
| A | 22 =   =  12 + 12 = 0 0 0 3 ax2 + by2 + cz2 + 2hxy + 2fyz + 2gzx
6 4 2    = 2, 2, 8
A~   a11 a12 a13 
0 4 8 3 Or sum of the eigen values of a matrix

4 18
 = 144  144 = 0
   If A = a 21 a 22 a 23 
0 0 0 0 = trace of the matrix
8 36
 a 31 a 32 a 33 
Now | A | 4  4 = 0 … As E transform
| A | 22 = 0 Then for
does not change rank of matrix 11. (C)
 | A | 11 = | 3 | = 3 ax2 + by2 + cz2 + 2hxy + 2fyz + 2gzx As R4 = 3R1
 rank  4
| A | 11  0 a11 = a a22 = b a33 = c  | A |44 = 0
1 2 3
 rank = 1 1
Now 0 0 3 = 12 i.e.  0 a12 = a21 = h = × (co-efficient of xy) 1 4 8
Or R2 = 2R1 2
0 4 8 Now 0 0 3
R3 = 4R1 1 4 2 3
 Rank = 3 a23 = a32 = f = × (co-efficient of yz)
2
 rank = 1 = 1(0  6)  4(0  12) + 8(0)
7. (A) 1
a31 = a13 = g = × (co-efficient of zx) = 6 + 48 = 42 i.e.  0
2. (B) A matrix is said to be orthogonal if 2
 rank = 3
This is like a property of triangular AT A = I
10. (B)
matrix.  1 1   2 1 
 0   0  Characteristic equation of A is
 2 2  6 3 12. (B)
3. (B)  2 1 1   1 1 1  | A  I | = 0
    | A | = 37
 6 6 6  2 6 3 6 2 2
4. (C)  1 1 1   1 1 1   2 3 1  = 0  1 4 6 
   
The diagonal matrix is  3 3 3  2 6 3 2 1 3 Adj. A =  4 16 13 
 6 13 1 
1 0 0   x1   1 0 0 by C3  C3 + C2
= [x1 …….xn]  0  2 .
 
0  :  = 0 1 0  = I  1 4 6 
. .    6 2 1 
4 16 13 
 0 0 n   x n   0 0 1 0 A1 =
 2 3   2   = 0 37 
 6 13 1 
= x12 + 2 x22 + …… + nxn2  A is orthogonal
2 1 2  
13. (C)
8. (A)
5. (B) R2  R2  R3 As A is not symmetric matrix.
For triangular matrix the eigen values
Q is obtained by interchanging C1 and C2
are same as principal diagonal
| Q | =  | P | = 8
elements.

233 234
Solutions  Linear Algebra Vidyalankar : GATE – Engineering Mathematics

14. (C) x x x Model Solution on Assignment  11


1 2 3 c bx a 0
b a cx 1. (C) 8. (C)
4 5 8
3 2 1 7 0 
1 1 1 2. (C) X+Y=   ….(i)
x c bx a 0 2 5 
= 1(5  16)  2(4  24) + 3(8  15)
3. (C)
b a cx 3 0 
= 11 + 40  21 XY=  ….(ii)

 x = 0 is one root of the given 4. (D) 0 3 
= 8 i.e.  0
equation. Addition of (i) and (ii) gives
 rank = 3 5. (C)
10 0 
2 3 2X =  
17. (D)   = 12  12 = 0  2 8
15. (D) 4 6
As R3 = R1 ax b c 5 0 
2 3   X=  
0 bx a 0   4 6  is not invertible.  1 4
 Value of determinant = 0  
0 0 cx
7 0  5 0 
 Y=    
bx a b c 6. (C) 2 5   1 4 
16. (C)
 (a  x) 0
0 cx 0 cx
ax c b 7. (C) 2 0 
 Y=  
c bx a 0 b c A = (aij) is 3  2 matrix  1 1
0 0
b a cx bx a
Elements are given as
R1  R1 + (R2 + R3)  (a  x) (b  x)(c  x)  0   0  0  0 aij = 2i  j i>j 9. (A)
ax cb ba = 2j  I ij Check with options
 (a  x)(b  x)(c  x) = 0
c  b x  a c  x
 x = a, b, c are the roots. Elements of A will be a11, a12, a21, a22,
0 a31, a32 10. (B)
c bx a
Elements having i > j = a21, a31, a32  1 2 3 
18. (C)  
b a cx  a21 = 2(2)  (1) = 3 Let A  0 1 2 
 0 0 1 
ab ab ab a31 = 2(3)  (1) = 5
c  x c  x c  x a32 = 2(3)  2 = 4 1 2 3
A  0 1 2
0 Elements having i  j ; a11, a12, a22
c bx a 0 0 1
 a11 = 2(1)  (1) = 1
b a cx 1 2 2 3 2 3
a12 = 2(2)  (1) = 3 =1 0 0
0 1 0 1 1 2
 a+b+c=0 a22 = 2(2)  (2) = 2
 A  1 0
 1 3
   A1 exists
 Matrix A  3 2 
 5 4 

235 236

Solutions  Linear Algebra Vidyalankar : GATE – Engineering Mathematics

Cofactors :  1 2 7  R2  R2  R1 ; R3  R3 R1 = 1  cos2(  )  cos2(  )  cos2(  )


1
1 2  0 1 2  .....( A  1) + 2{cos(  ) cos(  )} cos(  )
A11  (1)11 1 2 logam logam
0 1  0 0 1  logam
 logk 2logk = 1  cos2(  )  cos2(  )
0 2  1 2 7   cos2( ) + cos{(  )  (  )}
A12  (1)1 2 0 =  
0 1  A 1   0 1 2  3logk 3logk 3logk + cos{(  ) + (  )} cos(  )
 0 0 1 
1 3 0 1 = 1  cos2(  )  cos2(  )  cos2(  )
A13  (1) 0 6logk 6logk 6logk
0 0 + cos(  ) cos(  2 + )
11. (D) log am logam
2 3 log am + cos(  ) cos (  )
A 21  (1)2 1  2  logk 2logk
0 1
= 1  cos2 (  )  cos2 (   )
12. (B) = 36  
1 3
cos ( )(2 ) 
A 22  (1)2  2 1 logk logk logk
0 1  
13. (C) cos (  )(2 )
1 2 logk logk logk 
A 23  (1)2  3 0 a1, a2, ……. are in GP 2
0 0 =0
am 1 am  2 a m  3 = 1  cos 2 (  )  cos 2 (   )
2 3         k(say)
A 31  (1)3 1 7 am a m 1 a m  2 cos(2  2)cos(2 2)
1 2 14. (D) 
am+1 = amk 2
1 3 Given determinant, 
A 32  (1)3  2  2 am+2 = am+1k = amk2 = 1cos 2 (  )cos 2 ( )
0 2 3 1 cos(  ) cos(   )
am+3 = am+2k = amk cos2(  )cos 2(   )
1 2 = cos(  ) 1 cos(   ) 
A 33  (1)3  3 1 and so on 2
0 1 cos(   ) cos(   ) 1
log am logam 1 logam  2 22 cos2 (  )2 cos2 ( ) 
 1 0 0  = logam  3 logam  4 logam  5  1 cos(  ) cos(   )  
2 cos (  )1
2
  
 Cofactor Matrix =  2 1 0  logam  6 logam  7 logam  8 = cos(  ) 1 cos(   )  
2 cos (   )1
2

 7 2 1 cos(   ) cos(   ) 1 = 


log am log am 2
logam .... Since cos() = cos
Adj A = Cofactor matrix 
T
 logk 2logk =0
= 1(1  cos2(  ))
T
 1 0 0 logam logam log am
  =   cos(  )  15. (B)
=  2 1 0   cos(  )  
3logk 4logk 5logk cos(   ) cos(   )
 7 2 1 0 2 4
cos(  ) cos(   ) A   1 1 2 
 1 2 7  logam logam log am + cos(  )  
  6logk 7logk 8logk cos(   )   2 0 5 
Adj A = 0 1 2 
 0 0 1  = 1  cos2(  )  cos2(  ) Characteristic Equation of A is
+ cos(  ) cos(  ) cos(  ) A  I  0
adjA
 A 1 
A + cos(  ) cos(  ) cos(  )
 cos2(  )

237 238
Solutions  Linear Algebra Vidyalankar : GATE – Engineering Mathematics

0 2 4   = 2, 2, 3 Answer Key on Test Paper  1


 1 1  2  0 For  = 2 1. (B) 2. (C) 3. (C) 4. (B)
2 0 5
(3  2) 10 5   x1  0  5. (C) 6. (D) 7. (C) 8. (B)
 2 (3  2) 4  x 2  =  0  9. (A) 10. (C) 11. (D) 12. (A)
 2 4 
1 1  2  0  3 5 (7  2)  x 3   0  13. (B) 14. (C) 15. (C)
i.e.
2 0 5
 1 10 5   x1  0 
 2 5 4  x  =  0  Answer Key on Test Paper  2
1  2 1 2     2  
  2 1. (B) 2. (B) 3. (A) 4. (A)
0 5 2 5    3 5 5   x 3   0 
5. (B) 6. (B) 7. (D) 8. (B)
1 1  5 9. (B) 10. (D) 11. (C) 12. (B)
4 0  2  satisfies the above equation.
2 0 13. (C) 14. (A) 15. (A)
 
 5 
  (1  )(5  )  0  2  5    4
Answer Key on Test Paper  3
4  0  2(1   )  0 1. (C) 2. (C) 3. (D) 4. (A)
17. (D)
5. (C) 6. (A) 7. (B) 8. (D)
  5  6     2 1   
2
2 3
2 2 2 9. (D) 10. (C) 11. (C) 12. (B)
4  2  2   0 1 = 3 32 33  13. (B) 14. (C) 15. (A)
4 42 43
 5  6 2   3  2  2  8  8  0
1 2 22
Answer Key on Test Paper  4
 3 + 62  1 + 6 = 0 1. (B) 2. (C) 3. (C) 4. (C)
= 234 1 3 3  2

i.e. 3  62 + 1  6 = 0 1 4 42 5. (B) 6. (A) 7. (A) 8. (C)


9. (B) 10. (B) 11. (A) 12. (A)
1 2 4 13. (B) 14. (A) 15. (B)
16. (C)
= 24 1 3 9 
Characteristic equation: |A  I| = 0 1 4 16 Answer Key on Test Paper  5
(3  ) 10 5
1 = 24 1. (A) 2. (B) 3. (B) 4. (B)
  2 (3   ) 4  = 0
5. (C) 6. (C) 7. (B) 8. (A)
3 5 (7   ) 9. (B) 10. (D) 11. (B) 12. (A)
18. (D)
gives 3  72 + 16  12 = 0 13. (B) 14. (C) 15. (D)

Answer Key on Test Paper  6



1. (B) 2. (B) 3. (B) 4. (A)
5. (A) 6. (C) 7. (C) 8. (B)
9. (C) 10. (D) 11. (A) 12. (B)
13. (B) 14. (B) 15. (C)



239 240

Solutions  Linear Algebra Vidyalankar : GATE – Engineering Mathematics

Model Solution on Test Paper  1 9. (A) 10. (C)


We have equation as By taking product
1. (B) 31  52 3 = 0
1 1 1   x  9  c b   a ab ac 
2
0
41 + 22 + 33 = 0 2 5 7   y   52   
      AB =  c 0 a  ab b2 bc 
2. (C) 21 + 22 + 23 = 0  2 1 1 z  0   b a 0   ac bc c 2 
 
x+3=0  x = 3 By solving these equations we get AX = B
similarly y, z & a can be calculated. 1 = 1, 2 = 1, 3 = 2 0 0 0 
Now augmented matrix
= 0 0 0 
 Equation is x1 + x2  2x3 = 0 1 1 1 : 9 
0 0 0 
3. (C) [A : B] =  2 5 7 : 52 
11. (D)
8. (B)  2 1 1 : 0 
4. (B) f(x) = x2  5x + 6
Suppose these points are collinear on We reduce it in Echelon form
f(A) = A2  5A + 6 I
line ax + by + c = 0 R2  R2  2R1 R3  R3  2R1
5. (C) Then ax1 + by1 + c = 0 ……(i)  3 1  3 1  3 1
1 1 1 : 9  =     5  1 2 
Characteristic equation is ax2 + by2 + c = 0 …….(ii)  0 3 5 : 34   1 2   1 2   
 
4 1 ax3 + by3 + c = 0 ……(iii)  0 1 3 : 18   1 0
  =0 6  
1 4 Eliminate a, b, c, between (i),  0 1
R2
R2 
(4)2  1 =0 (ii) and (iii) 3 which gives

16  8 + 2  1 = 0 x1 y1 1  1 0 
1 1 1 : 9  f(A) =  
2  8 + 15 = 0 x 2 y 2 1 = 0  0 1 5 / 3 : 34 / 3   0 1
 
x3 y3 1
(  5) (  3) = 0  0 1 3 : 18 
 1 = 3, 3 = 5 x1 y1 1 R3  R3 + R 2 12. (A)
Thus rank of matrix x 2 y 2 1 is 1 1 1 : 9  A matrix is said to be idempotent if
Or following property can be used
x3 y3 1  0 1 5 / 3 : 34 / 3  A2 = A
Sum of the eigen values = trace  
less than 3  0 0 4 / 3 : 20 / 3 
 2 2 4   2 2 4 
Here,  1 3 4  1 3 4 
Conversely if rank of matrix A is less  Rank A = Rank [A : B] = 3    
6. (D)
than 3, then  1 2 3   1 2 3 
 Equations are consistent.
7. (C) x1 y1 1  2 2 4 
x 2 y 2 1 = 0 gives 1 3 4 
Relationship obtained by using
x3 y3 1  1 2 3 
1 x 1 + 2 x 2 + 3 x 3 = 0
 Area of triangle with vertices which is same as A.
 1(1, 3, 4, 2) + 2(3, 5, 2, 2)
(x1 , y1) (x2, y2) (x3, y3) is equal to 0  It is idempotent
+ 3(2, 1, 3, 2) = 0
 They are collinear points.
gives 4 equations
i.e. 1 + 32 + 23 = 0

241 242
Solutions  Linear Algebra Vidyalankar : GATE – Engineering Mathematics

13. (B) 14. (C) Model Solution on Test Paper  2


cos   sin  0  We have (A  I ) = 0
A =  sin  cos  0  1  1 1 1. (B) 6. (B)
 0 0 1 1 1  1 = 0 The given system of equation is
2. (B)
 |A| = 1 1 1 1  equivalent to the single matrix

1 adj A  (1  ) [(1  )2 1] 1[(1  )  1] 3. (A) equation


 A =
|A| + 1[1  (1  )] = 0 2 6 0  x 
4. (A)
Adj A = [cofactor matrix A]T  (1  ) [(1  )2 1] 1[(1  )  1] AX = 6 20 6  y 
2 3 4  0 1 2  0 6 18  z 
cos   sin  0  + 1[1  (1  ) ] = 0 2A + 3B = 2   +3 2 4 6 
 1 2 5   
=  sin  cos  0   (1  ) (2  2) + 2 = 0
 11
4 9 12    3  = B
 0 0 1 2
  (3  ) = 0 =  
8 8 28   1 
 cos  sin  0    = 0, 0, 3
We reduce this matrix into triangular
=   sin  cos  0  5. (B)
matrix by sequentially performing
 0 0 1 1 a bc
15. (C)
 = 1 b ca R2  R2  3R1,
 cos  sin  0  1
 A 1
=   sin  cos  0  We have C1 
8
C1 1 c ab R3  R33R2, we get

 0 0 1 Performing R2  R2  R1 , 2 6 0   x   11


 1 1 3 6 0 2 6   y   30 
 0 3 2 2 R3  R3  R1      
  0 0 0  z   91
 1 1 3 4  1 a bc
0 b  a ca  bc =0 We have 0x + 0y + 0z = 91, this
Now R3  R3 + R1
0 c  a ab  bc shows systems is not consistent
1 1 3 6 
0 3 2 2  which is in Echelon  (b  a) (ab  bc) 7. (D)
 
Using
0 0 0 10   (c  a) (ca  bc) = 0
(b  a). b(a  c) cos  cos   sin  sin  = cos ( + )
form so rank is equal to no. of nonzero
rows  (a  c) c(a  b) = 0 cos  sin  + sin  cos  = sin ( + )

Rank = 3 (a  c) (b  a) (b  c) = 0 cos  cos   sin  sin  = cos ( + )

 (a  b) is a factor of   cos(  ) sin(  ) 


we get  
  sin(  ) cos(  )

8. (B)
As A3 = A . A . A = 0
where A is given matrix
 It is nilpotent matrix

243 244

Solutions  Linear Algebra Vidyalankar : GATE – Engineering Mathematics

9. (B) 2 3 5   1 9 17  Model Solution on Test Paper  3


R3  R3 + R1 , R2  R2 + 2R1 gives = 0 3 1  0 4 5 
0 0 4  0 0 9  1. (C) 10. (C)
 1 2 3 
0 0 5  As for triangular matrix eigen values The given equation in Matrix Form:
  3 12 22 
= 0 7 6 
are principal diagonal elements 2 3 5   x  9 
0 0 10 
7 3 2  y  =  8 
0 0 13      
The determinant of above matrix is 0
2. (C) 2 3    z   
Checking for non-zero minor of order 2 Now for triangular matrix eigen values
As per definition of rank of matrix
AX = B
1 2 are same as principal diagonal
  =0 2 3 5 : 9 
2 4 element. 3. (D)
[A : B] = 7 3 2 : 8 
2 3  1 = 3, 2 = 7, 3 = 13 As columns of A  rows of B
  = 2  12 = 10  0 2 3  :  
4 1 are eigen values. So product is not possible
R3 R3 R1
 rank = 2 13. (C)
4. (A) 2 3 5 : 9 
10. (D) cos   sin   a 0  7 3 2 : 8 
Let A =   B = 0 b  
 sin  cos     5. (C)  0 0 (  5) : (  9)
11. (C)
Here AB = BA if a = b As these are properties of skew
As it is not a singular matrix If rank(A)  rank (A : B), system has no
symmetric matrix.
solution.
14. (A)
12. (B)
R4 = 3R1 6. (A)   = 5,   9
1 3 5 
Hence rank < 4 and 1 1
A = 0 2 1 e.g. A =   for 2  2 matrix 11. (C)
1 4 8 1 1
0 0 3  Just by the definition of the rank of
0 0 3 = 48  0 Rank of A = 1
 I + A + A2 matrix we find that the rank of given
4 2 3
 1 0 0  1 3 5  7. (B) matrix A is 3.
 Rank = 3
= 0 1 0   0 2 1
0 0 1 0 0 3  12. (B)
15. (A) 8. (D)
Characteristic equation of A is
1 3 5  1 3 5  As per property of idempotent matrix if
|A  I| = 0
 0 2 10 2 1 A & B are idempotent implies AB = BA.
0 0 3  0 0 3  1  0 2
9. (D)  0 2 1  =0
The determinant of 3  3 element of 2 0 3
 given matrix is not equal to zero or  (1  ) [(2  ) (3  )  0]
when reduced to echelon form, it gives + 2  2 (2  ) = 0
3 rowed matrix.  3  62 + 7 + 2 = 0

245 246
Solutions  Linear Algebra Vidyalankar : GATE – Engineering Mathematics

13. (B) 15. (A) Model Solution on Test Paper  4


By solving the above equations as R3  R3  (R1 + R2), we get
1. (B) 1
follows x2 2x  3 3x  4 a12 = a21 = h = × (co-efficient of xy)
As per the property of the skew 2
2 1 2 : 1 2x  3 3x  4 4x  5 = 0
0 1 3x  8 Hermitian matrix 1
1 1 0 : 0 a23 = a32 = f = × (co-efficient of yz)
2
1 k 6 : 3
By R2  R2  R1 & R1  R1 + R3 2. (C)
1
 That is, after reducing to Echelon a31 = a13 = g = × (co-efficient of zx)
x  2 2x  4 6x  12 As per the property of the eigen values 2
form, if we want unique solution, x  1 x 1 x 1  = 0 for orthogonal matrices. i.e. for Extension of this method
then K  2 0 1 3x  8 orthogonal matrix, the eigen value is
reciprocal of the original eigen value. 6. (A)
14. (C) 1 2 6
(x + 1) (x + 2) 1 1 1  =0 When reduced to Echelon form, non-
Hint : By just the definition 3. (C)
0 1 3x  8 zero rows obtained is 1.
i.e. transpose of all elements' cofactor Hint : The transpose of the matrix Or R2 = 6R1 & R3 = 5R1
R1  R1  R2
matrix. does not change the rank of the
OR [cofactor Matrix ]T 0 1 5 7. (C)
matrix.
(x + 1) (x + 2) 1 1 1  =0
0 1 3x  8 4. (C) 8. (C)

Now, As per the defitnition of nilpotent  2 1 1 


(x +1) (x + 2) [(3x + 8  5)] = 0 matrix for index K. A =  1 2 1
 1 1 2 
3 (x + 1) (x + 2) (x + 1) = 0
5. (B)
Characteristic equation of matrix
 x = 1, 1, 2
a h g  x  2 1 1

 x y zh b f  y   1 2 1  = 0
 g f c   z  1 1 2
= ax2 + by2 + cz2 + 2hxy + 2fyz + 2gzx
 3  62 + 9 + 4 = 0
i.e. XT AX gives,
ax2 + by2 + cz2 + 2hxy + 2fyz + 2gzx 9. (B)
 a11 a12 a13 
 If A =  a 21 a 22 a 23  10. (B)

 a 31 a 32 a 33 
11. (A)
Then for
2 2 2
Inverse of matrix A is,
ax + by + cz + 2hxy + 2fyz + 2gzx
1
a11 = a a22 = b a33 = c A1 =  Adj A
|A|

247 248

Solutions  Linear Algebra Vidyalankar : GATE – Engineering Mathematics

12. (A) 13. (B) Model Solution on Test Paper  5


0 is an eigen value of A   = 0 R2 = 3R1
1. (A) 8. (A)
satisfies the equation | A  I | =0 As one of the row is repeated, so the
For symmetric matrix A = AT For triangular matrix eigen values are
 | A | = 0  A is singular no. of rows for considering rank
same as major principal diagonal
Conversely  | A | = 0 reduces to 2. Now, for given row 2. (B)
elements.
matrix, determinant is non zero and We have, a + 3 = 2a + 1
 = 0 satisfies the equation
hence the matrix is 2.  a = 2 9. (B)
|AI|=0
 0 is eigen value of A Similarly, b = 1, 2 1x1 + 2x2 + 3x3 + 4x4 = 0
14. (A)
1(1, 2, 4) + 2(2, 1, 3) + 3 (0, 1, 2)
3. (B)
15. (B)  ab b2  + 4 (3, 7, 2) = 0
A=  
2 1 + 22  34 = 0
 a ab 
21  2 + 3 + 74 = 0
  ab b2   ab b2 
A.A=     41 + 32 + 23 + 24 = 0
2
 a ab   a 2 ab 
By solving these equations we get
0 0 
=   1 = 9 , 2 = 12, 3 = 5, 4 = 5
0 0 
2  relation is 9x1  12x2 + 5x3  5x4 = 0
A =0  K=2
 Index = 2 10. (D)

4. (B)
11. (B)
It is the definition of eigen values.
|A|=1
5. (C) T
 a  ib c  id 
For diagonal matrix, eigen values are adj A =  
 c  id a  ib 
different.
Adj A
A1 =
6. (C) |A|
7. (B) a  ib c  id
=  
C2 = 2C1  c  id a  ib 
 | A |3  3 = 0
 rank < 3
minor of order 2
3 2
  = 12  4 = 8  0
2 4

 rank = 2

249 250
Solutions  Linear Algebra Vidyalankar : GATE – Engineering Mathematics

12. (A) 14. (C) Model Solution on Test Paper  6


Idempotent matrix is one which determinant
1. (B) 6. (C)
satisfies A2 = A. But given matrix not 6 0 0
satisfy this relation 1 8 2 0 = 1[6 [80] ] = 48 | A | = a [b (d   )] = abd  As per theorem,

 It is not a idempotent matrix 1 4 4 1 a 1 1 1


2. (B) 1 1 b 1 1
 
13. (B) 1 1 1 c 1
15. (D) As [A] 3  3 [B] 3  2
| A | 33= 0 1 1 1 1 d
 [B] 3  2 [A] 3  3
 Rank < 3 BA is not possible as  1 1 1 1
= abcd  1     
 a b c d
1 1 columns of B  rows of A
Minor of order 2,   = 2
0 2 Now here a = b = c = d = 4

i.e.  0 3. (B)  = (4)4  (0) = 0

 rank = 2 A is obtained by substituting i by i


7. (C)

4. (A) By R4  R4  R3  R2  R1 , R1  R2,

3 1 2 R2  R2  2R1, R3  R3  3R1
| A | 3  3 = 6 2 4 = 0  1 1 2 4 
3 1 2 0 5 3 7 
Subsequently,  
 0 4 9 10 
3 1  
| A | 22 =   =0 0 0 0 0 
6 2
1 1 2
Similarly other minors of order 2 are 0
We see that 0 5 3  = 33  0
 | A | 11 = | 3 |  0
0 4 9
 rank = 1
 Rank = 3

5. (A) 8. (B)
Characteristic equation are Hint : Adjoint A = [ Cofactor matrix A]T
| A  I| = 0
9. (C)
5 3
  =0 Characteristic roots are obtained by
3 3  
| A  I | = 0
(5  ) (3  )  9 = 0
2
 1 2
15 +  + 2  9 = 0
1  1 = 0
2
  2  24 = 0 2 1 
  = 6, 4

251 252

Solutions  Linear Algebra Vidyalankar : GATE – Engineering Mathematics

2
 ( 1) 1( + 2) + 2(1 + 2) = 0 14. (B) a(1bcbc)  abcabc2(1abc)
  =
3 + 6  4 = 0  3 4  b(1acac)  1abc(1abc)
2 1 2  c(1a bab) 
AB =  1 1  
 3 + 02  6 + 4 = 0 1 3 4  =   ….From (i)
 2 0   (1a)(1bcbc)
abcabc22abc
By synthetic division =
aabacabcbabbc  1abc1abc
2 1 0 6 4  2 9 10   
  3 4 6  = 
abccacbcabc  2  a  b  c  abc
0 2 4 4 1bcbcaabacabc =
 4 2 4  2  a  b  c  abc
1 2 2 0
abcabc  = 1
 2 1  
 =2 B A =  1 3  
 3 1 2
= 2(abbcacabc)
4 1 0  1 a  b  c(abbcacabc)
2 + 2  2 = 0  2 4  

  = 1  3  2 3 4
 roots are 2, 1  3   9 4 2  
 10 6 4 

10. (D)
As for triangular matrix eigen values 15. (C)
are same as principal diagonal The given equations in matrix form
element.
1 a a  x  0 
b 1 b  y  =  0 
11. (A)     
 c c 1  z   0 

12. (B) For non trivial solution,


As C4 = 2C1 |A| = 0
 | A | 44 = 0 1 a a
 rank < 4  b 1 b = 0
c c 1
1 2 1
Minor of order 3 1 3 1 1(1  bc)  a(b  bc) + a(bc  c) = 0
2 4 3  1  bc  ab + abc + abc  ac = 0
= 1( 9  4)  2(3  2) + 1(4  6)  ab + bc + ac  abc = 1 + abc ….(i)
=1 0 a b c
 
 rank = 3 1 a 1 b 1 c

a(1  b)(1  c)b(1a)(1c) 


 
c(1a)(1b)
13. (B) =  
(1a)(1b)(1c)

253 254
Vidyalankar : GATE – Engineering Mathematics

Solutions  Calculus Answer Key on Assignment  4


1. (B) 2. (D) 3. (B) 4. (D)
Answer Key on Assignment  1
5. (D) 6. (B) 7. (A) 8. (A)
1. (A) 2. (D) 3. (C) 4. (D)
9. (C) 10. (D) 11. (D) 12. (C)
5. (C) 6. (C) 7. (C) 8. 2
13. (C) 14. (C) 15. (B) 16 (A)
9. (C) 10. (D) 11. 100.01 to 99.99 12. 5.9 to 6.1
17. (D) 18. (B)
13. 1.85 to 1.87 14. 1 15. 4.66 to 4.76 16 85.0 to 85.5

17. (B) 18. (C) 19. (C) 20. (C)


Answer Key on Assignment  5
1. (D) 2. (A) 3. (C) 4. (B)

5. (C) 6. (A) 7. (A) 8. (B)


Answer Key on Assignment  2
9. (A) 10. (A) 11. (D) 12. (B)
1. (B) 2. (B) 3. 0.01 to 0.01 4. (A)
13. (B) 14. (D) 15. (B) 16. (B)
5. (A) 6. (D) 7. (B) 8. 5.1 to 4.9
17. (B) 18 (D)
9. (B) 10. 2.0 to 2.0 11. (B) 12. (B)

13. 1.9 to 2.1 14. (C) 15. (B) 16 13 to 13

17. (D) 18. (B) 19. (A) 20. 0.99 to 1.01 Answer Key on Assignment  6
1. (A) 2. (A) 3. (B) 4. (D)

5. (B) 6. (A) 7. (A) 8. (B)

9. (C) 10. (A) 11. (B) 12. (C)


Answer Key on Assignment  3
13. (A) 14. (A) 15. (B) 16. (B)
1. 40 to 40 2. (A) 3. (C) 4. (C)
17. (C) 18. (C)
5. 0.35 to 0.30 6. (C) 7. (A) 8. (C)

9. (B) 10. 0.0 to 0.0 11. (A) 12. (A)

13. 0.95 to 1.05 14. 18 to 22 15. (D) 16 (A)

17. (A) 18. (D) 19. (C) 20. (A)

255 256

Solutions  Calculus Vidyalankar : GATE – Engineering Mathematics

Answer Key on Assignment  7 Model Solution on Assignment  1


1. (B) 2. (C) 3. (A) 4. (B) 1. (A) 5. (C)
xsinx 0 1  cos(x 2 ) 2 sin2 (x 2 2)
5. (A) 6. (D) 7. (A) 8. (B) lim =   lim = lim
x  0 1  cos x
0 x 0 2x 4 x 0 2x 4
9. (B) 10. (C) 11. (A) 12. (B)
Applying L. Hospital Rule, sin2 (x 2 2) 1
13. (A) 14. (D) 15. (B) 16. (A) = lim 
1cos x 0 x 0 (x 2 2)2 4
lim =  
17. (B) 18. (A)
x 0 sinx 0 2
1  sin(x 2 2) 
Once again, L. Hospital rule = lim 
4 x 0  (x 2 2) 
sinx 0
lim =   =0 1 1
Answer Key on Assignment  8 x 0 cos x  1 =
4
(1)2 =
4
2. (D) Alternate Method
1. (C) 2. (C) 3. (D) 4. (D)
We know that f(x) is continuous at x = a,
5. (D) 6. (D) 7. (B) 8. (C) 1  cos(x 2 ) 0
lim 
if limf(x) exists and equal to f(a). x 0 2x 4 0
x a
9. (C) 10. (D) 11. (A) 12. (A)
Using L Hospital Rule
3. (C)
13. (B) 14. (A) 15. (C) 16. (B)
(sin x 2 )2x 0
 1
x
lim 
17. (C) 18. (B) lim  1   x 0 8x 3 0
x 
 x
(cos x 2 )2x2x  (sin x 2 )2
1 lim
Let h x 0 24x 2
Answer Key on Assignment  9 x
as x  , h  0 (cos x )4x 2  2sin x 2
2
lim
1. (B) 2. (B) 3. (A) 4. (C) x
x 0 24x 2
 1 1

5. (B) 6. (D) 7. (B) 8. (A)  lim  1    lim 1  h  h ( sinx 2 )4x 2  cos x 2 (8x)
x 
 x h 0
 2

2(cos x )2x 
9. (A) 10. (C) 11. (A) 12. (B) =e = lim 
x 0 48x
13. (D) 14. (C) 15. (A) 16. (D) 4. (D)
17. (B) 18. (B)
(cos x 2 )2x 4x 2

  

a2 a3
We have ea = 1a  .... (sin x 2 )  8x  
2! 3!  
(sin x 2 )  2x  8x   12 cos x 2 

an  
 =  n! 2
( sinx )(2x)4x 
n0 = lim 
x 0 48
Put a = 1
12 1

1 = =
  e1e
n  0 n!
48 4

257 258
Solutions  Calculus Vidyalankar : GATE – Engineering Mathematics

6. (C) 9. (C) 12. 5.9 to 6.1 14. 1


Given: f(x) does not have a root in  4  f(x) = 2x3  9x2 + 12x  3 2/
 1  1
log (1  4x)  1  4x  4 I=   x 2 cos  x dx
[a, b]. That is from x = a to x = b, then lim e3x lim     f(x) = 6x2  18x + 12 2/

curve f(x) does not cross the X-axis.


x 0 e 1 x 0 3e3x 3
 f(x) = 12x  18  1
Hence the curve is entirely above Put  y
10. (D) f(x) = 0 gives 6(x2  3x + 2) = 0 x
X-axis or entirely below it.
 x = 1, x = 2  1
x 3  sinx    2 dxdy
 f(a) > 0 and f(b) > 0 OR f(a) < 0 limx  0  x 
x f(1) = 12  18 = 6
and f(b) < 0 Further, if x = 1/ then y =  and
 sin x  and f(2) = 24  18 = 6
= (limx 0 x 2 ) limx 0
 f(a) . f(b) > 0
 x   There is local maxima at x = 1 and if x = 2/ then y = /2

7. (C) = 0  1 = 1 local minima at x = 2 /2 

Now, f(0) = 3,


I=  cos y(dy) = / 2 cos ydy
The ranges are given by 0 < y < a and
11. 100.01 to 99.99
0 < x < y i.e. 0 < x < y < a. f(1) = 2  9 + 12  3 = 2 and = sin()  sin(/2) = 0  1 = 1
1 x3
This can also be written as 0 < x < a f(x) = x(x 2  3) = x f(3) = 54  81 + 36  3 = 6
3 3 15. 4.66 to 4.76
and x < y < a 5 /4 4.5
3x 2
8. 2
f(x) =
3
1 = x2  1 13. 1.85 to 1.87 Volume = 
 3

 / 8
 dzdd
z3
2 The length of the curve
 n x 1=0
 1 5 /4

 n.  2  =?
x=1
 /2
 dx   dy 
2 2
 dz 
2 =   (1.5)dd
n 0 =   dt    dt    dt  dt
     
 3  / 8

This can be solved using Z-transform f(x) = 2x 0


5

and also using algebra as arithmetic- f(1) = 2 > 0  /2


2
2 =  1.5( / 8)d
geometric series. We will use algebra.  at x = 1, f(x) has local minimum.
=  ( sin t)2  (cos t)2    dt

 3

0 = 1.5 × (/8) × 8

 1
n
1  1
2
f(1) = 2 < 0
 n.  2  = 0  1 1 22   2  ...  /2
4 = 1.5 = 4.7124
n 0  at x = 1, f(x) has local maximum =  sin2 t  cos2 t 
2
dt
2 0 16. 85.0 to 85.5
[a + (a + d)r + (a + 2d)r + …..infinite For x = 1, local minimum value
 /2
terms] 2 4 17. (B)
f(1) =
1
2 =  1
2
dt
a rd 3 3 0 y = 2x  0.1x2
=  (1 < r < 1)
1  r (1  r)2 Finding f(100) = 333433.33 4  /2 dy
= 1 .t 0 = 2  0.2x
f(100) = 333233.33 2 dx
1
For above series, a = 0, d = 1, r = .
2 ( x = 100, 100 are end points of d2 y
4  0
= 1 x = 1.8622 dx 2

 1
n
0 (1 2)1) interval) 2 2
   2  .n =
1
 2
=2
 Minimum occurs at x = 100  y maximizes at 2  0.2x = 0
n 0
1  1
2  1 2 
   x = 10
 y = 20  10 = 10 m

259 260

Solutions  Calculus Vidyalankar : GATE – Engineering Mathematics

18. (C) 20. (C) Model Solution on Assignment  2


Given function is f(x) = |x|  u 
Given x = cos   , 1. (B) 1+x=1
|x| is continuous at x = 0 but not  2 
differentiable e2x 1 0 x=0
 u  lim  is  
y = sin   0 ≤ u ≤ 1 x  0 sin4x
0
 2   There is a maxima at x = 0
19. (C) So, Applying L-Hospital Rule,  Maximum value = f(0) = n(1)00
dx   u 
To find maxima  sin  
du 2  2  2e2x 2
lim = = 0.5 4. (A)
We differentiate f(x) w.r.t. ‘x’ & equate x  0 4cos4x 4
dy  u
to zero  cos In order to find maximum value of the
du 2 2 2. (B)
df(x) function we have
  3x 2  18x  24  0 We know that surface area when the 2
dx
 (x  1) sin(x  1) f(x) = 0
2
curve revolved about X-axis of a
 x2  6x + 8 = 0 Let I = 0
2
dx  x ex + ex = 0
parametric curve is (x  1) cos(x  1)
(x  4) (x  2)  ex(1  x) = 0
1
 dx   dy 
2 2 We know that,
x = 2 or x = 4 = 2  y     du  x=1
0  du   du  2
(2x  1)2 sin(2x 1)
Now,
1
 (2x1) cos(2x1)dx
2 And f(x) < 0 at x = 1
 u  0
 Maximum value is f(1) = (1) e1 = e1
d2 f(x) = 2  sin  
 6x  18   6  0 (maxima)  2  2
(1x)2 sin(1x)
dx 2 x  2 0
=  (1x) cos(1x)dx
0
2 5. (A)
2 2
d2 f(x)    u      u   For negative value of x, f(x) will be
 6  0 (minimum)  sin     cos    du 2
(x  1) sin(x  1)2
dx 2 x  4  2  2   2  2  =  dx = I
2 positive
0 (x  1) cos(x  1)
1
 f(x) x  2 = (2)3  9(2)2  24(2)  5 u 2 For positive values of x, f(x) will be
= 2 sin  du  I+I=0
0
2 4 positive
= 8  36 + 48 + 5 = 25  2I = 0
 u
1  minimum value of f(x) will occur at
But since interval [1, 6] i.e. inclusive, = 2  sin dx  I=0
20 2 x=0
we have to find
3. 0.01 to 0.01
f(1) = 1  9 + 24 + 5 = 21  u 
1
6. (D)
 1cos
  
2 f(x) = n(1  x)x
f(6) = (6)3  9(6)2 + 24(6) + 5 = 2   7. (B)

= 216  324 + 144 + 5 = 41    f(x) =
1
 1 and f(x)
 2 0 The given function is
1 x
Thus, in the interval maximum value is F(x) = 1  x2 + x3,
2   1
41. = 2   cos cos 0  =  0
 2  (1  x)2 where x  [1, 1]
= 2[cos0 1] = 2  F(x) = 2x + 3x2
1
 Maxima f(x) = 0 gives 10
1 x By mean value theorem

1 F(1)F(1)
  1 F(x) =
1 x 1  (1)

261 262
Solutions  Calculus Vidyalankar : GATE – Engineering Mathematics

x 
2 3
Now F(1) = 1  (1) + (1) = 1 9. (B) 12. (B) 2
 x  1 x2
f(x) = x3  3x2  24x + 100 = lim
and F(1) = 1  (1)2 + (1)3 x 
10. 2.0 to 2.0 x2  x  1  x
= 1  1  1 = 1 where x  [3, 3]  1
1
1 x 1 
 F(x) =
1(1)
=
2
=1
I=  (1  x)
dx ;  f(x) = 3x2  6x  24 and = lim  x
=
1
1  (1) 2
0 x   1 1  2
f(x) = 6x  6 x  1   2  1
a a
 x x 
 F(x) = 1 Use:  f(x)dx f(ax)dx
0 0
f(x) = 0 gives 3(x2  2x  8) = 0
 2x + 3x2 = 1  (x  4) (x + 2) = 0  x = 2, 4 15. (B)
1
 2
3x  2x  1 = 0 1
I= 
0 (1  (1  x))
dx As x = 4 > 3, consider x = 2  [3, 3]
16. 13 to 13
 3x2  3x + x  1 = 0 f(2) = 12  6 < 0
1 1
f(x) = 2x3  x4  10 where x  [1, 1]
 3x(x  1) + 1(x  1) = 0 1  There is a maxima at x = 2
  x dx
1/ 2
= dx =  f(x) = 6x2  4x3 and
x
 (3x + 1) = 0 0 0
f(3) = 27  27 + 72 + 100 = 118
f(x) = 12x  12x2
1
or x1=0  x1/ 2  f(3) = 27  27  72 + 100 = 28
=   =2 f(x) = 0 gives 2x2(3  2x) = 0
1  1/ 2  0  Minimum value of f(x) = 28
 x=   x = 0. 1.5
3
11. (B) 13. 1.9 to 2.1 As x = 1.5 > 1, consider x = 0  [1, 1]
or x=1
2 x
1 f(0) = 0  There is a neither maxima
1
 e x circumference
xy
Now  lies between (1, 1) dydx
4 nor minima at x = 0
3 0 0

0 e  e 0 e dy dx


1 f(1) = 2  1  10 = 13
1 =
2
x x
x
y  xx4
 x=  4
3 f(1) = 2  1  10 = 9
2 2 
 Minimum value of f(x) = 13
 e  e  dx
x time = = 2 sec
 e (e 1)dx
x y x x
8. 5.1 to 4.9 = = 1.57
0
0 0
3 2
f(x) = 2x  3x where x  [1, 2] 17. (D)
2
2
 e2x  The function f(x) = x3 + 1 has a point of
 f(x) = 6x2  6x and f(x) = 12x  6 =  (e2x ex )dx =  e x 
2 0  2 0 inflection at x = 0, since in the graph
 f(x) = 0 gives 6x  6x = 0
e4 1 e4 1 sign of the curvature (i.e., the
 6x(x  1) = 0  x = 0, 1 = e2  1 = e2 
2 2 2 2 concavity) is changed.
f(0) = 6 < 0
1 4
 There is a maxima at x = 0 = (e 2e2 1)
2
f(1) = 12  6 = 6 > 0 14. (C)
1 2
= (e 1)2
 There is a minima at x = 1 2 lim x2  x  1 x
x 
f(1) = 2  3 = 1; f(1) = 2  3 = 5;
 x2  x  1  x 
f(2) = 16  12 = 4 = lim  x 2  x  1 x 
 Global minimum value of f(x) = 5
x 
 x2  x  1  x 
263 264

Solutions  Calculus Vidyalankar : GATE – Engineering Mathematics

18. (B) 1  24992 


= 32  32 Model Solution on Assignment  3
x
2 sin2   3  5 
 1  cos x  2 1. 40 to 40 4. (C)
lim  2   lim 2 24992
x 0
 x  x 0 x =
15 f  x  sinx 
x ((y 2  z2 )(2x)) Given, lim  
sin2   x x   x 
2 2
 2 lim 2    C  xy 2 dxdy = (24992)10 4 at x = 2, y = 1, z = 3 = (1 + 9) (4) = 40
x 0  x  5 1
Put x = and x   or h  0
 4
R
h
 2  = 0.99968  1 2. (A)
1 1 1 1  1
 2  OR
f(t) = et  2e2t where 0 ≤ t <   sin h   1hsin h 
4 2 Now, lim  h  = lim  
5  2x  h 0
 1  h0  1 
 f(t) = et  2e2t(2) = 4e2t  et
19. (A)  R
xy 2 dxdy =    xy 2 dy dx
  h 

x 1 y  0  and f(t) = et  8e2t
lim f(x)  lim (x  1)  2  f(3) 10
x 3  x 3  2x
f(t) = 0 gives 4e2t  et = 0 = =1
5
y  3
1
x3 = 1 x  3  dx
lim f(x)  lim    2  f(3)  et (4et  1) = 0
x 3  x 3 
 3  0
5. 0.35 to 0.30
 f(x) is continuous at x = 3 5  et = 0 or et = 1/4
8
3 1
=  x(x 3 )dx t   sin x  0 
 t =  or e = 4  t = , t = n(4) lim  form 
x 0  2 sinx  x cos x   0 
20. 0.99 to 1.01 5 f() = 0
8  x5    cos x 
=   
 R
xy 2 dxdy 3  5 1  Neither maxima or minima at t =  = lim  
x 0  2 cos x  cos x  x sin x 

1 1 1
 R xy dxdy R xy dxdy
2 2
= 8 24992 f (n4) = 8 =  0 (L  Hospital Rule)
 (3124) 4 16 4
1
15 15 1
5 2 5 2x  Maxima at t = n(4) =
24992 3
=   
x 1 y  0
xy 2 dxdy   
x 1 y  2
xy 2 dx  C xy dxdy =
15
10 4 6
2

R
3. (C) 6. (C)
5
 x2   y3  5  y3 
2 2x
2
=      x   dx = 2.4992 z = xyn(xy)xyn(x)xyn(y) /2
 cos x  isinx 
 2 1  3 0 1  3 2
5 I= 0  cos x  isinx dx
= 0.9968  1 z
5  = y[1n(x)]yn(y)
8 1 x /2 /2
= (12)   x(8x 3  8)dx  eix 
0  eix dx 0 e
2ix
3 3 1 = = dx
= yyn(xy)
1   x5   x2  
5 5

= 32 8   8    z  e2ix 


/2
1 i
and = xn(x)x[1n(y)] =  = [e e0 ]
3   5 1  2 1  y 
   2i  0 2i
= xxn(xy)
1
= [cos()i sin()1]
 z z 2i
 x y
x y i
= [ 101]i
2i2

265 266
Solutions  Calculus Vidyalankar : GATE – Engineering Mathematics

7. (A) 12 Now applying L Hospital rule 1


= 1 = 1  0.5774  x=
The partial derivative of x2y2 with 6  d  2
x   logx 
respect to y is 0 + 2y  2y. = 0.4226, 1.5774 lim   x dx  = log x For x = 0,
 0 d
The partial derivate of 6y + 4x with Now, x = 0.4226 < 1. 1
dx y2 =
respect x is 0 + 4 = 4. Consider x = 1.5774 4

Given that both are equal. f(1.5774) = 6(1.5774  1) > 0 1


13. 0.95 to 1.05  y=
2
 2y = 4  Minima at x = 1.5774
1
 y=2 f(1) = 0 and f(2) = 0 and for x=
2
 Maximum value is f(x) = 0
8. (C) 1
1
f(x) is discontinuous when 11. (A) y2 = 2 = 1
4 8
denominator = 0 3 sin x + 2 cos x
replacing sin x and cos x by its series, 1
 (x2 + 3x  4) = 0  y=
8
 x3 x5  Here rectangle is inscribed in ellipse.
 (x + 4) (x  1) = 0 = 3  x  
 3! 5!  Let the length and breadth of rectangle Now F(x) = 8  48 x2
 x = 4, 1
be 2x and 2y respectively.  F(x) < 0 for
 x2 x4 
2 1  .....
9. (B)  2! 4!  Now area of rectangle = (2x)  (2y) 1
x=
= 4xy 2
f(x) = x3  3x2 + 1 3x 3 3x 5
= 3x  .....
2
 f(x) = 3x  6x = 3x(x  2) 3! 5! Let F = (Area of rectangle)2  F(x) is maximum at x =
1
= (4xy)2 2
 Between 1 & 0, f(x) > 0 i.e. f(x) is 2x 2 2x 4
2  .... The maximum area of rectangle
2! 4! = 16 x2 y2
increasing
inscribed in ellipse is
Between 0 & 2, f(x) < 0 i.e. f(x) is x3
 1 x 2 
= 23xx 2  ..... = 16x 2  
2  4   1  1 
decreasing 4xy = 4   
12. (A)  2  8 
above 2, f(x) > 0 i.e. f(x) is increasing  2 1x 2 
y  fromequationofellipse  4
x  1  0   4  = =1
lim   form  4
10. 0.0 to 0.0  0   0  2 2
= 4x (1  x )
f(x) = x(x  1) (x  2) = x3  3x2 + 2x Hence applying L Hospital rule
Now F(x) = 0 14. 18 to 22
where x  [1, 2] Let y = x  x(1  2x2) = 0 1
2 
I =  (xy10)dxdy where D
 f(x) = 3x2  6x + 2 and f(x) = 6x  6  log y =  log x
 x=0 D
2
 f(x) = 0 gives 3x  6x + 2 = 0 1 dy  d denotes x2 + y2 ≤ 4
 = log x or 1  2x2 = 0
6  36  24 6  12 y dx x dx 2 Use polar co-ordinates. Put x = r cos ,
x= =  2x = 1
6 6 dy  d  y = r sin  dx dy = r dr d
 = x   log x 1
dx x dx   x2 =
2

267 268

Solutions  Calculus Vidyalankar : GATE – Engineering Mathematics

1
2 2
Model Solution on Assignment  4
2 r 0  (r cos r sin 10)rdrd
I= 
 0
1. (B) 4. (D)
2
1
 r(r sin r cos 100 dr
2

2 r 0
= 1  cos x  sin(x / 2)  1
2
ax  b a b
lim = lim  lim = lim 
 x  cx x  c x
x 0 x2 x 0
 (x / 2)  2
2
1 a a
2 r 0
=  (10r2)dr 1 = 0 =
= c c
2
2
r2  18. (D)  sin  
= 10   = 10 × 2 = 20  xlim  1
5. (D)
 2 0  0  
19. (C) ex  1
L im f (x) = L im =1
e 2. (D) x0 x0 x
15. (D)
1
x In (x) dx
0
Applying L’ Hospitals rule, we have But f (0) = , which is not defined.
 3  
e
3  0
16. (A) cos 
 x2   1 x2  lim0  cos 0  1
= In(x)        dx 1 Since L im f (x)  f (0) therefore f (x)
P = 50q  5q2 3 x 2 
x 0

2  2 1  3  has removable discontinuity at x = 0.
dp dp 3. (B)
 50  10q; 2  0
dq dq  3 3

e

= In(x)  x 2  2  4  x 2  x1/ 3  2
 p is maximum at 50  10q = 0 or,  3 9 1 L = lim 6. (B)
x 8 (x  8)
q=5 2 3 4  x2 
= e  Let x = a 3
 a2 ex   1  x  
Else check with options 9 9  2 
(a  2) lim
20. (A)  L = lim
x 0 x3
a 2 a3  8
Given  
17. (A) x2 x3
1
(sin1 x)2 Now by partial fractions,  1  x    ...  
The given curves are y = x and y = x 2 I=  dx  2! 3! 
0 1 x2 (a3  8) = (a  2) (a2 + 2a + 4)  
  1  x  x 
2
solving (1) and (2) we have 
  
1
(sin1 x)3  f n1(x)  1 1 2  
=  f n (x)f(x)dx 
x = 0, x = 1 3 0 
n1 
  L = lim
a 2

a 2  2a  4 12 = lim  3

x 0 x
1 1
 (x  x
2
Area = )dx = (sin1 )3 sin1 0  Alternate Method
0
3 = lim
1 x x2
   ... 
1
x n  an x 0 6 4! 5! 6
1    
1 3
x x  2
1 1 3 3 Since lim = nan1
=      =   0  = x a x  a
3  2  24 Else, use L'Hospital's rule
 2 3 0 2 3 
x1/ 3  2
1 L = lim 7. (A)
x  8 (x  8)
= sq units.
6
2
1 3 1 8. (A)
= (8) =
3 12
 9. (C)

269 270
Solutions  Calculus Vidyalankar : GATE – Engineering Mathematics

10. (D) 12. (C) 14. (C) 16. (A)

f (x) = x 3  6x 2  24 x  4 f (x) = (x  2)3 f(x) = 4x3  8x2 + 1 in [1, 1] dV


 4r 2
f(x) = 12x2  16x dt
2
f ‘(x) = 3x  12 x  24 f(x) = 3(x  2) 2

Equating f(x) to 0 4 3
f ‘’(x) = 6x  12 V= r
f(x) = 0 for maximum and minimum. 3
 12x2  16x = 0
For maxima and minima, f ‘(x) = 0 or x = 2 dV 4  2 dr  dr
3x2  4x = 0   3r  4r 2
or 3x 2  12 x  24 = 0 When x > 2, f ‘(x) = 3(x  2)2  0 dt 3  dt  dt
x(3x  a) = 0
or x = 2  2i When x < 2, f ‘(x) > 0 dr
4  4r 2  4r 2
Hence neither maximum nor minimum  x = 0, x= dt
 f (x) has no maximum and no 3
minimum at any real value of (x). at x = 2. dr
4 i.e. 4r 2  k 4r 2
Now, x [ 1,1]
 dt
3
11. (D) 13. (C) dr
So x = 0   k
Two parts are 30 and 30  x y = sin x (1 + cos x) dt
f(x) = 6x  4
f (x) = x 3 .(30  x) dy 17. (D)
= cos x  sin2 x  cos 2 x  f(0) = 4 < 0
dx
f (x) = 4x3  6x2  2x  4
f (x) = 30 x 3  x 4  f(x) in not minimum at x = 0.
= cos x + cos 2x
= 2 (x  2) (2x2 + x + 1)
f(x) = 90 x 2  4 x 3 Checking at endpoints :
d2 y
=  (sin x + 2 sin 2x) 3 2 Thus only critical number is x = 2
dx 2 f(1) = 4(1)  8(1) + 1
f (x) = 180 x  12 x 2 So f (0) = 3
dy =  4  8 + 1 = 11
For maximum and Minimum f ‘(x) = 0 For maximum and minimum, =0 3 2 f (2) = 9
dx f(1) = 4(1)  8(1) + 1
or 90 x  40 x
2 3
= 0 f (4) = 99
cos x + cos 2x = 0 = 4  8 + 1 = 3
90 45 18. (B)
or x = 0, x = = or 2 cos2 x + cos x  1 = 0  f(x) has minimum at x = 1 in [1, 1]
4 2  x  2   3x 2   x 3
1 f (x) =
But, x = 0 is impossible, hence or cos x = , cos x =  1 15. (B)
 x  2
3
2
45 4  dv dr
consider only x =  v= r   4r 2 2x 2  x  3 
2  x = or x =  3 dt dt  0
3
 x  2
2
2
 45  45  45  dr 2
f ''   = 180  2  12  2    4  = 4r
 2    But  (0, ) and   (0, ) dt x = 0 and x = 3 are critical.
3
 45  dr 1 1 However x = 3 does not lie in interval
    0.01
f ''   <0 d2 y
 0 at x =
 dt r 2 100
 2  so we list 0 and end points 1 and 1.
dx 2 3
45 Since diameter = 2  radius f(0) = 0, f (1) = 1, f (1) = 1/3
 f (x) is maximum at x = = 22.5
2 Hence diameter is decreasing at the
 Two parts of 30 are 22.5 and 7.5 rate of 0.02 cm /hr.



271 272

Solutions  Calculus Vidyalankar : GATE – Engineering Mathematics

Model Solution on Assignment  5 5. (C) 8. (B)


  dy dx
1. (D) But L im f (x) = 1 (Given) Put x  = t, so that, x = t 3  2x
x 2 2 dt dt
Lim ( x 2  1  x  1)  a = 1 (From eq. (2)) or 2x =  + 2t dx
x  When x = 3,  1foot
 / min
 dt
x2 when x  then t  0
( x  1  x  1)
2
 f (x) = 2
= L im( x 2  1  x  1) x 1 dy dy
x 
( x 2  1  x  1) 3 6  2ft / min
1  cos 2x dt dt
2 2 Now L im
(x 2  1)  (x  1) f (2) = = 0 x  (  / 2) (  2x)2
= L im  2 1
x
( x2  1  x  1) 9. (A)
1  cos(  2t)
= L im 4 3
x (x  1) 3. (C) t0 ( 2t)2 A = 4r2. v= r
= L im 3
x  1 1 1  Here 1  cos 2t
x  1 2  1  = L im dA dr dv dr
 x x x   12 22 32 t0 4t 2  8r  4r 2
n2  dt dt dt dt
L im  3  3  3  .............  3 
(x  1)
n 
n n n n  2 sin2 t
= L im = L im dr 1 dA
x   1 1 1 
t0 4t 2 10 = 4r 2  r 
 1 2  1   1  2  3  .....  n 
2 2 2 2
dt 2 dt
x = L im  
 x x  n  n3  sin t  1 1
2
  = L im  When r = 5,
 . = 2

t0
 t  2
= =   n(n  1)(2n  1)  1 dA
= L im   10 = 5
01 n 6n3 6. (A) 2 dt
 
  dA
 2 2 2 2 n(n  1)(2n  1)  Put   = y, as   , y  0   4cm2 / h
2. (A) 1  2  3  ....  n  6  2 2 dt
 
ax  b  2n2  3n  1 cot 
L im f (x) = L im  L im
= L im     (  / 2) ( / 2)   10. (A)
x0 x0 x 1 n
 6n 2

b2 = r2 + (h/2)2
L im ax  b tan(( / 2)  )
1  3 1 = L im
=
x0
= b ….(1) = L im 2  ( / 2)   v = r2 h =  (b2  h2 /4) h
6  n n2 
  (  / 2)
L im x  1 n
x0
dv
2 1 = L im
tan y
dh

  b2  3h2 / h  0  h  2b / 3
But L im f (x)  2 (Given) = = y0 y
x0 6 3
=1 d v 2
3h d v 2
 b=2 (from eq. (1))   2
4. (B) dh2 2 dh  2b / 3
ax  b 7. (A)
Also, L im f (x) = L im 2
x x x1  1
2n
  1 
n 3 2b
L im  1     L im  1    e 2 dy   b  ve
n   n  n    7  3x 2 hence rate of change of 2 3
n   dx
a  (b / x)
= L im Hence there is relative maxima at
x 1  (1/ x) d  dy  dx
slope is  6x
= a ….(2) dt  dx  dt h = 2b/ 3
=  6  3 4 =  72 r=b 2/3

273 274
Solutions  Calculus Vidyalankar : GATE – Engineering Mathematics

2
11. (D)  x (x  4x +4) = 0 15. (B) 18. (D)
Let x be thickness of ice  x (x2)2 = 0 If 2 points on line are A1(0, 1) and 
 x2 2

v = 300 [  (10 + x)2  100]  x = 0, 2, 2 A2(1, 0)


Let I = 0 0 e e  y dxdy

dv  dx  d2 y y  y 1 y1  y 2 
 300  x)  = 36x2  96x + 48  Equation of line is   (x 2  y 2 )
dt  dt  dx 2 x  x 1 x1  x 2  I= 0 0 e dxdy

dv At x = 0, y 1 1 0
 2  Put x = r cos 
dt x  0 0  (1)
d2 y y = r sin 
= +48
dx 1 dx 2 y 1
  1 Then J = r
dt 300 10  x  At x = 2, x
Now 0  x   and 0 ≤ y  
1 1 d2 y y=x+1
  = 36  4  96  2 + 48 = 0  region of integration is the first
300 10  2  3600 dx 2 2 2
I   y dx =  (x  1)dx quadrant.
12. (B) so only 1 local minima. 1 1
 0r, 0    /2
Let length be 2x and breadth be 2y either x = 0 or complex root  x2 
2
1 
=   x  = (2  2)    1 /2 
so only value = 0 r 2
0 0 e
2 2
x + y = 1  dy/dx = x/y 2 1 2   I=  rdrd
A = (2x) (2y) = 4xy 3 5
14. (D) = 4 = = 2.5 /2  
dA 4 2 2  r 2 2r
  0 d 0 e 0 e
r
  x 2  y 2  0 /4
1  tan x  = rdr = dr 
2 2
dx y 0 1  tan x  dx

 x=y=1/ 2 16. (B)  r 2  2 
e  2r dr =  er 
4 0
/4 =
cos x  sin x 4 0
d A 2
4x I = 0 dx 1 1 1 1
cos x  sin x
0 e dx = x 0 e dx 0 1 0 e dx
x x x
 3
dx 2 y 
Let t = cos x + sin x =  0  (1)
4
 xe   e 
2 1 1
d A 4  1 2 x x
= (e  0)  (e  1) = 1
   8  dt = (sin x + cos x) dx 
1 
0 0
dx 2 1 2,1 2 2
3
 I=
dt cos x  sinx 4
 = dx
t cos x  sinx
d2 A 17. (B)
 = 16
dx 2 1 /4
dt
 lnt 0
/4
2,1 2
 I= 0 t
1 1
 Area = 4xy = 4   2
 ln cosxsinx 0
/4
2 2

13. (B)  ln cos / 4sin / 4 
Given : y = 3x4  16x3 + 24x2 + 37
For maximum value or minimum
 ln  2   ln2 1/ 2 1
 ln2
2
dy
= 12x3  48x2 + 48x = 0
dx

275 276

Solutions  Calculus Vidyalankar : GATE – Engineering Mathematics

Model Solution on Assignment  6 6. (A) 8. (B)


y = log (sin x) The series 1 + 2 + 3 + ….. is an A.P
1. (A) 3. (B)
dy 1 d 1
Here f (0) = 0 Here f (x) = 1 = (sin x) Sn  n  n  1
dx sinx dx 2
Right Hand Limit Also, Left Hand Limit
1
=
cos x
= cot x lim Sn  lim n  n  1  
 |x|  L im 1 = 1 sin x n n 2
lim f (x) = lim   f (x)  3 
x  0 x 0  x 
hence the series is divergent.
Right Hand Limit  
x For the interval  0,  , cot x is +ve
= lim   = 1 L im (ax  b) = 3a + b  2 9. (C)
x 0  x  x 3 
dy   The series 1  2 + 3  4 +  6 + . ……
|x|  x if x  0  Also, f (3) = 1   0 on  0,  .

dx  2 is an alternatory series
   x if x  0  Since f (x) is continuous, therefore
  Sn = (1  2) + (3  4 ) + (5  6) +. . . .+
Left Hand Limit 3a + b = 1 ….(1)  log (sin x) is increasing on  0, 
 2 [(n  1)  n]
 |x|  Also, f (s) = 7
lim f (x) = lim   Again cot x is negative on the interval 1
x  0 x 0  x  and L im  f (x) = L im (ax  b) = (1) + (1) + ........... + (1) = n
f (x)  5 x 5   2
 x   2 , 
= lim   =  1 = 5a + b    1 
x 0  x  lim sn  lim   n   
Since f (5) is continuous at x = 5, dy
n n
 2 
 R.H.L.  L.H.L.  < 0
therefore dx Series can also be written as
The function f (x) is discontinuous at
5a + b = 7 ….(2)  log (sin x) is decreasing. Sn = 1  (2  3)  (4  5)  (6 7)……..
x = 0 and the discontinuity at x = 0 is
Solving eq. (1) and eq. (2), we get  ((n1)  n))
of first kind because R.H.L. and L.H.L. 7. (A)
a = 3, b =  8. n1
both exist finitely but are unequal. 1 1 1 = 1 + 1 + 1 + 1 ………..+ 1 =
The series 1     ..... is a G.P 2
4. (D) 2 22 23
1
2. (A) The given function is of straight line
lim Sn  lim n  1  
11   1/ 2   2   1n 
n n n 2
Here f (1) =  1 Sn =    1
 
hence no maximum or minimum.  1  3   2   Since limit does not exist because the
Also, Right Hand Limit 1  
5. (B)  2 sum of infinite forms of series is not
L im f (x) =  1 unique
x 1 y = log x 2   1 
n

lim S n  lim 1     
n 3
Left Hand Limit dy 1 n
  2   10. (A)
=
dx x The series is alternatory series
L im f (x) = a  2 2 2
x 1
Since log x is defined only for +ve  1  0 
3 3 1 1 1
Now, L.H.L. = R.H.L. 1 >   .......
values of x. 2 3 4
Hence the series is convergent
 a2=1  a=1 1 dy
So, >0  > 0  y = log x is 1
lim 1n  lim 0
x dx n n n
increasing. here the series is convergent

277 278
Solutions  Calculus Vidyalankar : GATE – Engineering Mathematics

11. (B)   17. (C) 18. (C)


 
n 1 Since f(x) = 1/x is not continuous in f(x) = x3  f (x) = 3x2
Here Un = = lim  
n  
1  2 n 1 [ 3, 3] [4, 2 ] or [1, 1] the point of
 1 3/ 2  1  f (b)  f(a) = (b  a) f (c)
 n 
n discontinuity is 0. only in [ 2, 3] the b3 a3 = (b a)  3c2
lim Un  lim 0
n n 1  2 n 1 1 function is continuous and differentiable.

1 1

2 b  a   a2  ab  b2 
hence the series is divergent.  3c2 =
ba
Hence it is convergent.
12. (C) a 2  b2  ab
c=
15. (B) 3
The infinite series
1
1 1 1 here Un = Sin
    .....is n
np 1p 2p 
3
i) convergent of p > 1 1 1  1 1 1
  ...   3  .....
n 3  n  n 6n
ii) divergent of p  1
1
Take Vn =
13. (A) n

1 1 1 Vn 1 1  n
lim  lim   ...  
The series 1   
3 9 27
......is a G.P. n Vn n  n 6n3  1

Where a = 1 ,  1 
 lim  1  2  ...   1
1
n
 6n 
r=  Which lie between 1 and 1
3 series Vn n a divergent. Hence  Un
Hence the given series is convergent. is also divergent.

14. (A)
16. (B)
Un = n3  1  n3
n1 n 1
Un = , Take Vn = 2 
1 n2 n n

n3  1  n3
  n  1 
1 Vn  2 
n 
Take Vn = lim  lim   1
n3/ 2 n V
n
n  1 
 n 
 1   
Un  3 3 
By comparison test Un &  Vn behave
lim  lim  n  1  n 
A  V n  1 
n alike.
 n3/ 2 
 
1
Now Vn =  is a divergent series
n
hence Un is also divergent.

279 280

Solutions  Calculus Vidyalankar : GATE – Engineering Mathematics

Model Solution on Assignment  7 5. (A) 9. (B)

f (x) = x 3  12 x 2  45 x  11 let f(x) = ex. Then fn(x) = ex.


1. (B) 4. (B)
Hence fn (o) = 1 for all n  0
f (x) = 3x 2  24 x  45
f (x) = e (1/ x)  
f()  5 cos   3 cos      3 
f n (0) n 
xn
f ‘(x) = e (1/ x)
  1  3 f (x) = 6x  24  Maclaurin series  n!
x 
n  0 n!
 2 n0
x    f (x) = 0 or 3x 2  24 x  45  0
f()  5 sin   3 sin     10. (C)
Since e (1/ x) > 0 and x 2 > 0,  3
x = 3, x = 5 Let f(x) = sin x.
for all x  0  
f()  5 cos   3 cos     f (5) = (6  5)  24
 3 Then f(/4) = 2 / 2.f    / 4 
 f ‘(x) < 0, for all x  0, x > 0
=6 > 0
 f (x) is decreasing in the interval Equating f() to 0  2
 f (x) is minimum at x = 5.  2 / 2f    / 4  
2
[0, ]  
5 sin   3 sin      0
 3 6. (D) f    / 4  
  2  .Thus
Hence f (x) = e 1/ x , x > 0 is a
2
decreasing.  
 5 sin   3 sin      0 7. (A) Taylor series for sin x about /4 is
 3
2. (C) Using lagrange’s mean value theorem
2  x   / 4  x   / 4
2
 1 3 1  
1  5 sin   3 sin    cos   0 f  x   f  0 ex  1 2  1! 2!
f (x) = x  2 2  f (c) = c
or e = ….(i) 
x  x0 x
 x   / 4 
3
1 2
f (x) = 1  2 ; f(x) = 3 3 3 Now 0 <<< x  e0 < ec < ex .…(ii)   ......
x x  5 sin   sin   3 cos   0 3! 
2 2 From equation (i) and (ii)
for maximum and minimum, f(x) = 0
13 3 3 ex  1 11. (A)
1  sin   cos   0 1  ex
or 1  = 0 2 2 x Let f(x) = ln (1x). Then f(0) = 0.
x2
3 3  x <ex 1 < xex f (0)  1,f (0)  1,f   0   12
or x = 1  tan   
13  1 + x < ex < 1 + xex
f( 1) =  2 < 0 f(4) = 1.2.3 and fn (0) =  (n 1) !
 3 3 
 f (x) is maximum at x =  1.    tan1  Thus for n  1 fn (0)/n! = 1/n and
 13  8. (B)
  1/x x2 x3 x4
3. (A) x is decreasing function then Maclaurin series is x    ...
  21.79 2 3 4
f (x) =  x 2  2x  1  1  loge x 
f(21.79) < 0 f (x) = x1/x   12. (B)
 x2 
f ‘(x) =  2x + 2; f ‘’(x) =  2 1
 maximum at  = 21.79 Let f(x) = Then
f ‘(x) = 0 f(x) is decreasing if f (x) < 0 x
f(21.79) = 5 cos (21.79)  1  log x  1 2 2.3
[for maximum and minimum] i.e. x1/x  x 2   0 f   x   2 ,f   x   3 ,f   x   4
+ 3 cos (21.79 + 60) x x x
 2x + 2 = 0
f(21.79) = 10 234
x = 1  1  loge x < 0  loge x > 1 f4  x  andingeneral
x5
f (x) =  2 < 0 x>e
n!
f n  x    1 sof n 1   1 n!
n n

 f (x) is maximum at x = 1 x n 1

281 282
Solutions  Calculus Vidyalankar : GATE – Engineering Mathematics

Thus Taylor series is Hence Model Solution on Assignment  8


x f n 1 x x4 x6
  x  1    1  x  1 ln (1 + x2) = x 2 
n n n
  ..... 1. (C) Right Hand Limit :
n0 n! n 0 2 3
f (x) = 1 + k x2 h h
…  h  h for h  0 
2n
13. (A) x lim   lim
  1
n 1
..... h h
n For x = 2 to be critical h 0 h 0
Let f(x) sec x. Then
f (2) = 1 + k (2)2 = 0 =1
f   x   Secxtan x, f   x  x 2n
Thus term is  1
n 1

n  k = 4 h h
 lim   lim 

 sec x 1  2 tan2 x  17. (B) Hence h 0 h h 0 h

4 8 h
f   x   sec x tan x(5  6 tan2 x) 1 x f (x) = 1  ,f (x)  3  lim does not exist.
f(x) = x2 x h 0 h
1 x3
 f(0)  1f   0   0,f   0   1
1  f (2) = 1 4. (D)
from   xn
f (0)  0,f 4  5 1  x n 0 Hence relative maximum at x = 2 f x  2
lim  0  lim f  x   2
The Maclaurin series is 1 1 2. (C) x 1 f x  2 x 1
&sof  x   x
1 5 4 1 x3 1 x3 Let x and y be dimension  f(x) = 2x
1 + x2  x  ....
2 24  
2x + 2y = 16  A = x(8  x) = 8x  x2
=  x 3n
  x 3n1 hence lim f  x   2
x 1
14. (D) n 0 n 0
dA d2 A
 8  2x, 2  2
In general, From Maclaurin expansion, dx dx 5. (D)
fn  0  f 36  0  d2 A 
an = .so f 33  0   33!a 33  33!233  a 36  1 Critical number is x = 4    2
6. (D)
n! 36! dx 2  4
1
36
Hence f (0) = 36! Let A = lim
15. (B) here x = 4 and y = 4 n 0 enlogn

x 2n1 1
tan1 =   1
n
 18. (A) 3. (D)  loge A  lim logee
n0 2n  1 n 0 nlogn
Let f(x) = tan x . then x2
fn  0  lim 1/ n 1/ n2
Sincean  ,f 99  0   99!a99 f (x)  sec 2 x,f   x   2 tanx sec 2 x
x 2 x2 = lim  lim
n! n 0 logn n0 1/ n
Put x  2 = h
1 1 f (x)  2(sec 4 x  2 tan2 x  tan4 x) 1
But a99 = (1)49  as x  2, h0  lim  
99 99 n 0 n
So f(0) = 0,
x2 h
Thusf 99  0     99! / 99  98!  lim  lim
f   0   1,f   0   0,f   0   2 x 2 x2 h 0 h
7. (B)
16. (A)
Thus Maclaurin series is Let f(x) = nx . then
Left Hand Limit :
x2 x3 1 1
ln (1 + x) = x    ..... 1
x  x3 
2 5
x  ... h h f  x   ,f   x    2 ,f   x 
2 3 3 15 lim  = lim x x
h 0 h h 0 h

xn 2 4 2.3
  1
n 1
 ...for | x | 1 …  h  h for h  0   f  x   4 & in general
n x3 x
 = 1

283 284

Solutions  Calculus Vidyalankar : GATE – Engineering Mathematics

n  1 !
0  2x  x 
11. (A) 15. (C) 1
2
fn(x) = (1)n+1 =  x 3 dx
xn
f   x   4x  4,f   x  N D
1 2x  x 
C 0
So f(2) = n2 , B M  2
 x 3 dx
 4 & f n  x   0forn  2 x2 = 4y
n  1 ! 1 0
fn(2) = (1)n+1 2n Thus f(1) = 3, A O E x3 x4 x3 x4 3
= x2    x2   =
3 4 3 4 2
f (1)  8,f  1  4 & f  x  Required area
0 1
 fn  2
  x  2
n
Thus Taylor series is
 3  8  x  1  2  x  1
2
n 0 n! 2  x  2 x 2  18. (B)
BOD =  1
 4 4 
 dx
x  sin x
x2 1
  n2      x  2   .......
2
12. (A)
I=  1  cos x dx
 2  8 2
1  x2 x3  9
=   2x    x sinx
13. (B) 4 2 3  1 8  dx   dx
8. (C) 1  cos x 1  cos x

2x = exn2 I= 0 log 1  cos     d 16. (B) x
2 sin cos
x
The given curve passes through the x 2 2
Now, e =  . x x  n

=  2 sin 2
dx   2

n  0 n!
2I = 0 log 1  cos   d point (2, 3) and (4,
3
)
x/2 2 sin x / 2

2 x x x
 =  cos ec 2 dx   cot dx
  n2    log 1  cos   d
n

2 2 2
Therefore 2x = exn2 =  xn 0 4  8 
n 0 n! Required area = 2  1  x2  dx x
0 log 1  cos   d

= 2 =  x cot C
2
9. (C) 4
 8


 2  log sin d = x    4
1  x 2
ex =  xn
n  0 n!
0

  
= 4   log2  17. (C)
Hence   
Required area
1 2n f  0 
 100
1 I = log 2
f x   x ,  G100 
n 0 n! 100  50  !
14. (A)
100!
Hence f100(0) =
50! /2     1,0 0, 0 (2,0)
I= 0 sin2   x  log tan   x  dx
2  2 
x =1
n! 2 x = 1
n
f (0) = for f(x) = ex
(n 2)! /2 sin2x log(tanx) 
2I = 0   dx
 sin2x logcot x  
10. (D) /2
= 0 sin2x log  tanx cot x  dx
The Maclaurin series for f(x) is the
/2

polynomial 2  x 
4 2 12 3
x  x = 0 sin2x log(1)dx  0
2! 3!
Thus f(x) = 2 + x + 2x2+ 2x3

285 286
Solutions  Calculus Vidyalankar : GATE – Engineering Mathematics

Model Solution on Assignment  9 6. (D)


=
1
 cos ec  x   / 4  dx
 1  2  ....  n  n  n  1 2
1. (B) log  cosecx  lim    nlim
 lim
n
 n   2n 1 x 
2x 2x x 0 1  log tan     C
 x   1 x  n1 2 2 
let A = lim    lim   x  
x  1  x
  x 
 x  2
2x
ByLHospitalrule 10. (C)
 1 7. (B)
= lim  1   1 sec 2 x dx
1
x 
 x  cosecx cot x  /2 I =  1  3 sin2 x   sec2 x  3 tan2 x
2 = lim cosecx
1
I= 0 cos4 x sin3 x dx
  1 
x x 0
(Divide numerator and denominator by
  lim  1    x2 /2


x 
 x   = 0 cos4 x sin2 x sin xdx
cos2x)
x2
 lim = lim 2x / sec 2 x
1
  sec 2 xdx
/2
= [e]2 =
e2
x  0 tan x x 0  0 cos4 x 1  cos2 x sinxdx =  1  tan2 x  3 tan2 x
ByLHospitalrule let cos x = t
1 sec 2 xdx
4   1 2

1 t 1  t  dt
0
2. (B)  lim 2x cos2 x I= 4 2

 2   tan x
x 0 2

 1
tan x  
A = lim   loge A = 0  A = e0 = 1  t5 t7 
1
x  x
2
      Put tan x = t
 5 7  0 35
 loge A  lim tan x log1/ x 1 dt
4  1/ 2 2  t 2
x 0 4. (C) =
8. (A)
log1/ x 1 2 1 n  n  1 / 2
= lim lim  2  2  ...    lim Put x = sin 
x 0 cot x n  n n2 tan1 2t
n
 n n  C
1  sin2  2

x 1/ x 2
 1 1 I =  cos d
 lim = lim 1 1  sin2  tan1  2 tan x 
x   cos ec 2 x n 2  n  = C
2
 1  sin  d
2
ByLHospitalrule 1
=
 1  0   1/ 2 11. (A)
2  1 sin2
sin2 x =+  C
= lim sec 4 
x 2 2 2
 tan4  1d
x 0
5. (B) I =
2 sinxcos x 3 1 1
 lim 0 L.  lim f  x   lim 2a  2  h  b  4a  b = sin x  x 1  x 2  C
x 0 1 x 2 h 0 2 2 1  tan  sec
2 2

  d
ByLHospitalrule R. lim f  x   lim a  2  h   b  4a  b
x 2 h 0
2
9. (A)
1  tan4 
Put tan  = x
loge A = 0  A = e0 = 1 1 dx
L. lim f  x   R lim f  2 
x 2 x 2
I=
2
 1 1 1  x  dx  1  1/ x  dx
2 2
cos sinx
3. (A) 4a  b = 2 and 4a + b = 2 2 2 I =  1 x 4  1
2

A = lim  cosecx 
x x  x   2
 a = 1/2, b = 0 1 dx  
x 0  
2 sin  x   / 4 
loge A = lim x log  cosecx  1  1
x 0
Put x   t   x  2  dx  dt
x  x 

287 288

Solutions  Calculus Vidyalankar : GATE – Engineering Mathematics

dt 14. (C) 16. (D)


I =  1 1 x2
 2 = lim
2
t2  For x  (0, /2), 0 < sin x < x 
e  x log x xn1 x 0
In =  dx x 1 1 x2
sinx nsin x 0
u dv
1 t  x  1/ x  0  1 0   n …(1) dx
 tan1  tan  x2
C x x = lim
2 2  2   x 0
 nsin x   xn  x 1 1 x2
  x   n 1 …(2) In = e  x log x 
1  tan2   1   n 0
= tan1  C  x
2  2 tan   = lim ,
Divide by x 
 e x  xn
x 0
x 1 1 x2
   e x log x  dx
sin x 0
x  n  xifx  0
1 0 …(3)
12. (B) x because x 2
 xifx  0
1 n  1 !
sin xdx nsin x =0+ In+1 
I =  3 sinx  4 sin 3
x
 0
x
n n n
=
1
 2

dx  nsin x   e x xn1dx = (n  1)!
  x   n 1 …(4) 1
3  4 sin2 x   0
But f(0) = 
2

dx (2) & (4) 1 n  1 !
 In = In+1  ….(1)  F is not differentiable of x = 0.
3 cos2 x  sin2 x  nsinx  n n
lim    n1 Replace
sec 2 xdx x 0
 x 
= 
18. (B)
3  tan2 x n  1 !
15. (A) n  n + 1 : (n+1) In + 1  .…(2)
Put tan x = t n z z
 5y  8x  2 ,  5x  2y  1
x y
dt 1  3 t I= 3
 
 t sin  2t  dx  t dy  t dz
2
(1) & (2) : In + 2 – (2n+1)In+1 + n2In = 0
I = 3t 2

2 3
log 
 3  t 
C C
z
 0,
z
0 x=
1 18
  ,y=
dx x y 41 41
 2 cos  2t  17. (B)
1  3  tanx  dt
= log  C Here r = 8, s = 5, t = 2
2 3  3  tanx  f(0) = 0
  dx  2 cos  2t  dt  rt  s2 < 0
lim f  h   0
x 0
Since dy = dt and dz = 2tdt  1 18 
 f(x) is continuous at x = 0   ,  is a saddle point.
13. (D)  41 41 
 t sin2t   2 cos2t 
1

Integrating by parts I =    dt


1 
3
t  2t
3
 1 1 x2  0
f(0+) = lim
 x sinxdx x
2
I= 1 1
x 0

= 2  t sin  4t  dt   t 3 dt
  x 2 cos x   2x cos xdx 1 1 
= 1
 sin x dx
2
= x cos x + 2x sin x  2

2 / 2
=  x cos x + 2x sin x +2 cos x + C 0

=2

289 290
Solutions  Calculus Vidyalankar : GATE – Engineering Mathematics

Answer Key on Test Paper  1 Answer Key on Test Paper  5


1. (B) 2. (D) 3. (A) 4. (D) 1. (D) 2. (B) 3. (B) 4. (A)

5. (A) 6. (A) 7. (B) 8. (D) 5. (A) 6. (A) 7. (B) 8. (B)

9. (B) 10. (C) 11. (B) 12. (D) 9. (B) 10. (C) 11. (D) 12. (B)

13. (B) 14. (B) 15. (B) 13. (C) 14. (D) 15. (A)

Answer Key on Test Paper  2 Answer Key on Test Paper  6


1. (D) 2. (B) 3. (A) 4. (D) 1. (C) 2. (A) 3. (A) 4. (B)

5. (A) 6. (D) 7. (C) 8. (D) 5. (D) 6. (B) 7. (C) 8. (C)

9. (D) 10. (D) 11. (B) 12. (A) 9. (B) 10. (C) 11. (A) 12. (B)

13. (D) 14. (D) 15. (A) 13. (C) 14. (A) 15. (C)



Answer Key on Test Paper  3


1. (C) 2. (D) 3. (A) 4. (B)

5. (A) 6. (B) 7. (C) 8. (A)

9. (A) 10. (B) 11. (A) 12. (A)

13. (A) 14. (B) 15. (C)

Answer Key on Test Paper  4


1. (C) 2. (C) 3. (D) 4. (D)

5. (C) 6. (B) 7. (B) 8. (C)

9. (D) 10. (A) 11. (D) 12. (B)

13. (A) 14. (D) 15. (C)

291 292

Solutions  Calculus Vidyalankar : GATE – Engineering Mathematics

Model Solution on Test Paper  1 10. (C) 13. (B)


1 1 
x 1 2x 1 1 
1. (B) 5. (A) A= 0 1 x2
dx   
2 0 1 x2
dx A=   x  x  1 dx
1
3
3 sinx  4 sin x  sin x Put 1 + x = y, so that x  0  y  1
Lim 1/ 2 1    n x   n x  1 1

x 0 sinx (1 x)n  1 yn  1
=
  1 x2
    =1
 0
= Lim(2  4 sin2 x) = 2 x y 1 = n2
x 0

(1 x)n  1 yn  1 11. (B)


2. (D)  L im = L im 14. (B)
x0 x y 1 y  1
By Stoke’s formula There is a discontinuity at x = 2
1
Limsin does not exist because = n(1) n 1
 
x
 
x 0 2 3 2 3
V =  e 2x
dx  e 4x
dx dx dx dx
1 =n 0 0
So x2  x2 x2
sin oscillates. 1 1 2
x v
6. (A)     n x  2 1    n x  2 2
v 2 3
 1
=  lim  e 4x dx   lim   e 4x  
3. (A) v 
0
v 
 4 0
7. (B) = neither limit exists.
sin3x sin3x 3x  
Now, L im
x0 5x
= L im
x0 3x
.
5x = 
4 v 

lim e 4v  1 
4
 Therefore integral diverges.
8. (D)
sin3x 3 15. (B)
= L im . 
1

1
2 x n2 x  2  nx 
2
x 0 3x 5  dx 12. (D)
x By successive application of L'Hospital
3 3 There is discontinuity at x = 1. Then
  nx 
p
=1× = 1 
5 5 =     nx   9
dx
1
dx
9
dx rule, lim 0.
 2 x
  x  1   x  1  x 
=
4. (D)  x  1
2/3 2/3 2/3
0 0 1
  nx 
p
1
=    n      n2  
1 1
Left Hand Limit, Hence  1 for sufficiently large
 n2 1/ 3 1 9
x
L im [x] = L im [2  h] = 3  x  1   3  x  11/ 3 
0 1
x 2 h 0 x. Thus for some x0.
9. (B) =3+6=9
= L im 1 1 1
 n x
p
h 0 There is discontinuity at x = 2. Thus If x  x0  x,  .
  nx 
p
x
=1 5
dx
2
dx
5
dx
Right Hand Limit,
 3
x2
  3
x2
3
x2

dx

dx
1 1 2 So   nn x  p
  nn x   .
L im [x] = L im [2  h] 2 5 e e
3 2/ 3  3 2/3 
x 2 h 0
   x  2  +   x  2 
2 1  2 2 Hence the integral is divergent.
= L im 2
h 0

=2 
3
2
 3
9 1  

 L.H.L. L im [x]  R.H.L. L im [x]


x  x 2

293 294
Solutions  Calculus Vidyalankar : GATE – Engineering Mathematics

Model Solution on Test Paper  2 L.H.L. L im f (x) = R.H.L. L im f (x)  f (a)


xa x a
= tan1    3   tan1  0  3 

1. (D) L im f (x)  f (2)  f (x) is discontinuous at x = a, 


x2
  tan1 3
2
Left Hand Limit, where ‘a’ is a rational number.
 f (x) is continuous at x = 2.
L im f (x) = L im 2 x = 4 Similarly we can prove that at x = b (an 8. (D)
x 2  x 2
3. (A) irrational number) f (x) is discontinuous. 
dx 1 xa

Right Hand Limit,


Left Hand Limit,  f (x) is discontinuous at every real
A=  2
a 1 x  a
2

2a
n
x  a a 1
L im f (x) = L im x 2 = 4 point.
x 2  x 2 L im f (x) = L im(1 h)  [1 h] 1
x 1 h0 =  n 2a  1
Thus, L im f (x) = 4 2a
x2 = L im (1  h)  0 = 1 5. (A)
h0

But,f (2) = 2  L im f (x) Left Hand Limit, 9. (D)


x2 Right Hand Limit,
1 1
Thus the given function is L im f (x) = L im (1 h)  [1 h] L im f (4  h) = L im 4(4  h)  3 1 1
x 1 h0 h0 h0
x
0
k
dx  lim
v 0  x
v
k
dx
discontinuous at x = 2. = 19 ….(1)
= L im (1 h)  1 1
But if we change f (x) = 4 at h0
Right Hand Limit, 1 
 lim x  k 1 
x = 2, then the given function f = L im h = 0 v 0  k  1 0
h 0 L im f (4  h) = L im 4(4  h)  3
h0 h0
becomes continuous at x = 1. 1  1 
L.H.L. L im f (x)  R.H.L. L im f (x) = lim 1  k 1  .
Hence 2 is the removable x 1 x 1 = 19 ….(2) v 0  1  k  v 
discontinuity. f (x) = x  [x] is discontinuous at x = 1. From eq. (1) and eq. (2),
If k > 1 this limit is + whereas if k < 1
 L im f (x) = 19
x 4 1
4. (D) the limit is
2. (B) Also, f (h) = 3 . 4 + 7 1  k 
At x = a (say), where ‘a’ is any rational
Left Hand Limit, = 19
number.
L im f (2  h) = L im (2  h  1) L im f (x) = f (4) = 19 10. (D)
h0 h0
Given, f (x) = 1, when x is rational. x4
a a
=1 ….(1) Left Hand Limit,  f (x) is continuous at x = 4. dx 1 ax
Right Hand Limit,
a 2
 x2
=
2a
 n
ax 0
L im f (x) = L im f (a  h) 0
xa ha
L im f (2  h) = L im  [2(2  h)]  3  6. (D) 1
h0 h0 = L im ( 1) =  n a  x
h0
/2
/2 2a
=1 ….(2) I= 0 cot xdx  ln  sinx  0 =+
(because a  h is an irrational number) 1 
a
From eq. (1) and eq. (2),   n  a  x 
Right Hand Limit, 2a 0
L.H.L. = R.H.L. 7. (C)
L im f (x) = L im f (a + h) =+
L im f (x) exists. x a h0 
dx

dx
x  x  3
x2
A=  Thus the integral diverges.
= L im ( 1) =  1 2
 6x  10 2
1
f (2) = value of f (x) h0 0 0

=2.23=1 But, f (a) = 1  tan1  x  3  


295 296

Solutions  Calculus Vidyalankar : GATE – Engineering Mathematics

11. (B) 14. (D) Model Solution on Test Paper  3


The value of mod function cannot be f(x) = x5  5x4 + 5x3  1
1. (C) a x  bx
less than 0 f (x) = 5x4  20x3 + 15x2  L im
2 x 16  1
x0 x
f(x) = | x  5x + 2 | = 0 3
f (x) = 20x  60x + 30x2
L im 5
x 1 x 1 a x 1 b x 1
f (x) = 60x2  120x + 30 = L im  L im
x 0 x x 0 x
12. (A)  x  1 x  1 16 5
for maxima or minima f (x) = 0 = L im    a
 sinx  cos x  x 1
 x 1 x 1 = log a  log b = log  
let f(x) =  sin  x    4  x4  4x3 + 3x2 = 0  x = 0, 1, 3 b
2 x 16  1 x5  1
f (0) = 0, f (0) = 30  0 = L im  L im
since maximum value of sin  is 1 x 1 x 1 x 1 x  1 4. (B)
f (1) =  10 < 0
Left Hand Limit,
 maximum is 1 = 16  5
f 3 = 90 > 0
|0  h|
Hence f (x) is neither maximum and 16  xn  an  L im f (x) = L im
13. (D) =  na n  1  x 0 h0 0 h
 Lx 
im
minimum at x = 0 f (x) is maximum at 5  0 xa 
let y = xx h
x = 1 and minimum at x = 3 = L im =1
log y =  x log x h0 h
Hence 1 maxima and 1 minima 2. (D)
dy/dx = 1/xx (1 + log x) Right Hand Limit,
2 2 x2  4
d y/dx =  [ y/x + (1 + log x) L im |0  h| h
15. (A) x0
x  2  3x  2 L im f (x) = L im = L im = 1
{  y (1 + log x)}] x 0 h0 0h h0 h

f(x) = x3/3  3x2/2 + 2x x2  4 x  2  3x  2


for maxima and minima dy/dx = 0 = L im  L.H.L. L im f (x)  R.H.L. L im f (x)
f (x) = x2  3x+ 2 x 2 x  2  3x  2 x  2  3x  2 x0 x0

 1/xx (1 + log x) = 0  x = 1/e


f (x) = 2x  3 Hence f (x) is discontinuous function of
(x  2)(x  2) ( x  2  3x  2)
  = L im first kind at x = 0.
d2 y  1  2 1 For maxima minima x2 (x  2)  (3x  2)
  0  
dx 2  x  1  1 1/e  1 f (x) = 0 x3  3x + 2 = 0
e (x  2)(x  2) ( x  2  3x  2)
e
   e  = L im 5. (A)
 (x  1) (x  2) = 0 x2  2 (x  2)
Left Hand Limit,
e 1/e
 e   0  x = 1, 2
(x  2)( x  2  3x  2)
= L im f (0  h)  f (0)
Hence maximum at x = 1/e f (1) = 2  3 = 1 < 0 x2 2 f ‘(0) = L im
h0 h
1 f (2) = 2  23 = 1 > 0 4 ( 4  4)
Maximum value =  e1/ e = =8 1
 1
1/e
f(1) = 1/3  3/2 + 2 = 5/6 2 = L im h sin =0
h0 h
e
  f(2) = 8/3  6 + 4 = 4/6 = 2 / 3 Right Hand Limit,
3. (A)
f (0  h)  f (0)
ax  bx (a x  1)  (b x  1) f ‘(0) = L im
Now, = h0 h
 x x
1
h2 sin 0
a x 1 b x 1 h
=  = L im
x x h0 h

297 298
Solutions  Calculus Vidyalankar : GATE – Engineering Mathematics

1 2y 12. (A) 14. (B)


= L im h sin = 0 fy = ,
h0 h x2  y2 If z = x f(y/x) then A normal vector to tangent plane will
 L.H.L. f ‘(0) = R.H.L. f ‘(0) = 0
x 2
y 2
  2   2y  2y   2  x  y  2 2
x
z
y
z be (2x, 2y, 2z) = 1,1, 2  
fyy  x y
x y  x  y 
2 2
2 2 2 2
Hence equation of tangent plane is
6. (B)
Hence fxx + fyy = 0 
 x  xf   y / x  y / x 2  f  y / x  
    1  1  1 
x
y= x  x 2    y  2   2 z  0
e xyf   y / x  1/ x      2
9. (A)
log y = log x  x  xf  y / x   z Or x + y + 2z  2
By chain rule
1 1 dy 1  dz dx dy
dy / dx   1   y   1  0
y x dx x   fx  fy 15. (C)
dx dx dx 13. (A)
x=1 1  dh
2

 4x  3y   3x  2y  cos x
3
 let c (x, y) = f (x, y) xy, V =
3
r h
dt
1 d2 y 1  dy  1  c 2
 2  1  1  0 then
y dx 2 y 2  dx 
then V dr v dh
x 10. (B) xy  
r dt h dt
1 d y 2
d y 2 dz z dx z dy c (x, y) = A (x) + B (y)
 
  0  1  2   e  0 dt x dt y dt 
2 1
rh  2   r 2  3 
e dx   x 1 dx  x 1  f (x, y) = A (x) + B(y) + xy 3 3
Hence maxima at x = 1 x dx 2y dy dx
  since 5 4 
8 dt 9 dt dt =    5000   2500 
1 3 
Maximum value  ymax = and
e
12500
dy  mm3 / s
5x + y 0 3
dt
7. (C) 
sowhenx  2andy  1,
Assume that there is such a function.
Then fxy and fyx will be continuous dy / dt  10and

everywhere. Hence fxy = fyx Thus  ex dz 2 2 125


  5    10  cm / s
sin y = ex sin y = 0 for all y which is dt 8 9 36

false
11. (A)

8. (A) f k
Let k (x, y) = , then 0
x x
2x
fx = , then k (x, y) = g(y).
x2  y2
f
Hence = g(y) then
fxx 
x 2
 y2   2   2x  2x   2  y  x  2 2
x

x  y  x  y 
2 2
2 2 2 2
f(x,y) = xg(y) + h(y) for a suitable
function h

299 300

Solutions  Calculus Vidyalankar : GATE – Engineering Mathematics

Model Solution on Test Paper  4 10. (A) dy



a2
/2 a cos  dx 2 x  a  x 3 / 2
 / 2 d0 a  r rdr
2 2
v=
1. (C) 7. (B)
/2 a  sec  cos   2 2 2  a2 x 
2. (C) Putt  a  r 2
 fromcurvey 
A=2 0 a sec  rdrd
 a x

a sin 
3. (D) a  sec  cos   a sin  2 t3 
= 
/2
t dt  
t2
 0 r 2 
a sec 
d a

3 a V = a2 
 ax
dx
s = 3t 3   15 0
x
0  sec   cos  
/2
2 = a2
2
 sec 2  d 1
= a 3 1  sin3   Put x = a sin2 
ds 3
speed 9t 2  t /2
dt =a 2
  2  cos2   d /2 2 a 3
0
11. (D)
V = 2a3  0
cos2  d 
2
2 2
ds ds 1
 18t1 2  0 t 5a 2 2 1 x 2
=
1dx 2 1x 1  x  y / 4  dy
dt 2 dt 18 v =
1 2 2 14. (D)
4 2
3
ds  ds
again; 3  18  0, hence speed 8. (C) 2 1 x2
S = 2  yds  2  y d
dt  y3 
 d
 
1
4 x/2 =   1 x2 y  
has maxima. Mass =k 0 dx x 3/2
ydy 1
 12 2 1 x2
 4  y
 ds
d
ds
4 0 d
8
 3 1  x 
Maximum speed = 1 2
3/2
dt t  1 x/2 = dx  
 y2  1/ 2
ds  dx   dy  
4 1 2 2
18 =k  0
  3 / 2 dx
 2 x
        2a cos   2
2 d  d   d  
 1 1 4
12. (B)
 9   
 18  18 k  x3 x4  2
4
ax   
     k
0
v =   y 2 dx   x 2
0
dx S = 4  2a sin2 2a cos d
1 1 2  12 64  0 3 a a ax
0 2 2
=  
36 18 put z = a  x so 32a 2
9. (D) = .
2 1 3
 a  z   2a  z 
2
= 2a
Equation of circle r = 2acos 
36 v= 0 z
dz
/2 2a cos 
15. (C)

1
speed units. Mass = k   / 2
d 
0
r 2 dr
2a  2a
3

36     5a 2  4az  z2  dz 4 0
k /2 2acos
0
 z  V = 2 
2 x  x  2   8
 5  x  dx dy
3  / 2   0
4. (D) = d r 3 
3
= 2a [log2  2/3] 4
5. (C)
8a 3k  / 2
= 2 2  5  x  8  x  x  2 dx
3 0
2 cos3 d
6. (B) 13. (A) 4
2 40  18x  3x  x 3  dx
2
= 2

V = 2    a  x  dy
2
2 2x 2  4x  7 32 3
A =  
1 3x 2  x  3
dx dy =
9
ka 0
= 2  216 = 432 

  dy
2 a
= 5  dx,  x 2  x  2  2   a  x 
2
dx
1 0 dx
2
 x3 x2  45 
= 5    2x  =
 3 2  1 2

301 302
Solutions  Calculus Vidyalankar : GATE – Engineering Mathematics

Model Solution on Test Paper  5 6. (A) 10. (C)


By Cauchy’s mean value theorem, y
2
4 2
1. (D)
 f (e) =
2  3 1
 3 0 f b   f  a  f  c 
I=  x dxdy   2
dy 
y2 / 4
xdx
1 1 e3 e 
Let f(x) = x +  f (x) = 1  2 g  b   g  a  g  c  y / 22
x x f(x) is maximum at x = e 4  x2 
Choosing f(x) = ex, g(x) = cx
=  2
dy  
 2  y2 / 4
 f   x   2 / x 3 loge 1
maximum value f (e) =  eb  ea ec ab
e e   c 4
For maxima and minima f (x) = 0 eb  e a e c 2 1  y3 2 y5  72
=   y  4y   
1 2  12 80  2 5
 1 =0x= 1 4. (A)
x2
7. (B)
2  x  x2
Total SP = x  3  3x 11. (D)
f   x   20  1000  1000 f   x  = F (x)
13 b b

f(x) is minimum at x = 1 x let (x) = f(x)  F(x)


I= 0 0 y  2b  y dx dy
Total CP =  200
2 (x) = f (x)  F (x) = 0 b
1 b  y3 
minimum value = f (1) = 1 + =2
0 by
2
Profit,f(x)SP CP =   dx
1 3 0
 (x) = constant
5 x2
 x  200 b  3 b3  4b4
2. (B) 2 1000
8. (B)
=  0
b 
 3
dx 
 3
Let  be semi vertical angle of 5 2x
 f '(x) = 
inscribed triangle 2 1000
9. (B) 12. (B)
Let S be the area 2
f ''(x) 0 /4 2 sin 
 0 
4
1000 r cos drd I=2 r 3 drd
1
Then s =  2x sin 2  (r + r cos 2)
2 5 2x  a 1 cos   4
  0 x1250 /4

dS 2 1000
 0 cos 0 r drd =8 0 sin4 d
= 2r2 (cos 4 + cos 23) = 0
d 5
1 a 1  cos 2 
 2

0 1  cos 
5
 f''(1250) negative = cos d 8
 
d
  = /6 500 5  4
S is maximum at  =  / 6 So, profit is maximum when x = 1250 32a 5  10   2   /2
0 cos 2  2 cos 2  1 d 0 1  cos  ....    2
2
 = d
5
vertical angle = 2 = /3
5. (A) 64 5  / 2  1   3
= a   2cos12   cos10  d =  2    2
5 0 2 2  
3. (B) By implicit differentiation w.r.t. x

1  log x
z z  
y y x z 0 …   
f (x) = x x 2 
x2
z z y  z 21 5
 1  log x = 0  x = e   x  y   (y  z)  = a (by reduction formula)
x x y  x 16
2log x  3
f (x) =
x3

303 304

Solutions  Calculus Vidyalankar : GATE – Engineering Mathematics

13. (C) 15. (A) Model Solution on Test Paper  6


 a 1 cos   a 2a  x
I= 0 0 2r 4 sin3  dr d 0
dx  2
x /a
xy dy
1. (C) 7. (C)
2a 5  1
a
 x4  2 4  2y

 1  cos   1
5
  x dx (2a  x)2  2 
3
= sin  d
5 0
20  a  2. (A)
I=  
0 0 2y  y 2
dxdy

2a 5
  2z 
2
1   x5 
6 a
=  z7 dz 4  2y

2 0 
2 3
5 0   4a x  4ax  x  2  dx 3. (A)
2   x
a  =   
0  2y  y 
2
dy
….(z = 1 + cos) 1 0
1 4 1 a4  lim  x  a  cos
26   2a 4  a 4  a 4   x a x a 2
= a 5 2 3 4 6 2 2y
35 1
  y 2y
dy
4  limhcos [x  a  h] 0
3a h 0 h
 2
14. (D) 8 = 2  2y1/ 2   4
x2 = 0  some finite a + y = 0 0
y
x log  x  a   y 2 
a
a a
I= 0
dx
x  a  2 
2   (a, a) 8. (C)
2ax  x 2 4. (B)
y=2ax 3 2

1 a x log  x  a  2 V=   3x  4y  dA     3x  4y  dxdy
2 0
= dx  a  2ax  x 2 
x  a  5. (D)
2 R 0 1

3 2
3 
Integrating with parts, we get 6. (B) =   x 2  4yx  dy
0
2 1
1 a
x log  x  a  dx
2 0
1 5x
=
  y dydx
2
I= 3
9  9
3
 63
  2  4y  dy   2 y  2y
2
0 2x =   2
a 2
0 0
= 2loga  1 1
1 3 1
5x 1
8
= 0 3 y  dx  3 0 125x  8x dx
3 3
  9. (B)
  2x
2 4

 y dA    y dydx
2 2
1 V=
39 4  39
= x   R 0 0
4 0 4
2 4 2
1  64
   y 3  dx   dx
0
3 0 0
3
5
64 128
= 2 
3 3

10. (C)
1 3  2x
A  1dA    dydx
1 R 1/ 2 1/ x

1
 1

1/ 2
  3  2x  x  dx

305 306
Solutions  Calculus

Solutions  Probability and Statistics


1
=  3x  x 2   nx 
1/ 2 13. (C)
3 1 1 a cos 2 
  3  1 0      n 
/4
I=  d    r   2

2 4 2 /4  0 Answer Key on Assignment  1


1. (C) 2. 0.65 to 0.68 3. (C)
 
/4
5 
= 2     n2   
 / 4
d a 1  cos 2  a
4  4. 0.4 to 0.4 5. (D) 6. 0.25
/4
 2 cos   1d
=
3
  n2
=a 
 / 4   7. (A) 8. 0.07 to 0.08 9. (A)
4  
= a  2 sin    10. 0.55 to 0.55 11. (D) 12. (B)
 
11. (A)
13. 0.43 to 0.45 14. 0.26 to 0.27 15. (B)
 
 a  2   / 2   2a 1  
V   2x  2y  1 dA
R
 4 16. 1.06 to 1.07 17. (B) 18. 6
2 x
19. 0.32 to 0.34 20. (A)
    2x  2y  1 dydx
0 0
14. (A)
1 x 2x y

2
x
I=  0
dx  2 dy 
x 0
dz
Answer Key on Assignment  2
  2xy  y 2  y  dx
0 1
dx  2 dy  z 0
x 2x y 1. 0.64 to 0.66 2. 0.1276 to 0.1372 3. 0.79 to 0.81
0 =  0 x

 1 
2 4. (B) 5. 0.96 to 0.98 6. 0.23 to 0.25
1 x
=  x3  x2  =  dx  2  2  x  y  dy
 2 0 0 x
7. 99.6 to 99.8 8. (A) 9. 2.5 to 2.5
= 8  2 = 10  1 x x4 
=   2  x  x    2  x  x 2  dx 10. (C) 11. 0.25 to 0.27 12. (B)
0
 2 2
13. (B) 14. 0.25 to 0.28 15. (D)
12. (B) 11
= 16. (B) 17. (D) 18. 0.39 to 0.43
2  1 +cos 30
A=  dA    rdrd
19. 54.0 to 54.0 20. 0.33 to 0.34
R 0 15. (C)
1 cos 
x 
2 2 1
1 2
 dy  Answer Key on Assignment  3
2
I=  y 2 dx
 
0
r
2  0
d 1 y 1

y 1. 49 to 51 2. 49.9 to 50.1 3. (B)


2  x3 


1 = dy   y 2 x 
=
2 0

1  2 cos   cos2  d  1
3  y 1 4. (B) 5. (C) 6. (C)

7. 0.9 to 1.1 8. 54.49 to 54.51 9. 0.5 to 0.5


1  y  1 
3

1  1  cos 2  2
   y2   y 2  y  1  dy
2  
 1  2 cos    d =
2 
1 3 3  10. (B) 11. (A) 12. (D)
 
1  3  3 2 13. 0.35 to 0.45 14. (B) 15. (D)
=  2   y 2y 3  y  14 y 4  2
2  2  2 =     = 16. 4 17. 6 18. (A)
 3 3 12 4 3
1
19. (A) 20. 2.0 to 2.0


307 308

Solutions  Probability and Statistics Vidyalankar : GATE – Engineering Mathematics

Answer Key on Assignment  4 Model Solution on Assignment  1


1. (A) 2. (D) 3. (B) 1. (C) Alternative
We know that mode is the value of the Suppose there are total 10 families.
4. (C) 5. (A) 6. (C)
data which occurred most of 10
7. (C) 8. (A) 9. (B)
 17 is mode.
10. (D) 11. (C) 12. (B) One child per  5 5 Two child
2. 0.65 to 0.68 family per family
13. (A) 14. (C) 15. (A)
N Number of children in Number of children in
If there are N families, then have these 5 families is 5 these 5 families is 10
16. (B) 17. (C) 18. (C) 2
N  Total children = 15
single child per family and have two
2
Answer Key on Assignment  5 Required probability =
10
=
2
= 0.67
children per family. 15 3
1. (A) 2. (C) 3. (D) N 3. (C)

4. (A) 5. (D) 6. (C) 1 1


P(Head) =  P(Tail) =
Single child N N Two children 2 2
7. (A) 8. (C) 9. (D) per family 2 2 per family
P(Fourth Head in tenth toss)
10. (A) 11. (D) 12. (A) 3 6
9  1  1 1
= C3       0.082
13. (A) 14. (A) 15. (C) N N 2 2 2
 Total children = 2 = N
2 2
16. (D) 17. (C) 18. (C)
N 4. 0.4 to 0.4
 Total number of children = N
2
Given, probability density function of
Answer Key on Assignment  6 3
= N evaporation E is
2
1. (B) 2. (D) 3. (C) 1
If a child picked at random should  , 0E5mm / day
f(E) =  5
4. (C) 5. (A) 6. (C) have a sibling, then that child should  0, otherwise
7. (C) 8. (B) 9. (A) come from the family which has 2
As, the probability has to be find
10. (D) 11. (C) 12. (A) child. There are N such children.
between 2 and 4 mm/day, therefore
N 2
13. (B) 14. (A) 15. (A)  Required probability = = 4 4 1
3
N
3 P(2 < E < 4) = 
2
f(E)dE = 
2 5
dE
16. (B) 17. (D) 18. (D) 2
1 4 4 2 2
= 0.67 = E 2 = =
5 5 5
  P(2 < E < 4) = 0.4

309 310
Solutions  Probability and Statistics Vidyalankar : GATE – Engineering Mathematics

5. (D)

a
+
b
=
2  0.5 = 1  P(Y) E(x2) = x 2
P(x)
We have Probability distribution 2 3 3  P(Y) = 1  0.5 = 0.5
= 1  0.3 + 4  0.6 + 9  0.1
function of Normal Distribution  3a + 2b = 4 ….(2) Then,
= 0.3 + 2.4 + 0.9 = 3.6
(x  )2 Solving (1) & (2) we get, a = 0, b = 2 P(X  Y) = P(X) + P(Y)  P(X  Y)
1 2 Variance v(x) = E(x2)  2
f(x) = e 2 … (1) 0.5 0.5
= P(X) + P(Y)  P(X) P(Y)
 x P(X < 0.5) =  f(x) dx =  2x dx = 3.6  (1.8)2
2 0 0 = 0.4 + 0.5  0.4 × 0.5
Variance =  is lowest S.D() =  v(x)
 x2 
0.5
 P(X  Y) = 0.7
  also lowest
=  2   = (0.5)2 = 0.25 =  3.6(1.8)2 = 0.36
 2 0
 If  decreases  f(x) increases 10. 0.55 to 0.55
= 0.6
( from (1)) There are equal number of bulbs of
 Curve will have highest peak 7. (A) two different types. 12. (B)

8. 0.07 to 0.08 Probability that the bulb selected is of Given  = 5.2


6. 0.25 1 Let x be random variable which follows
P(Head) = 0.3  P(Tail) = 0.7 Type 1 P(T1 )   0.5
As f(x) is a pdf, 2 Poission's distribution
P (getting Head first time in fifth toss)
1 Probability that the bulb selected is of P(x < 2) = P(x = 0) + P(x = 1)
= 0.7  0.7  0.7  0.7  0.3
 f(x) dx = 1
1 e  0 e  1
0
= 0.072 type 2 P  T2    0.5 =  
2 0! 1!
1
 
0
(a + bx) dx = 1 9. (A) Let E be the event that a bulb selected = e5.2 (6.2)
P(X) = 0.40 at random lasts more than 100 hours.
= 0.0055 × 6.2 = 0.034
1
 x2  P(X  Yc) = 0.7 P(E/T1) = 0.7
  ax  b  = 1
 2 0  X and Y are independent events P(E/T2) = 0.4 13. 0.43 to 0.45

b P(X  Y) = P(X) . P(Y) ….(1) By total probability Parcels pass sequentially through two
a+ =1
2  X and Yc are also independent P(E) = P(T1) P(E/T1) + P(T2) P(E/T2) post offices.
 2a + b = 2 ….(1) P(X  Yc) = P(X) . P(Yc) ….(2) = 0.5  0.7 + 0.5  0.4 Probability that parcel reaches first
2 Now,  P(E) = 0.55 post office
Also, E(X) =
3 P(X  Yc) = P(X) + P(Yc)  P(X  Yc) P(01) = 1
11. (D)
1
2 P(X  Yc) = P(X) + P(Yc)  P(X) P(Yc) Probability that parcel is lost by first
  x f(x) dx =
3 c c
Given
post office
0  0.7 = 0.4 + P(Y )  0.4 P(Y )
x 1 2 3
1 c
p(x) 0.3 0.6 0.1 1
2  0.7  0.4 = P(Y ) [1  0.4] P(L/01) =
  x (a + bx) dx =
3  0.3 = 0.6 P(Yc)
5
0
mean () = exp(x) Parcel will reach second post office if it
 x2
1 0.3 = 1  0.3 + 2  0.6 + 3  0.1
x3  2 P(Yc) = 0.5 is not lost at first post office.
 a  b  = 0.6
 2 3 0 3 = 0.3 + 1.2 + 0.3 = 1.8 Probability that parcel reaches second
 P(Yc) = 1  P(Y)
post office

311 312

Solutions  Probability and Statistics Vidyalankar : GATE – Engineering Mathematics

4 (out of 100) are considered, of the 5 18. 6 I) Red ball in first draw
P(02) =
5 students get more than 90% Var(x) = E(x2)  (E(x))2 5
P(red in first draw) =
Probability that parcel is lost by  5 out of 20 is the probability  1 12
 x. 2| x | e dx
|x|
E(x) =
second post office Or 1/4 is the answer. When the red ball is placed back with
1 Alternative Method: 1 0 2 x 1  another red ball, there will be total 13
2 
P(L/02) = =  x e dx  x 2 .e x dx
5 Let the student pass the examination 2 0 balls with 6 balls of red colour.
By Total Probability, be A and student pass the examination =0 6
P(red/red in first draw) =
Probability that parcel is lost and got about 90% marks be B 1  13
E(x) =  x . | x |e |x| dx
2 2

P(L) = P(01) P(L/01) + P(02) P(L/02)  2 II) Green Ball in first draw
Now P(A) = 20% and P(A  B) = 5%
1 4 1 9 1   7
= 1   =  B  P(AB) 5% 1 = 2   x 2 .x.e  x dx  P(green in first draw) =
5 5 5 25  P     2 0  12
A P(A) 20% 4
By Baye's Theorem, knowing that the  When the green ball is placed back
16. 1.06 to 1.07
= 
0
x 3 .e x dx
parcel is lost, probability that it was with another green ball, there will be
2 
lost by second post office e x  total 13 balls with 5 balls of red colour.
Mean = 0 x f(x) dx = x 3 .
1 0
(3x 2 )(e x )
0

P(02)P(L 02) 4 25 4 5
P(02/L) =   P(red/green in first draw) =
P(L) 9 25 9 2   13
1  x3 x5  6x(e x ) 6e  x
= 4   0 0
P(02/L) = 0.444 4  3 5 0 5 6 7 5
=6  P(red) =   
12 13 12 13
14. 0.26 to 0.27 1  32 32 
=  = 1.0666  Var(x) = 6  0 = 6
4  3 5  =
30

35
Given, mean  = 5 156 156
The probability that there will be less 17. (B) 19. 0.32 to 0.34 65
 P(red) =
than 4 penalties in a day is 156
20. (A)
P(y < 4) = P(y = 0) + P(y = 1)
5 red and 7 green balls
+ P(y = 2) + P(y = 3)
5 0 5 1 5 2 5 3
Total 12 balls.
e 5 e 5 e 5 e 5
=    A ball is drawn and placed back along
0! 1! 2! 3!
with another ball of same colour, this
5 5 5 5 
0 1 2 3
= e5      can be done in two ways:
 0! 1! 2! 3! 
= 0.006737 [1 + 5 + 12.5 + 20.833] 
= 0.006737  39.33 = 0.2649
 Required probability
15. (B) 1 2 1 3 1 4 1
=      
6 6 6 6 6 6 6
Given that the student is passing
15 5
exam, i.e., if only 20 students  
36 12

313 314
Solutions  Probability and Statistics Vidyalankar : GATE – Engineering Mathematics

Model Solution on Assignment  2 10. (C) Here, n  3 = 1 is not possible


P(p < 3) = P(p = 0) + P(p = 1) Taken n=3
1. 0.64 to 0.66 1 2
Put x =y  x dx = dy + P(p = 2)  S = {HHH, HHT, HTH, HTT, THH,
Let M  men 2
e   e  e    THT, TTH, TTT}
W  women

1 =  
 E( X ) = 2  0 2
e  y dy 0! 1! 2! and X = 3, 1, 1, 2
E  employed (where µ = 3) Here n  3 = 0 is not possible
U  unemployed 1
=2 (1) 3 3e 3  9 Similarly, if a coin is tossed n times
2 = e e 3
Given P(M) = 0.5 2 then the difference between heads
P(W) = 0.5 = 0.7979
 9  17 and tails is n  3 is not possible.
= e 3  1  3    3
P U M  = 0.20  2  2e
4. (B)  Required probability is 0.
P U W  = 0.50 The probability of getting 'tail' when the 11. 0.25 to 0.27
14. 0.25 to 0.28
1 2 1
By Total probability, coin is tossed again is p= =
6 3 Given: Poisson distribution,
2
 U M P(W)P
P(U) = P(M)P  U M 
5. 0.96 to 0.98 1 2 e  x
q = 1 = p(x) = ; x = 0, 1, 2, ...
= 0.5 × 0.20 + 0.5 × 0.50 = 0.35 3 3 x!
P(at least one will meet specification)
Required probability = P(E) Using Binomial distribution  = 240 per hour = 4 per minute
= 1  P(none will meet specification)
= 1  P(U) p(x ≥ 2) = 2 per 30 seconds
= 1  (1  0.8) x (1  0.7) x (1  0.5)
= 1  0.35 = 0.65 2 1 3 0 e 1
= 1  0.2 x 0.3 x 0.5  1  2   1  2  P(X = 1) = = 2  e  2 = 0.2706
= 3C2     3C3     1!
2. 0.1276 to 0.1372 3 3 3 3
= 1  0.03 = 0.97
By Poission's distribution 15. (D)
6 1 7
6. 0.23 to 0.25 =  =
p(h≥8) = e 8 27 27 27 Player A Starts the game
E(X) = (0q) + (1p) = p Player A can with the game in the
900 12. (B)
= = 0.25
3600 E(X2) = (02q) + (12p) = p following manner :
Variance = E[X2]  [E[X]]2
80.25 V(X) = E(X2)  [E(X)]2 A wins by getting 6 in the first try)
p(h≥8) = e 0.1354 We know that, variance  0
= p  p2 = p(1  p) = pq 2 2
A loses, B loses, A wins
3. 0.79 to 0.81  E[X ]  [E[X]]  0
= 0.60.4 = 0.24 A loses, B loses, A loses, B loses, A
 E[X2]  [E[X]]2
1 wins and so on,

 x2 / 2
E( X ) = x e dx
 2 7. 99.6 to 99.8 P(A winning)
13. (B)

1 For SN distribution, percentage of area 1 5 5 1 5 5 5 5 1
= 2   | x | e  x
2
/2
dx Let X = difference between the number =          .....
2 from 3 to 3 is 99.74 6 6 6 6 6 6 6 6 6
0
of heads and tails.
1  25 25 25 
….(even function) = 1    ....
8. (A) Taken x=2 6  36 36 36 

1 2
 S = {HH, HT, TH, TT}
=2 0 x e x 9. 2.5 to 2.5
/2
dx Which forms a G.P.
2
and X = 2, 0, 2

315 316

Solutions  Probability and Statistics Vidyalankar : GATE – Engineering Mathematics

 P(A winning) From above table, Model Solution on Assignment  3


  P{X + Y + Z}  P < Z  B
1  1   9  P{Z  1} = P{X = 0 and Y = 1} 1. 49 to 51 P(only one) = P(Ram)  P(not Ramesh)
=   ....  Sn 
6  25   1  r  Given M = 500 + P(Ramesh)  P(not Ram)
1 + P{X = 1 and Y = 0}
 36 
+ P{X = 1 and Y = 1}  = 50 1 7 1 5
1 1 =   
=  P(X > 500) = ? 6 8 8 6
6 11 = 1  P{X = 0 and Y = 0}
36 = 1  pq 12 1
= =
48 4
6
 P(A winning) =
11 18. 0.39 to 0.43 4. (B)
Use Binomial distribution:
16. (B) p(x) = nCx px q n  x 5. (C)
where X follows normal distribution
Use Binomial distribution: Here n = 5, p = 0.1, q = 0.9 1
We know that standard normal
p(x) = nCx px q n  x P(XY) 12 1
P(packet is replaced) variable P(Y/X) = = 
P(X) 1 3
Here n = 10, p = 0.1, q = 0.9 = P(X  1) = 1  p(0) x M 4
P(3 out of 10 are defective) z=
= 1  qn = 1  (0.9)5 
10 3 7 6. (C)
= C3(0.1) (0.9) = 0.0574
= 0.40951 500500
z= =0 Here A and B are two independent
50
events
17. (D) 19. 54.0 to 54.0  (X > 500) = P(z > 0) = 0.50
 P(A  B) = P(A) P(B)
P{x = 0} = P  P{x = 1} = 1  p E(X) = 5 (see figure)
and P(A/B) = P(A)
P{y = 0} = q  P{y = 1} = 1  q  E(X2) = 30, where X  P(),  = 5
2. 49.9 to 50.1 and P(B/A) = P(B)
Let Z = X + Y  E[(X + 2)2] = E(X2) + 4E(X) + 4
Let X be a positive odd number less Also if A and B are independent event
X Y Z = 30 + 20 + 4 = 54 than 100 then A and B are also independent
0 0 0 ( V(X) = E(X2)  (E(X))2)  X = 1, 3, 5, ..., 99 each with event.
0 1 1
20. 0.33 to 0.34
probability
1
50
  
P AB = P(A)P(B)
1 0 1
Hence option (A), (B) and (D) are
1 1 2 1
 E(X) = (1 + 3 + 5 + ... + 99)
50 correct.

 1 Now,
= 10025 = 50
50 P(A  B) = P(A) + P(B)  P(A  B)

3. (B) = P(A) + P(B)  P(A) P(B)

1 1  Option (C) is false.


P(Ram) = ; P(Ramesh) =
6 8
7. 0.9 to 1.1

317 318
Solutions  Probability and Statistics Vidyalankar : GATE – Engineering Mathematics

8. 54.49 to 54.51 W ~ N (0, 2) ie., W has mean  = 0 Probability of selecting BBR 16. 4
Observed speeds (in km/hr) : 66, 62, and variance,   2 2 6 5 4 
=  
45, 79, 32, 51, 56, 60, 53 and 49. 10 9 8
E(e 3X/4) = 0 e 3x / 4 e  x dx
W 0
 p(W  0) = p  
Number of observations = 10     Probability of selecting BRB 

 Median will be the mean of middle = p(Z  0), Z is standard normal variants =
6 4 5
 
= 0 e  x / 4 dx = 4

two values in the ordered set. 10 9 8


1
= 0.5 = P(Red = 1) = sum of above three
Arranging in ascending order : 32, 45, 2
17. 6
49, 51, 53, 56, 60, 62, 66, 79 probabilities = 0.5
11. (A) Let S: getting 3 on a die
53  56 Let 'x' be no. of defective pieces. 13. 0.35 to 0.45 1 5
 Median Speed = = 54.5 p = P(S) = , q = 1  p =
2 6 6
P(0.5 < x < 5)
x 0 1 2 X = number of times die is thrown to obtain
5
9. 0.5 to 0.5 1 2 1 S for the first time
P(x) =  f(x)dx
1 6 3 6 0.5
Given: f(x) = ;a<x<1 X 1 2 3 4 ….
x2 Mean () = E(x) = (x) P(x) 1 4 5
p(X) p qp q2p q3p ….
1
=  0.2 dx   0.1 dx   0 dx
 1  1  1 0.5 1 4
 f(x) dx = 1 =  0  1  2 
 6  3  6 E(X) =  x (qx  1 p) = p  x q x  1
a = 0.2[x]10.5  0.1[x]14  0 1 1
=p = =6
1 2 1 1  q 
2
p
= 0  = 1 = 0.2  0.5 + 0.1  3
 a x  2 dx = 1 3 3
2 2
 P(0.5 < x < 5) = 0.4
1
E(x ) =  x P(x) 18. (A)
 x 1  14. (B)
   =1  1  2   1
 1  a =  0  1  4 
 6  3  6 1
n = 4; p = 19. (A)
1 1 1 6
 1 + =1  =2 a= 2 2 4 4
= 0  = 1 5
a a 2 3 3 3  q = 1 = P(X  4) =  0.25 dx
6 6 1
10. (B) Variance, V(x) = E(x2)  (E(x))2
p(x  2) = 0.25(4  1) = 0.75
p(3V  2U) = p(3V  2  0) = p(W  0), 4 1
= 1 = = 1  p(x < 2)
W = 3V  2U 3 3
= 1  [p(x = 0) + p(x = 1)] 20. 2.0 to 2.0
U, V are independent random 12. (D) Given X(t) = U + Vt
 0
 1  5 
4
 1  5  
1 3
Re d Black = 1  4 C0      4 C1      X(2) = U + 2V
 1 Given,
variables and U ~ N  0,  6
   6 6 6
     E[X(2)] = E[U + 2V]
 4 4 6 
19 = E(U) + 2E[V] = 0 + 2 × 1 = 2
 1 The selection will be RBB or BBR of 
V ~ N  0,   144
 9 BRB

 1 1 Probability of selecting RBB 15. (D)


 W = 3V  2U ~ N  0,9   4  
 4 9 4 6 5 
=  
10 9 8

319 320

Solutions  Probability and Statistics Vidyalankar : GATE – Engineering Mathematics

Model Solution on Assignment  4  Number of favourable case Let A be the event that the flower is
= 13 + 3 = 16 red and B the event that the flower is a
1. (A) 5. (A)
Sample space = 52 rose.
The possible cases in the throw of a Sample space = 6
Probability of getting a spade or an ace  A  B is the event that the flower is
die are six i.e. 1, 2, 3, 4, 5, 6 4, 5, 6 are greater than 3
16 4 4 a red rose.
Cases of even numbers 2, 4, 6 are 3 1 = = 
 Probability =  52 13 9  4 16
three. 6 2 n(A) = 16  P(A) =
Odds against winning the bet are 100
 Probability of getting an even 6. (C)
9 to 4. 10
Sample space = 62 = 36 n(A  B) = 10  P (A  B) =
3 1 100
number = 
6 2 Favourable cases are (5, 6), (6, 5), 9. (B)
P(B/A) = probability that a selected
(6, 6) Events are mutually exclusive
2. (D) flower is a rose red is colour
3 1 odd in favour of horse H1 = 1/3
When we toss a coin, there are two Probability =  P  A  B 10 / 100 5
36 12 1 1 P(B/A) =  
possible outcomes i.e. Head or Tail. P(H1) =  PA 16 / 100 8
1 3 4
7. (C)
 Number of possible cases = 2
There are 366 days in a leap year and it 1 1 11. (C)
The outcome of Tail is favourable Similarly P(H2) =  ,
1 4 5 A throw amounting to 18 must be
has 52 complete weeks and 2 days
event
1 1 made up of 6, 6, 6 and this can occur
 Probability of getting a tail = 1/2
extra. P H3   
1 5 6 in 1 way. 17 can be made up of 6, 6, 5
They are
1 1 which can occur in 3 ways. 16 may be
3. (B) (1) Sunday and Monday P(H4) = 
1 6 7 made up of 6, 6, 5 and 6, 5, 5 each of
Total number of equally likely and (2) Monday and Tuesday
P H1  H2  H3  H4   P H1   P H2  which arrangements can occur in 3
exhaustive cases = n = 6 + 9 = 15 (3) Tuesday and Wednesday
(4) Wednesday and Thursday P H3   P H4  ways.
Number of favourable cases
(5) Thursday and Friday  The number of favourable cases is
= 9 C1  9 1 1 1 1
= + + + = 1 + 3 + 3 + 3 = 10 and the total
(6) Friday and Saturday 4 5 6 7
[ number of black balls = 9] number of cases is 63 = 216
(7) Saturday and Sunday 319
Probability of drawing a black ball =
sample space = 7 420 10 5
 Required chance = 
9 3 216 108
=  Number of favourable cases = 2 10. (D)
15 5
 Probability = 2/7 Suppose there are 100 flowers
12. (B)
4. (C) 8. (A) Number of roses = 40; Number of
Probability that A wins (P1) = 1/6
Number of clubs in a pack = 13 In a pack of 52 cards, 1 card can be carnations = 60
and that B wins (P2) = 1/10
13 1 drawn in 52 ways 25% of 40 = 10 roses are red and
Probability of getting a club =  and that C wins (P3) = 1/8
52 4 10% of 60 = 6 carnations are red
Since there are 13 spades and 3 aces As a dead heat is impossible, these
(one ace is present in spade) are mutually exclusive events, so the

321 322
Solutions  Probability and Statistics Vidyalankar : GATE – Engineering Mathematics

chance that one of them will win the The probability that wife is not selected In the second bag we have five white Number of cases with a total of 9 = 4
race is 1 4
=1 
and 6 black balls and probability of  5,4  ,  4,5  ,  3,6 ,  6,3 
1 1 1 47 5 5 5
P1  P2  P3 i.e.    getting a white ball is .  Total number of cases in which a
6 10 8 120 Probability that only husband is 11
doublet or a total of 9 appear = 6 + 4
1 4 4  Required probability
13. (A) selected =   = 10
7 5 35 1 5 5
The chance of choosing the first bag is =   P(a doublet or a total of 9) = P(A)
Probability that only wife is selected 15 11 165
1/2 and if the first bag be chosen the 10 5
1 6 6 Third way : = 
chance of drawing a red ball from it is =   36 18
5 7 35 One black and one white ball have
5/12 hence the chance of drawing a P(neither a doublet nor a total of 9)
Probability that only one of them is been transferred from first bag to the
red ball from the first bag is = 1  P(A)
4 6 10 2 second so that the probability for this
1 5 5 selected =    5 13
  35 35 35 7 is = 1 
2 12 24 18 18
16. (B) 4
C1  2 C1 8
Similarly the chance of drawing a red  18. (C)
6 15
There are 3 mutually exclusive and C2
ball from the second bag is Let P(A), P(B) be the probability of A &
exhaustive way in which 2 balls are In the second bag we have 6 white B speaking the truths, then
1 3 1
  . Hence, as these events
2 15 10 transferred form first bag to second bag. and 5 black balls and the probability of 75 3 80 4
P(A) =  , P(B) = 
are mutually exclusive, the chance First Way : drawing a white ball is 6/11. 100 4 100 5
 Required probability
required is
5

1

37 Two white balls are transferred from
 
P A  P ( A tells a lie) = 1  P(A)
24 10 120 first bag to second bag so that 8 6 48
=   3 1
4 15 11 165 = 1 
14. (C) C2
probability for that is = . In the 4 4
6
C2 Since these three cases are mutually
9 cards of a suit can be selected in
second bag we have 7 white and 4
exclusive,  
P B = P(B tells a lie) = 1  p(B)
13
C9 ways, 4 cards can be selected
 The required probability of drawing 4 1
black balls and the probability of =1 
39
from the remaining in C4 ways. a white ball 5 5
7
getting a white ball is =
The suit can be selected in 4 ways 11 42 5 48 95 Now P(A and B will contradict)
=   
165 165 165 165
 Required probability  Required probability  
= P  A  P B  P B  P A  
13
C9  39 C4  4 4
C27 6 7 42
= = 6     17. (C) 3 1 4 1 7
52
C13 C2 11 15 11 165 =      35%
Total number of cases in a throw of 4 5 5 4 20
15. (A) Second way : two dice = 36
Two black balls have been transferred
The probability that husband is not Number of cases with doublets = 6
from first bag to the second bag so
1 6
selected = 1   2
1,1 ,  2,2  ,  3,3  ,  4,4  ,  5,5  ,  6,6 
7 7 C2 1
that probability for that is 6
 
C2 15

323 324

Solutions  Probability and Statistics Vidyalankar : GATE – Engineering Mathematics

Model Solution on Assignment  5 6. (C) 10. (A)


P = probability of getting 1 in a throw Probability that any person selected at
1. (A) 4. (A) of a die = 1/6 1
random is a rice eater = . Probability
Probability of 1 appearing on upper P(A) and P(B/A) denote the probability 1 5 2
 q=1p=1 
1 of drawing a black ball in the first and 6 6 1
face = of a non rice eater = . Let A0, A1, A2,
6 second attempt Probability of getting at least one die 2
1  Probability of drawing a black ball in 3 A3 denote the event that none, one,
Probability of 6 appearing = 5 91
6 the first attempt is = 1  q3 = 1     two or three persons respectively are
6 216
 The probability 1 or 6 appear 3  3  rice eaters out of 10. Then
P(A) = 7. (A)
8 5  3
1 1 1   0 10
=  
6 6 3 P(A0) = 10C0  1   1  ,
8. (C) 2 2
Probability of drawing the second
2. (C) Let X = defective items. We seek P 1 9
black ball given the first ball drawn is
Probability of the winning of the horse (C/X) the probability that an item is P(A1) = 10C2  1   1 
black 2 2
1 produced by machine C, given that the
A= and the probability of the horse 2  2  2 8
5 P(B / A)  5  2 item is defective P(A2) = 10C2  1   1  ,
7   2 2
1 By Bayes’ theorem,
B= Probability that both the balls drawn 3 7
6 P CP  X / C  1  1
P(C/X) = P(A3) = 10C3    
are black is P  A  P  X / A   2 2
 P(A + B) = P(A) + P(B)
 
1 1 11
P(AB) = P(A) P(B/A)  P B  P  X / B   Now, the required probability
=    
5 6 30 3 2 3  P   
C P X / C  = P(A0 + A1 + A2 + A3)
=  
8 7 28 = P(A0) + P(A1) + P(A2) + P(A3)
3. (D)  0.10  0.04 
5. (D) = 1
 0.60  0.02   = [10C0 + 10C1 + 10C2 +10C3]
First draw : Probability of getting a Probability of getting a head in a toss   210
  0.30  0.03  
4 1 1   1

  0.10  0.04  
queen = of a coin = ; = [ 1 + 10 + 45 + 120]
52 13 2 210
Second draw : After drawing the first Probability of getting tail in each case 4
= 176
25 = = 0.17 = 17 % (approx)
queen we are left with 51 cards with 3 1024
1
= 9. (D)
queens 2
11. (D)
 Probability of getting a queen in Probability of getting a tail in all four Probability of six occurring in one toss
Probability that the first person lives till
3 1 1 1 1 1 1 = 1/6
second draw =  cases =     8
51 17 2 2 2 2 16 Probability of six occurring in 180 toss he is 75 years =
14
 Probability of both the cards are  Probability of getting at least one 1
= 180  = 30 Probability that the second person
6
1 1 1 1 15 4
queen =   head = 1   lives till he is 80 years =
13 17 221 16 16 9

325 326
Solutions  Probability and Statistics Vidyalankar : GATE – Engineering Mathematics

Probability of the compound event that  Variance = npq 16. (D) 18. (C)
both the persons live 40 years hence = 500  0.1  0.9 = 45 The probability that face 1 or face 2 Combinations are CMC, CMM, CCC,
8 4 32 16  Standard deviation = 45 = 6.7 turn up = 0.10 + 0.32 = 0.42 CCM.
=   
14 9 126 63 The probability that face 1 turns up Probability of
Probability that at least one of them 14. (A) = 0.10 CMC = 0.6  0.4
would die without living 40 years  =  pi xi  The required probability = 0.24  Favourable
16 47 144 24 1 0.10 10 5 CMM = 0.6  0.6 = 0.36
hence = 1   =0.  1.  2. =  
63 63 169 169 169 0.42 42 21 CCC = 0.4  0.4
26 2
12. (A)   = 0.16  Favourable
169 13 17. (C)
The probability of getting a head in a CCM = 0.4  0.6 = 0.24
2 =  pixi2  ()2 Since odds against A are 8 : 3
single toss of a coin is 1 / 2 Total Probability = 1
144 24 1 4 3 3
= 0.  1.  4.  P(A) =  Favourable = 0.24 + 0.16 = 0.4
1 1 169 169 169 169 83 11
 p = 1/2, q = 1 p = 1  =
2 2 So P = 0.4/1
28 4 24 2 2
=   P(B) = 
Also we are given n = 10, N = 100 52 7
169 169 169
and r =7 P(A) + P(B) + P(C) = 1
24 4.9
The required frequency  =  = 0.377 {mutually exclusive}
169 13
= N nCr pr qnr 3 2
15. (C) Or  + P(C) = 1
7 3 11 7
 1  1
= 100  10 C7     np = 12 npq = 2
2 2 3 2 34
 P(C) = 1   
10
 npq = 4 11 7 77
10!  1 
100.  odds against C are 77  34 = 34
7!3!  2 
npq 4 1 1
  q=
np 12 3 3 i.e. 43 : 34
10
10  9  8  1  375
= 100   1 2
3  2  1  2  32 p=1q =1 =
3 3 
= 11.7 = 12
Also np = 12
2
13. (A) n = 12
3
Let (q + p)n, q + p =1, be the binomial
2
distribution  n = 18 & p=
3
P = 0 1, n = 500
Mean = np = 0.1  500 = 50
Now p = 0.1
 q = 1  p = 1  0.1 = 0.9

327 328

Solutions  Probability and Statistics Vidyalankar : GATE – Engineering Mathematics

Model Solution on Assignment  6 6. (C) 11. (C)


52 1
Two cards can be selected in C2 P(A  B ) = P(A)  P(A  B) =
1. (B) 1,2  , 1,4 , 1,6 ,  2,1 ,  2,3 , 3
ways
S = {Sun, Mon, Tue, Wed, Thu, Fri, Sat}  2,5  ,  3,2  ,  3,4  ,  3,6  ,  4,1 , P(A  B) = P(A) + P(B)  P(A  B)
A=  n(S) = 52
C2  26  51
n(s) = 7  4,3  ,  4,5  ,  5,2  ,  5,4  ,  5,6  , P(A  B) = [P(A)  P(A  B)] + P(B)
Let A be the event that the child is born  6,1 ,  6,3  ,  6,5  Let A be the event that 2 cards selected
P(A  B) = P(A  B ) + P(B)
on Sunday or a Saturday is Jack and an Ace.
n(A) = 18 2 1
 n(A) = 2 Jack can be selected in 4 C1 ways and = + P(B)
nA 
18 1 3 3
nA   PA   
 P(A) = 
2 n  S  36 2 the Ace can also be selected in 4 C1 2 1 1
nS 7  P(B) =  =
4. (C) ways. 3 3 3
2. (D) 5 students can be selected from 10 by  n(A) = 4 C1  4 C1  16 12. (A)
S = {1, 2, 3, 4, 5, 6 } 10 1
C5 ways nA 16 8 Probability P of a white flower =
n(S) = 6  PA    4
10! n S 26  51 663
Let A be the event that the die shows a  n(S) = 10
C5 = 1 3
5!5! q = 1 P = 1  =
7. (C) 4 4
multiple of 2
10 × 9 × 8 × 7 × 6 The required probability n=3 N = 64
 A = {2, 4, 6} = = 252
5×4×3×2 r 3 r
 n(A) = 3  12 C11  (0.99875)11 (0.01125) + 12C12 3 1  3 
Let A be the event that the committee p(r) = Cr    
4 4
nA 3 1 includes exactly 2 girls and 3 boys. The (0.99875)12 = 0.9923
P(A) =   Number of beds with zero white flowers
nS 6 2 4 8. (B)
two girls can be selected in C2 ways
0 3
Required probability  1  3 
3. (C) and the three boys can be selected in = 64  3 C0    
1 1 4 1 4 4 4
S= 1,1, 1,2  , 1,3  , 1,4 , 1,5 , 1,6  6
C3 ways = .  . 
5 3 5 4 15 27
 64  = 27
 2,1 ,  2,2  ,  2,3  ,  2,4  ,  2,5  ,  2,6  n(A) = 4 C2 × 6 C3 = 6 × 20 = 120 9. (A) 64

 3,1 ,  3,2  ,  3,3  ,  3,4  ,  3,5  ,  3,6  13 4 Beds with 1 white flower
nA  120 10 P(A) =
C1

C1

1
P(A) =   52 51
nS 52
C1 C1 1 2
 4,1 ,  4,2  ,  4,3  ,  4,4  ,  4,5  ,  4,6  252 21  1  3 
= 64  3 C1     = 27
10. (D) 4 4
5. (A)
 5,1 ,  5,2  ,  5,3  ,  5,4  ,  5,5  ,  5,6 
Let A be an event that ball drawn is red 2 Beds with 2 white flowers
Probability of first event = P
 6,1 ,  6,2  ,  6,3  ,  6,4  ,  6,5  ,  6,6  or white
3 2
 1  3  9
= 64  3 C2      64  =9
n(S) = 36 7 4 Mutually exclusive events 4 4 64
 n(A) = C1  C1  7  4  11
Let A be the event that the score on the 2 3 Beds with 3 white flowers
11 P+   P=1 Or P = =3:5
dice is odd  PA  3 5 3 0
20  1  3  1
Hence odds in favor of the other are = 64  3 C3      64  =1
4 4 64
[ n  S   20 C1  20 ]
3 : (5  3) i.e. 3 : 2

329 330
Solutions  Probability and Statistics Vidyalankar : GATE – Engineering Mathematics

13. (B) 16. (B) Answer Key on Test Paper  1


1 1 1 1 1
P (A  B) = ; P(A)  ; P(B)  P(B) = 1  P( B)  1  
2 3 3 2 2 1. (D) 2. (D) 3. (A)

1 2 and P(A  B) = P(A) + P(B)  P(A  B) 4. (C) 5. (A) 6. (C)


Now, P(A) = 1  P(A)  1  
3 3 5 1 1
or = P(A) +  7. (C) 8. (B) 9. (A)
1 2 6 2 3
P(B) = 1  P(B)  1   10. (D) 11. (A) 12. (A)
3 3 2
or P(A)
 P(A  B) = P(A)  P(B)  P(A  B) 3 13. (C) 14. (A) 15. (C)

2 2 1 2 1 1
=    P(A) P(B) =   = P(A  B)
3 3 2 3 2 3
Hence A and B are independent.

4 1 83
  Answer Key on Test Paper  2
3 2 6
5 5 2 10 17. (D) 1. (B) 2. (D) 3. (D)
=   
6 6 2 12 n(s) = 24 = 16 since each of the four 4. (C) 5. (B) 6. (A)
places in a determinant of order 2 is to
14. (A) 7. (D) 8. (A) 9. (A)
filled by 0 or 1 favourable number of
To be one step away from the starting 10. (A) 11. (A) 12. (D)
ways is 3.
point the Man is to take 6 steps forward
13. (A) 14. (A) 15. (C)
and 5 steps backward or 5 steps 1 0 1 0 1 1
  or   or  
0 1 1 1 0 1
forward and 6 steps backward
 The required probability 3
Hence P =
= 11C6 (0.4)6 (0.6)5 + 11C5 (0.4)5 (0.6)6 16 Answer Key on Test Paper  3
= 11C5 (0.4)5 (0.6)5 {0.4 + 0.6}
18. (D) 1. (D) 2. (A) 3. (D)
= 11C5 (0.24)5
100 50 50 100 51 49
C50 p (1  p) = C51 p (1  p) 4. (B) 5. (A) 6. (D)
15. (A) 1 p 100! 50! 50! 50 7. (B) 8. (B) 9. (D)
or   
Required probability p 51! 49! 100! 51
10. (B) 11. (D) 12. (A)
1 2 1 3 1 or 51  51p = 50 p
=     13. (C) 14. (B) 15. (D)
2 5 2 5 2
51
giving P=
101





331 332

Solutions  Probability and Statistics Vidyalankar : GATE – Engineering Mathematics

Model Solution on Test Paper  1 6. (C) 8. (B)


R stands drawing a red ball and B for 15
c 2 27 c 2
1. (D) Let C be the event that both are P1 = 42
drawing black ball. Then required c4
2 3 selected probability is
P(A) = P(A) = 15  14  27  26  1 2  3  4
5 5  C be the event that both are 
= RRR + RBR + BRR + BBR 1 2  1 2  42  41 40  39
2 1 selected.  6 5 6   6 6 5 
P(B) = P(B) = 27
3 3 =        
 C =AB  10 11 10   10 11 10  82
3 2  P(C) = P(A  B)
P(C) = P(C) =  4 4 7   4 7 6  30
c 4 54 c 4
5 5          P2 =
 P(C) = P(A)  P(B) as A and B are  10 11 10   10 11 10  84
c8
Probability that only one of them hits
1 2 2 6400 32 30.29.28.27.54.53.52.51.8!
the target independent events =   =  
5 7 35 1100 55 4!4!84.83.82.81.80.79.78.77
= probability that A hits the target but
4. (C) 17.29.45.53
not B and C + probability that B hits =
7. (C) 11.79.82.83
12
the target but not A and C Two balls are selected from 12 in C2
6 1
P1 =  P1 27 11.79.82.83 33.79.83
+ probability that C hits the target ways. Two white balls can be selected 36 6   
P2 82 17.29.45.53 29.53.85
but not A and B 7
in C2 ways. [ out of total of 36 ways both the
216381
= P  A  B  C   P  A   B  C  7 persons can throw equal values in = >1
C 130645
 Required probability = 12 2
P  A   B  C  C2 6 ways]
Hence P1 > P2
To find p2 the total number of ways
2 1 2 2 3 2 3 3 1 21 7
=          
5 3 5 3 5 5 5 5 3 66 22 n  64 and the favorable number of
9. (A)
ways, M =15  8 = 120
4 12 9 25 1 5. (A) P(AB) = P(A) + P(B)  P(A  B)
    
75 75 75 75 3 Since any two numbers out of 6 can
Three balls can be selected from 9 in 6
P(A  B)  1
be selected in c2 i.e. 15 ways and
2. (D) 9
C3 ways  P(A) + P(B)  P(A  B)  1
corresponding to each of these ways,
Probability of selecting any bag is 1/2 P(A) + P(B)  1  P(A  B)
3 black balls can be selected from 5 in there are 8 ways e.g. corresponding to
Probability of getting a white ball P(A  B)  P(A) + P(B)  1
5
C3 ways the numbers 1 and 2 the eight ways
1 6 C1 1 4 C1 5 are (1, 1, 1, 2), (1, 1, 2, 1), (1, 2, 1, 1),
=     C 5
2 9 C1 2 9 C1 9  Required probability = 9 3 10. (D)
C3 (2, 1, 1, 1) , (2, 2, 2, 1) (2, 2, 1, 2) (2,
We first find the total number of cases.
1, 2, 2) (1, 2, 2, 2).
3. (A) 10 5 For each element of set A with n
  120 5
84 42 Hence P = 
Let A be the event that A is selected element, the number of possible
64 54
and B be the event that B is selected images is n.
1 5
 P(A) = 1/5 and P(B) = 2/7 Since  , we have P1 > P2
6 54  The total number of cases = nn.

333 334
Solutions  Probability and Statistics Vidyalankar : GATE – Engineering Mathematics

When next find the number of There are 6 way to go to city F Model Solution on Test Paper  2
favourable cases. For the first element through C.
we have n choices. For the seconds Total no. of paths = 10
1. (B)  The probabilities of Dayanand,
There are 6 possible ways in which the Ramesh, Naresh not solving the
element we have (n  1) choices and  Hence, required probability
die can fall and of these two are problem are
so on. = 6/10 = 3/5
favourable events required. 1 1 1 2 1 3
 The number of favourable cases 1  ;1   ;1  
2 1 2 2 3 3 4 4
15. (C)  Required chance = 
= n  n  1 n  2  ...2  1  n! 6 3 respectively.
x
Hence the required probability Mean x = 2. (D)  The probability that the problem is
N
n!  n  1 ! x The possible number of cases is not solved by any one of them is
=  n1  90 =
nn n 200 6  6 = 36 1 2 3 1
  
An ace on one die may be associated 2 3 4 4
 x = 200 × 90 = 18000
11. (A)
with any of the 6 numbers on the other  The probability that the problem
 Sum of observations is 18000
12. (A) die and the remaining 5 numbers on will be solved by at least one of
Of these, two wrong observations are
13. (C) the first die may be associated with the 1 3
15 and 80. them = 1  
ace on the second die, thus the 4 4
14. (A) Subtracting 15 and 80 from 18000, we
number of favorable cases is 11. 5. (B)
get 17905
Distinct paths from A to F are given The probability of drawing a black ball
Adding the correct observations to 11
below.  Required chance =
36 from the bag = 3/8
17905,
1) ABDF The probability that the drawn ball is
i.e. 17905 + 40 + 87 = 18032 3. (D)
2) ACEF
Various Digits in the log table are 0, 1, 3 5
 Correct sum of 200 observation is not black = 1 
3) ABF 8 8
18032 2, 3, 4, 5, 6, 7, 8, 9
4) ABEF 6. (A)
i.e. a total of 10 digits are used.
5) ACDF 18032
 Correct mean is = 90.16 In “PROBABILITY” there are
200 The number of favourable cases for
6) ABCDEF O A I : 3 distinct vowels
getting 1 out of the 10 all equally likely
7) ACDEF P R B L T Y : 6 distinct consonants
cases is one
8) ABDEF Total number of distinct letters = 9,
9) ABCDF  Probability of getting 1 is 1/10.
3 1
Required probability = 
10) ABCEF 4. (C) 9 3
The probabilities of Dayanand, 7. (D)
 Ramesh and Naresh solving the Let P(A), P( A ) be probabilities of A’s
problem are getting the head and not getting the
1 1 1 1
, , respectively head respectively, then P(A) =
2 3 4 2

335 336

Solutions  Probability and Statistics Vidyalankar : GATE – Engineering Mathematics

1 1 8. (A) 12. (D) 21 20  19


 P( A ) = 1 P(A) = 1  = = = 7  10  19
2 2 n(S) = 36 Var (x) = 1 3  2 1
1 1 The outcomes which have odd number  Required probability
Similarly, P(B) = and P( B ) =
2 2 and multiple of 3 are 13. (A) 1045 209
= =
Let A start the game. He can win it in (1, 3) (1, 6) (3, 1) (3, 3) (3, 5) (3, 6) 7  10  19 14  19
st rd th th
The probability that event A occurred,
the 1 throw 3 throw 5 throw, 7 (5, 3) (5, 6) (6, 1) (6, 3) (6, 5) 11 19 11
provided that B took place will be = 
throw and so on. Probability of A’s favourable cases = 11 14  19 14
1 denoted byP( A B ). Now the probability
winning in 1st throw = P(A) = 11
2 Probability = that event A occurred, provided that B
36 15. (C)
Probability of A’s winning in 3rd throw took place will be denoted by
Probability that the item is produced by
9. (A)
= P( A ) P( B ) P(A) Let A = The event that students are P(A/ B ).The event A is the non machine A, P(A) = 0.25
3
taller than 18m occurrence of the event A. Probability that the item produced by A
1 1 1  1
= . . =  
2 2 2 2 P(w /A) = Probability that a student is  We have P(A/ B ) + P( A / B ) = 1 is faulty, P(F/A) = 0.05
Probability of A’s winning in 5th throw a woman given A Similarly,
Hence P( A / B ) = 1  P(A / B )
= P( A ) P( B ) P( A ) P( B ) P(A) P(w)P(A / w) P(B) = 0.30
=
5 P(w)P(A / w)  P(M)P(A / M) P(F/B) = 0.04
1 1 1 1 1  1 14. (A)
= . . . . =   P(C) = 1  0.25  0.3 = 0.45
2 2 2 2 2 2 (0.60)(0.01) 3
=  Two points are always collinear so we
(0.60)(0.01)  (0.40)(0.04) 11 P(F/C) = 0.03
Probability of A’s winning in 7th throw can say a triangle consists of two
By total probability,
= P( A ) P( B ) P( A ) P( B ) P( A ) collinear points and one point which is
10. (A) Probability that the item selected is
P( B ) P(A) not collinear to both of them
n = total number of ways = 6  6 = 36 faulty
7 simultaneously.
1 1 1 1 1 1 1  1 The numbers higher than 9 are 10, 11, = P(A) P(F/A) + P(B) P(F/B)
= . . . . . . =   So here, we have no. of triangles
2 2 2 2 2 2 2 2 + P(C) P(F/C)
12 in the case of two dice = No. of triangles having 2 points on
Mutually exclusive cases = 0.25 × 0.05 + 0.30 × 0.04 + 0.45
 m = favourable number of ways line 1 and one point on line 2
 Probability of A’s winning the × 0.03
=3+2+1=6 + No. of triangles having 2 points
game first is = 0.038
M 6 1 on line 2 and one point on line 1
1 Hence P =  
3 5 7 n 36 6 By Baye's Theorem,
10
1  1  1  1 2 = C2  11  11C2 = 45  11 + 55  10
          ....  2  11. (A) Probability that the faulty item was
2 2 2 2 1 3
1 = 1045
4 Total number of cards =12 produced by machine C,
 Probability of B winning the game Total number of points = 10 + 11 = 21
Probability of at least one are P(C)F(F / C) 0.450.03
P(C/F) = =
2 1  No. of 3 points group P(F) 0.038
first = 1   = 1 probability no are
3 3 = 21
C3  P(C/F) = 0.355
8
C2 14 19
= 1 12
 1 
C2 33 33


337 338
Solutions  Probability and Statistics Vidyalankar : GATE – Engineering Mathematics

Model Solution on Test Paper  3 7. (B) 10. (B)


2 3 Here np + npq = 10 , n = 18
1. (D) 4. (B) P= q= n=5
5 5 18p + 18pq = 10  18p(1 + q) = 10
Probability of getting both the balls When all the letters are taken they can
5 r r
10 n nr r 3 2  p + q = 1  p = 1 q
even numbered be arranged in p10 ways or 10 ! P(r) = Cr . q p = 5
Cr   . 
5 5 18(1  q) (1 + q) = 10
P(EE) = P(E)P(E)
5. (A)  The required chance 10
12 12 144 1  q2 =
=   Here the balls are drawn without = P(4) 18
25 25 625
replacement. Let P(W) and P(G) 4 10 8
P(at least one odd) = 1  P(EE) 3 2  q2 = 1 
denote the probability of drawing white = 5 C4 . .   18 18
144 5 5
=1 ball and green ball respectively. 8 4
625 48 48 48 q2 = 
P(W and G) = P(W) P(G) + P(G) P(W) = 5 =  18 9
481 55 54 625
 10 8 8 10 80 2
625 =     q =
18 17 18 17 153 8. (B) 3
2. (A)
Total number of ways in which two 2
The total number of ways in which four 6. (D)  q= (ve sign rejected),
cards are drawn out of 52 is 52
C2 . 3
persons can be selected out of 9 The largest number on the selected
A king can be drawn in 4 ways and 1
persons is 9 C4 . coupon is 9, and hence the selection is p=
3
to be made from the coupons Queen can be drawn in 4 ways.
For favourable cases, we want that 2 Hence Binomial distribution is
numbered 1 to 9. Since there are 15 44 8
out of the four selected should be Required probability = 52
 18
C2 663  1 2
children. Two children can be selected possible cases for selecting a coupon 3  3 {(p +q)n}
 
and seven coupons are selected the
out of 4 in 4
C2 ways. The other two 9. (D)
total number of possible cases = 157 11. (D)
are to be selected out of 5 persons (3 In a single throw of two dice, the
The number of favorable cases
men and 2 women). Two persons can probability of throwing a doublet is
= 97  87. Observe that out of 97 cases 12. (A)
6 1
be selected out of 5 in 5 C2 ways. 
87 cases do not contain the number 9 36 6
 The number of favourable cases The required probability  The probability of throwing three 13. (C)
4
C2  5 C2 Number of favourable cases doublets in three throws is
=
Total number of cases
 Required probability 1 1 1 1
. .  14. (B)
4
C2  5 C2 10 97  87 6 6 6 216
=  = Out of the 5 women, 3 women can be
9
C4 21 157  Hence the required probability 5
invited in C3 ways. Nothing is
3. (D) 1 215
=1  mentioned about the number of men
216 216
 A  P  A  B  1/ 4 3 that he has to invite. He can invite one,
P    
B P B  1/ 3 4
two, three, four or even number of

339 340

Solutions  Probability and Statistics

men. Out of 4 men, he can invite them 1


P(AB) =
in the said manner in (2)4 ways. Thus, 8

P(A B) 18 1
the total number of ways is 5 C3  (24 ) P(A B) = = 
P(B) 12 4
= 10  16 = 160.
1
 P(A B) = is false.
2
15. (D) If A  B , A  B = A
P(A) = 1/4
then P(A  B) = P(A)
P(A B) 1
P(B/A) = = 1 1
P(A) 2 but P(A  B) = and P(A) =
8 4
1  This is false.
 P(A  B) =
8
1 1 3
P(AB) 1 P(A B)P(A B) =  =
P(A/B) = = 4 2 4
P(B) 4
 P(A B)P(A B) = 1 is false
1
 P(B) =  None of the options (A), (B) or (C)
2
is true.
P(AB) = P(A)  P(A  B)

1 1
= 
4 8



341

You might also like